Download as pdf or txt
Download as pdf or txt
You are on page 1of 237

MICROBIOLOGY

USMLE WORLS STEP 1 MICROBIOLOGY

Question List
Microbiology Q No: 1 Musculoskeletal Microbiology Q No: 43 Pulmonology
Microbiology Q No: 2 Hepatobiliary system Microbiology Q No: 44 Dermatology
Microbiology Q No: 3 Genitourinary Microbiology Q No: 45 Genitourinary
Microbiology Q No: 4 Genitourinary Microbiology Q No: 46 Musculoskeletal
Microbiology Q No: 5 Cardiology Microbiology Q No: 47 Dermatology
Microbiology Q No: 6 Hematology Microbiology Q No: 48 Pulmonology
Microbiology Q No: 7 Head and neck Microbiology Q No: 49 Gastrointestinal system
Microbiology Q No: 8 Cardiology Microbiology Q No: 50 Neurology
Microbiology Q No: 9 Neurology Microbiology Q No: 51 Pulmonology
Microbiology Q No: 10 Genitourinary Microbiology Q No: 52 Pulmonology
Microbiology Q No: 11 Genitourinary Microbiology Q No: 53 Head and neck
Microbiology Q No: 12 Neurology Microbiology Q No: 54 Musculoskeletal
Microbiology Q No: 13 Pulmonology Microbiology Q No: 55 Pulmonology
Microbiology Q No: 14 Hepatobiliary system Microbiology Q No: 56 Musculoskeletal
Microbiology Q No: 15 Hepatobiliary system Microbiology Q No: 57 Pulmonology
Microbiology Q No: 16 Head and neck Microbiology Q No: 58 Head and neck
Microbiology Q No: 17 Hematology Microbiology Q No: 59 Genitourinary
Microbiology Q No: 18 Pulmonology Microbiology Q No: 60 Hepatobiliary system
Microbiology Q No: 19 Pulmonology Microbiology Q No: 61 Head and neck
Microbiology Q No: 20 Genitourinary Microbiology Q No: 62 Gastrointestinal system
Microbiology Q No: 21 Neurology Microbiology Q No: 63 Hepatobiliary system
Microbiology Q No: 22 Head and neck Microbiology Q No: 64 Genitourinary
Microbiology Q No: 23 Oncology Microbiology Q No: 65 Musculoskeletal
Microbiology Q No: 24 Genitourinary Microbiology Q No: 66 Hepatobiliary system
Microbiology Q No: 25 Hematology Microbiology Q No: 67 Head and neck
Microbiology Q No: 26 Neurology Microbiology Q No: 68 Head and neck
Microbiology Q No: 27 Neurology Microbiology Q No: 69 Pulmonology
Microbiology Q No: 28 Cardiology Microbiology Q No: 70 Neurology
Microbiology Q No: 29 Pulmonology Microbiology Q No: 71 Gastrointestinal system
Microbiology Q No: 30 Hematology Microbiology Q No: 72 Gastrointestinal system
Microbiology Q No: 31 Musculoskeletal Microbiology Q No: 73 Head and neck
Microbiology Q No: 32 Hepatobiliary system Microbiology Q No: 74 Gastrointestinal system
Microbiology Q No: 33 Pulmonology Microbiology Q No: 75 Head and neck
Microbiology Q No: 34 Cardiology Microbiology Q No: 76 Musculoskeletal
Microbiology Q No: 35 Pulmonology Microbiology Q No: 77 Cardiology
Microbiology Q No: 36 Pulmonology Microbiology Q No: 78 Hepatobiliary system
Microbiology Q No: 37 Gastrointestinal system Microbiology Q No: 79 Neurology
Microbiology Q No: 38 Head and neck Microbiology Q No: 80 Neurology
Microbiology Q No: 39 Gastrointestinal system Microbiology Q No: 81 Genitourinary
Microbiology Q No: 40 Pulmonology Microbiology Q No: 82 Neurology
Microbiology Q No: 41 Head and neck Microbiology Q No: 83 Pulmonology
Microbiology Q No: 42 Pulmonology Microbiology Q No: 84 Cardiology
Microbiology Q No: 85 Musculoskeletal Microbiology Q No: 128 Blood vessels
Microbiology Q No: 86 Gastrointestinal system Microbiology Q No: 129 Head and neck

136
USMLE WORLS STEP 1 MICROBIOLOGY

Microbiology Q No: 87 Hepatobiliary system Microbiology Q No: 130 Hematology


Microbiology Q No: 88 Musculoskeletal Microbiology Q No: 131 Musculoskeletal
Microbiology Q No: 89 Neurology Microbiology Q No: 132 Gastrointestinal system
Microbiology Q No: 90 Head and neck Microbiology Q No: 133 Cardiology
Microbiology Q No: 91 Hematology Microbiology Q No: 134 Pulmonology
Microbiology Q No: 92 Pulmonology Microbiology Q No: 135 Neurology
Microbiology Q No: 93 Reproductive system Microbiology Q No: 136 Gastrointestinal system
Microbiology Q No: 94 Neurology Microbiology Q No: 137 Genitourinary
Microbiology Q No: 95 Pulmonology Microbiology Q No: 138 Genitourinary
Microbiology Q No: 96 Head and neck Microbiology Q No: 139 Hepatobiliary system
Microbiology Q No: 97 Head and neck Microbiology Q No: 140 Head and neck
Microbiology Q No: 98 Genitourinary Microbiology Q No: 141 Hepatobiliary system
Microbiology Q No: 99 Neurology Microbiology Q No: 142 Hepatobiliary system
Microbiology Q No: 100 Neurology Microbiology Q No: 143 Gastrointestinal system
Microbiology Q No: 101 Musculoskeletal Microbiology Q No: 144 Pulmonology
Microbiology Q No: 102 Head and neck Microbiology Q No: 145 Head and neck
Microbiology Q No: 103 Pulmonology Microbiology Q No: 146 Gastrointestinal system
Microbiology Q No: 104 Neurology Microbiology Q No: 147 Pulmonology
Microbiology Q No: 105 Neurology Microbiology Q No: 148 Pulmonology
Microbiology Q No: 106 Dermatology Microbiology Q No: 149 Genitourinary
Microbiology Q No: 107 Genitourinary Microbiology Q No: 150 Gastrointestinal system
Microbiology Q No: 108 Reproductive system Microbiology Q No: 151 Hepatobiliary system
Microbiology Q No: 109 Pulmonology Microbiology Q No: 152 Genito urinary
Microbiology Q No: 110 Cardiology Microbiology Q No: 153 Pulmonology
Microbiology Q No: 111 Genitourinary Microbiology Q No: 154 Gastrointestinal system
Microbiology Q No: 112 Pulmonology Microbiology Q No: 155 Pulmonology
Microbiology Q No: 113 Hepatobiliary system Microbiology Q No: 156 Dermatology
Microbiology Q No: 114 Neurology Microbiology Q No: 157 Pulmonology
Microbiology Q No: 115 Pulmonology Microbiology Q No: 158 Pulmonology
Microbiology Q No: 116 Hepatobiliary system Microbiology Q No: 159 Cardiology
Microbiology Q No: 117 Neurology Microbiology Q No: 160 Hepatobiliary system
Microbiology Q No: 118 Pulmonology Microbiology Q No: 161 Genitourinary
Microbiology Q No: 119 Pulmonology Microbiology Q No: 162 Neurology
Microbiology Q No: 120 Pulmonology Microbiology Q No: 163 Pulmonology
Microbiology Q No: 121 Gastrointestinal system Microbiology Q No: 164 Hematology
Microbiology Q No: 122 Gastrointestinal system Microbiology Q No: 165 Musculoskeletal
Microbiology Q No: 123 Cardiology Microbiology Q No: 166 Neurology
Microbiology Q No: 124 Head and neck Microbiology Q No: 167 Gastrointestinal system
Microbiology Q No: 125 Gastrointestinal system Microbiology Q No: 168 Neurology
Microbiology Q No: 126 Hepatobiliary system Microbiology Q No: 169 Pulmonology
Microbiology Q No: 127 Pulmonology Microbiology Q No: 170 Hepatobiliary system

137
USMLE WORLS STEP 1 MICROBIOLOGY

Microbiology Q No: 171 Head and neck Microbiology Q No: 181 Pulmonology
Microbiology Q No: 172 Musculoskeletal Microbiology Q No: 182 Hematology
Microbiology Q No: 173 Dermatology Microbiology Q No: 183 Neurology
Microbiology Q No: 174 Pulmonology Microbiology Q No: 184 Pulmonology
Microbiology Q No: 175 Pulmonology Microbiology Q No: 185 Genitourinary
Microbiology Q No: 176 Head and neck Microbiology Q No: 186 Dermatology
Microbiology Q No: 177 Head and neck Microbiology Q No: 187 Pulmonology
Microbiology Q No: 178 Musculoskeletal Microbiology Q No: 188 Pulmonology
Microbiology Q No: 179 Head and neck Microbiology Q No: 189 Pulmonology
Microbiology Q No: 180 Genito urinary Microbiology Q No: 190 Cardiology

138
USMLE WORLS STEP 1 MICROBIOLOGY

Q NO 1: A 10-year-old Caucasian male is brought to the ER with a several day


history of high fevers and chills. He also complains of dull pain just above the
left knee. There is no joint effusion. X-ray films show soft tissue swelling, bone A.
destruction, and periosteal reaction over the lower end of femur. Which of the
following organisms is most likely responsible for this patient’s symptoms?
Staphylococcus aureus
B. Staphylococcus epidermidis
C. Streptococcus pyogenes
D. Streptococcus agalactiae
E. Streptococcus pneumoniae
F. Streptococcus faecalis
G. Moraxella catarrhalis

Explanation:
Hematogenous osteomyelitis is predominantly a disease of children that most
frequently affects the long bones. When this condition occurs in adults, it is
frequently due to a predisposition for bacteremia such as IV drug abuse or dialysis.
The presenting symptoms of hematogenous osteomyelitis are vague, and an
increased clinical suspicion is required to arrive at the diagnosis. Commonly,
symptoms are similar to those of any systemic infection: malaise, fevers and pain
over one of the long bones. Bone pain is caused by abscess formation within the
bone leading to periosteal disruption and bone necrosis. Staphylococcus aureus is
implicated in most cases of acute hematogenous osteomyelitis in otherwise healthy
children.
(Choice B) Staphylococcus epidermidis is commonly isolated in cultures due to its
ubiquitous nature, and in these cases it is necessary to rule out culture
contamination. S. epidermidis can colonize indwelling intravenous catheters leading
to bacteremia and sepsis, and it can also cause infections of foreign bodies such as
prosthetic heart valves and orthopedic hardware.
(Choice C) After Staphylococcus aureus, Streptococcus pyogenes (Group A
Streptococcus) is isolated from most cases of hematogenous osteomyelitis in
children. Group A Streptococci are also responsible for streptococcal pharyngitis and
skin infections such as impetigo and necrotizing fasciitis. Following either of these
infections, a patient has the potential to develop rheumatic fever and/or post-
streptococcal glomerulonephritis.
(Choice D) Streptococcus agalactiae, commonly known as Group B Strep, colonizes
the vagina of young women, and infants born vaginally to colonized mothers can
become infected. The most common neonatal infections include sepsis, pneumonia,
and meningitis. This is the reason for testing expecting mothers for vaginal
colonization with Group B Streptococci and treating those who are colonized shortly
prior to delivery.
(Choice E) Streptococcus pneumoniae is the most common etiologic agent of
community-acquired pneumonia. It also causes otitis media in children, sinusitis,
meningitis, and sepsis.
(Choice F) Streptococcus faecalis is more commonly known as Enterococcus faecalis.
Diseases caused by this agent include subacute bacterial endocarditis and urinary
tract infections.
(Choice C) Moraxella catarrhalis is a part of the normal flora of the upper respiratory
tract of healthy adults, children and the elderly. It causes otitis media and sinusitis
in healthy individuals and is commonly responsible for bronchitis and pneumonia in
elderly patients with chronic obstructive pulmonary disease.
Educational Objective:

139
USMLE WORLS STEP 1 MICROBIOLOGY

Hematogenous osteomyelitis most commonly occurs in male children and is most


frequently caused by Staphylococcus aureus after some bacteremic event.
Symptoms are vague and include fever, malaise and pain in the long bones most
frequently.

140
USMLE WORLS STEP 1 MICROBIOLOGY

Q NO 2: Liver biopsy of a 36-year-old Caucasian male with a long history of


constitutional symptoms stains positive for HBsAg using immunoperoxidase. A.
Which of the following best describes the viral genome replication in this One-
patient’s liver cells?
stranded DNA — Template double-stranded DNA — Progeny one-stranded DNA
B. Double-stranded DNA — Template double-stranded DNA — Progeny double-
stranded DNA
C. Double-stranded DNA —? Template +RNA — Progeny double-stranded DNA
D. One-stranded +RNA — Template -RNA — Progeny one-stranded +RNA
E. One-stranded +RNA — Template double-stranded DNA — Progeny one-stranded
+RNA
F. One-stranded -RNA — Template +RNA — Progeny one-stranded -RNA

Explanation:
A member of the DNA-containing family Hepadnaviridae, the mature hepatitis B
virus is a spherical double-layered “Dane particle” that has a hexagonal core covered
with an outer surface envelope of protein, lipid, and carbohydrate. The HBV genome
is a partially double stranded circular DNA molecule, and replication of this genome
is accomplished through a reverse transcriptase DNA polymerase that creates an
intermediate + single-stranded RNA template. The progeny of this process is double-
stranded DNA.
(Choice A) This replicative sequence characterizes parvovirus B19.
(Choice B) This replicative sequence characterizes papovavirus, adenovirus,
herpesvirus, and poxvirus.
(Choice D) This replicative sequence characterizes poliovirus.
(Choice E) This replicative sequence characterizes the retroviruses (eg, HIV).
(Choice F) This replicative sequence characterizes influenza virus, measles virus, and
rabies virus.

Educational Objective:
The hepatitis B virus replicates via the following sequence: double-stranded DNA —
template ±RNA — progeny double-stranded DNA.

141
USMLE WORLS STEP 1 MICROBIOLOGY

Q NO 3: A 34-year-old male presents to clinic with white plaques on his buccal


mucosa that were discovered incidentally while brushing his teeth. He is found
to be HI V-positive with a CD4+ T lymphocyte count of 280/µL. Two years A.
later, he returns with itching and pain in the perirectal area. Examination
shows an ulcerative mass. Which of the following pathogens is most likely
responsible for his current condition?
Herpes simplex virus-2
B. Cytomegalovirus
C. Human papilloma virus
D. Epstein-Barr virus
E. Adenovirus
F. Chlamydia trachomatis
G. Candida albicans

Explanation:
This patient most likely has squamous cell carcinoma of the anus. Anogenital
squamous cell carcinomas and their proposed precursors, squamous intraepithelial
lesions, have been linked to infection with human papilloma virus (HPV)
intraepithelial neoplasia of the cervix vulva, penis, and anus have a clear and well-
documented association with HPV types 16 and 18. However a definitive correlation
between HPV and invasive carcinoma has only been established for cervical cancer.
Immunodeficiency states (eg, AIDS) increase the host’s susceptibility to HPV
infection. As a consequence HIV infection is associated with a higher incidence of
anogenital carcinomas. HIV-positive homosexual males are more prone to
developing anal squamous cell carcinoma, and HI V-positive females are more prone
to developing cervical squamous cell carcinoma.
(Choice A) Although AIDS increases the prevalence of herpes simple x virus type 2
(HSV-2) infection and the frequency of symptomatic genital herpes recurrences
HSV-2 is not known to cause anogenital carcinoma.
(Choice B) HI V-positive patients who have CD4+ cell counts of less than 100
cells/mm3 are at significantly increased risk of developing cytomegalovirus (CMV)
infection. CMV most frequently causes retinitis in AIDS patients. Gastrointestinal
tract involvement can include esophageal ulcers and colitis. However CMV is not
associated with the development of anogenital carcinoma.
(Choices D and G) HIV-positive patients often experience reactivation of latent
Epstein-Barr virus (EBV) infection. EBV replication in such patients is associated with
large cell non-Hodgkin’s lymphomas and oral hairy leukoplakia (OHL). OHL clinically
presents as single or multiple white plaques on the lateral tongue margins. (This
patient’s white buccal plaques in contrast are more likely due to candidiasis.) EBV is
not associated with the development of anogenital carcinoma.
(Choice E) Adenoviruses can cause severe upper respiratory illnesses pneumonia
and disseminated infection in the immunosuppressed. However these viruses are not
associated with the development of anogenital carcinoma.

Educational Objective:
Human papilloma virus (HPV) types 16, 18, and 31 are strongly associated with anal
and cervical squamous cell carcinoma. HIV infection increases the prevalence of HPV
infection and increases the risk of anal carcinoma in HIV positive homosexual males.

142
USMLE WORLS STEP 1 MICROBIOLOGY

Q NO 4: A 24-year-old female presents to your office with burning urination,


urgency and frequency. She is sexually active. Urine cultures show catalase-
positive, gram-positive cocci. The organism responsible for this patient’s A.
symptoms is most likely to be:
Coagulase positive
B. Hemolytic
C. Novobiocin resistant
D. DNase positive
E. Yellow pigment producer

Explanation:
The Staphylococci are Gram-positive cocci that form clusters, pairs and, rarely, short
chains. The catalase test (with 3% hydrogen peroxide) differentiates Streptococci
(catalase-negative) from Staphylococci (catalase-positive). The ability to clot blood
plasma (slide and tube coagulase tests) separates Staphylococci into two groups:
the coagulase positive Staphylococci which constitutes the most pathogenic species
Staphylococcus aureus, and coagulase negative staphylococci (CNS) which
constitutes S. epidermidis, S. Saprophyticus, S. haemolyticus, and 30+ other
species. The coagulase-negative staphylococci exist as part of the normal flora on
the skin and in the throat and nose, and only some species can cause infections.
S. saprophyticus a common cause of urinary tract infection: itis responsible for
almost half of all UTIs in sexually active young women. S. saprophyticus is resistant
to novobiocin. When catalase-positive, coagulase-negative gram- positive cocci in
clusters are isolated from urine specimens of the above group of patients the
laboratory performs a novobiocin test to distinguish this organism from other similar
pathogens.

143
USMLE WORLS STEP 1 MICROBIOLOGY

(Choice A) Coagulase positivity is a characteristic of S. aureus; this is how S. aureus


is differentiated from the other species of Staphylococci that do not express
coagulase.
(Choice B) Hemolysis is a typical feature of Streptococci (streptolysin O and
streptolysin S) and Staphylococcus aureus (hemolysin). S. saprophyticus does not
cause hemolysis.
(Choice D) DNase is produced by group A streptococcus. DNase degrades DNA in
pus to facilitate spread of the organism. Anti DNase can be used as a laboratory test
in patients who have had streptococcal infection followed by glomerulonephritis.
DNase is not produced by S. saprophyticus.
(Choice E) Yellow pigment is produced by Staphylococcus aureus. Staphylococcus
mucus usually does not cause urinary tract infections. If S. aureus is cultured from
the urine you should suspect a metastatic infection from another location in the body
(an abscess or infective endocarditis etc.)

Educational Objective:
S saprophyticus is responsible for almost half of all UTIs in sexually active young
women. Staphylococcus saprophyticus belongs to coagulase negative staphylococci
and is unique among these because it is resistant to novobiocin.

144
USMLE WORLS STEP 1 MICROBIOLOGY

Q NO 5: An 8-year-old Caucasian boy is brought to your office with throat pain fever
and malaise. Physical examination reveals white exudates on his tonsils and swollen
anterior cervical lymph nodes. If the boy returns in a month with fatigue joint pain
and chest pain and later in life he develops a heart murmur, which of the following
would be the most likely explanation for his condition?

A. Protein A-mediated opsonization block


B. Bacterial and human epitope homology
C. Immune complex deposition
D. Exotoxin-induced T-cell receptor activation
E. Coronary artery aneurysm formation

Explanation:
The patient described in this question exhibits a clinical picture consistent with acute
rheumatic fever as well as long term sequelae of rheumatic fever. Rheumatic fever is
a syndrome of fever arthritis, subcutaneous nodules characteristic rash (erythema
marginatum), involuntary rh4hmic movements of the extremities (Sydenham
chorea) and myocarditis leading to valvular in sufficiency of the mitral or aortic
valves. It follows untreated Group A Streptococcal (GAS) pharyngitis. The symptoms
of rheumatic fever result from structural homology between antigenic determinants
(epitopes) on GAS and on human cardiac CNS and cutaneous tissue (Choice B).
(Choice A) Protein A is a cell wall component of Staphylococcus aureus, which
consists of a single polypeptide chain. This polypeptide chain binds to the Ec portion
of lgG causing the epitope binding sites of the lgG to face away from the bacterial
cell shielding the cell from complement fixation and phagocytosis. It does not play a
role in the pathogenesis of acute rheumatic fever.
(Choice C) Immune complex deposition with subsequent complement fixation is the
general mechanism behind all Type Ill hypersensitivity reactions. Post-streptococcal
glomerulonephritis is a specific example of a Type Ill hypersensitivity reaction.
(Choice D) Exotoxin-induced T-cell receptor activation is caused by superantigens.
This is the mechanism of action of the toxic-shock syndrome exotoxins produced by
both Staphylococcus aureus and Streptococcus pyogenes. This is not the etiology of
acute rheumatic fever.
(Choice E) Coronary artery aneurysm formation is a feared sequela of Kawasaki
disease (mucocutaneous lymph node syndrome). Kawasaki disease is a vasculitis of
children characterized by high fever palmoplantar erythema with periungual
desquamation, oral mucosal and conjunctival inflammation and cervical
lymphadenopathy.

Educational Objective:
Rheumatic fever is an autoimmune reaction that occurs following untreated
Streptococcus pyogenes (GAS) pharyngitis. Antigenic similarity between bacterial
antigens and normal “self” antigens in the heart and CNS are believed to cause
formation of anti-self antibodies resulting in RE.

145
USMLE WORLS STEP 1 MICROBIOLOGY

Q NO 6: A 5-year-old Caucasian male is brought to the ER with somnolence,


lethargy and oliguria. He developed diarrhea several days ago that later
became frankly bloody. Laboratory studies show elevated blood urea A.
nitrogen and creatinine. Peripheral blood smear reveals fragmented
erythrocytes. This patient’s condition is most likely related to consumption
of which of the following foods?
Custard
B. Mayonnaise
C. Undercooked beef
D. Fried rice
E. Oysters
F. Eggs
G. Canned beans

Explanation:
Hemolytic uremic syndrome (HUS) is characterized by microangiopathic hemolytic
anemia thrombocytopenia and renal insufficiency. It can occur after gastrointestinal
infection caused by the Escherichia coil strain 0157:H7, a special strain of
enterohemorrhagic E. coil (EHEC). The usual clinical picture associated with EHEC is
that of a hemorrhagic colitis with hemorrhagic diarrhea and severe abdominal
cramping caused by EHEC’s ability to secrete a toxin similar to that of Shiga toxin.
HUS tends to occur most commonly in children under l0years old and in association
with treatment of EHEC gastroenteritis with antibiotics.
Most cases of HUS associated with EHEC 0157:H7 have been associated with eating
undercooked, contaminated ground beef. Person-to-person contact in families and
childcare centers is also an important mode of transmission. Infection can also occur
after drinking raw unpasteurized milk and after swimming in or drinking sewage-
contaminated water. Thoroughly cooking ground beef avoiding unpasteurized milk,
and washing hands carefully during food preparation can prevent infection.
(Choices A and B) The preformed heat stable enterotoxin of Staphylococcus aureus
can be present most commonly in custard, mayonnaise and processed or salted
meats and cause food poisoning leading to abdominal cramping, vomiting and
diarrhea. The onset of symptoms is rapid in these cases as the toxin is preformed,
and the diarrhea is watery and nonbloody. Symptoms usually resolve within 24
hours.
(Choice D) Fried rice ingestion followed by gastroenteritis is most commonly
associated with Bacillus cereus emetic type food poisoning.
(Choice E) Consumption of raw oysters is associated with Vibrio Parahaemolyticus
(voluminous watery diarrhea, like cholera), Vibrio vulnificus (sepsis with up to a
50% mortality), as well as other Vibrio infections.
(Choice F) Raw egg consumption as well as raw chicken consumption and improper
food handling is associated with Salmonella gastroenteritis.
(Choice G) Canned beans or any other canned or jarred food can be contaminated
with Clostridium botulinum, especially if home-canned.

Educational Objective:
Escherichia coil strain 0157:H7 can cause hemolytic-uremic syndrome (HUS). HUS is
a rare condition affecting mostly children under the age of 10. This strain of E. coli is
most commonly contracted after eating undercooked ground beef.

146
USMLE WORLS STEP 1 MICROBIOLOGY

Q NO 7: A 47-year-old Caucasian male undergoing treatment for chronic myeloid


leukemia (CML) complains of headaches and scant nasal discharge. Physical
examination reveals tenderness over the paranasal areas. Biopsy of the sinus
mucosa reveals the findings below.

Which of the following is the most likely cause of this patient’s condition?
A. Malassezia furfur
B. Microsporum canis
C. Rhizopus species
D. Aspergillus fumigatus
E. Candida albicans
F. Cryptococcus neoformans
G. Blastomyces dermatitidis
H. Histoplasma capsulatum
I. Coccidioides immitis
J. Sporothrix schenckii

Explanation:
The image above shows fungal hyphae branching at acute angles in a V-shaped
pattern. This is the characteristic histologic appearance of Aspergillus fumigatus.
This fungus is widely distributed in the environment and commonly grows on
decaying vegetables. It is monomorphic, existing only in mold form (with
multicellular hyphae).
Aspergillus can cause the following conditions:
1. Invasive aspergillosis develops in immunosuppressed patients. The neutropenia
associated with leukemias and lymphomas is strongly associated with invasive
aspergillosis. The lung is the area most commonly affected. Patients present with
hemoptysis and lung granulomas. Aspergillus has a predilection for blood vessels,
spreading hematogenously and potentially causing tissue infarcts in the skin
paranasal sinuses, kidneys endocardium and brain. Diagnosis is made by light
microscopy of tissue specimens, which reveal V shaped, branching septate hyphae
invading the tissue. Amphotericin B is used to treat invasive aspergillosis.
2. Aspergillus can grow in old lung cavities (produced by tuberculosis or
bronchiectasis), forming aspergillomas, also called ‘fungus balls.” Fungus balls grow
inside the cavity only; they do not invade the surrounding lung tissue. Aspergillomas
can be surgically removed.
3. In patients with asthma Aspergillus can cause allergic bronchopulmonary
aspergillosis (ABPA). Patients present with wheezing and migratory pulmonary
infiltrates. Increased serum IgE and increased titers of antibodies against Aspergillus
are characteristic. ABPA is treated with steroids.

147
USMLE WORLS STEP 1 MICROBIOLOGY

(Choice C) Rhizopus and Mucor cause mucormycosis. The typical presentation is a


paranasal infection in a diabetic patient. Unlike Aspergillus, Mucor and Rhizopus form
broad non-septate hyphae that branch at right angles.
(Choices G, H and I) Blastomyces, Histoplasma, and Coccidioides are dimorphic
fungi. They grow as molds in nature, but exist in the human body in yeast form.
Thus, there would not be hyphae seen in biopsy specimens.
(Choice E) Candida albicans is a component of normal human mucous membrane
flora. On light microscopy, it appears as oval budding yeast and pseudhyphae.
(Pseudhyphae are not true hyphae; rather, they are formed by elongated yeast.)
(Choice F) Cryptococcus neoformans causes meningitis and lung infections in AIDS
patients. It is an oval budding yeast with a thick capsule. India ink stain of the CSF
reveals a wide clear zone around the nucleus (corresponds to unstained capsule).

Educational Objective:
Neutropenic patients are at risk for infection with Aspergillus fumigatus. This fungus
produces septate hyphae with V shaped branching. It causes invasive aspergillosis,
aspergillomas and allergic pulmonary aspergillosis (ABPA).

148
USMLE WORLS STEP 1 MICROBIOLOGY

Q NO 8: An 8-year-old male who recently moved to the United States from Asia is
brought to the emergency room with throat pain and difficulty breathing. Soon
after being admitted to the hospital, he dies of severe heart failure. In the
laboratory, bacterial isolates from this patient’s tonsils are found to cause rapid
death of 8 out of 10 exposed guinea pigs. Two of the 10 experimental animals
survive. The surviving animals most likely possess which of the following
antibodies?
A. IgA against adhesion proteins
B. Opsonizing 1gM
C. Complement-fixing 1gM
D. Immobilizing lgG
E. lgG against circulating proteins

Explanation:
Diphtheria is an acute bacterial disease that is rare in the United States due to
widespread administration of the Diphtheria-Pertussis-Tetanus (DPT) childhood
vaccine. The DPT vaccine contains diphtheria toxoid, which stimulates production of
neutralizing antibodies against the binding component (B subunit) of the diphtheria
exotoxin. Antibody binding prevents the exotoxin from attaching to host cell
membrane receptors, thus preventing disease (choice E).
Corynebacterium diphtheriae colonizes the or oropharynx of non-immunized children
and can cause pharyngitis. Patients may have a dark pseudomembrane in the
posterior pharynx (composed of C. diphtheriae bacteria leukocytes, fibrin and
necrotic mucosal epithelial cells) on physical exam. This pseudomembrane is
characteristically tightly adherent and will cause bleeding if avulsed.
The bacteria, however, are not the direct cause of the major clinical sequelae of
diphtheria; instead, it is the C. diphtheriae exotoxin that exerts these effects.
Exotoxin is released into the bloodstream and deactivates elongation factor-2(EF-2),
a protein that facilitates the movement of the forming peptide chain on the human
ribosome. This effectively inhibits human protein synthesis. The clinical consequence
is cardiac and neural toxicity resulting in cardiomyopathy and heart failure as well as
neuropathy, paralysis, coma and death in 10% of patients.
(Choice A) IgA is the primary immunoglobulin found on mucosal surfaces and in
secretions. C. diphtheriae expresses K antigen, an antiphagocytic molecule, on its
cell wall. This allows the organism to colonize the posterior pharynx and produce
exotoxin despite the action of IgA. (Choices B and C) Disease caused by C.
diphtheriae is not due to bacterial invasion of the host; rather, it is due to circulating
exotoxin. Neither 1gM opsonization nor complement fixation would affect circulating
exotoxin, thus neither would prevent the development of severe disease and death.
(Choice D) Corynebacteria are nonmotile organisms, so an immobilizing antibody
would not be relevant here. Furthermore, C. diphtheriae causes disease via its
exotoxin, which would not be affected by an immobilizing lgG.
Educational Objective:
C. diphtheriae causes diphtheria, an acute bacterial disease that initially affects the
oropharynx. The organism is spread by respiratory droplet trans mission and causes
disease via its AB exotoxin. The B (think: binding) subunit allows penetration of the
A (think: active) subunit into the cell, to inhibit ribosome function. Neural and
cardiac toxicity are serious potential sequelae. Immunization with the diphtheria
toxoid induces production of circulating lgG against the exotoxin B subunit,
effectively preventing disease.

149
USMLE WORLS STEP 1 MICROBIOLOGY

Q NO 9: CSF cultures from a neonate with fever and poor feeding reveal motile
Gram-negative rods that form pink colonies on MacConkey agar. Which of the
following is the most important bacterial factor in the development of this A.
infection?
Capsule
B. Verotoxin
C. antigen
D. Fimbrial antigen
E. Lipid A

Explanation:
The neonate in the above vignette suffers from meningitis secondary to an E. coil
infection. E. coil is a motile gram- negative bacillus that is facultatively anaerobic
and able to ferment both lactose and glucose. It grows well on blood, MacConkey,
and eosin methylene blue (EMB) agar plates. MacConkey agar is a selective and
differential medium used to isolate gram-negative organisms from contaminants in
clinical specimens. When an organism ferments lactose on MacConkey agar, the
local drop in pH causes the colony to take on a pink-red appearance. The bile salts
present in MacConkey agar prevent the growth of Gram-positive organisms;
therefore any CSF culture showing growth on MacConkey agar implies CNS infection
with enteric bacteria—specifically, infection by E. coil.
Group B Streptococcus is the most common cause of neonatal meningitis in the
United States, followed by E. coil Listeria monocytogenes, and Klebsiella
pneumoniae. Hemophilus influences type b also remains an important cause of
meningitis in non immunized infants.
The K-1 capsular antigen is present in 20-40% of intestinal E. coil isolates. E. coil
can invade the blood stream of infants from the nasopharynx or GI tract and can
then travel hematogenously to the meninges. The K-i antigen is considered the
major determinant of virulence among strains of E. coil that cause neonatal
meningitis. The K-1 antigen inhibits complement phagocytosis, and other host
responses. The capsule is immunogenic and anti capsular antibodies are protective.
(Choice B) Verotoxin is another name for the shiga-like toxin synthesized by
enterohemorrhagic E. coil (EHEC). Exotoxin expression is not present in any of the
pathogens commonly associated with neonatal meningitis.
(Choice C) The O antigen is simply a cell wall outer membrane polysaccharide
antigen used to classify gram- negative bacteria.
(Choice D) Fimbriae, or pili, are a virulence factor that allows bacteria to adhere to
the target tissue thereby establishing infection. Examples of organisms that use pili
are Neisseria is meningitidis, uropathogenic and diarrheogenic E. coil and Vibrio
cholerae.
(Choice E) The Lipid A in LPS is similar for all enterobacteria; Lipid A causes
activation of macrophages, which leads to widespread release of IL-i and TNF-alpha,
which in turn cause the signs and symptoms of septic shock.

Educational Objective:
E. coli is a frequent cause of neonatal meningitis second only to Group B
Streptococci (GBS). The capsule synthesized by some E. coil (K-1 antigen) is a
virulence factor that allows the bacteria to survive hematogenous spread and to
establish meningeal infection. Most strains of E. coil causing neonatal meningitis do
possess this K-1 antigen.

150
USMLE WORLS STEP 1 MICROBIOLOGY

Q NO 10: A 24-year-old male presents to your office with an extremely painful


vesicular rash on the shaft of his penis. He has never had such symptoms
before. Tzanck smear of the lesion is positive for multinucleated giant cells. A.
The lesion heals in ten days without any treatment but reappears two months
later. Which of the following had the greatest potential to prevent a recurrence
of this patient’s condition?
Regular condom use after the first episode
B. Short course of acyclovir during the first episode
C. Immunoglobulin during the first episode
D. Daily valacyclovir after the first episode
E. Daily lamivudine after the first episode

Explanation:
This previously asymptomatic patient’s new vesicular rash and positive Tzanck
smear are consistent with a primary genital herpes infection. Most herpes infections
below the waist are caused byHSV-2 and occur due to reactivation of latent infection
within the 3-2. 3-3, and 3-4 dorsal root (sensory) ganglia. Recurrence of genital
herpes can be suppressed or minimized with daily oral valacyclovir, acyclovir, or
famciclovir. Such suppressive therapy can reduce the number of clinical recurrences
by approximately 50% and the number of viral culture-positive recurrences by up to
70%.
(Choice A) Although condom use may prevent transmission of HSV-2 to a sexual
partner, it does not reduce the frequency with which HSV-2 genital herpes recurs in
the host. Condom use might have prevented this patient from acquiring his primary
genital HSV-2 infection, however.
(Choice B) Acyclovir treatment during a primary herpetic episode usually reduces the
duration of viral shedding time for lesional healing constitutional symptoms and local
pain. However treatment with oral acyclovir for 7 to 10 days during the primary
herpetic infection does not appear to alter viral latency or recurrence.
(Choice C) Immunoglobulin therapy is not commonly used for genital herpes simplex
infections. Such therapy can be used to combat varicella-zoster infections in
immunocompromised individuals, in neonates whose mothers developed a perinatal
varicella infection and as prophylaxis in pregnant females exposed to varicella.
(Choice E) Lamivudine is a nucleoside analog reverse transcriptase inhibitor active
against HIV-1 and HIV-2. It also has some activity against hepatitis B virus but is
not known to be effective against HSV-2.

Educational Objective:
The combination of new onset genital vesicular rash with a positive Tzanck smear in
a previously asymptomatic patient is suggestive of prima genital herpes due to HSV-
2. Recurrences of genital herpes can be reduced through daily treatment with oral
valacyclovir, acyclovir, or famciclovir; these drugs suppress reactivation of latent
HSV infection. Condom use can help prevent a prima genital HSV infection but does
not prevent reactivation of latent infection.

151
USMLE WORLS STEP 1 MICROBIOLOGY

Q NO 11: A group of investigators is trying to develop an anti-gonococcal vaccine.


Their attempts to use pilus components to induce long-lasting immunity would
most likely fail because of:
A. Antigenic mimicry
B. Low molecular weight
C. Non-protein structure
D. Antigenic variation
E. Local immunosuppression

Explanation:
N. gonorrhoeae have pili which are hair-like protein polymers that project from the
surface of the cell and are involved in the attachment of the organism to mucosal
surfaces. Those gonococci that have pili are able to adhere to susceptible cells and
thereby begin the infectious process. When the host produces antibodies directed
against gonococcal pili, adherence to the mucosa is inhibited. In a given strain at a
given time, only a single pilus gene is functional, so only one pilus type is expressed,
but the pilus genes are known to undergo antigenic variation at a high frequency.
Antigenic variation is a process by which the structural genes for pilus proteins
undergo recombination with each other to produce new antigenic types of pili, and
the array of different antigenic pilus types produced by this mechanism theoretically
may be quite large. This diversity of pilus protein expression is one reason why
development of an effective vaccine directed against the gonococcal pilus is so
challenging.
(Choice A) Antigenic mimicry is the sharing of antigenic sequences between the
microorganism and the host cell, thus leading to autoimmune disease if the host
mounts an immune response against the infectious epitope and that epitope has
sufficient sequence homology with a host antigen.
(Choice B) Pilin proteins do not represent a low molecular weight structure that
would be poorly immunogenic.
(Choice C) “Non-protein structures” does not describe pili as gonococcal pili are
protein polymers. This statement is also incorrect because vaccines have been
developed against non-protein structures. The meningococcal and the pneumococcal
vaccines are both examples of polysaccharide vaccines used in clinical practice.
(Choice E) Local immunosuppression does not describe a potential failure for
vaccination. Immunosuppression can blunt the effect of a vaccine to induce an
adequate antibody response requiring multiple inoculations in order to induce
immunity. This is the case in patients chronically immunosuppressed with
corticosteroids. Often after vaccination they will develop sufficient immunity, but
antibody titers should be checked to be sure.

Educational Objective:
The gonococci use their pili to mediate adherence to the mucosal epithelium. An
antibody against the specific pilus protein expressed by a gonococcus would prevent
mucosal adherence and initiation of infection, but each gonococcus possesses the
ability to modify the pilus protein that it expresses by the process of antigenic
variation and thus avoid host defense to some degree as well as make vaccination
directed against the pilus protein difficult.

152
USMLE WORLS STEP 1 MICROBIOLOGY

Q NO 12: A 34-year-old male is hospitalized with difficulty swallowing and blurred


vision of sudden onset. He has never had symptoms like this before. Routine
nerve stimulation studies show normal nerve conduction velocity but decreased
compound muscle action potential. The patient should be carefully questioned
about:
A. Missed vaccinations
B. Recent travel
C. Unprotected sexual contacts
D. Home-canned food consumption
E. Family history
F. Recent antibiotic use

Explanation:
Botulism is a rare but serious paracytic illness caused by a nerve toxin produced by
the bacterium Clostridium botulinum. There are three main types of botulism. Food-
borne botulism results when an individual eats foods containing the botulism toxin.
Wound botulism is the result of toxin production in a wound infected with C.
botulinum. Infant botulism occurs when a baby consumes C. botulinum spores which
then mature into vegetative toxin-producing cells in the intestine. (Contaminated
honey is a frequent cause of infant botulism.) All forms of botulism can be fatal and
are considered medical emergencies.
Botulinum neurotoxin is among the most toxic substances known. The anaerobic
environment within a can of food contaminated by C. botulinum spores allows spore
germination and organism growth. The botulinum toxin produced by the vegetative
C. botulinum bacteria remains intracellular until autolysis releases the potent
neurotoxin into the food. (This toxin IS NOT actively secreted by the bacteria.) The
toxin is readily destroyed by heating but if food containing the pre-formed
neurotoxin is not cooked well the toxin will be able to exert its effects. The most
common clinical manifestations are diplopia, dysphagia and dysphonia (three ‘Ds’),
which occur within 12-46 hours of neurotoxin consumption.
(Choice A) There is no vaccine against botulism. Botulism is treated with antitoxin
that blocks the action of circulating toxin. However, the antitoxin is unable to
inactivate toxin that has already gained access to nerve cells. Botulism is further
treated with supportive measures like intubation and mechanical ventilation for
diaphragmatic paralysis.
(Choice B) Recent travel is associated with numerous diseases including infectious
diarrhea, malaria, hepatitis, yellow fever, dengue fever and Hantavirus. Botulism is
not associated with travel, but obviously food can be improperly prepared or stored
in any country.
(Choice C) Unprotected sexual contacts invite sexually transmitted diseases. STDs
are caused by bacteria (Neisseria gonorrhoeae, Chlamydia trachomatis, Mycoplasma
hominis, Ureaplasma, urealyticum, Treponema pallidum, and Hemophilus ducreyi),
viruses (HIVI HSVI Hepatitis B, HPV) and protozoans (Trichomonas vaginalis).
(Choice E) A family history of recurrent infections can suggest a heritable
immunodeficiency such as X-linked agammaglobulinemia of Bruton or Wiskott-
Aldrich syndrome. Botulism is a toxin-mediated disease, not an organism- mediated
disease, so immunodeficiency would not predispose to it.
(Choice F) Recent antibiotic use can upset the intestinal bacterial balance, allowing
the multiplication of Clostridium difficile in C. difficile colitis. Antibiotics can also
disturb the vaginal flora, predisposing to vaginal candidiasis.

153
USMLE WORLS STEP 1 MICROBIOLOGY

Educational Objective:
There are three main kinds of botulism: food-borne botulism, wound botulism and
infant botulism. In food-borne botulism, botulinum neurotoxin in food (produced by
the organism Clostridium botulinum) prevents release of acetylcholine from nerve
terminals at the neuromuscular junction, thereby preventing muscular contraction.

154
USMLE WORLS STEP 1 MICROBIOLOGY

Q NO 13: A 34-year-old female who suffers from bronchial asthma is found to have
recurrent transient pulmonary infiltrates. Complete blood count shows eosinophilia.
A chest CT scan reveals proximal bronchiectasis. This patient’s condition is most
likely related to colonization with:

A. Pseudomonas aeruginosa
B. Streptococcus pneumoniae
C. Legionella pneumophila
D. Adenovirus
E. Aspergillus fumigatus
F. Strongyloides stercoralis

Explanation:
Aspergillus fumigatus is a low virulence fungus that generally does not cause
significant infections except in immunocompromised or debilitated patients. It may,
however, colonize the bronchial mucosa and complicate asthma or cystic fibrosis via
a hypersensitivity reaction. The result is allergic bronchopulmonary aspergillosis
(ABPA). ABPA occurs in 5% to 10% of steroid-dependent asthmatics. Patients with
this condition have very high serum IgE levels, eosinophilia, and IgE plus lgG serum
antibodies to Aspergillus. There is intense airway inflammation and mucus plugging
with exacerbations and remissions. Repeated exacerbations may produce transient
pulmonary infiltrates and proximal bronchi ectasis.
(Choice A) Pseudomonas aeruginosa is an opportunistic aerobic Gram-negative
bacillus that is a frequent, sometimes deadly, pulmonary pathogen in patients with
cystic fibrosis or neutropenia.
(Choice B) Although asthma may be a risk factor for the development of
pneumococcal lung colonization and pneumonia pneumococcal disease generally
results in resolution with preservation of lung lobular architecture. Potential
complications of pneumococcal bronchopneumonia or lobar pneumonia include lung
abscess, empyema, or lung fibrosis. Bronchiectasis is not a common outcome of
pneumococcal pulmonary disease.
(Choice C) Asthmatics do not have a predisposition to lung colonization with
Legionella.
(Choice D) Although viral respiratory infections can cause asthma exacerbations and
adenovirus colonization of the lung may be associated with asthma chronicity, pure
viral pneumonias do not progress to bronchiectasis.
(Choice F) Strongyloidiasis can cause pulmonary symptoms including cough,
asthma-like bronchospasm, and hemoptysis. It can also produce a transient
pneumonia as larvae migrate through the lungs on their way to the laryngopharynx.
In rare cases, chronic lung involvement may cause features of obstructive and/or
restrictive pulmonary disease. Progression to bronchiectasis is not commonly
reported.

Educational Objective:
Allergic bronchopulmonary aspergillosis (ABPA) due to Aspergillus fumigatus may
complicate asthma. ABPA can result in transient recurrent pulmonary infiltrates and
eventual proximal bronchiectasis.

155
USMLE WORLS STEP 1 MICROBIOLOGY

Q NO 14: A 34-year-old IV drug user who was an asymptomatic HBV carrier is


hospitalized with severe hepatitis following infection with delta agent. The A.
delta agent was able to cause hepatitis with direct HBV assistance in:
Intracellular survival
B. Cytoplasmic penetration
C. Replication of viral RNA
D. Translation of viral transcripts
E. Coating of viral particles

Explanation:
Often referred to as the delta agent or the hepatitis delta virus, hepatitis D virus is a
35-nm, double-shelled particle that resembles the Dane particle of HBV. The internal
polypeptide assembly of HDV is designated HDAg. Associated with this antigen is a
very short circular molecule of single-stranded RNA. HDAg is considered replication-
defective because it must be coated by the external coat antigen HBsAg of the
hepatitis B virus. Therefore HDV infection can arise either as an acute coinfection
with hepatitis B virus (with the HBV established first to provide the HBsAg for the
HDV) or as a superinfection of a chronic HBV carrier.
(Choices A, B, C, and D) Once coated with HBsAg, the hepatitis D virus is able to
penetrate the hepatocyte, survive within the cell, replicate its viral RNAI and
translate its genome into protein.

Educational Objective:
The HBsAg of hepatitis B virus must coat the HDAg of hepatitis D virus before it can
infect hepatocytes and multiply.

156
USMLE WORLS STEP 1 MICROBIOLOGY

Q NO 15: A small outbreak of hepatitis A infection is reported by epidemiological


services. Which of the following treatments is most likely to destroy the virus in
the contaminated products?
A. Drying
B. Treating with 20% diethyl ether
C. Treating with acid (pH = 1.0)
D. Heating to 60°C for an hour
F. Boiling for 1 minute

Explanation:
Because hepatitis A virus (HAV) is transmitted through the fecal-oral route,
improved sanitary conditions (frequent handwashing, appropriate food heating, and
the avoidance of food and water in endemic areas) serve to limit outbreaks of
infection. When HAV contamination is a concern, the virus can be inactivated with
water chlorination, bleach (1:100 dilutions) formalin, ultraviolet irradiation, or
boiling to 85°C for one minute.
(Choice A) When the HAV virus is dried itis actually stable at room temperature for
weeks.
(Choice B) Because HAV lacks a lipid-soluble envelope, it can withstand exposure to
20% diethyl ether, chloroform, or 50% trichlorofluoromethane.
(Choice C) Treatment with acid does not cause sufficient damage to the hepatitis A
virus which is not surprising given that it can easily withstand the acidic gastric
environment.
(Choice D) Heating to 60°C for an hour will not inactivate HAV, though boiling to
85°C for one minute is sufficient. Also recommended is autoclaving (120° for 20
minutes) which is quite effective.

Educational Objective:
Hepatitis A virus can be inactivated water chlorination bleach (1:100 dilution)
formalin, ultraviolet irradiation or boiling to 85°C for one minute.

157
USMLE WORLS STEP 1 MICROBIOLOGY

Q NO 16: A developing country includes infant vaccination with the Haemophilus


influenzae type b conjugate vaccine into its routine immunization schedule. If
effectively implemented, this change in the vaccination schedule would most
likely affect the epidemiology of:
A. Otitis media
B. Sinusitis
C. Acute bronchitis
D. Meningitis
E. Conjunctivitis

Explanation:
There are six serotypes of Haemophilus influenzae (a-f); capsular type b is the most
invasive strain of H. influenzae and can cause sepsis, meningitis, pneumonia and
other diseases. Additionally, there are unencapsulated strains referred to as
nontypable Haemophilus influenzae because serotyping is based on antigens in the
polysaccharide capsule. From the time that the H. influenzae type b (Hib) protein-
polysaccharide conjugate vaccine became available in 1987 for childhood
immunization beginning at 2 months of age, there has been a dramatic decrease in
the incidence of invasive disease caused by Haemophilus influenzae type b.
immunity to this and other infectious disease is accomplished during the first months
of life by lgG antibodies acquired transplacentally from the mother, but this
protection is only transient. H. influenzae type b conjugate vaccines prevent disease
by the induction of active B- lymphocyte mediated humoral immunity and may
decrease oropharyngeal carriage of H. influenzae type b. Before the availability of
the vaccine, slightly less than 50% of all cases of acute bacterial meningitis in the
U.S. were caused by Haemophilus influenzae type b.
(Choice A, B and E) The most common bacterial causes of acute otitis media,
sinusitis AND bacterial conjunctivitis in childhood are: 1. Streptococcus peroneae, 2.
Nontypable Haemophilus influenzae and 3. Moraxella catarrhalis, in that order. The
Hib vaccine has no effect on nontypable strains of H. influenzae as the Hib vaccine
induces immunity specifically against the type b polysaccharide capsule, and
nontypable H. influenzae do not produce capsules.
(Choice C) Acute bronchitis is almost always viral in etiology with influenza A & B,
coronavirus, respiratory syncytial virus and human meta pneumovirus being the
most commonly implicated. Patients with viral bronchitis will often present with
cough and demand antibiotics. Bacterial causes of acute bronchitis are rare and
include Mycoplasma pneumoniae and Bordetella pertussis.

Educational Objective:
The Haemophilus influenzae type b vaccine is composed of cell wall polysaccharide
conjugated with protein toxoid from either diphtheria or tetanus. This vaccination
can be given as early as 2 months of age and has drastically reduced the incidence
of clinical disease caused by H. influenzae such as meningitis, pneumonia, sepsis
and epiglottitis.

158
USMLE WORLS STEP 1 MICROBIOLOGY

Q NO 17: A 14-year-old African-American female with sickle cell anemia complains


of progressive exertional dyspnea after a minor febrile illness. Laboratory
evaluation reveals a hematocrit of 18% and reticulocyte count of 0.5%. Which of
the following viruses is most likely responsible for this patient’s current condition?

A. Enveloped double-stranded DNA virus


B. Non-enveloped double-stranded DNA virus
C. Non-enveloped single-stranded DNA virus
D. Non-enveloped (+) strand RNA virus
F. Enveloped (+) strand RNA virus
F. Enveloped (-) strand RNA virus

Explanation:
This patient has developed severe anemia after a minor febrile illness. If her bone
marrow were able to respond appropriately to the degree of anemia, the reticulocyte
count would be elevated (normal reticulocyte count is 0.5-1.5% of red cells).
Instead, the patient’s reticulocyte count persists at the low end of normal. This
scenario describes an aplastic crisis, which in sickle cell patients is usually secondary
to parvovirus B19 infection of erythroid precursor cells in the bone marrow.
Destruction of the erythroid precursor cells by this virus diminishes the number of
reticulocytes available to replace the deformed and/or removed erythrocytes.
Parvoviruses are non-enveloped, single-stranded DNA viruses.
(Choices A) Epstein-Barr virus is an enveloped, double-stranded DNA virus that can
cause hematopoietic depression and some measure of aplastic anemia. However itis
not the most common cause of these findings especially in a patient with sickle cell
anemia.
(Choice B) Non-enveloped, double-stranded DNA viruses include adenoviruses and
papovaviruses, which are not known to commonly cause hematopoietic depression
or aplastic anemia.
(Choices D and E) Hepatitis C virus (an enveloped single-stranded positive-sense
RNA virus) and hepatitis E virus (a non-enveloped single-stranded positive-sense
RNA virus) can cause hematopoietic depression and some measure of aplastic
anemia. HIV is another enveloped single-stranded positive-sense RNA virus that can
cause aplastic anemia. However these viruses are not the most common cause of
hematopoietic depression and aplastic anemia especially in a patient with sickle cell
anemia.
(Choice F) Important enveloped negative-sense RNA viruses include
orthomyxoviruses (eg, influenza), paramyxoviruses (eg, measles and mumps) and
rhabdoviruses (eg, rabies). These viruses are not known to infect erythroid
progenitor cells in sufficient numbers to significantly impact erythropoiesis, however.

Educational Objective:
In patients with sickle cell anemia and other chronic hemolytic disorders the most
common viral cause of an aplastic crisis is infection of erythroid progenitor cells with
parvovirus B19, a non-enveloped single-stranded DNA virus.

159
USMLE WORLS STEP 1 MICROBIOLOGY

Q NO 18: Microscopic examination of lung tissue obtained from a 45-year-old


Caucasian male with pulmonary infiltrates shows spherules packed with A.
endospores. This patient’s history is likely to reveal:
Recent travel to Arizona
B. Exposure to pigeon droppings
C. Cave exploration in Ohio
D. Previous cavitary tuberculosis
E. Chemotherapy for leukemia
F. Long history of asthma

Explanation:
Coccidioides immitis is a dimorphic fungus that has a mold form (hyphae) at 25-
30°C and a yeast form (spherules with endospores) at body temperature (37-40°C).
C. immitis is endemic to the southwest Unites States (southern and central
California, Arizona. New Mexico and western Texas), as well as to Northern Mexico
and some regions of Central and South America. Patients with coccidioidomycosis
are likely to live in or have recently traveled to an endemic area.
C. immitis is transmitted by spore inhalation. Spores are formed by fragmentation of
hyphae. Once inside the lungs, the spores turn into spherules that contain
endospores. The spherules subsequently rupture and release endospores that
disseminate to other organs and tissues. Each endospore is capable of forming a
new spherule.
In immunocompetent hosts, C. immitis causes lung disease, which may be
asymptomatic or cause flu-like symptoms (cough, fever myalgias) accompanied by
erythema nodosum. Immunosuppressed patients may develop systemic cocci di o i
do mycosis.
(Choice B) They east form of Cryptococcus neoformans is present in pigeon
droppings. This fungus causes pulmonary disease and meningoencephalitis in
immunocompromised patients.
(Choice C) Histoplasma capsulatum is endemic to the Mississippi and Ohio River
basins found in bird and bat droppings. Patients with histoplasmosis often have a
history of cleaning bird coops or caving.
(Choice D) Aspergillus fumigatus can colonize old lung cavities (e.g. formed by
tuberculosis) to form a “fungal ball” (aspergilloma). Symptoms include cough,
dyspnea and hemoptysis.
(Choice E) Neutropenic patients are at high risk for developing opportunistic
mycoses. Candida albicans, Aspergillus fumigatus, Mucor and Rhizopus species can
all cause severe disease in this population.
(Choice F) Patients with asthma are at risk for developing an allergic reaction to
Aspergillus fumigatus, called allergic bronchopulmonary aspergillosis (ABPA). Signs
and symptoms include cough, dyspnea, wheezing, fever, and migratory pulmonary
infiltrates.

Educational Objective:
Coccidioides immitis is a dimorphic fungus endemic to the southwestern U.S. It
exists in the environment as a mold (with hyphae) that forms spores. These spores
are inhaled and turn into spherules in the lungs.

160
USMLE WORLS STEP 1 MICROBIOLOGY

Q NO 19: A 45-year-old Caucasian male with cough and fever is found on chest x-
ray to have a lung infiltrate, hilar adenopathy, and a right-sided pleural effusion.
Lung tissue obtained from this patient reveals the findings below.

Which of the following is the most likely cause of this patient’s condition?

A. Rhizopus species
B. Aspergillus fumigatus
C. Candida albicans
D. Cryptococcus neoformans
E. Blastomyces dermatitidis
F. Histoplasma capsulatum
G. Coccidioides immitis
H. Sporothrix schenckii

Explanation:
The image above shows a large spherule filled with small round endospores.
Spherules are the tissue form of Coccidioides immitis. This is a dimorphic fungus
that exists in the environment in the form of mold (hyphae). It is endemic to the
desert areas of the United States and Mexico. Coccidioides can cause lung disease in
immunocompetent people and disseminated disease in the immunocompromised.
Microscopic examination of body fluids, sputum, and tissue samples in 10% KOH or
silver stain shows thick-walled spherules packed with endospores. Culture on
Sabouraud’s agar and serology are also important in making the diagnosis.
(Choice B) Aspergillus fumigatus commonly causes pulmonary disease in
immunocompromised patients. It has a mold form only. In tissue specimens
Aspergillus is seen as septate hyphae that branch at 45° angles.
(Choice E) The characteristic appearance of Blastomyces dermatitidis is that of
round yeast with broad-based budding. These yeasts have thick, doubly refractive
walls.
(Choice F) Histoplasma capsulatum would be seen as small oval yeast forms within
macrophages.

Educational Objective:
Coccidioides immitis infection can be asymptomatic or it can cause pulmonary
disease ranging from a flu-like illness to chronic pneumonia. It causes disseminated
disease in immunocompromised patients. Spherules containing endospores are
found in tissue samples.

161
USMLE WORLS STEP 1 MICROBIOLOGY

Q NO 20: A 23-year-old Caucasian female is brought to the ER with fever, vomiting,


diarrhea and muscle pain. Her blood pressure is 90/50 mm Hg and pulse is
120/mm. Physical examination reveals erythroderma, and pelvic exam reveals a
tampon in the vagina. The activation of which of the following cells is primarily
responsible for this patient’s condition?

A. Mast cells and eosinophils


B. Basophils and macrophages
C. Neutrophils and B lymphocytes
D. Macrophages and T lymphocytes
E. Platelets and mast cells

Explanation:
Fever, vomiting diarrhea, muscle pain and erythroderma are the symptoms of Toxic
Shock Syndrome (TSS). It can rapidly progress to severe hypotension and
multisystem dysfunction. Desquamation particularly on the palms and soles can
occur 1-2 weeks after the onset of illness. TSS has been associated with the use of
tampons, nasal packing etc.
Staphylococcus aureus strains producing toxic shock syndrome toxin (TSST-1) are
responsible for most cases of TSS. TSST acts as a superantigen. Itis called a
superantigen because in contrast to usual antigen which activates few helper T cells
it activates large numbers of helper T cells. These toxins interact with major
histocompatibility complex molecules on antigen presenting cells and the variable
region of the T lymphocyte receptor to cause a nonspecific widespread activation of
T lymphocytes. Activation of T cells is responsible for the release of interleukin-2 (IL-
2) from the T cells and IL-1 and TNF from macrophages. These interleukins cause
capillary leakage, circulatory collapse hypotension shock fever, skin findings, and
multiorgan failure.
(Choice A) Eosinophil activation is associated with reactions to allergens and
parasites. Peripheral eosinophilia can also be found in hypersensitivity reactions to
drugs as well as with hematologic malignancies.
(Choice B) Basophils are not involved in TSS.

162
USMLE WORLS STEP 1 MICROBIOLOGY

(Choice C) B lymphocytes respond to activation by replicating and synthesizing


antibodies specific for the antigen with which they were presented. B lymphocytes
are not responsible for TSS.
(Choice E) Mast cells when stimulated release histamine prostaglandins, leukotrienes
and other inflammatory mediators. They are stimulated by the cross-linking of IgE
present on their cell membrane by specific antigens and are active in the
pathogenesis of allergic reactions and anaphylaxis.

Educational Objective:
Enterotoxins Exfoliative Toxins and Toxic Shock Syndrome Toxin (TSST-i) are the
toxins with superantigen activity. Superantigens interact with major
histocompatibility complex molecules on antigen presenting cells and the variable
region of the T lymphocyte receptor to cause nonspecific “widespread” activation of
I-cells resulting in the release of interleukin-2 (IL-2) from the T cells and IL-i and
TNF from macrophages. The immune cascade in turn is responsible for the effects of
TSS.

163
USMLE WORLS STEP 1 MICROBIOLOGY

Q NO 21: A 42-year-old male presents to the emergency room following a seizure


episode. Physical examination reveals oral thrush and cervical lymphadenopathy.
An MRI of his brain with contrast is shown below.

Which of the following is the most likely cause of his symptoms?

A. Neurocysticercosis
B. Mycobacterium tuberculosis
C. Toxoplasmosis
D. Aspergillus fumigatus
E. Glioblastoma multiforme

164
USMLE WORLS STEP 1 MICROBIOLOGY

Explanation:
This contrast MIRI shows multiple ring-enhancing lesions, as indicated by the
arrows. The patient also has oral thrush and lymphadenopathy. This constellation of
signs and symptoms points to HI V-associated toxoplasmosis as the most likely
diagnosis. Seizures are a common complication of such brain lesions and their
surrounding edema.
(Choice A) Neurocysticercosis can also cause multiple brain lesions but fits less well
with the clinical picture here than toxoplasmosis. Neurocysticercosis is uncommon in
the United States.
(Choice B) Mycobacterium can also cause similar lesions but is not very common.
(Choice D) Aspergillus fumigatus can cause brain abscesses in the
immunocompromised patient, but is more common in neutropenic patients and
immunosuppressed transplant recipients than individuals with HIV. Aspergillus is not
a leading opportunistic pathogen in AIDS patients.
(Choice E) While ring-enhancing, CNS glioblastomas are usually solitary with a
characteristic butterfly appearance.

Educational Objective:
The finding of multiple ring-enhancing lesions in an HIV patient is most likely due to
toxoplasmosis.

165
USMLE WORLS STEP 1 MICROBIOLOGY

Q NO 22: A 6-year-old immigrant from Eastern Europe is brought to the emergency


room with difficulty breathing and a low- grade fever. Physical examination reveals
neck swelling, palatal paralysis, and a gray pharyngeal exudate. The parents are
unable to provide information regarding the child’s vaccination history. Which of the
following interventions is most likely to improve this patient’s prognosis?

A. Antibiotic therapy
B. Active immunization
C. Passive immunization
D. Adequate hydration
E. Anti-inflammatory medications

Explanation:
Diphtheria is caused by Corynebacteria diphtheriae. Acute infection of the naso- and
oropharynx causes pseudomembranous pharyngitis. The diphtheria exotoxin, an AB
exotoxin specific for neural and cardiac tissue causes CNS and cardiac sequelae in
some patients as well. This toxin ribosylates and deactivates elongation factor- 2,
thus inhibiting human protein synthesis. Clinical signs and symptoms of diphtheria
infection include sore throat, fever lymphadenopathy, upper airway dyspnea, and
odynophagia. The disease is transmitted via respiratory droplets.
Treatment of an acute C. diphtheriae infection requires (in order of importance): 1.
administration of diphtheria antitoxin, 2. administration of penicillin or erythromycin,
and 3. administration of the DPT vaccine. The diphtheria antitoxin inactivates all
circulating toxin, but is ineffective against toxin that has already gained access to
cardiac or neural cells. Thus rapid administration of antitoxin is essential. Antibiotic
therapy kills the bacteria, halting release of new exotoxin into the bloodstream and
preventing disease transmission. Immunization with the DPT vaccine provides
lasting immunity against future diphtheria infection.
(Choice A) Antibiotic therapy is an essential component of diphtheria treatment, but
has much less effect on prognosis than antitoxin administration. This is because the
exotoxin, not the bacteria, causes the major clinical sequelae. Penicillin is the
antibiotic of choice in treating C. diphtheriae infection.
(Choice B) Active immunization with the diphtheria toxoid (given as part of the
childhood DPT vaccine) prevents diphtheria. Diphtheria toxoid can also be given as
part of tetanus boosters in adults. In patients who are acutely ill. Vaccination does
not help to eliminate the bacteria or the exotoxin from the body.
(Choice D) While adequate hydration is an important supplementary measure in
many diseases, it does not specifically combat the disease-causing agents in
diphtheria.
(Choice E) Anti-inflammatory medications do not play a major role in the treatment
of diphtheria.

Educational Objective:
Diphtheria infection is associated with a 3% mortality rate. Cardiomyopathy is the
most common cause of death. Treatment includes diphtheria antitoxin, antibiotics
and immunization. Of these, the antitoxin is the most important and has the
greatest effect on prognosis.

166
USMLE WORLS STEP 1 MICROBIOLOGY

Q NO 23: A 24-year-old female presents to clinic complaining of recurrent genital


itching and vaginal discharge that responds well to topical antifungal therapy.
Laboratory testing reveals that she is HIV-positive and has a CD4+ I lymphocyte
countofl85/pL. The patient is at the greatest risk of developing which of the
following malignant proliferations?
A. H. pylon-induced B-lymphocyte proliferation
B. Epstein-Barr virus-induced B-lymphocyte proliferation
C. Human T lymphotrophic virus-induced B-lymphocyte proliferation
D. Hepatitis B virus-induced I-lymphocyte proliferation
E. BK virus-induced monocyte proliferation

Explanation:
Latent Epstein-Barr virus (EBV) infection is present in up to 90% of normal
individuals, with reactivation common in the immunosuppressed (eg, those with
AIDS). EBV is tropic for B lymphocytes, mediating their transformation into long-
term proliferating cell lines. As a result, AIDS patients have an increased incidence
of EBV-associated non- Hodgkin’s lymphomas, including the aggressive diffuse large
B-cell lymphomas and Burkitt’s lymphoma. In HIV positive patients, the
development of EBV-associated non-Hodgkin’s lymphoma is preceded by a decrease
in the number of functional EBV-specific CD8+ cytotoxic lymphocytes. The use of
highly active antiretroviral therapy to replenish CD4+ T-cell counts in HI V-positive
patients can mitigate the increased risk of developing HI V-induced non- Hodgkin’s
lymphoma.
(Choice A) Helicobacter pylon infection has been implicated in the pathogenesis of
gastric adenocarcinoma and mucosa-associated lymphoid tissue tumor (MALT0ma).
It has not been established whether HIV infection influences the incidence of H.
pylon-induced gastric lymphoma.
(Choice C)The retrovirus HTLV-I is associated predominantly with adult T-cell
leukemia and lymphoma, not B- lymphocyte proliferations.
(Choice D) Hepatitis B virus (HBV) infection is associated with hepatocellular
carcinoma not T-lymphocyte proliferation.
(Choice E) The BK virus is a DNA virus in the Papovaviridae family that primarily
causes one of two diseases in the immunocompromised: nephropathy (typically
post-transplant, when the latent infection is reactivated) or hemorrhagic cystitis. A
causal relationship between BK virus infection and malignant monocyte proliferation
in HI V-positive patients has not been established.

Educational Objective:
HI V-positive patients often experience reactivation of latent EBV infections with a
resulting increased incidence of EBV-induced lymphoproliferative disorders, including
the aggressive non-Hodgkin’s diffuse B-cell lymphomas.

167
USMLE WORLS STEP 1 MICROBIOLOGY

Q NO 24: A 34-year-old Caucasian female presents to your office with perineal


pruritus and vaginal discharge. Her past medical history is significant for acute
sinusitis one week ago as well as allergic rhinitis. Pelvic examination shows A.
erythematous vulva and thick adherent, “cottage cheese-like” vaginal
discharge. Microscopic examination of the vaginal discharge reveals budding
cells. This patient’s condition was most likely preceded by:
Decreased thickness of the vaginal epithelium
B. Increased glycogen concentration in the vaginal epithelium
C. Decreased number of Gram-negative bacteria in the vagina
D. Increased alkaline secretions by the cervical mucosa
E. Depressed T-lymphocyte response to mucosal stimuli

Explanation:
Gram-negative lactobacilli comprise a major part of the normal vaginal flora. Other
important colonizers include Corynebacteria, fungi (Candida) Streptococcus group B,
and E. coil. These species exist in balance at the normal vaginal pH of 3.8 - 4.2.
Changes in vaginal pH, damage to the vaginal microflora, and epithelial injury can all
lead to increased growth of Candida and symptoms of vulvovaginitis. Candida
vaginitis is the second most common cause of vaginitis.
Common triggers for Candida vaginitis include:
1. Antibiotic use suppresses normal bacterial flora, facilitating Candida overgrowth.
This patient’s history of acute sinusitis suggests recent antibiotic treatment.
2. Contraceptives both oral and typical alter vaginal pH.
3. Corticosteroid therapy.
4. Uncontrolled diabetes mellitus.
5. Any other cause of immunosuppression, including HIV.
Patients with Candida vaginitis complain of vulvar pruritus and white, thick
discharge. Physical examination reveals erythema of the vulvar and vaginal mucosa
and “cottage-cheese”-like discharge. Vaginal pH is normal or slightly higher than
normal (>4.5). Wet mount examination is diagnostic: it reveals budding yeast and
pseudohyphae.
(Choices A and B) Decreased thickness of the vaginal epithelium and decreased
amounts of glycogen in epithelial cells are characteristic of postmenopausal and
lactating women. These changes are caused by low estrogen levels and lead to
atrophic vaginitis. Yeast would not necessarily be isolated from the vaginal discharge
of these patients.

Educational Objective:
Use of broad-spectrum antibiotics suppresses the normal bacterial flora of the vagina
and facilitates Candida overgrowth. Antibiotic use is the most common cause of
Candida vaginitis. Other potential causes include contraceptives, steroids, diabetes
mellitus and immunosuppression.

168
USMLE WORLS STEP 1 MICROBIOLOGY

Q NO 25: Viral particles have been isolated from the peripheral blood of an 8-year-
old male. The particles are non-enveloped and contain single-stranded DNA. The
patient most likely suffers from:

A. Acute hepatitis
B. Bronchiolitis
C. Rubella
D. Erythema infectiosum
E. Chickenpox
F. Mumps

Explanation:
There is only one clinically significant viral family in which the members are both
non-enveloped and contain single- stranded DNA: Parvoviridae, which includes the
smallest DNA nucleocapsid viruses. Particularly renowned as a human pathogen is
parvovirus B19, which is associated with erythema infectiosum (“fifth disease”)
aplastic crises in those with sickle cell anemia, and hydrops fetalis (particularly when
infection occurs before the 20th week of gestation).
(Choice A) The major pathogens responsible for acute viral hepatitis are shown in
the table below.

The only virus with a DNA genome is hepatitis B, but this virus is enveloped and
contains double-stranded DNA.
(Choice B) The major viruses responsible for bronchitis/bronchi colitis are influenza
A, respiratory syncytial virus (RSV)I and coronavirus. These are all enveloped RNA
viruses.

(Choice C) Rubella (German measles) arises secondary to infection with a togavirus.


The Togaviridae family consists of enveloped RNA-containing viruses.
(Choice E) Chickenpox is due to varicella zoster virus (VZV). This is a member of the
herpesvirus family, which consists of enveloped virions that all contain double-
stranded DNA genomes.
(Choice F) Mumps is caused by a member of the paramyxovirus family, which
consists of enveloped virions that contain an RNA genome.
Educational Objective:
There is only one clinically significant viral family in which the members are both
non-enveloped and contain single stranded DNA: Parvoviridae. Parvovirus B19
causes erythema infectiosum (fifth disease), aplastic crises in sickle cell anemia, and
hydrops fetalis.

169
USMLE WORLS STEP 1 MICROBIOLOGY

Q NO 26: Blood cultures from a 54-year-old male recently diagnosed with Hodgkin’s
disease reveal motile Gram positive rods that produce a very narrow zone of j3-
hemolysis on sheep blood agar. Which of the following processes is most important
in eliminating these bacteria from the body?

A. Terminal complement cascade


B. Eosinophil action
C. Immunoglobulin secretion
D. Neutrophil oxidative burst
E. Cell-mediated immunity

Explanation:
Listeria monocytogenesis a Gram positive rod that produces a very narrow zone of
beta-hemolysis on sheep blood agar, similar to the pattern produced by colonies of
β-hemolytic Streptococci. L. monocytogenes shows tumbling motility at 22°C but
can be cultured at temperatures as low as 4°C. It is a facultative intracellular
parasite and the only Gram positive bacteria to produce lipopolysaccharide (LPS)
endotoxin. Listeria can cause serious disease (meningitis septicemia) in newborns,
pregnant women, the elderly and immunocompromised patients, but rarely causes
disease in immunocompetent individuals.
Macrophages ingest L. monocytogenes, which survives phagocytosis and can go on
to multiply in the cytoplasm. In healthy individuals, the cell-mediated immune
response stimulates the production of cytokines (interferon gamma, tumor necrosis
factor-beta and interleukin-12) that induce a cytotoxic T-cell response and
macrophage activation and killing of intracellular Listeria. In patients with
compromised cell-mediated immunity, however, the organism survives and disease
results (Choice E).
There are two types of listeriosis, neonatal and adult. In neonatal listeriosis, the
organism is transmitted from the mother to the fetus transplacentally or during fetal
passage through the vaginal canal. In both adults and neonates listeriosis can be
transmitted by the consumption of contaminated food.
(Choice A) Patients with inherited deficiencies of the terminal complement cascade
(C5b-C9) are unable to form the membrane attack complex (MAC) and will have a
predisposition to recurrent Neisseria infections. (Choice B) Eosinophil action is
important for eradicating parasitic infections from the body. (Choice C) X-linked
agammaglobulinemia of Bruton is a condition causing a pure defect of
immunoglobulin synthesis and secretion. Patients lack humoral immunity but have
intact cell mediated immunity. These patients are therefore at increased risk for
bacterial infections by Streptococci and Staphylococci but have normal responses to
infections by Listeria, viruses and fungi. (Choice D) Defects in the neutrophil
oxidative burst lead to chronic granulomatous disease (CGD). This is an X linked
disorder that causes decreased NADPH oxidase activity and a failure of the
myeloperoxidase system resulting in an inability to form hydrogen peroxide and
oxygen free-radicals in concentrations high enough to kill catalase producing
organisms like S. aureus.
Educational Objective: 1. Intact cell-mediated immunity is essential for the
elimination of Lister/a monocytogenes from the body. Neonates up to 3 months of
age are especially vulnerable because they have yet to fully develop their cell
mediated immunity. Listeria rarely causes disease in normal healthy adults.
2. This bacterium produces a very narrow zone of beta-hemolysis on sheep blood
agar (resembling the pattern produced by β-hemolytic Streptococci), shows tumbling
motility at 22°C, and can be cultured at temperatures as low as 4°C.

170
USMLE WORLS STEP 1 MICROBIOLOGY

Q NO 27: A 54-year-old male is brought to the ER with a one-week history of


headaches and progressive confusion. He was hospitalized six months ago A.
with viral esophagitis and two months ago with pneumocystic pneumonia.
Lumbar puncture is performed, and shows a moderate increase in CSE
protein concentration and CSE pleocytosis. The latex agglutination test is
positive for soluble polysaccharide antigen. Light microscopy of this patient’s
CSF is most likely to reveal:
Nonseptate hyphae
B. Germ tubes
C. Budding yeast
D. Spherules
E. Sporangium

Explanation:
A recent history of viral esophagitis and pneumocystis pneumonia is virtually
diagnostic of HIV infection. The patient’s headache, confusion, and inflammatory
CSF changes are typical symptoms of meningitis. CNS infection in an HIV (+) patient
is likely to be caused by Cryptococcus neoformans, an opportunistic fungus that
causes meningitis (or meningoencephalitis) in immunosuppressed patients.
For study purposes, the medically important facts regarding C. neoformans are
summarized below:

(Choice A) Nonseptate hyphae that branch at wide angles are characteristic of Mucor
and Rhizopus species. These fungi cause infection of the paranasal sinuses
(mucormycosis) in immunosuppressed patients.
(Choice B) Candida albicans forms germ tubes (sprouts of true hyphae from yeast
cells) if incubated in 37°C serum for3 hours. This test helps to differentiate C.
albicans from other Candida species. Candida albicans does not usually cause
meningitis.
(Choice D) Spherules (round structures containing endospores) are found in the
tissue form of Coccidioides immitis. This fungus causes lung disease and
disseminated mycosis, but is not commonly associated with meningitis (rarely it can
cause meningitis).

171
USMLE WORLS STEP 1 MICROBIOLOGY

(Choice E) Sporangium is a structure that produces and contains spores. It is


present in mold fungi. Cryptococcus has only a yeast form and thus does not have
sporangia.

Educational Objective:
Cryptococcus neoformans causes meningoencephalitis in HIV (+) patients. The latex
agglutination test detects the polysaccharide capsule antigen of Cryptococcus and is
used for diagnosis. India ink staining of the CSF shows round or oval budding yeast.

172
USMLE WORLS STEP 1 MICROBIOLOGY

Q NO 28: A 38-year-old male intravenous drug user hospitalized for high-grade


fever, fatigue and dyspnea dies in the intensive care unit (ICU). His lung
autopsy findings are shown on the slide below.

This patient most likely suffered from:

A. Mycotic aortic aneurysm


B. Tricuspid valve endocarditis
C. Severe small airway obstruction
D. Miliary tuberculosis
E. Venous thromboembolism

Explanation:
The above slide illustrates wedge-shaped hemorrhagic pulmonary infarcts.
Pulmonary infarcts are almost always hemorrhagic due to the dual blood supply to
the lungs (pulmonary and bronchial arteries). The question stem describes a young
male patient with a history of intravenous drug abuse dying of sepsis. Intravenous
drug abusers most commonly form endocarditis of the right heart. Recall that S.
aureus is the most common endocarditis associated with IVDU. Pulmonary infarction
in this case likely resulted from septic embolization from the tricuspid valve to a
distal pulmonary artery.
(Choice A) Mycotic aneurysm could also result from endocarditis. However the above
showed multiple infarcts in a patient with high grade fevers favor right sided valve
endocarditis as the primary cause. Despite their name, mycotic aneurysms are only
very rarely associated with fungal infections.
(Choice C) Severe small airway obstruction is not associated with infection and
would not cause the clinical picture described. It would additionally not cause
pulmonary infarction. Small airway obstruction is characteristic of asthma.
(Choice D) Pulmonary tuberculosis classically affects the apical sections of the lungs
as well as the hilar lymph nodes after initially establishing infection by inhalation and
establishment of a Ghon complex in the middle or lower lobes. Progression of
disease leads to the formation of caseating granulomas and the formation of cavitary
pulmonary lesions.
(Choice E) Venous thromboembolism would not cause high-grade fevers.

Educational Objective:
S. aureus is the most common cause of tricuspid endocarditis in intravenous drug
users. P. aeruginosa is the second most common cause in this patient population.
These patients can develop multiple septic emboli in lungs. Pulmonary infarcts are
almost always hemorrhagic due to the dual blood supply to the lungs (pulmonary
and bronchial arteries).

173
USMLE WORLS STEP 1 MICROBIOLOGY

Q NO 29: In a research experiment, Strain of Streptococcus pneumoniae produces


no effects when injected into the mouse peritoneal cavity. However, when grown
on media alongside a virulent strain of S. pneumoniae (Strain 2), Strain acquires
the ability to cause death. Which of the following processes most likely accounts
for this acquisition of virulence?
A. Pilus-mediated transfer
B. Chromosomal fragment uptake from media
C. Phage-mediated DNA transfer
D. Spontaneous mutation
E. Transposon-mediated DNA transfer

Explanation:
Certain strains of S. pneumoniae express capsular polysaccharides that inhibit
phagocytosis, making it a successful pathogen. Strains lacking the capsule are not
pathogenic. S. pneumoniae is able to undergo transformation, a process involving
uptake and expression of chromosomal fragments from the environment, when
neighboring bacteria die and lyse (Choice B). Bacteria capable of taking up free DNA,
that is, capable of undergoing transformation, are said to be ‘competent.’
Streptococcus pneumoniae, Haemophilus influenza and Neisseria meningitidis are
bacteria that have this ability. Through this process, non-virulent, non-capsule-
forming strains of S. pneumoniae can acquire the genetic material that codes for the
capsule and thus gain virulence.
(Choice A) Conjugation is pilus-mediated DNA transfer. Conjugation was first
described in E. coil, but we now know that most bacteria are capable of the process.
To initiate conjugation, the donating bacterium must produce a sex pilus, which
creates a direct connection with the receiving bacterium. Only bacteria with genetic
material coding for conjugative ability (e.g. the F plasmid) can initiate conjugation in
this way. Next, the donating bacterium synthesizes a new DNA strand, which is
passed through the pilus into the recipient organism. In the receiving organism, the
complementary DNA strand is synthesized.
(Choice C) In transduction, a bacteriophage (virus) transfers DNA from one bacterial
cell to another. While replicating within the infected host bacterium, the
bacteriophage may incorporate host bacterial DNA into the phage particle. When the
phage is subsequently released, it transfers both phage DNA and DNA from the
previously infected bacterium into the newly infected cell. By this mechanism,
bacteria can acquire genes for virulence and antibiotic resistance.
(Choice D) A mutation is a change in the nucleotide sequence of a gene. Mutational
changes may be spontaneous or induced. Changes in the nucleotide base sequence
can cause changes in the transcribed mRNA base sequence, potentially altering the
amino acid sequence of the protein product. Through this mechanism, a bacterium
may begin to form proteins with potentially useful functions to aid in bacterial
survival.
(Choice E) Transposon-mediated DNA transfer is a mechanism by which DNA from
plasmids or phage transfer can be incorporated into the host bacterium’s DNA
genome, genetic material can be moved from one position to another within the
genome, or DNA can be removed from the genome and placed onto a plasmid. The
location of a gene in the genome is important because it determines its proximity to
promoter or suppressor regions.

174
USMLE WORLS STEP 1 MICROBIOLOGY

Educational Objective:
S. pneumoniae is able to undergo transformation, which is the uptake and
expression of chromosomal fragments from the environment made available when
another bacterial cell dies and undergoes lysis.
Conjugation is pilus-mediated transfer of DNA. Conjugation occurs in most bacteria
but was first described in the gram negative rod E. coil.
In transduction, DNA from one bacterial cell is transferred to another bacterial cell
by a bacteriophage (virus).

175
USMLE WORLS STEP 1 MICROBIOLOGY

Q NO 30: Monoclonal antibodies against complement receptor CD21 on the surface


of B-lymphocytes could prevent infection with which of the following viruses?

A. Parvovirus B19
B. Human immunodeficiency virus
C. Epstein-Barr virus
D. Cytomegalovirus
E. Adenovirus

Explanation:
Essential to viral invasion of cells and the viral tropism for specific tissues is the
initial attachment of the virion envelope or capsid surface proteins to the
complementary host cell surface receptors. Many viruses bind to normal host cell
plasma membrane receptors in order to enter host cells.
Epstein-Barr virus (EBV) is the ubiquitous herpesvirus responsible for acute
infectious mononucleosis, nasopharyngeal carcinoma lymphomas, and Burkitt’s
lymphoma. More than 90% of the normal adult population is seropositive for EBV,
which is primarily transmitted through contact with oropharyngeal secretions. The
EBV envelope glycoprotein gp350 binds to the cellular receptor for the C3d portion of
component (CR2 or CD21). CD21 is normally present on the surface of B cells and
nasopharyngeal epithelial cells. Thus, a monoclonal anti-CD21 antibody could
interfere with the attachment of EBV to cells.
(Choice A) Parvovirus B19 is thought to attach to human erythroid cells via the blood
group P antigen (globoside). The P antigen is expressed by mature erythrocytes,
erythroid progenitors, megakaryocytes, placenta, and the fetal liver and heart.
Immature cells of the erythroid family are most vulnerable to parvovirus B19
infection, which is why adult bone marrow and fetal liver are principal targets. Viral
replication causes cell death.
(Choice B) HIV viruses attach to their major target host cells (CD4+ T cells)
primarily via the binding of viral envelope glycoprotein gpl2O to the cellular CD4
transmembrane glycoprotein and the coreceptor (CCR5 orCXCR4). The HIV envelope
then undergoes a conformational change that activates gp4l and initiates membrane
fusion.
(Choice D) Like the other herpesviruses, cytomegalovirus (CMV) requires initial
contact with glycosaminoglycan chains on host cell surface proteoglycans for entry
into the host cell.
(Choice E) The adenovirus has hexon and penton capsomeres on its surface. Rodlike
structures (“fibers”) that project from the penton base capsomeres are responsible
for mediating adsorption to host cells. The cell receptor for most adenovirus fibers is
a transmembrane protein member of the immunoglobulin superfamily.

Educational Objective:
Essential to viral invasion of cells and the viral tropism for specific tissues is the
initial attachment of the virion envelope or capsid surface proteins to the
complementary host cell surface receptors. Many viruses bind to normal host cell
plasma membrane receptors in order to enter host cells. Known host cell receptor—
virion/virion protein binding specificities include: CD4 and HIV gp120; CD21 and EBV
gp350; and erythrocyte P antigen and parvovirus B19.

176
USMLE WORLS STEP 1 MICROBIOLOGY

Q NO 31: A 54-year-old male presents to the ER with a sore on his right shoulder.
Physical examination demonstrates an ulcer with central black eschar surrounded
by edema. Exudate microscopy reveals chains of large Gram-positive rods. Which
of the following would be most helpful in making the diagnosis?
A. Family history
B. Occupation
C. Recent contact with foreigners
D. Allergies
E. Current medications
F. Sexual history
G. Prior ER visits

Explanation:
Humans become infected with Bacillus anthracis most commonly by exposure
through contact with infected animals or animal products or through use of B.
anthracis as a biological weapon. For this reason an occupational history of exposure
to animals or animal products is extremely important; if cutaneous anthrax is
suspected in a patient without the risk of occupational exposure, then the potential
for bioterrorism should be suspected and public health authorities contacted. The
disease is rare in the Unites States; it is most common in areas where vaccination of
livestock and animal carriers is not done or not possible.
Growth of the vegetative organisms at the site of inoculation results in formation of
a characteristic edematous wound with central necrosis leading to formation of a
black eschar. B. anthracis spreads via lymphatics to the bloodstream, and the
organism multiplies in the blood and tissue. Cutaneous anthrax is the most common
form of this disease; pulmonary anthrax accounts for approximately 5% of cases,
and gastrointestinal anthrax is a rare occurrence. Cutaneous anthrax commonly
occurs on exposed surfaces of the arms or hands, and sometimes on the face and
neck.
(Choice A) Family history would not be helpful in making the diagnosis of anthrax as
there is no genetic predisposition to this disease.
(Choice C) Recent contact with foreigners would not be a consideration in diagnosing
anthrax as this organism is not transmitted from person to person.
(Choice D) Allergies are not a reasonable choice, because formation of a necrotic
skin wound is not a common allergic response.
(Choice E) Current medications would be a consideration as drugs can induce a
variety of cutaneous findings, but they generally do not cause discrete areas of
eschar as mentioned in this question stem.
(Choice F) Sexual history is not a consideration because anthrax is not a sexually
transmitted disease.
(Choice G) Prior ER visits are not a factor in diagnosing anthrax because anthrax is
not a nosocomial infection and a visit to the ER would not be an exposure risk.

Educational Objective:
Cutaneous anthrax leads to the formation of a necrotic skin wound with an
erythematous and edematous border and a necrotic center after inoculation of
spores of Bacillus anthracis into the skin. Anthrax is most commonly acquired
occupationally by those who handle livestock that have not been immunized for the
disease as well as those who handle the hides of such animals. Anthrax is also used
as a biological weapon due to the near 100% mortality of the pulmonary form.

177
USMLE WORLS STEP 1 MICROBIOLOGY

Q NO 32: A 25-year-old male with a several month history of fluctuating levels of


alanine aminotransferase (ALT) and aspartate aminotransferase (AST) is found to
have high titers of lgG directed against hepatitis C envelope protein. These
antibodies do not confer effective immunity against the infection because:
A. The antibodies do not have neutralizing properties
B. Envelope proteins vary their antigenic structure
C. Envelope proteins have low immunogenicity
D. Envelope proteins are lost after recurrent replications
E. Envelope proteins are sequestered within the hepatocytes

Explanation:
Hepatitis C virus has six or more genotypes and multiple sub genotypes, as
demonstrated by the genetic differences in the encoding of its two envelope
glycoproteins. This genetic variation has led to the development of a hypervariable
region of the envelope glycoprotein that is especially prone to frequent mutation.
Moreover, there is no proofreading 3’ — 5’ exonuclease activity built into the virion-
encoded RNA polymerase. As a result, the RNA polymerase makes many errors
during replication, with several dozen subspecies of hepatitis C virus typically
present in the blood of an infected individual at any one time. Because of this
remarkable variety in the antigenic structure of the HCV envelope proteins,
production of host antibodies lags behind the production of new mutant strains of
HCV and effective immunity against infection is not conferred. The tremendous
antigenic variety of HCV has significantly slowed efforts to develop a vaccine against
the virus.
(Choice A) Host antibodies against hepatitis C virus do have neutralizing properties
and are effective in eliminating the more established hepatitis C strains. It is the
constantly emerging mutant strains that pose the greatest challenge to the host
immune system.
(Choice C) The envelope proteins of hepatitis C virus are highly immunogenic.
(Choice D) The envelope proteins of hepatitis C virus are not lost after recurrent
replications.
(Choice E) The envelope proteins of hepatitis C virus are not sequestered within
hepatocytes. Rather, these envelope proteins are present on the outermost surface
of the virus and facilitate entrance into hepatocytes.

Educational Objective:
Because of the remarkable variety in the antigenic structure of hepatitis C virus
envelope proteins, the production of host antibodies lags behind the production of
new mutant strains of HCV and effective immunity against infection is not conferred.

178
USMLE WORLS STEP 1 MICROBIOLOGY

Q NO 33: A 34-year-old immigrant from Eastern Europe presents to your office with
a three-month history of productive cough, night sweats and low-grade fever.
Sputum cultures grow budding yeast that form germ tubes at 37°C. The most likely
site of this organism before entering the sputum is which of the following:

A. Fibrous lung cavities


B. Inflamed lung parenchyma
C. Bronchioles and small bronchi
D. Large bronchi and trachea
E. Oral cavity

Explanation:
Candida albicans gives rise to true hyphae, termed “germ tubes.” when incubated at
37°C for 3 hours. Germ tubes are specific for C. albicans; they are not seen with any
other Candida species. Morphologically, all Candida fungi are yeasts, seen in tissue
sections as single cells with pseudohyphae.
Candida albicans is the most common opportunistic mycosis. It is also a frequent
colonizer of human skin and mucous membranes. (Candida contributes to the
normal flora of skin, mouth, vagina, and intestine.)
Superficial Candida infections are associated with antibiotic use, corticosteroid use,
diabetes mellitus, HIV and other immunosuppressing illnesses. These superficial
infections include oral thrush vulvovaginitis and cutaneous candidiasis. Disseminated
candidiasis occurs in neutropenic patients and most often affects the esophagus,
heart, liver and kidney.
Candida does not usually cause lung disease. This patient’s pulmonary symptoms
(cough, night sweats and low- grade fever) as well as the fact that he has come
from Eastern Europe, are consistent with a diagnosis of tuberculosis. The Candida in
his sputum indicates only that his oral cavity is colonized: it does not contribute to
the diagnosis.
(Choice A) Aspergillus fumigatus is a fungus that colonizes already-existing lung
cavities. It forms a “fungus ball” (aspergilloma) that may cause cough and
hemoptysis.
(Choice B) Many pathogens, including bacteria viruses and fungi, can cause
inflammation of the lung parenchyma. Candida-associated pulmonary disease is
uncommon, however.
(Choices C and D) The trachea and large and small bronchi are normally sterile.

Educational Objective:
Candida albicans is a normal inhabitant of the 01 tract (including the oral cavity) in
up to 40% of the population. Thus it is a common contaminant of sputum cultures.
The presence of Candida in sputum does not indicate disease.

179
USMLE WORLS STEP 1 MICROBIOLOGY

Q NO 34: Infective endocarditis is known to be caused by many different bacterial


species. Which of the following scenarios is most consistent with infective
endocarditis caused by Streptococcus bovis?
A. A 34-year-old female with known mitral stenosis develops low-grade fevers and
negative blood cultures
B. A 45-year-old male complains of fatigue and exertional dyspnea three weeks after
tooth extraction
C. A 62-year-old female has a persistent fever after being diagnosed with colon
cancer
D. A 64-year-old male with repeatedly negative blood cultures has small mitral
vegetation on transesophageal echocardiogram
E. A 29-year-old male with a persistent fever has tricuspid vegetations and tricuspid
regurgitation on transthoracic echocardiogram

Explanation:
Among non enterococcal group D streptococci, Streptococcus bovis is the one main
human pathogen. Streptococcus bovis is a part of the normal flora of the colon, and
bacteremia or endocarditis caused by S. bovis is associated with colonic cancer in
approximately 25% of cases. S. bovis causes a subacute bacterial endocarditis with
symptoms similar those of S. viridans SBE. In addition to its association with colonic
malignancy, S. bovis endocarditis is unique in that the majority of cases that occur
do so in patients with no preexisting valvular abnormality. The role of S. bovis as
causative agent or marker of the disease in colon cancers is unclear, but every
patient with S bovis bacteremia with or without endocarditis should be examined for
a G1 tract malignancy.
(Choice A) This clinical picture is most consistent with a patient who has suffered
from acute rheumatic fever in the past which has left her with mitral stenosis. It is
likely that she has recently experienced yet another episode of rheumatic fever.
(Choice B) This scenario is most consistent with Streptococcus viridans subacute
bacterial endocarditis. S. viridans is the most common etiologic agent in subacute
bacterial endocarditis (SBE) following dental work.
(Choice D) This case is most consistent with culture-negative endocarditis. These
organisms include, but are not limited to Bartonella, Coxiella, Mycoplasma,
Histoplasma, Chlamydia and the HACEK organisms (Haemophilus, Actinobacillus,
Cardiobacterium, Eikenella and Kingella).
(Choice E) Right-sided endocarditis commonly occurs in IV drug users and is
typically caused by S. aureus.

Educational objective:
S. bovis endocarditis and S. bovis bacteremia are associated with GI lesions (colon
cancer) in more than 25% of cases. When S. bovis is cultured in the blood workup
for colonic malignancy with colonoscopy is essential.

180
USMLE WORLS STEP 1 MICROBIOLOGY

Q NO 35: A 3-year-old male experiences several episodes of otitis media. Middle ear
exudate cultures grow H. influenzae although the patient’s immunizations are up-to-
date. Which of the following explains H. influenzae infection in this patient despite
immunizations?

A. No vaccine is effective against H. influenzae


B. The strains responsible for this patient’s disease produce exotoxin
C. The strains responsible for this patient’s disease do not produce capsule
D. H. influenzae infection despite immunizations means defective cellular immunity
E. H. influenzae infection despite immunizations signifies defective neutrophil
function

Explanation:
H. influenzae along with Streptococcus pneumoniae are the most frequent causes of
acute otitis media. This infection is primarily seen in children between 6 months and
12 years of age. More than 90% of H. influenzae strains isolated from middle ear
aspirates of infected children are nontypable; the remaining 10% are H. influenzae
type b. H. influenzae is classified or typed, based on the antigens in its
polysaccharide capsule, and strains of H. influenzae that do not produce a capsule
are referred to as nontypable. Nontypable H. influenzae strains are part of the upper
respiratory tract normal flora, but they can also cause otitis media, sinusitis and
bronchitis in adolescents and adults as well as children and vaccination with the H.
influenzae type b (Hib) vaccine does not confer immunity to any strain except type
b.
It is recommended that all children receive the Hib vaccine beginning at 2 months of
age as H. influenzae type b is associated with disseminated disease, pneumonia
epiglottitis and meningitis.
(Choice A) The protein-conjugated Hib vaccine is very effective in inducing immunity
to H. influenzae type bin children as young as two months old.
(Choice B) Exotoxin is not produced by any strain of H. influenzae, typable or not.
The vaccine for H. influenzae is directed against capsular polysaccharide, not toxin
or toxoid.
(Choice D) H. influenzae infection specifically with H. influenzae type b despite
immunization with the Hib vaccine does not indicate defective cellular immunity as a
T-cell response is generally not induced even by the conjugate vaccine. The
conjugate vaccine effectively stimulates B-cells leading to the generation of memory
B-lymphocytes and humoral immunity to Hib.
(Choice E) H. influenzae infection despite immunizations does not necessarily signify
defective neutrophil function although poor neutrophil function or low neutrophil
count can predispose to bacterial disease. Neutrophil function is impaired in chronic
granulomatous disease Chédiak-Higashi syndrome and other inherited diseases.

Educational Objective:
Nontypable strains of Haemophilus influenzae are strains of H. influenzae that do not
form an antiphagocytic capsule. They are part of the normal flora of the upper
respiratory tract, but can cause otitis media sinusitis and bronchitis. Immunity to
nontypable strains as well as capsular strains other than type b is not conferred by
vaccination with the Hib vaccine.

181
USMLE WORLS STEP 1 MICROBIOLOGY

Q NO 36: An 18-month-old male is seen on an outpatient basis for fever, runny


nose, and sore throat. Two days later, he is brought to the emergency A.
department with persistent fever, brassy cough and di1ticultc breathing.
Physical examination reveals stridor. Which of the following pathogens is most
likely responsible for this patient’s condition?
Rhinovirus
B. Paramyxovirus
C. Togavirus
D. Parvovirus
E. Calicivirus

Explanation:
This child initially presented with fever, rhinitis, and pharyngitis, symptoms of an
upper respiratory tract infection (URI). Most URIs (with or without pharyngitis) are
caused by viruses, with roughly 15-20% of childhood cases of pharyngitis and 5-
10% of adulthood cases of pharyngitis caused by bacteria. The various viral causes
of URIs are listed below:

When a URI-stricken child such as this one develops a brassy cough and breathing
difficulties, it is likely he has developed acute laryngotracheobronchitis (croup). The
dyspnea associated with croup occurs when inflamed subglottic tissue obstructs the
upper airway, and the characteristic stridor of croup is a sign that the URI has
spread to include the larynx. Croup is typically caused by the standard URI viruses
with the parainfluenza viruses (members of Paramyxoviridae) most commonly
responsible.

(Choice A) Rhinovirus is the most common viral cause of upper respiratory infections
but is an uncommon cause of viral croup.
(Choice C) Togaviruses are responsible for rubella (German measles) and Eastern
and Western equine encephalitis. (Choice D) Parvoviruses are responsible for
aplastic crises in sickle cell anemia, erythema infectiosum (fifth disease), and
hydrops fetalis.
(Choice E) Caliciviruses (hepatitis E virus and Norwalk virus) are responsible for viral
gastroenteritis.
Educational Objective:
Brassy cough with dyspnea in a child with a recent URI history is suggestive of viral
laryngotracheobronchitis (croup). The most common cause of croup is parainfluenza
virus type 1.

182
USMLE WORLS STEP 1 MICROBIOLOGY

Q NO 37: A 32-year-old male presents to his physician complaining of diarrhea,


flatulence, foul-smelling stools, and abdominal cramping. He recently returned
home from a hiking trip. A smear of his stool sample is shown below:
Which of the following is the drug of choice for this condition?

A. Ciprofloxacin
B. Metronidazole
C. Mebendazole
D. Albendazole
E. Loperamide
F. Clindamycin
G. Erythromycin

Explanation:
When examined for ova and parasites (stool smear with iodine staining) this
patient’s stool reveals ellipsoidal cysts with smooth well-defined walls and 2+ nuclei
consistent with Giardia lamblia. Giardia is the most common enteric parasite in the
U.S. and Canada. When an individual drinks contaminated water from an endemic
area without first boiling it, Giardia can colonize the duodenal and jejunal mucosal
lining. Giardiasis causes chronic diarrhea malabsorption and flatulence.
Metronidazole is an accepted treatment for this protozoan flagellate. This drug can
also be used to treat Helicobacter pylori infections amebiasis and trichomoniasis.
(Choice A) Inflammatory traveler’s diarrhea (marked by stool containing blood and
mucus) can be treated with ciprofloxacin or another fluoroquinolone.
(Choice C) Mebendazole is an anti-helminthic drug that acts on the microtubules of
roundworms like Ancylostoma, Ascaris and Enterobius.
(Choice D) Albendazole is used to treat the tapeworm (cestode) Echinococcus
granulosus. Echinococcus does not commonly cause diarrhea, but rather produces
cysts in the liver and elsewhere.
(Choice E) Loperamide is an opiate antimotility drug that may be prescribed in some
cases of traveler’s diarrhea where there is no fever or blood in the stool.
(Choice G) Erythromycin can be used in lieu of fluoroquinolones to treat
Campylobacter jejuni, as this organism has shown increasing worldwide resistance
to fluoroquinolones.

Educational Objective:
This iodine-stained stool smear shows Giardia lamblia cysts. Giardia is the most
common enteric parasite in the U. S. and Canada and is a common cause of diarrhea
in campers/hikers. Metronidazole is the treatment of choice.

183
USMLE WORLS STEP 1 MICROBIOLOGY

Q NO 38: Cytomegalovirus (CMV) is isolated from the urine sample of a 26-year-old


male. Which of the following manifestations of CMV disease is most commonly seen
in immunocompetent adults?

A. Vesicular skin rash


B. Mononucleosis
C. Viral pneumonia
D. Retinitis
E. Hepatitis
F. Esophageal ulcer

Explanation:
Cytomegalovirus (CMV) is a rare cause of disease in the immunocompetent, with the
virus more typically responsible for subclinical infection. When primary CMV infection
does result in clinically evident illness afflicted individuals appear to have a systemic
mononucleosis-like syndrome characterized by fever, malaise, myalgia, atypical
lymphocytosis, and elevated liver transaminases. In contrast to EBV mononucleosis,
heterophil antibodies are not usually present in patients with CMV mononucleosis.
(Choice A) A vesicular skin rash is frequently seen in association with herpes simplex
and herpes zoster.
(Choice C) CMV pneumonitis is common after organ transplantation.
(Choice D) Retinitis is the most common manifestation of CMV infection in patients
who are HI V-positive.
(Choice E) Disseminated CMV can cause diarrhea and hepatitis in the
immunocompromised.
(Choice F) Odynophagia (painful swallowing) associated with linear esophageal
ulcers is often reported by HIV positive patients.

Educational Objective:
CMV is typically associated with subclinical infection in the immunocompetent, with
the occasional individual developing a mononucleosis-like syndrome that is Monospot
negative. In the immunocompromised, prima or reactivated CMV infection can result
in severe retinitis, pneumonia, esophagitis, colitis, and/or hepatitis.

184
USMLE WORLS STEP 1 MICROBIOLOGY

Q NO 39: An 8-year-old male is brought to the ER with a two day history of fever,
abdominal pain and diarrhea. Careful history taking reveals that the patient’s pet
puppy had diarrhea one week ago. Aside from the present episode, the patient
has no significant past medical history and all of his vaccinations are up-to-date.
His stool is negative for ova and parasites. Which of the following is the most
likely cause of this patient’s symptoms?
A. Staphylococcus aureus
B. Shigella
C. Campylobacter
D. Bacillus cereus
E. Vibrio parahemolyticus
F. Giardia lamblia

Explanation:
Of the enteric pathogens listed in the answer choices, only Campylobacter species
can be transmitted from domestic animals to humans. Campylobacter is a curved
Gram negative rod with a filament that allows it to move in a characteristic
“corkscrew” fashion. Campylobacter jejuni is the most common cause of acute
gastroenteritis in children and adults in industrialized countries. Transmission is via
the fecal-oral route. The organism can be acquired from:
1. Domestic animals, such as cattle, sheep, dogs, and chickens. This route of
transmission is common in farm and laboratory workers.
2. Contaminated food, such as undercooked poultry and unpasteurized milk.
Campylobacter species cause inflammatory diarrhea (initially watery, later bloody),
accompanied by abdominal cramping, tenesmus and leukocytes in stool. The
abdominal pain may mimic appendicitis. Campylobacter is the most common
infectious agent associated with Guillain-Barré syndrome.
(Choice A) Gastroenteritis caused by S. aureus results from the action of a
preformed exotoxin and is associated with consumption of certain precooked foods
dairy products, custard and mayonnaise. This disease is more commonly associated
with vomiting and abdominal cramps than diarrhea.
(Choice B) Shigella is a human pathogen transmitted via the fecal-oral route by dirty
hands, fomites in daycare centers, and food contaminated by unhygienic handlers.
Domestic animals do not carry this pathogen.
(Choice D) Bacillus cereus survives on steamed and fried rice where it produces a
heat-stable enterotoxin. (Choice E) Vibrio parahemolyticus is transmitted by
consumption of contaminated shellfish.
(Choice F) Giardia lamblia infection is commonly acquired by drinking contaminated
water. A history of camping, hiking in the mountains and white-water rafting are
classically associated with giardiasis.

Educational Objective:
Campylobacter infection is a common cause of inflammatory gastroenteritis and can
be acquired from domestic animals (cattle, chickens, dogs) or from contaminated
food. The diarrhea is inflammatory and is accompanied by fever, abdominal pain and
tenesmus. Campylobacter infection is associated with Guillain-Barré syndrome.

185
USMLE WORLS STEP 1 MICROBIOLOGY

Q NO 40: A 78-year-old nursing home resident develops high fevers, chest pain and
cough productive of rusty sputum. Sputum microscopy reveals numerous lancet-
shaped Gram positive cocci in pairs. The patient dies despite aggressive hydration
and antibiotic treatment. A vaccine containing which of the following might have
prevented this outcome?

A. Recombinant surface protein


B. Capsular polysaccharide
C. Live attenuated bacteria
D. Killed bacteria
E. Inactivated toxin
F. Synthetic oligopeptides

Explanation:
There are more than 80 serotypes of S. pneumoniae based on variants of the
capsular polysaccharide. The diversity of serotypes makes vaccine development a
complex task. In adults, S. pneumoniae types 1-8 are responsible about 75% of the
time. The 23-valentpneumococcal polysaccharide single-dose vaccine is
recommended for all adults over age 65 and for other patients at high risk for
pneumococcal sepsis (e.g. HIV patients asplenic patients, chronic obstructive
pulmonary disease patients, and immunosuppressed patients). It is 50-90%
efficacious (Choice B).
There is also a 7-valent conjugated vaccine available for use in children less than 2-
years-old, which is about 90% efficacious in these patients. The conjugated vaccine
contains polysaccharide antigens that are protein-coupled in order to stimulate the
T-cell dependent immune (memory) response.
The N. meningitidis and H. influenzae vaccines are other examples of polysaccharide
vaccines.
(Choice A) The Hepatitis B vaccine is a recombinant surface protein vaccine.
(Choice C) Live attenuated bacterial vaccines include the BCG vaccine used outside
of the United States to immunize against tuberculosis and the typhoid vaccine
(against Salmonella typhi). Live attenuated vaccines are very effective at inducing
immunity because the organisms are actively growing within the inoculated host.
The risk associated with these vaccines is the live organism’s potential to revert to a
virulent strain. Live attenuated vaccines are more commonly used for viruses, and
include the vaccines for measles mumps, rotavirus, and Varicella-zoster, as well as
the Sabin polio vaccine.
(Choice D) Killed bacteria vaccines include those for anthrax (Bacillus anthracis),
cholera (Vibrio cholera), and pertussis (Bordetella pertussis) and plague (Yersinia
pestis). Killed vaccines usually require multiple inoculations in order to induce
immunity, but they do not possess the risk of virulent reactivation. This vaccination
method is also more commonly used in viral vaccines, including the hepatitis A,
influenza, rubella, rabies and Salk polio vaccines.
(Choice E) Inactivated toxin vaccines include the diphtheria and tetanus vaccines.
Inactivated toxin vaccines allow the body to immunologically recognize the toxin and
inactivate it by antibody binding.
(Choice F) Synthetic oligopeptides are presently an experimental mode of
vaccination. Researchers are analyzing the structures of proteins expressed by
infectious vectors and attempting to identify oligopeptide epitopes that could be
synthetically produced and used to generate an immune response. As yet no
vaccines using this mechanism have been produced.

186
USMLE WORLS STEP 1 MICROBIOLOGY

Educational Objective:
The pneumococcal polysaccharide vaccine is recommended for all adults over 65
years of age and for patients with COPD, asplenia, or immunosuppression.
Vaccination does not completely prevent pneumonia, as this vaccine only contains
antigen from 23 of the more than 80 different capsular serotypes known. The adult
pneumococcal vaccine is an unconjugated polysaccharide vaccine that, unlike the
infant vaccine, does not stimulate a T-helper response.

187
USMLE WORLS STEP 1 MICROBIOLOGY

Q NO 41: A 6-year-old male is brought to the pediatric emergency room with fever
and sore throat. The parents tell you that the child has not received any
immunizations. Physical exam reveals a grey pharyngeal exudate and Gram stain
shows scant Gram positive organisms. Which of the following culture types would
facilitate the growth of this organism?

A. MacConkey agar
B. Thayer-Martin VON medium
C. Blood agar containing bile and hypertonic saline
D. Cysteine-tellurite agar
E. Bordet-Gengou medium

Explanation:
Corynebacteria (koryne, club) are Gram positive catalase-positive, aerobic or
facultatively anaerobic club-shaped rods. The genus is composed of the species
Corynebacterium diphtheriae and the non diphtheria corynebacteria, also known as
diphtheroids. (Once thought to be clinically insignificant diphtheroids are now
recognized as pathogens that cause diseases like septicemia, urinary tract infections
erythrasma and endocarditis in immunocompromised hosts.)
C. diphtheriae will grow on cysteine-tellurite agar as dark black slightly iridescent
colonies. It can also be cultured in Loffler’s medium where it will develop cytoplasmic
metachromatic granules (visualizable after staining with an aniline dye such as
methylene blue). Because culturing the organism may take days and because
diphtheria has high mortality that warrants immediate treatment more rapid
diagnostic mechanisms such as the immunochromatographic strip assay are being
developed.
(Choice A) MacConkey agar is used to grow many of the enteric bacteria. MacConkey
is a bile salt-containing agar that restricts the growth of most Gram positive
organisms.
(Choice B) Thayer-Martin VCN medium will encourage growth of Neisseria species
while prohibiting growth of other organisms. This heated blood agar, or chocolate
agar is supplemented with the antimicrobial agents Vancomycin, Colistin
(polymyxin), and Nystatin (VCN) which restrict the growth of Gram positive
organisms Gram negative organisms other than Neisseria and yeast respectively.
(Choice C) Blood agar containing bile and hypertonic saline can be used to culture
enterococci and to differentiate enterococci from the non enterococcal Group D
Streptococci. The enterococci include E. faecalis and E. faecium and are able to grow
in the presence of both bile salts and 6.5% hypertonic saline. The non enterococci
include Streptococcus bovis and Streptococcus equinus. Non enterococcal Group D
streptococci grow in the presence of bile but NOT in the presence of hypertonic
saline.
(Choice E) Bordet-Gengou medium is used to culture the veri sensitive Bordetella
pertussis, the causative agent in whooping cough.

Educational Objective:
C. diphtheriae is cultured on cysteine-tellurite agar. The resultant colonies are black
in color. The bacterium produces intracellular polyphosphate granules called
metachromatic granules that can be detected on microscopy after methylene blue
staining.

188
USMLE WORLS STEP 1 MICROBIOLOGY

Q NO 42: A 7-year-old child with a history of recurrent granulomatous skin


infections and a prior episode of Aspergillus pneumonia undergoes a partial A.
hepatectomy to treat a poorly draining liver abscess. The patient is thought
to have a mutation of a structural component of a neutrophil oxidase. This
child also has an increased vulnerability to infections caused by:
Pseudomonas cepacia
B. Mycobacterium tuberculosis
C. Cryptococcus neoformans
D. Herpes simplex type 1
E. Streptococcus pyogenes
F. Giardia lamblia
G. Corynebacterium diphtheriae

Explanation:
This patient has chronic granulomatous disease (COD), a condition caused by
mutations in neutrophil NADPH oxidase. Patients with COD are at increased risk for
infections caused by
• Staphylococcus aureus
• Pseudomonas cepacia (Burkholderia cepacia)
• Serratia marcescens
• Nocardia species
• Aspergillus species
These organisms are all catalase positive. Catalase decomposes H202 (2 H202 — +
2 H20). In COD patients the production rate of H202 (and the downstream
microbicidal HOCL) is impaired. It appears that catalase positivity is necessary but
not sufficient for opportunistic infectivity in COD, since the risk of infection with
other catalase positive microbes (such as Mycobacterium tuberculosis, Cryptococcus
neoformans, and Corynebacterium diphtheriae) is not increased in COD.
(Choice E) Streptococci are catalase negative bacteria. COD patients are predisposed
to infections with catalase positive organisms.

Educational Objective:
Chronic granulomatous disease (COD) results from a genetic defect in NADPH
oxidase. Normally, NADPH oxidase participates in the killing of microbes within
neutrophil phagolysosomes. Patients with COD develop recurrent pulmonary,
cutaneous lymphatic and hepatic infections with a tendency toward granuloma
formation, usually beginning in childhood. These infections are predominantly
caused by:
• Staphylococcus aureus
• Pseudomonas cepacia (Burkholderia cepacia)
• Serratia marcescens
• Nocardia species
• 4spergillus species

189
USMLE WORLS STEP 1 MICROBIOLOGY

Q NO 43: A 23-year-old male is being evaluated for persistent cough and a


pulmonary infiltrate detected on chest X-ray. 3 ml of this patient’s blood is A.
added to an anticoagulated tube and placed into iced water. Several
minutes later clumping is detected inside the tube, but it rapidly un
coagulates when the tube is warmed while being held it in the observer’s
hand. Which of the following organisms is most likely responsible for this
patient’s condition?
Streptococcus pneumoniae
B. Klebsiella pneumoniae
C. Haemophilus influenzae
D. Mycobacterium kansasii
E. Legionella pneumophila
F. Coxiella burnetii
G. Mycoplasma pneumoniae
H. Coccidioides immitis
I. Histoplasma capsulaturn
J. Pneumocystis carinii

Explanation:
The question stem is describing the effect of cold agglutinins. As their name implies,
cold agglutinins cause the agglutination, or clumping, or red blood cells when a
sample of blood containing cold agglutinins is chilled. Cold agglutinins are antibodies
that are produced in response to Mycoplasma pneumoniae infection. These
antibodies are directed against antigens in the cell membrane of M. pneumoniae that
happen to be homologous to antigens that are present on the surface of human
erythrocytes. The test described can be easily done at bedside with blood drawn into
an EDTA-containing tube and a cup of ice. These cold agglutinins are responsible for
the transient anemia that can be documented in many patients with a M.
pneumoniae infection. Cold agglutinins are also associated with Epstein-Barr virus
infection and hematologic malignancies in addition to infection with Mycoplasma
pneumoniae.

Educational Objective:
Infection with Mycoplasma pneumoniae can result in the formation of cold
agglutinins. Other illnesses resulting in cold agglutinin formation include EBV
infection and hematologic malignancy. Cold agglutinins are antibodies specific for red
blood cells that only cause agglutination, or clumping, of red blood cells at low
temperatures.

190
USMLE WORLS STEP 1 MICROBIOLOGY

Q NO 44: Bacteria are injected into the skin of several healthy volunteers as part of
an experiment. Over several weeks, the bacteria migrate slowly through the skin
from the site of injection outward. On physical examination, each patient develops
a flat red skin lesion with a central clearing. Which of the following bacteria was
most likely injected?

A. Mycobacterium scrofulaceum
B. Treponema pallidum
C. Leptospira interrogans
D. Borrelia burgdorferi
E. Rickettsia rickettsiae
F. Bartonella henselae

Explanation:
These cutaneous lesions are consistent with erythema chronicum migrans (ECM) the
classic initial skin lesion of Lyme disease. ECM occurs at the site of Borrelia
burgdorferi (Choice D) inoculation by an Ixodes tick bite. ECM begins as an
erythematous macule that enlarges with an advancing erythematous border and
central clearing. The classic lesion is erythematous (red) and ring-shaped (annular).
Often the central clearing does not appear, and the rash remains an erythematous
macule or patch.
(Choice A) Mycobacterium scrofulaceum is the etiologic agent of scrofula, a disease
characterized by lymphadenitis (usually cervical lymphadenitis) that occurs with
greatest frequency in children.
(Choice B) Treponema pallidum is the etiologic agent of syphilis. Syphilis causes a
painless ulceration, or chancre, at its site of inoculation. In secondary syphilis
condyloma lata and a generalized macular eruption may appear.
(Choice C) The spirochete Leptospira interrogans is the etiologic agent of
leptospirosis. L. interrogans produces no characteristic cutaneous manifestations.
Infection is usually asymptomatic and self-limited, though serious cases can
progress to Weil’s syndrome. In Weil’s syndrome hepatic dysfunction produces a
conjugated hyperbilirubinemia, renal dysfunction, thrombocytopenia, and bleeding.
(Choice E) Rickettsia rickettsia is the etiologic agent of Rocky Mountain spotted fever
(RMSF). RMSF is a tick- borne illness characterized by a palmoplantar erythematous
macular or petechial eruption that spreads proximally to eventually involve the
trunk. Patients also experience headache, fever and conjunctival hyperemia.
Diagnosis is based on clinical findings and patient history prompt treatment with
doxycycline is required.
(Choice F) Bartonella henselae is the Gram negative etiologic agent of cat scratch
fever. Cat scratch fever, as the name implies, occurs after a cat scratch or bite.
Clinical manifestations include regional lymphadenopathy, malaise and fever. Other
than regional lymphadenopathy, B. henslae infection is not associated with any
characteristic cutaneous lesions.

Educational Objective:
The characteristic cutaneous lesion in B. burgdorferi infection (Lyme disease) is
erythema chronicum migrans (ECM). Typically, ECM begins as an erythematous
macule at the site of the inoculating tick bite. The macule then spreads outward
leaving a zone of central clearing. ECM is characteristic of the first stage of Lyme
disease, but can recur at various distant skin sites in the second stage of Lyme
disease as well.

191
USMLE WORLS STEP 1 MICROBIOLOGY

Q NO 45: A 7-year-old Caucasian boy is brought to your office with blisters on his
face. Some of the blisters have broken and are covered with golden yellow crusts.
Exudate microscopy reveals Gram-positive cocci in chains. Which of the following
would be a component of the clinical syndrome that may follow such an infection?
A. Joint pain and eve redness
B. Fatigue and heart murmurs
C. Face puffiness and dark urine
D. Bilateral facial nerve palsy
E. Abdominal pain and jaundice
Explanation:
Staphylococcus aureus
and Streptococcus pyogenes(so called pyogenic cocci) cause prima diseases of the
skin and subcutaneous tissues, frequently infecting wounds, abrasions and burns,
Impetigo, a vesicular, blistering eruption eventually leading to formation of a golden
yellow crust, is usually seen in children and newborns, frequently occurs periorally,
and can be caused by either Staphylococcus aureus and/or Streptococcus pyogenes.
Acute rheumatic fever (ARE) and Acute Post Streptococcal Glomerulonephritis
(APSGN) are the late sequelae of untreated or partially treated S. pyogenes (Group
A Strep) infections. APSGN can follow a streptococcal skin infection (impetigo) or a
streptococcal pharyngeal infection. If APSGN occurs, it will occur 1-5 weeks after the
onset of the streptococcal infection. Rheumatic fever, on the other hand, is
associated only with Group A Streptococcal (GAS) throat infection.
Hypertension hematuria, nephritic range proteinuria, and RBC casts in the urine
following a GAS infection suggest acute post streptococcal glomerulonephritis.
Patients frequently present with facial edema and dark colored (cola colored) urine
(Choice C). The renal damage is due to immune complex deposition on the
glomerular basement membrane and activation of complement. Complement
activation is responsible for the massive inflammatory response and glomerular
basement membrane structural damage seen in APSGN.
(Choice A) Joint pain and eye redness can occur with many conditions including
Sjogren syndrome lupus and Reiter syndrome. Reiter syndrome is an HLA-B27
related illness that occurs most classically in relation to an episode of Chlamydia
trachomatis urethritis. It is an inflammatory arthritis that usually affects large joints
and causes uveitis and conjunctivitis. The triad of arthritis, uveitis/conjunctivitis and
urethritis defines Reiter syndrome.
(Choice B) Fatigue and heart murmurs following a GAS pharyngeal infection can be
due to acute rheumatic fever. When rheumatic fever occurs, it is usually 4-6 weeks
following an untreated episode of streptococcal pharyngitis; it DOES NOT occur in
relation to streptococcal skin infections.
(Choice D) Bilateral facial nerve palsy is extremely rare. Infectious causes most
classically include Lyme disease (Borrelia burgdorferi) and, less commonly, HIV.
(Choice E) Abdominal pain and jaundice in a child can indicate acute viral hepatitis
most likely due to Hepatitis
A. Hepatitis A is transmitted by the fecal-oral route, and outbreaks have been
described in child care settings.
Educational Objective: Rheumatic fever and acute post-streptococcal
glomerulonephritis are the late sequelae of Group A Streptococcal (Streptococcus
pyogenes) infections. Post-streptococcal glomerulonephritis can follow either a skin
infection (impetigo) or an episode of streptococcal pharyngitis, whereas rheumatic
fever is associated only with streptococcal throat infection.

192
USMLE WORLS STEP 1 MICROBIOLOGY

Q NO 46: A toxic substance produced by C. perfringens induces massive hemolysis


and tissue necrosis when injected into mice. The lethal effect observed in the
experiment correlates with the substance’s ability to split

A. Collagen
B. Hyaluronic acid
C. Phospholipids
D. Carbohydrates
E. Plasminogen

Explanation:
Lecithinase is the main toxin of C. perfringens; its concentration correlates with its
lethal and necrotic effects. Lecithinase, also known as phospholipase C or alpha toxin
is an enzyme that catalyzes the splitting of phospholipid molecules. (Recall that
human phospholipase A2 is the enzyme that catalyzes arachidonic acid release from
phospholipid cell membranes in the first step of leukotriene, thromboxane, and
prostaglandin synthesis.) Phospholipase C hydrolyzes lecithin-containing lipoprotein
complexes in cell membranes causing cell lysis (including RBC hemolysis), tissue
necrosis and edema. C. perfringens has atleastl2toxins of which Alpha toxin is the
most injurious.
(Choice D) C. perfringens uses carbohydrates for energy. Its rapid metabolism of
muscle tissue carbohydrate produces significant amounts of gas, which can be
demonstrated radiographically by either plain film X-ray or CT scan.
(Choice E) Plasminogen activators like streptokinase, urokinase and tissue
plasminogen activator (tPA) convert plasminogen to plasmin. Of the three enzymes
mentioned, only streptokinase is a bacterial product, and as the name implies, itis
an exotoxin released by Streptococcus pyogenes (Group A Streptococcus).

Educational Objective:
Lecithinase, also known as alpha toxin, is the main toxin produced by C. perfringens.
Its function is to degrade lecithin, a component of cellular phospholipid membranes,
leading to membrane destruction, cell death and widespread necrosis and hemolysis.

193
USMLE WORLS STEP 1 MICROBIOLOGY

Q NO 47: A 34-year-old immigrant from Africa has patchy areas of skin anesthesia
and hypopigmentation on his upper extremities. Nerve biopsy evaluated under
light microscopy shows many bacteria invading Schwann cells. This patient’s
disease is most likely caused by:

A. Borrelia burgdorferi
B. Treponema pallidium
C. Corynebacteria diphtheriae
D. Mycobacteria leprae
E. Campylobacter fetus

Explanation:
Leprosy, or Hansen disease, is a deforming infection primarily of the skin and nerves
that is caused by Mycobacterium leprae. Transmission is believed to occur through
the respirators’ route, although direct cutaneous contact has not been excluded as a
mode of transmission. Infection has been also associated with armadillo contact in
the southwest United States. Leprosy has a wide range of clinical manifestations that
vary depending on the strength of the cell-mediated immune response to the
organism.
The most severe form of leprosy is termed “lepromatous leprosy” and occurs in
patients with a weak cell-mediated (Th1) immune response. Without an appropriate
cell-mediated response macrophages are never given the signal to kill the
mycobacterial organisms. As a result M. leprae multiplies and disseminates widely.
M. leprae grows best at temperatures that are slightly lower than core body
temperature, which partially explains why the features of leprosy are primarily
observed in the skin superficial nerves, eyes and testes. Lepromatous leprosy
clinically manifests as diffuse skin thickening, cutaneous hypopigmentation in
plaques (often accompanied by hair loss), leonine facies, paresis and regional
anesthesia of motor and sensory nerves, and testicular destruction and blindness.
The least severe form of leprosy is often self-limited and is called “tuberculoid
leprosy.” In this subtype, the mycobacterial infection is limited by an intact cell-
mediated immune system. Mild skin plaques develop that are associated with
hypopigmentation, hair follicle loss, and focally decreased sensation.
Features intermediate between the tuberculoid and lepromatous forms are often
seen, demonstrating the true spectrum of this disease.
(Choice A) Borrelia burgdorferi is the spirochete responsible for Lyme disease.
Symptoms of Lyme disease involve a characteristic skin rash (erythema chronicum
migrans, which occurs most frequently at the site of the tick bite) fever myalgias
and malaise. Systemic disease can progress to cause arthritis, facial paresis, and/or
cardiac inflammation (which may be associated with conduction abnormalities).
Prolonged untreated disease can lead to CNS effects similar to those of tertiary
syphilis.
(Choice B) Treponema pallidum is the spirochete responsible for syphilis. Syphilis is
a sexually transmitted disease that initially causes a painless ulceration (chancre) at
the site of inoculation, most commonly the genitalia. Systemic illness is described as
secondary syphilis and results in the diffuse eruption of erythematous macules over
the entire body, including the palms and soles (which differentiate it from many
other rashes) and the formation of condyloma lata. Tertiary syphilis results in
gummas of the skin and bone ascending aortitis, and neurosyphilis.
(Choice C) Corynebacterium diphtheriae is the Gram-positive rod responsible for
diphtheria.

194
USMLE WORLS STEP 1 MICROBIOLOGY

(Choice E) Campylobacter fetus is a Gram-negative rod responsible for mild enteritis


in immunocompetent patients and mild systemic bacteremic illness in
immunocompromised patients.

Educational Objective:
Leprosy, or Hansen disease, is a systemic illness caused by Mycobacterium leprae.
The severity of disease depends on the strength of the cell-mediated immune
response, with tuberculoid leprosy the milder form and lepromatous leprosy the
more severe form.

195
USMLE WORLS STEP 1 MICROBIOLOGY

Q NO 48: A 74-year-old previously healthy Caucasian male comes to his physician’s


office complaining of abrupt on set fever, headache, myalgias, malaise, cough and
throat pain. His two granddaughters missed several days of school because of
similar symptoms. Examination demonstrates mild hyperemia of the throat without
any exudate, and the patient is sent home on conservative management. Five days
later, he is admitted to the hospital with progressive dyspnea, chest pain, and
productive cough. Which of the following pathogens is most likely to be isolated
from this patient’s sputum?

A. Listeria monocytogenes
B. Klebsiella pneumoniae
C. Staphylococcus aureus
D. Nontuberculous mycobacteria
E. Cytomegalovirus

Explanation:
This patient’s presenting signs and symptoms and report of similarly ill household
children are consistent with influenza infection. Outbreaks of influenza A can affect
50-75% of school-aged children, many of whom then spread the virus to family
members. Individuals infected with influenza A tend to experience abrupt onset
fever, headache myalgia, and malaise: signs and symptoms gradually improve over
a period of two to five days.
A subset of patients stricken by influenza go on to develop secondary bacterial
pneumonia characterized by recurrent fever, dyspnea, and productive cough. The
elderly are particularly at risk for this complication. Physical examination and chest
radiograph demonstrate pulmonary consolidation. Patients recently infected with
influenza are vulnerable to secondary bacterial infection because of virally-induced
damage to the mucociliary clearance mechanisms of the respiratory epithelium. In
order, the pathogens most often responsible for secondary bacterial pneumonia are
Streptococcus pneumoniae, Staphylococcus mucus, and Haemophilus influenzae.
(Choice A) Listeria monocytogenes is an occasional cause of septicemia and purulent
meningitis in neonates.
(Choice B) Klebsiella pneumoniae is most commonly responsible for nosocomial
urinary tract infections nosocomial pneumonia, and pneumonia in alcoholics and IV
drug abusers.
(Choice D) Mycobacterium avium intracellulare causes disseminated disease in AIDS.
Mycobacterium kansasii may yield pulmonary tuberculosis-like symptoms.
Mycobacterium leprae is responsible for cutaneous leprosy. Mycobacterium
scrofulaceum causes cervical lymphadenitis in children.
(Choice E) Cytomegalovirus (CMV) may cause pneumonia particularly in the
immunocompromised (eg, those with HIV infection).

Educational Objective:
Patients older than 65 are particularly prone to developing secondary bacterial
pneumonia after influenza infection. In order, the pathogens most often responsible
for secondary bacterial pneumonia are Streptococcus pneumoniae, Staphylococcus
aureus, and Haemophilus influenzae.

196
USMLE WORLS STEP 1 MICROBIOLOGY

Q NO 49: A 22-year-old male ingests a solution containing 13C-labeled urea. He is


then asked to blow into a tube, and the labeled carbon is detected in his breath
samples. This test is most likely part of the evaluation for which of the following
conditions?

A. Bronchiectasis
B. Pulmonary fibrosis
C. Congenital heart disease
D. Chronic hepatitis
E. Duodenal ulcer
F. Acute pancreatitis
G. Chronic diarrhea

Explanation:
The test described is a screening assay for the presence of urease activity, an
indirect means of detecting the presence of Helicobacter pylori, a major cause of
duodenal ulcer. Culturing the organism from gastric biopsy specimens is considered
the definitive confirmatory test. The noninvasive urease breath test involves
consuming a solution containing isotopically-labeled urea. When present urease (a
product of H. pylon) degrades the urea into carbon dioxide and ammonia. The
isotopically-labeled carbon dioxide is absorbed into the bloodstream and exhaled in
the patient’s breath. Typically, breath samples are collected 30 minutes after the
labeled urea is ingested. This test has excellent sensitivity and specifically for both
the initial diagnosis of H. pylon infection and for monitoring treatment success.
Antibiotic or proton pump inhibitor use during the 2-4 weeks prior to the test may
cause false negative results.
(Choice A) High-resolution chest CT is the test of choice for diagnosing
bronchiectasis.
(Choice B) High resolution CT scan chest X-ray, pulmonary function testing, lung
biopsy, and serologies may all play a role in diagnosing pulmonary fibrosis, but none
is considered a gold standard.
(Choice C) Congenital heart diseases (e.g. patent foramen ovale, ventricular septal
defect) are most commonly diagnosed by echocardiography.
(Choice D) Viral hepatitis is diagnosed when viral components and antibodies against
the hepatitis virus are detected in the blood.
(Choice F) Acute pancreatitis is usually diagnosed by correlating a patient’s clinical
presentation (i.e. the characteristic epigastric pain boring through to the back) with
elevated amylase and lipase levels.

Educational Objective:
The urease breath test is used to noninvasively detect H. pylon infection. The patient
consumes 13C-labeled urea and his breath is then monitored for the presence of
13C-labeled carbon dioxide which would indicate the presence of the H. pylon
product urease in the stomach.

197
USMLE WORLS STEP 1 MICROBIOLOGY

Q NO 50: A 3-month old infant is irritable, feeds poorly and frequently vomits. His
current weight is at the 45th percentile and his head circumference as at the 96th
percentile. Funduscopy reveals white-yellow chorioretinal lesions in both eyes.
Head CT shows enlarged ventricles and scattered intracranial calcifications. Which
of the following is the most likely cause of this patient’s condition?
A. Chromosomal abnormality
B. Single gene disease
C. In-utero infection
D. Intrapartum infection
E. Postpartum infection

Explanation:
Hydrocephalus, intracranial calcifications and chorioretinitis form the classic triad of
congenital toxoplasmosis. Hydrocephalus occurs due to CNS inflammation and is
evidenced by macrocephaly and enlargement of the ventricles. Chorioretinitis s
refers to inflammation of the choroids and the retina that can leave cotton-like
white/yellow scars on the retina visible on funduscopy. Affected neonates also have
hepatosplenomegaly, rash and multiple neurological abnormalities such as seizures,
altered muscle tone and ocular movement defects. Occasionally microcephaly occurs
when hydrocephalus is not severe.
Congenital toxoplasmosis is transmitted transplacentally. The fetus is affected only if
the mother is infected with toxoplasmosis during the first six months of pregnancy.
As the majority of healthy individuals are exposed to Toxoplasma early in childhood,
most adults are immune and primary infection in pregnancy is rare. Pregnant
women, however, should be warned not to handle cat litter in order to prevent
contact with Toxoplasma which is often found in cat feces.
(Choice D) Infection with herpes simplex is acquired by a neonate during delivery
(intrapartum). Ophthalmia neonatorum refers to neonatal conjunctivitis caused by a
number of agents such as Chlamydia, Neisseria (which can cause blindness), and
viruses; these infections are acquired during birth. Group B streptococcal sepsis is
also acquired during birth.
(Choice E) Postpartum infections include any infections acquired after birth and are
numerous.

Educational Objective:
Congenital toxoplasmosis is a transplacental infection (acquired in utero). Its classic
triad includes hydrocephalus, intracranial calcifications and chorioretinitis. Expecting
mothers should avoid cat feces to help prevent exposure to Toxoplasma.

198
USMLE WORLD STEP 1 MICROBIOLOGY

Q NO 51: A 34-year-old male dies from a fulminant S. pneumoniae infection. His


past medical history was significant only for a motor vehicle accident two years
ago in which he sustained blunt abdominal trauma and required emergent
laparotomy. Impairment of which of the following protective mechanisms most
likely contributed to the severity of this patient’s infection?
A. Complement fixation
B. Interferon release
C. Bacterial clearance
D. Opsonization
E. Intracellular killing

Explanation:
This patient likely experienced splenic rupture secondary to abdominal trauma two
years ago with splenic remnants subsequently removed during the laparotomy. The
spleen is a part of the systemic lymphoid system that receives roughly 6% of the
cardiac output. Its red pulp filters and destroys aged erythrocytes and serves as an
emergency source of erythrocytes, granulocytes and platelets that can be instantly
delivered into the circulation when needed. The white pulp consists of lymphoid
tissue with T- and B-cell regions. Splenic mononuclear phagocytes and dendritic cells
ingest circulating bacteria particularly un opsonized organisms, and present them to
B- and T-cells residing in the spleen in order to quickly mount an immune response.
Nearly half of the body’s total immunoglobulins are produced by the splenic B-
lymphocytes.
Asplenic patients are predisposed to sepsis with encapsulated bacteria such as S.
pneumoniae, H. influenzae, and N. meningitidis due to a decreased ability to
recognize and clear these organisms. Vaccination against these three organisms is
recommended for asplenic patients.
(Choice A) Deficiencies of key components of the complement cascade can cause
defects in complement fixation. C3 deficiency predisposes to recurrent infections
with encapsulated organisms. Deficiencies of C5-9, the components of the
membrane attack complex, can lead to recurrent infections with Neisseria
meningitidis or N. gonorrhoeae.
(Choice B) Interferon is released from virus-infected cells and causes macrophage
and cytotoxic T-lymphocyte activation and destruction of virus-infected cells. Defects
in interferon release lead to increased susceptibility to viral infections.
(Choice D) Opsonization is the process by which antibodies and complement flag
foreign material for phagocytosis and destruction. Complement 3 deficiency is an
example of an opsonic defect; this condition predisposes to recurrent infections with
encapsulated organisms.
(Choice E) Chronic Granulomatous Disease (CGD) is an example of an intracellular
killing defect. This X-linked disorder is characterized by failure of the phagocyte
oxidative burst due to absence of or defects in NADPH-oxidase.
In patients with CGDI catalase-positive organisms like Staphylococcus aureus are
phagocytosed but not killed.
Patients experience recurrent suppurative infections (e.g. abscesses, pneumonias.)

Educational Objective:
The spleen serves both as a site of antibody synthesis and as a reservoir of
phagocytic cells capable removing circulating pathogens. Asplenic patients are prone
to infections caused by encapsulated organisms such as S. pneumoniae, H.
influenzae and N. meningitidis.
Q NO 52: Several students returning from a cave exploration trip to the eastern
United States develop fever, cough and malaise. Pulmonary infiltrates and hilar
adenopathy are apparent on chest x-rays. Lung tissue specimens would most 199
likely show:
USMLE WORLD STEP 1 MICROBIOLOGY

A. Multinucleated spherules
B. Ovoid cells within macrophages
C. Budding yeast with a thick capsule
D. Pseudohyphae and blastoconidia
E. Septate hyphae with dichotomous branching

Explanation:
Histoplasma capsulatum is a dimorphic fungus that is found as a mold in soil. It is
also present in bird and bat droppings, and is endemic to the Mississippi and Ohio
River basins. Patients may report a history of exploring caves (exposure to bats) or
cleaning bird cages or coops.
H. capsulatum is transmitted by the respiratory route when bird or bat droppings
containing fungal spores are inhaled. In the lungs, the fungus is ingested by
macrophages, and is seen on light microscopy as small intracellular oval bodies. The
immune reaction to Histoplasma closely resembles that induced by M. tuberculosis:
a cellular response with formation of granulomata. Because the fungus targets
histiocytes and the reticuloendothelial system, it may cause lymphadenopathy and
hepatosplenomegaly.
While the majority of immunocompetent hosts remain asymptomatic, some may
develop acute pulmonary disease (cough, fever, pleuritic chest pain and pulmonary
infiltrates). Furthermore, individuals with underlying lung disease may develop
chronic pulmonary histoplasmosis, a condition that clinically resembles tuberculosis
(patients present with cough, malaise, weight loss and cavitations in the upper lung
lobes). Disseminated disease occurs in immunocompromised individuals.
(Choice A) Spherules packed with endospores are found in Coccidioides immitis
infection. Like Histoplasma, Coccidioides can cause pulmonary disease in
immunocompetent individuals. Coccidioides is endemic to the southwestern U.S. and
is not associated with cave exploration.
(Choice C) Cryptococcus neoformans takes the form of a budding yeast with a thick
capsule. This yeast also grows abundantly in soil containing bird (pigeon) droppings.
However, this fungus tends to cause disease (meningoencephalitis and pulmonary
disease) in the immunocompromised.
(Choice D) Candida species are yeasts that form pseudohyphae. Blastoconidia are
spores that grow as buds on the fungal hyphae. Candida infection is usually not
associated with pulmonary infiltrates or lymphadenopathy.
(Choice E) Aspergillus fumigatus causes pulmonary disease in immunocompromised
patients. This fungus is seen in tissue sections as septate hyphae with V-shaped
branching.

Educational Objective:
Histoplasma capsulatum is a dimorphic fungus located intracellularly within
macrophages. It affects the lungs and reticuloendothelial system. It is present in
bird and bat droppings, and is endemic to the Mississippi and Ohio River basins.

200
USMLE WORLD STEP 1 MICROBIOLOGY

Q NO 53: Clustered Gram positive bacteria are isolated from the tonsillar exudates
of a 6-year-old boy. On microscopic examination, the bacteria have polar granules
that stain deeply with aniline dyes. The pathogenicity of these organisms is the
result of:

A. Widespread T-cell activation


B. Blockade of neurotransmitter release
C. Activation of electrolyte transport
D. Impairment of protein synthesis
E. Cellular membrane disruption

Explanation:
The question stem describes Corynebacterium diphtheriae, a non-motile
unencapsulated, Gram positive rod. The organisms are often found in clumps
resembling Chinese letters or joined in V- or Y-shaped chains on microscopic
examination. Their cytoplasm contains metachromatic granules that stain with
aniline dyes like methylene blue.
C. diphtheriae produces a classic two subunit AB exotoxin. The B (binding) subunit
binds specifically to the heparin-binding epidermal growth factor receptor found on
cardiac and neural cells, and is responsible for the toxin’s affinity for heart and
nervous tissue. The B subunit induces endocytosis of the toxin, and the subsequently
released A (active) subunit inhibits host cell protein synthesis by catalyzing the ADP-
ribosylation of protein elongation factor 2 (EF-2). (EF-2 is necessary for tRNA to
insert new amino acids into the growing protein chain during translation.) By
inhibiting cell protein synthesis (choice D), the toxin causes cell death.
(Choice A) Superantigens such as the staphylococcal and streptococcal toxic shock
toxins cause widespread T-cell activation. Superantigens bind nonspecifically to T-
cell receptors and MHC class II molecules on antigen presenting cells, leading to
widespread T-cell stimulation and inflammation.
(Choice B) Toxins released by Clostridium botulinum and Clostridium tetani cause
blockade of neurotransmitter release. Botulinum toxin is a neurotoxin that blocks
exocytosis of acetylcholine-containing synaptic vesicles from peripheral nerve
terminals, causing cranial and peripheral nerve palsies, muscle weakness and
respiratory paralysis. The tetanus toxin inhibits the release of the inhibitory
neurotransmitters GABA and glycine, resulting in tetany.
(Choice C) Certain virulent enteric bacteria, such as Enterotoxigenic E. coil (ETEC),
Campylobacter jejuni, Vibrio cholera, and Yersinia enterocolitica, release exotoxins
that activate electrolyte transport. These toxins are all AB exotoxins with one A
subunit and five B subunits. The B subunits are responsible for cell binding and toxin
endocytosis. The A subunit then stimulates a G-protein to increase intracellular
concentrations of cAMP, which causes active efflux of Na and C ions and resultant
cell water loss into the gut lumen, producing watery diarrhea and dehydration.
(Choice E) Clostridium perfringens releases a cytolytic toxin, called alpha-toxin or
phospholipase C, that degrades cell membrane phospholipids, causing cell
destruction.

Educational Objective:
Diphtheria exotoxin inhibits host cell protein synthesis by catalyzing the ADP-
ribosylation of host cell elongation factor-2 (EF-2).

201
USMLE WORLD STEP 1 MICROBIOLOGY

Q NO 54: A 46-year-old Caucasian male presents to your office with the skin
findings pictured below. The lesions are determined to be fungal in etiology.
Which of the following scenarios is most likely a component of this patient’s
history?

A. Immune system disease


B. Recent antibiotic use
C. Animal contact
D. Thorn prick
E. Exposure to sea water
F. Bat guano exposure

Explanation:
The subcutaneous nodules pictured above are consistent with sporotrichosis, a
subcutaneous mycosis caused by Sporothrix schenckii. Sporothrix is a dimorphic
fungus found in the natural environment in the form of mold (hyphae). It resides on
the bark of trees, shrubs and garden plants, and on plant debris in soil. It enters the
body through breaks in the skin (often via thorn prick) and spreads along the
lymphatics. Sporotrichosis is common in gardeners.
The initial lesion — a reddish nodule that later ulcerates — appears at the site of the
thorn prick or other skin injury. Biopsy of the lesion would reveal a granuloma
consisting of histiocytes, multinucleated giant cells, and neutrophils, surrounded by
plasma cells. From the site of inoculation, the fungus spreads along the lymphatics
forming subcutaneous nodules and ulcers.
The diagnosis of sporotrichosis is made by culturing the affected area and isolating
Sporothrix schenckii.
(Choice A) Immune system disease predisposes to systemic mycoses (candidiasis,
aspergillosis and mucormycosis). Sporotrichosis, in contrast, commonly affects
immunocompetent individuals. Disseminated sporotrichosis is very rare.
(Choice B) Recent antibiotic use is associated with superficial Candida disease, such
as oral thrush or vulvovaginitis.
(Choice C) Animal contact is associated with the dermatophytoses caused by
Microsporon species. Microsporon canis causes tinea capitis.
(Choice E) Aeromonas and Vibrio vulnificus may cause cellulitis related to freshwater
or seawater exposure.
(Choice F) Exposure to bats (e.g. during cave exploration) or to bird droppings in the
Ohio and Mississippi areas is associated with histoplasmosis. H. capsulaturn typically
causes lung disease resembling tuberculosis.

Educational Objective:

202
USMLE WORLD STEP 1 MICROBIOLOGY

Sporothrix schenckii is a dimorphic fungus that causes a subcutaneous mycosis. It is


often transmitted by a thorn prick. The disease manifests with nodules that spread
along lymphatics.

203
USMLE WORLD STEP 1 MICROBIOLOGY

Q NO 55: A 5-year-old boy is brought to the FR with breathing difficulty, dysphagia,


drooling and fever. His temperature is 39.4C (103.0 F). White blood cell count is
23,000fcmm with many band forms. Laryngoscopy in the operating room shows a
swollen and cherry-red epiglottis. This patient most likely:

A. Traveled recently
B. Missed vaccination
C. Was stung by a bee
D. Has penicillin allergy
F. Has characteristic family history

Explanation:
Acute epiglottitis is a rapidly progressive infection of the epiglottis leading to severe
inflammation and edema of the epiglottis and larynx and potentially acute
obstruction of the airway especially during laryngoscopy. Small children typically
present with fever and dysphagia, while older children and adults complain of sore
throat. Inspiratory stridor and anxiousness due to compromised diameter of the
larynx occur frequently, and patients may present with drooling. Diagnosis is
confirmed by the presence of an edematous epiglottis that classically appears cherry
red though inspection of the epiglottis should not be done unless the team is
prepared to provide a surgical airway by tracheostomy. The most likely pathogen in
children is H. influenzae, but the introduction of the H. influenzae type b (Hib)
vaccine has led to a dramatic decrease in the incidence of invasive disease caused
by Haemophilus influenza type b including epiglottitis, meningitis, sepsis and other
diseases commonly caused by this bacterium.
The Haemophilus influenzae b vaccine is now a part of the routine vaccination
schedule and is administered in the first few months of life. Immunity during the
first couple months of life prior to this vaccination is acquired by transient maternal
transplacental lgG antibodies. If a child were to miss a vaccination or not be
vaccinated at all, then the child would be at an increased risk of H. influenzae type b
infection, though Hib epiglottitis has been shown to occur in fully immunized children
as well.
(Choice A) Recent travel can alert the physician to suspect diseases that are
endemic to the area that the patient traveled. Epiglottitis is not associated with
travel. Malaria is associated with travel in Africa, HIV is rampant in Africa and Haiti,
Coccidioides immitis is common in the southwest United States, Blastomycosis in the
Mississippi river valley and Histoplasmosis common in the Ohio River valley, but this
list is by no means comprehensive of all diseases associated with geography.
(Choice C) Bee stings are not associated with bacterial infections of any kind unless
the wound left by a sting becomes somehow super infected. Bee stings can induce
clinical disease ranging from local allergic reactions around the site of the sting to
anaphylaxis and death. Bee venom contains hyaluronidase, phospholipase, and other
proteins which initiate an IgE-mediated response.
(Choice D) Penicillin allergy is not associated with epiglottitis. Adverse reactions to
penicillin can range from rash and hives to angioedema, bronchospasm, and
anaphylaxis.
(Choice E) Family history is not a factor in epiglottitis except for cases of heritable
immune system defects that predispose to recurrent bacterial infections, but these
disorders would not predispose purely to epiglottitis.

204
USMLE WORLD STEP 1 MICROBIOLOGY

Educational Objective:
Rapidly progressing fever, severe sore throat, drooling and progressive airway
obstruction potentially accompanied by stridor are the presenting symptoms of acute
epiglottitis. This illness is most commonly caused by H. influenzae type b, but the
Hib vaccine has dropped the incidence of this disease considerably. H. influenzae
type b can still cause disease in un immunized or improperly immunized patients as
well as fully immunized patients in some cases.

205
USMLE WORLD STEP 1 MICROBIOLOGY

Q NO 56: A 23-year-old Caucasian student who recently returned home from a trip
to the seaside complains of white spots over the otherwise suntanned skin of his
chest and shoulders. He has no other medical problems and does not take any
medications. He denies any substance abuse. Which of the following is the most
likely cause of this patient’s condition?

A. Malassezia furfur
B. Microsporum can/s
C. Rhizopus species
D. Aspergillus fumigatus
E. Candida albicans
F. Cryptococcus neoformans
G. Blastomyces dermatitidis
H. Histoplasma capsulatum
I. Coccidioides immitis
J. Sporothrix schenckii

Explanation:
All medically important fungi may be divided into a few groups according to the area
of involvement
1. Cutaneous mycoses include dermatophytosis and pityriasis versicolor.
2. Sporotrichosis is an example of a subcutaneous mycosis.
3. Mycoses with systemic involvement (most often affecting the lungs) are
histoplasmosis, coccidioidoses and blastomycosis.
4. Opportunistic mycoses mainly affect immunosuppressed patients. These include
Candida, Aspergillus, Mucor and Rhizopus species.
Hypopigmented or hyperpigmented skin patches or papules are characteristic of
pityriasis versicolor (also called tinea versicolor), a benign condition caused by the
fungus Malassezia furfur. The infection is localized to the stratum corneum of the
skin. It is more common in areas with hot and humid climates. Hypopigmented
areas are commonly located on the back and chest, and are more visible on
suntanned skin because affected areas do not tan. Most cases of tinea versicolor
occur in healthy individuals with no immunologic deficiencies.
Pityriasis versicolor is usually asymptomatic and is only problematic in terms of its
cosmetic appearance. The diagnosis is made by KOH treatment of skin scrapings.
Malassezia furfur forms spores and hyphae, producing the characteristic “spaghetti
and meatballs” appearance on light microscopy. The hyphae have a short “cigar-
butt” appearance. This condition is treated with topical antifungal agents or
selenium-containing shampoo.

Educational Objective:
Hypo- or hyperpigmented skin patches that become more visible after tanning are
characteristic of pityriasis versicolor, a condition caused by Malassezia furfur. KOH
preparation of skin scrapings reveals a “spaghetti and meatballs” appearance on
light microscopy.

206
USMLE WORLD STEP 1 MICROBIOLOGY

Q NO 57: Gram-positive bacteria are inoculated under the skin of experimental


animals and then the infection is treated with antibiotics. Bacteria isolated from
the injection site several days later assume a spherical configuration when A.
placed in an isotonic solution and disintegrate rapidly when placed in a
hypotonic solution. Which of the following antibiotic was most likely used in this
experiment?
Chloramphenicol
B. Azithromycin
C. Ciprofloxacin
D. Cefuroxime
E. Doxycycline

Explanation:
The question stem describes the inoculation of Gram-positive organisms into an
experimental animal and treatment with an antibiotic that causes these organisms to
be destroyed when exposed to a hypotonic solution. Recall that Gram positive
organisms have a cytoplasmic membrane composed of a phospholipid bilayer as well
as a peptidoglycan cell wall outside of that cell membrane. The peptidoglycan cell
wall provides the shape of the bacterium as well as resistance to osmotic stress.
Under normal circumstances, Gram-positive organisms would not be destroyed by
variations in tonicity within a certain range due to their intact peptidoglycan cell wall.
Because the organisms described in this question were destroyed by placement in a
hypotonic solution one can infer that there was damage to the peptidoglycan cell
wall by the antibiotic used. The only antibiotic listed that acts against cell wall
synthesis is Cefuroxime (Choice D). Cefuroxime is a second generation
cephalosporin. Cephalosporins are Beta lactam antibiotics and are related to
penicillins. Vancomycin is another example of an antibiotic that inhibits cell wall
synthesis.
(Choice A) Chloramphenicol is an anti-ribosomal antibiotic that acts to inhibit the
50S bacterial ribosomal subunit. Its use is limited by its toxicity which includes a
dose-related myelosuppression leading to anemia as well as aplastic anemia, the
most feared side effect of this drug.
(Choice B) Azithromycin is a macrolide antibiotic that acts by inhibiting the bacterial
50S ribosomal subunit as do all other macrolide antibiotics, chloramphenicol and
linezolid.
(Choice C) Ciprofloxacin is an antibiotic in the fluoroquinolone class of antibiotics.
The fluoroquinolones act by inhibiting bacterial DNA gyrase thereby leading to
bacterial DNA damage and bacterial death. These agents have no effect on the cell
wall.
(Choice E) Doxycycline, tetracycline and minocycline are antibiotics that act by
inhibiting the bacterial 30S ribosomal subunit. The aminoglycosides are the other
major class of antibiotics that act by this mechanism. Neither the cyclines nor the
aminoglycosides would act to disrupt the bacterial cell wall.

Educational Objective:
Penicillins, cephalosporins and vancomycin are able to disrupt the peptidoglycan cell
wall of Gram-positive and Gram-negative organisms. The peptidoglycan cell wall of
these organisms gives them the ability to survive osmotic stress; this ability is lost
after treatment with these antibiotic agents.

207
USMLE WORLD STEP 1 MICROBIOLOGY

Q NO 58: A 34-year-old male develops fever myalgias, malaise and progressive


fatigue over a two-week period. He denies any sore throat. He recently received
a blood transfusion while hospitalized for a bleeding duodenal ulcer. Physical A.
examination reveals mild splenomegaly. Lymphocytosis is identified in the
peripheral blood with 30% atypical lymphocytes. The patient’s serum fails to
agglutinate sheep erythrocytes. The agglutination test is repeated one week
later and again no agglutination is noted. Which of the following is the most
likely cause of this patient’s condition?
Epstein-Barr virus
B. Cytomegalovirus
C. Coxsackievirus A
D. Hepatitis C virus
E. JC virus
F. Parvovirus B19

Explanation:
This patient presented with a mononucleosis-like illness after a recent blood
transfusion. Because his Monospot test was negative, Epstein-Barr virus (EBV) is an
unlikely cause of this patient’s symptoms. Research indicates that 75- 90% of EBV
mononucleosis cases are associated with serum heterophil antibodies (which are
detected with the Monospot test). Cytomegaly virus (CMV), human herpesvirus 6
(HHV-6), and toxoplasmosis are all known to cause mononucleosis-like syndromes
characterized by negative Monospot tests. In this case then CMV is the most likely
causative factor. A heterophil antibody-negative mononucleosis syndrome is the
most frequent clinical manifestation of CMV infection in the immunocompetent. Such
an infection can be acquired during the transfusion of leukocyte laden blood
products, as CMV infects leukocytes of granulocyte-macrophage lineage.
(Choice C) Coxsackie A viruses cause aseptic meningitis and herpangina in children.
(Choice D) Hepatitis C may cause post-transfusion chronic hepatitis, although the
increasing sensitivity of blood donor screening tests has reduced the transmission
rate to less than per 500000 units transfused. While atypical lymphocytosis is a
possibility with hepatitis C, this is not the most likely diagnosis.
(Choice E) JC virus is the agent responsible for progressive multifocal
leukoencephalopathy (PML) a “slow” viral infection of the CNS resulting in
demyelination. Clinical manifestations include hemiparesis, visual field defects (eg,
hemianopsia), and cognitive impairment.
(Choice F) Parvovirus B19 is responsible for the childhood illness erythema
infectiosum, which is characterized by red, flushed cheeks (the “slapped cheeks”
appearance). It also can cause aplastic crises in those with sickle cell anemia and
can cause hydrops fetalis in the fetus.

Educational Objective:
CMV is the most common cause of heterophil antibody-negative (ie, Monospot-
negative) mononucleosis. The Monospot test is positive in up to 90% of cases with
EBV-associated mononucleosis.

208
USMLE WORLD STEP 1 MICROBIOLOGY

Q NO 59: A 29-year-old female presents to clinic with malaise and an extremely


painful rash in her genital region. She has never had such symptoms before. A.
Tzanck smear of the genital lesion is positive for multinucleated giant cells. A
drug that can cause rapid resolution of this patient’s symptoms would most
likely:
Prevent viral attachment to target cells
B. Inhibit viral protease activity
C. Incorporate into newly replicated viral DNA
D. Prevent virion release from infected cells
E. Up-regulate interferon synthesis in infected cells

Explanation:
This previously asymptomatic patient’s new genital rash and positive Tzanck smear
are consistent with a primary genital herpes simplex infection, likely due to HSV-2.
Antiviral drugs currently recommended for the treatment of primary genital herpes
include acyclovir, valacyclovir, and famciclovir. Acyclovir treatment during a primary
episode usually reduces the duration of viral shedding time for lesional healing,
constitutional symptoms, and local pain.
In infected host cells, acyclovir (a nucleoside analog) is converted into acyclovir
monophosphate principally via virus- encoded thymidine kinase. Cellular enzymes
then convert the monophosphate into acyclovir triphosphate. When viral DNA
polymerase incorporates acyclovir triphosphate into the viral DNA chain, viral DNA
synthesis is terminated.

209
USMLE WORLD STEP 1 MICROBIOLOGY

Acyclovir does not significantly impact uninfected cells in vitro because uptake into
these cells is poor, phosphorylation in the absence of viral thymidine kinase is
minimal, and cellular a-DNA polymerase has significantly less affinity for acyclovir
triphosphate than does viral DNA polymerase.
(Choice A) Prevention of viral attachment to target cells is the mechanism of action
for drugs such as enfuvirtide, an anti retroviral used in HIV treatment.
(Choice B) Inhibition of viral protease activity is the mechanism of action for drugs
such as indinavir, an antiretroviral used in HIV treatment.
(Choice D) Prevention of virion release from infected cells is the mechanism of action
for drugs such as neuraminidase inhibitors, which is used in the early treatment of
influenza.
(Choice E) Up regulation of interferon synthesis in infected cells is accomplished by
many interleukins.

Educational Objective:
Antiviral drugs currently recommended for the treatment of primary genital herpes
include the nucleoside analogs (eg, acyclovir). These nucleoside analogs are
incorporated into newly replicated viral DNA and ultimately terminate viral DNA
chain synthesis.

210
USMLE WORLD STEP 1 MICROBIOLOGY

Q NO 60: A 22-year-old male is found to have serum anti-HAV lgG antibodies


although he denies any vaccination against hepatitis. The laboratory finding A.
most likely indicates: An
anicteric viral infection in the past
B. An icteric illness in the childhood
C. Chronic viral hepatitis
D. Asymptomatic viral carrier state
E. Reinfection with hepatitis A virus
F. Immunity against hepatitis 0 infection

Explanation:
Hepatitis A virus infection is most often silent or subclinical (“anicteric” with no
jaundice observed) in children but can also present as an acute, self-limited illness
characterized by jaundice malaise, fatigue, anorexia, nausea, vomiting. right upper
quadrant pain, or an aversion to smoking. Clinical disease is typically less common
but more severe in adults. As living conditions improve in many countries, however,
a diminishing number of adults have antibodies to HAV and therefore remain
susceptible to infection. Itis currently estimated that4O-7O% of adults within the
United States have anti-HAV lgG antibodies, with most positive individuals having
never experienced an icteric illness.
(Choice B) Anicteric (rather than icteric) hepatitis A is especially prevalent among
children.
(Choices C and D) Hepatitis A infection is a self-limiting disease and does not
progress to chronic hepatitis cirrhosis, or hepatocellular carcinoma. It is also not
associated with an asymptomatic viral carrier state.
(Choice E) Reinfection with the hepatitis A virus would be associated with the
presence of anti-HAV 1gM antibodies not anti-HAV lgG antibodies. Moreover
individuals who have persistent anti-HAV lgG antibodies typically enjoy immunity to
repeat hepatitis A infection.
(Choice F) immunities against hepatitis D infection is signaled by anti-HDV lgG
antibodies not anti-HAV lgG antibodies.

Educational Objective:
Hepatitis A virus infection is most commonly silent or subclinical (“anicteric”) in
children but can also present as an acute, self-limited illness characterized by
jaundice malaise fatigue anorexia nausea vomiting right upper quadrant pain or an
aversion to smoking.

211
USMLE WORLD STEP 1 MICROBIOLOGY

Q NO 61: A 35-year-old man presents to your office complaining of a hard mass


under his jaw. He reports that the mass developed three months ago, shortly
after he had a tooth extracted. The mass has since grown in size, and has A.
recently begun to drain yellowish pus through the overlying skin. Which of the
following is the most likely cause of this patient’s condition?
Borrelia burgdorferi
B. Pasteurella multocida
C. Sporothrix schenckii
D. Actinomyces Israelii
E. Bacillus anthracis
F. Rickettsia rickettsiae

Explanation:
This patient’s history of a slowly growing mass that began in the setting of oral
trauma and has recently been draining yellow pus through the skin makes
actinomycosis the most likely diagnosis. Actinomyces are Gram positive components
of the normal oral flora capable of causing cervicofacial actinomycosis, a condition
characterized by the formation of chronic face and neck abscesses complicated by
cutaneous sinus tracts. Treatment consists of a prolonged course of parenteral
penicillin and surgical debridement.
Actinomyces have a growth pattern similar to that of the mycelial form of fungi
hence their fungus-like name. A notable feature of this organism is its ability to form
‘sulfur granules, which are yellow aggregations of organisms bound together by
proteins. (Sulfur granules do not actually contain sulfur.)
(Choice A) Borrelia burgdorferi is the etiologic agent of Lyme disease. Lyme disease
does not commonly cause abscesses.
(Choice B) Pasteurella multocida is a Gram negative rod commonly found as part of
the normal oral flora of cats and dogs. P. multocida typically causes a rapidly
progressive soft tissue infection following an animal (usually cat) bite. Local soft
tissue infection can lead to draining cutaneous sinus tracts, lymphadenopathy,
osteomyelitis and septic joints. This infection is acute, not chronic as described in
the question stem.
(Choice C) Cutaneous sporotrichosis begins as an ulcerating papule at the site of
inoculation and spreads proximally along the lymphatics causing additional lesions as
it spreads.
(Choice E) Cutaneous anthrax occurs after B. anthracis spores inoculated into the
subcutaneous tissue germinate into vegetative cells. Release of the toxins lethal
factor and edema factor leads to local edema and formation of a papule at the site of
inoculation. This papule eventually evolves into a painless necrotic wound covered
with black eschar. Treatment is with ciprofloxacin.
(Choice F) The cutaneous lesions classically associated with Rocks Mountain Spotted
Fever are palmoplantar erythematous macules that migrate centripetally toward the
trunk.

Educational Objective:
Actinomyces Israelii is a Gram positive organism that is best known for causing
cervicofacial actinomycosis in patients following dental manipulation or other oral
trauma. The disease is characterized by a slowly growing and firm feeling abscess in
the face or neck region that eventually forms cutaneous sinus tracts. Long-term
penicillin treatment and surgical debridement are required.

212
USMLE WORLD STEP 1 MICROBIOLOGY

Q NO 62: Volunteer studies are used to determine the infective dose of Salmonella
required to cause gastroenteritis. The curve obtained during the studies is shown
on the diagram and is labeled as ‘2’. Which of the following organisms could be
represented by line ‘1’ on this diagram?

A. Shigella flexneri
B. Vibrio cholerae
C. Vibrio parahaemolyticus
D. Clostridium perfringens
E. Enterotoxigenic E. coil

Explanation:
Shigella is highly adapted to surviving the acidity in the stomach, as well as the
bacteriostatic action of bile. In fact, depending on the age and condition of host, as
few as 10 cells of Shigella can cause disease. Even healthy adults will contract
shigellosis with an inoculum as small as 200 organisms via the oral route. The
superior infectivity of Shigella may also be due to its unique binding site on intestinal
mucosal M cells. These sites are usually unoccupied by the normal flora of the gut,
which allows easier attachment and invasion. The incubation period for shigellosis is
24to 72 hours. Disease begins with watery diarrhea which progresses to abdominal
pain, cramps, diarrhea with blood, mucous and pus fever vomiting, and tenesmus.
Tenesmus is a painful spasm of the rectum that is associated with an urge to
defecate, yet little passage of stool occurs. A much larger inoculum of Salmonella
(approximately 10^7) is required for successful infection in a susceptible host.
(Choice B) Approximately 10^6 toxigenic Vibrio (01 and 0139 serotypes) are needed
for successful infection in the susceptible host.
(Choice C) Vibrio parahaemolyticus: >10^5 to 107^7 organisms are needed for
successful infection in the susceptible host.
(Choice D) Clostridium perfringens: approximately 500 organisms are needed for
successful infection in the susceptible host.
(Choice E) Enterotoxigenic F. coli approximately 10^8 to 10^10 organisms are
needed for successful infection in the susceptible host.

Educational Objective:
Depending on the age and condition of host, as few as 10 cells of any Shigella
species can cause infection, although the infectious dose is usually much higher.
Other organisms that can cause diarrhea with only a small inoculum include
Campylobacter jejuni (500), Entamoeba histolytica (as few as one organism), and
Giardia Iamblia (as few as one organism).

213
USMLE WORLD STEP 1 MICROBIOLOGY

Q NO 63: A cell line is infected with virus obtained from a 45-year-old male.
Analysis of the progeny viruses demonstrates that the phospholipid
composition of the viral particle surface is highly similar to that of the cell A.
nuclear membrane. The patient is most likely infected with:
Rhinovirus
B. Hepatitis C virus
C. Cytomegalovirus
D. Mumps virus
E. Adenovirus
F. Human papilloma virus

Explanation:
A virion with a phospholipid-containing particle surface is likely an enveloped virus.
Most enveloped nucleocapsid viruses acquire lipid bilayer envelopes by budding
through the plasma membrane of the host cell. However the herpesviruses (which
include cytomegalovirus) bud through and acquire the lipid bilayer envelope from
the host cell nuclear membrane.
(Choice A) The rhinovirus is a non-enveloped picornavirus and therefore would not
have surface phospholipids.
(Choice B) Hepatitis C virus is a flavivirus that obtains its envelope by budding
through the plasma membrane of the host cell.
(Choice D) Mumps virus is a paramyxovirus that obtains its envelope by budding
through the plasma membrane of the host cell.
(Choice E) The adenovirus is non-enveloped and therefore would not have surface
phospholipids.
(Choice F) The human papilloma virus is a non-enveloped papovavirus and therefore
would not have surface phospholipids.

Educational Objective:
Most enveloped nucleocapsid viruses acquire their lipid bilayer envelope by budding
through the plasma membrane of the host cell. Exceptions include the
herpesviruses, which bud through and acquire their envelope from the host cell
nuclear membrane.

214
USMLE WORLD STEP 1 MICROBIOLOGY

Q NO 64: A 22-year-old female presents with severe pain and swelling in her right
knee, left elbow, and left wrist. Joint aspiration of the knee reveals an opaque
exudate with high neutrophil content and intracellular organisms. This patient’s
symptoms could have most likely been prevented by:
A. Methotrexate therapy
B. Proper sore throat treatment
C. Timely vaccination
D. Barrier contraception
E. Avoiding intravenous drug abuse

Explanation:
This patient most likely has septic arthritis caused by Neisseria gonorrhoeae.
Arthritis is a complication of disseminated N. gonorrhoeae infection and is the most
common cause of septic arthritis in sexually active young adults; therefore, barrier
contraception would help prevent this disease. In the initial stage, patients
experience polyarthralgia that primarily involve the knees, elbows, and more distal
joints. Asymmetric polyarthritis of large joints with fever in a sexually active young
adult is characteristic of this disease.
Besides septic arthritis, other manifestations of gonococcal infection are urethritis in
men and cervicitis, pelvic inflammatory disease (PID), infertility, and ectopic
pregnancy in women. Additionally, it is important to remember that the Neisseria are
facultative intracellular organisms and are often seen within neutrophils.
(Choice A) Disease modifying anti-rheumatic drug (DMARD) therapy is used in the
treatment of rheumatoid arthritis, an autoimmune syndrome characterized by
bilateral symmetrical arthritis with morning stiffness caused by immune- mediated
inflammation and destruction of the joint synovium.
(Choice B) The primary bacterial pathogen for pharyngitis is Group A, beta-hemolytic
Streptococcus (S. pyogenes). Untreated GAS pharyngitis can lead to the
development of rheumatic fever which, among other symptoms, causes a migratory
polyarthritis. This is unlikely in this patient as there is no history of pharyngitis and
the incidence of rheumatic fever has decreased considerably due to the development
of and widespread use of antibiotics.
(Choice C) There is no vaccine available for prevention of Neisseria gonorrhoeae.
(Choice E) IV drug use can lead to S. aureus bacteremia with subsequent septic
arthritis at any age. S. aureus is the most common cause of septic arthritis in
children and adults over age 50. This infection requires prompt attention from an
orthopedist for drainage. In this case the question stem mentions intracellular
organisms. This is a characteristic of gonococcal infections, not S. aureus.

Educational Objective:
Acute bacterial arthritis in sexually active young adults is most commonly caused by
N. gonorrhoeae. S. aureus septic arthritis is most common in children and non-
sexually active adults.

215
USMLE WORLD STEP 1 MICROBIOLOGY

Q NO 65: A 35-year-old male hospitalized with e4ensive trauma following a tractor


accident complains of severe pain in his injured right leg. The leg swells rapidly,
and radiographs reveal gas in the tissues. The microorganism most likely
responsible for this patient’s condition is also commonly associated with:

A. Persistent cough
B. Transient watery diarrhea
C. Polyarthritis
D. Urinary frequency and burning
E. Relentless headaches and nausea

Explanation:
About 95% of cases of gas gangrene are due to Clostridium perfringens. The spores
of this Gram positive bacillus are abundant in soil, and patients suffering penetrating
injuries can be inoculated with spores. Upon entering the host, the C. perfringens
spores germinate in the anaerobic environment into vegetative toxin-producing cells.
The toxin is a phospholipase that attacks cell membranes and causes cell death.
Extensive tissue damage, necrosis, and reduction of blood supply to the affected
area result, and the disease continues to spread in the enlarging anaerobic
environment. The organism rapidly metabolizes carbohydrate, producing a
significant amount of gas that can be visualized on plain film radiographs. Treatment
involves emergent debridement and intravenous antibiotics, but even with prompt
therapy the prognosis of clostridial myonecrosis, or gas gangrene, is poor and tissue
loss is often considerable.
C. perfringens can also cause a late-onset food poisoning characterized by a
transient watery diarrhea. This gastroenteritis is caused by toxin formed when large
quantities of clostridial spores are ingested. The spores germinate in the digestive
tract and then begin to elaborate toxin, hence the diseases delayed onset. (This is in
contrast to the early-onset food poisoning caused by the preformed toxins of S.
aureus and B. cereus.)
(Choice A) C. perfringens does not cause cough.
(Choice C) Polyarthritis can be caused by gonorrhea, the most common cause of
septic arthritis in sexually active adults. It can also result from rheumatic fever, a
syndrome that can occur following untreated Group A Streptococcal infection. C.
perfringens does not cause arthritis.
(Choice D) Urinary frequency and burning are caused by urethritis cystitis. The
urethritis of urinary tract infections is usually due to E. coli, whereas sexually
transmitted types of urethritis can be due to gonorrhea, Chlamydia, and Ureaplasma
urealyticum. C. perfringens does not cause dysuria.
(Choice E) Relentless headaches and nausea can be caused by any organism that
causes meningitis. Specifically, subacute or chronic infections causing these illnesses
include M. tuberculosis, Acanthamoeba, Cryptococcus neoformans.

Educational Objective:
C. perfringens causes late-onset food poisoning and clostridial myonecrosis (gas
gangrene). The food poisoning is toxin-mediated, late-onset, and clinically causes a
transient watery diarrhea.

216
USMLE WORLD STEP 1 MICROBIOLOGY

Q NO 66: HCV strains isolated from a single individual demonstrate significant


variability. This genetic instability of HCV is attributed in large part to its RNA- A.
dependent RNA-polymerase, which lacks:
Nucleotide specificity
B. 3’ — 5’ exonuclease activity
C. 5’ — 3’ exonuclease activity
D. Ligase activity
E. Glycosylase activity
F. Primase activity

Explanation:
Hepatitis C virus has six or more genotypes and multiple sub genotypes, as
demonstrated by the genetic differences in the encoding of its two envelope
glycoproteins. This genetic variation has led to the development of a hypervariable
region of the envelope glycoprotein that is especially prone to frequent mutation.
Moreover there is no proofreading 3’ — 5’ exonuclease activity built into the virion-
encoded RNA polymerase. As a result, the IRNA polymerase makes many errors
during replication, and several dozen subspecies of hepatitis C virus are typically
present in the blood of an infected individual at any one time.
(Choice A) Nucleotide specificity refers to the importance of inserting the correct
nucleotide (eg, adenine, guanine, cytosine, thymine, uracil) into a strand of DNA or
RNA. It is not the cause of instability in the hepatitis C virus.
(Choice C) 5’ — 3’ exonuclease activity is important in allowing enzymes such as
DNA polymerase Ito replace nucleotides in the repair of a growing strand of DNA.
This enzyme is not the cause of instability in the hepatitis C virus.
(Choice D) DNA ligase is used to join together two fragments of DNA and is
therefore useful in both DNA repair and replication. This enzyme is not the cause of
instability in the hepatitis C virus.
(Choice E) DNA glycosylase is important in base excision repair, as it removes the
nitrogen base from the sugar- phosphate backbone. An endonuclease then
completes the remainder of the repair. This enzyme is not the cause of instability in
the hepatitis C virus.
(Choice F) DNA primase is a form of RNA polymerase. In bacteria, it binds with DNA
helicase and synthesizes a short RNA primer to which nucleotides can be added by
DNA polymerase. This enzyme is not the cause of instability in the hepatitis C virus.

Educational Objective:
The hepatitis C virus is genetically unstable because it lacks proofreading 3’ — 5’
exonuclease activity in its IRNA polymerase and its envelope glycoprotein contains a
hypervariable region prone to frequent genetic mutation.

217
USMLE WORLD STEP 1 MICROBIOLOGY

Q NO 67: Non-pathogenic strains of Corynebacterium diphtheriae can acquire


pathogenicity and thus the ability to cause severe pseudomembranous A.
pharyngitis through which of the following mechanisms?
Transformation permitting capsule formation
B. Bacterial conjugation permitting pili expression
C. Phage conversion permitting exotoxin production
D. Chromosomal mutation permitting endotoxin production
F. Environmental influences permitting toxin synthesis

Explanation:
Diphtheria is an acute toxin-mediated disease, but not all strains of 0. diphtheriae
express the disease-causing exotoxin. C. diphtheriae acquires virulence via
bacteriophage-mediated “infection” with the Tox gene (choice C), which codes for
the diphtheria AB exotoxin. The bacteriophage responsible is called Corynephage
beta. The phage Tox gene incorporates into the bacterial chromosome as a prophage
and codes for toxin production by C. diphtheriae. This process, whereby a
bacteriophage infects a host bacterium and integrates its genome into the host
bacterium’s genome, is termed lysogenization.
(Choice A) Streptococcus pneumoniae bacteria acquire the ability to produce
capsules via the process of transformation. The capsule is the major virulence factor
for S. pneumoniae: strains without the capsule are not pathogenic.
(Choice B) Conjugation is the direct transfer of plasmids from one bacterium to
another through a sex pilus. E. coil bacteria acquire the ability to form pili via
bacterial conjugation. Genes for antibiotic resistance are often transmitted in this
manner.
(Choice D) Chromosomal mutations can theoretically lead to endotoxin production,
though more commonly, mutations lead to bacterial antibiotic resistance.
(Choice E) The Gram positive spore-forming rods Bacillus and Clostridium
demonstrate toxin synthesis under environmental influences. The A. anthracis spore,
the causative agent in anthrax survives in soil for extended periods of time in a state
of dormancy. Only upon warming to 37°C in the presence of appropriate
concentrations of CO2 and protein do the spores germinate and begin to synthesize
toxin. The case is similar for C. tetani spores. Which only germinate and produce
toxin in anaerobic environments, such as necrotic wounds.

Educational Objective:
Non-pathogenic Corynebacterium can cause severe pseudomembranous pharyngitis
after acquiring the Tox gene via lysogenization by a temperate bacteriophage.

218
USMLE WORLD STEP 1 MICROBIOLOGY

Q NO 68: A 38-year-old Caucasian male undergoing treatment for acute


myelogenous leukemia (AML) complains of severe right-sided headaches.
Physical examination reveals right-sided proptosis and periorbital
tenderness. Biopsy of the right maxillary sinus mucosa reveals the
following:
Which of the following is the most likely cause of this patient’s condition?

A. Malassezia furfur
B. Microsporum can/s
C. Rhizopus species
D. Aspergillus fumigatus
E. Candida albicans
F. Cryptococcus neoformans
G. Blastomyces dermatitidis
H. Histoplasma capsulatum
I. Coccidioides immitis
J. Sporothrix schenckii

Explanation:
Mucor, Rhizopus and Absidia species are saprophytic fungi present in the
environment. They are transmitted by spore inhalation and cause mucormycosis.
Patients with underlying immunosuppression (e.g. solid organ transplantation
patients, patients with hematologic malignancies, patients undergoing corticosteroid
treatment) are at high risk. Mucormycosis is also very strongly associated with
diabetic ketoacidosis.
Mucormycosis tends to affect the paranasal sinuses. Patients complain of facial and
periorbital pain, headache and purulent nasal discharge. The fungi proliferate in the
walls of blood vessels and cause necrosis of the corresponding tissue. Black eschar
(necrotic tissue) may be seen on the palate or nasal turbinates.
Mucormycosis is diagnosed by light microscopy of a tissue specimen. Mucor,
Rhizopus and Absidia fungi exist in mold form only. They form broad nonseptate
hyphae that branch at wide (often 90° angles). The typical histologic appearance is
shown on the slide above. Mucormycosis is treated by surgical debridement of
necrotic tissue and amphotericin B.
(Choice D) Invasive aspergillosis involving the paranasal sinuses can show close
clinical resemblance to mucormycosis. Light microscopy of the affected tissue will
differentiate these two diagnoses. Unlike Mucor and Rhizopus, Aspergillus has
septate hyphae that branch at 45° angles (V-shaped branching).
(Choice E) Candida albicans can cause a variety of symptoms in immunosuppressed
patients. Right-sided endocarditis, renal abscesses, pneumonia and esophagitis are
among the most common. Budding yeasts and pseudohyphae are seen on light
microscopy.
(Choices G, H and I) Blastomyces, 1-Iistoplasma and Coccidioides cause lung
disease in healthy patients. Disseminated disease occurs in immunosuppressed
individuals. Isolated paranasal involvement is not typical.

219
USMLE WORLD STEP 1 MICROBIOLOGY

(Choice A and B) Malassezia furfur and Microsporum canis cause cutaneous mycoses
and do not affect the paranasal sinuses.
Educational Objective:
Mucormycosis is an opportunistic infection caused by Rhizopus, Mucor and Absidia
species. The classic clinical picture is paranasal sinus involvement in a diabetic or
immunosuppressed patient. The fungi form broad nonseptate hyphae that branch at
right angles.

220
USMLE WORLD STEP 1 MICROBIOLOGY

Q NO 69: A 4-year-old immigrant boy is brought to the pediatric emergency room


with a swollen right knee accompanied by fever and malaise. He is hypotensive
and tachycardic. His past medical history is nothing significant except for one
episode of otitis media. Some of his vaccinations are not up-to-date. Synovial
fluid and blood cultures grew pleomorphic gram negative rods on hematin
containing medium. The pathogenesis of the organism responsible for his
condition is most likely related to which of the following?
A. Endotoxin
B. Fimbriae
C. Capsule
D. Cytotoxic exotoxin
E. Hemolysins
F. Hyaluronidase

Explanation:
H. influenzae is a gram-negative rod that can be either encapsulated or
unencapsulated. Herophilus influenzae is a blood loving organism and requires both
X factor (exogenous hematin) and V factor (NAD+) to support growth. There are six
capsular serotypes classified as types a-f. Most infections are caused by H.
influenzae strains belonging to capsular serotype b. Type b is the only H. influenzae
capsular serotype that contains a ribose rather than hexose, as the carbohydrate
component of the capsule, and this feature may be related to the virulence of the
organism. The type b capsular material is a linear polymer consisting of ribose,
ribitol and phosphate which is known as poly ribosyl-ribitol-phosphate (PRP). The
PRP capsule prevents phagocytosis and intracellular killing by neutrophils and is
essential for virulence of this organism (Choice C). Antibodies to serotype bare
shown to promote opsonization, complement fixation. and phagocytosis and killing
of H. influenzae type b.
Unencapsulated (nontypable) H. influenzae are part of the normal flora of the upper
respiratory tract. They rarely cause disease, and when they do it is only local
infection such as sinusitis, otitis media or bronchitis; without a capsule they do not
cause invasive disease. (Choice A) As gram-negative organisms, H. influenzae do
possess LPS endotoxin in their cell membranes, but endotoxin is not the major
virulence factor in H. influenzae. Endotoxin release by gram-negative bacteria during
bacteremia leads to sepsis and this is most pronounced with N. meningitidis,
Salmonella and E. co/i. (Choice B) Fimbriae are not present in H. influenzae. These
proteinaceous projections from bacterial cells mediate attachment to target tissues
during the process of establishing infection. This method of attachment is used by
Neisseria gonorrhoeae, N. meningitidis and E. coli.
(Choice D) There are no strains of H. influenzae known to produce an exotoxin of
any kind. (Choice E) Hemolysins are not present in H. influenzae.
(Choice F) The enzyme hyaluronidase is not present in H. influenzae. Hyaluronidase
is used by bacteria to digest ground substance and enhance their ability to spread.
Itis produced by Staphylococci, Group A Streptococci and C. difficile most notably.
Educational Objective:
The pathogenicity of H. influenzae is dependent on the presence of the
antiphagocytic polysaccharide capsule. The type b strain is the most invasive and
virulent; it has a capsule with a ribose as the sugar rather than a hexose as is used
in the other strains of encapsulated H. influenzae and this may be a reason for the
increased virulence of Hib compared to the others. Unencapsulated (nontypable) H.
influenzae are part of the normal flora and cause only local infections.

221
USMLE WORLD STEP 1 MICROBIOLOGY

Q NO 70: Foreign double-stranded DNA fragments in the neuron bodies of a 22-


year-old female’s sacral sensory ganglia are strongly associated with which of A.
the following conditions?
Chronic demyelinating disease
B. Recurring painful genital rash
C. Invasive cervical carcinoma
D. Tabes dorsalis
E. High-grade non-Hodgkin lymphoma

Explanation:
This young woman appears to have a viral infection of the primary afferent neuron
cell bodies in her sacral dorsal root ganglia. Viruses known to invade the dorsal root
sensory ganglia include the herpes simplex viruses (HSV-1 and HSV-2) and the
varicella zoster virus (WV). Among these HSV-2 produces a recurrent, painful genital
rash (“herpes genitalis”) when the latent virus is reactivated in the sacral sensory
ganglia. HSV- also has the potential to cause a recurrent genital rash, but more
commonly infects the sensory ganglia-innervating dermatomes above the waist (eg,
trigeminal ganglia).
The natural history of VZV infection of the dorsal root ganglia involves reactivation
that results in a painful, vesicular skin eruption along the sensory dermatomes (a
phenomenon called herpes zoster, or “shingles”). Shingles arises most commonly in
the thoracic or trigeminal dermatomes.
(Choice A) The double-stranded DNA viruses known to be neurotropic include the
herpesviruses and the JC papovavirus. The JC virus can cause chronic CNS
demyelination through infection of oligodendrocytes (ie, progressive multifocal
leukoencephalopathy). JC virus is not known to significantly invade the neurons in
sensory gangli a.
(Choice C) Some human papillomaviruses (HPV) are double-stranded DNA viruses
that contribute to the development of cervical carcinoma. Because human
papillomaviruses are epitheliotropic, they almost exclusively infect the skin and
mucous membranes.
(Choice D) “Tabes dorsalis” is a term used to describe the degeneration of dorsal
spinal columns and nerve roots in patients with tertiary syphilis.
(Choice E) Double-stranded DNA viruses that are oncogenic and associated with
high-grade non-Hodgkin lymphomas include Epstein-Barr virus (EBV) and
herpesvirus Wpe-8 (HHV-8). However, EBV and HHV-8 generally do not infect the
sensory ganglia. HHV-8 is associated with Kaposi sarcoma.

Educational Objective:
Infection of the sacral sensory ganglia with a double-stranded DNA virus is likely to
eventually result in a recurrent, painful genital rash (genital herpes) secondary to
reactivation of the latent herpes simplexvirus (HSV-2).

222
USMLE WORLD STEP 1 MICROBIOLOGY

Q NO 71: An outbreak of water-born gastroenteritis with a high mortality is


reported in some Asian countries. Itis caused by oxidase-positive, gram-
negative rods that grow well on highly alkaline media. Which of the A.
following patients would most likely require the smallest infective dose to
initiate the infection?
Patients previously exposed to the bacteria
B. Patients with chronic pancreatitis
C. Patients on omeprazole treatment
D. Patients with chronic hepatitis
E. Patients with peptic ulcer disease

Explanation:
Vibrio cholerae is only known to infect humans and does not cause invasive disease
of any kind. The organism must be swallowed in either food or water and survive
passage through the acidic pH of the stomach to colonize the small intestine in order
to cause disease. V cholerae is extremely acid sensitive, and under normal gastric
acid conditions an individual must ingest 1010 or more V. cholerae to become
infected from ingesting water. If V. cholerae is contained in food then as few as io6
V cholerae are needed because of the buffering capacity of food. Achlorhydria is a
condition where there in inadequate gastric acid production to maintain the normal
gastric pH of less than 4 even with maximal hormonal stimulation. It can be
pharmacologically induced with long-term proton pump inhibitor therapy or can be a
result of gastritis. In patients with achlorhydria there is insufficient acid to kill the
organism so very few V. cholerae organisms are needed to cause disease.
(Choice A) Patients previously exposed to the bacteria would not be very susceptible
if their immune response is intact and they produce secretory lgA antibody to V.
cholerae. Vaccines exist for V. cholerae, but they are infrequently used in the United
States because the disease is not endemic here and travelers can avoid infection by
ensuring proper water treatment with boiling.
(Choices B and D) Patients with chronic pancreatitis or chronic hepatitis are not
more susceptible because these organs are not involved in the disease process.
(Choice E) Patients with peptic ulcer disease are not more susceptible because there
is not a change in the acidity of the stomach or small intestine.

Educational Objective:
The minimal infectious dose for cholera infection is usually quite high with W10
organisms required to cause infection after ingestion of contaminated water. V
cholerae is very sensitive to gastric acidity and any condition that increases gastric
pH will lower the minimum infective dose of V cholerae by many orders of
magnitude. Gastric pH can be increased by achlorhydria, food ingestion and antacid
ingestion.

223
USMLE WORLD STEP 1 MICROBIOLOGY

Q NO 72: A 5-month-old Hispanic boy is brought to the ER with complaints of poor


feeding weakness and complete loss of extremity muscle tone. All of his
vaccinations are up to date and there is no significant past medical history. He
receives formula as his sole source of nutrition with the exception of occasional
fruit juice and honey. He has also received vitamin D supplementation. Which of
the following tests is most likely to establish the diagnosis in this patient?

A. Stool for bacterial toxins


B. Blood for liver enzymes
C. Blood for viral titers
D. Urine for glucose and ketones
E. Urine for amino acids

Explanation:
This 5-month-old baby is consuming honey, a food notorious for contamination with
C. botulinum spores. In studies, more than 12% of tested honey samples contained
C. botulinum spores. Infant botulism results when a baby consumes C. botulinum
spores, which then germinate in the infant Gl tract. Intracellular toxin production
bacteriolysis resulting in toxin release, and mild systemic absorption of toxin ensue.
In infant botulism, constipation usually precedes the characteristic signs of
neuromuscular paralysis by a few days or weeks. Other symptoms include mild
weakness, lethargy, and reduced feeding. Some infants, however, show more severe
symptoms such as weakened suck, swallowing, and crying: generalized muscle
weakness: and diminished gag reflex. In severe cases, the generalized muscle
weakness and loss of head control can cause the infant to appear ‘floppy.” (In
contrast, adult botulism, which results from ingestion of preformed toxin, is almost
always very severe.)
Infant botulism can usually be diagnosed based on the patient’s clinical presentation
and food consumption history. Culture and isolation of the organism and bioassay of
toxins are time-consuming procedures, but rapid in vitro procedures have been
developed for the detection of types A, B, E, and F botulinum toxin-producing
organisms and their toxins. The tests are based on ELISA methodology and
polymerase chain reaction techniques.
(Choice B) Measurement of blood liver enzyme levels is a nonspecific test that can
indicate infectious liver disease such as that caused by viral hepatitis. Liver enzyme
levels are typically in the thousands in acute viral hepatitis and in the hundreds in
chronic disease.
(Choice C) Serum viral titers are frequently used to monitor the effectiveness of
HAART therapy in patients with HIV. Serologies are used more commonly in the
evaluation of viral hepatitis and in patients with suspected Epstein- Barr and
cytomegalovirus infections.
(Choice D) High levels of urine glucose and ketones can indicate diabetic
ketoacidosis.
(Choice E) Patients with amino acids in their urine may have a disorder of renal
amino acid resorption, such as Fanconi syndrome or Hartnup disease.

Educational Objective:
Infant botulism is frequently due to honey consumption. More than l2% of honey
samples contains low numbers of C. botulinum spores. Whereas infant botulism
results from consuming C. botulinum spores, adult botulism results from consuming
preformed toxin, typically in canned food. Symptoms of infant botulism include
constipation, mild weakness, lethargy, and poor feeding.

224
USMLE WORLD STEP 1 MICROBIOLOGY

Q NO 73: A small outbreak of self-limited infection is reported in a summer camp.


Several children develop low-grade fever, throat pain, red eyes and cervical
lymphadenopathy. Which of the following is the most likely cause of the outbreak?
A. Calicivirus
B. Paramyxovirus
C. Parvovirus
D. Adenovirus
E. Arenavirus
F. Poxvirus

Explanation:
The outbreak described here is typical of the pharyngoconjunctival fever caused by
adenoviruses. Specifically this condition is characterized by acute, self-limited febrile
pharyngitis, cough, nasal congestion, conjunctivitis, and enlarged cervical nodes.
This condition accounts for only 2-4% of the acute viral diseases of the upper
respiratory tract in civilian populations. However pharynx conjunctival fever is
epidemiologically much more common in small groups of individuals who are living in
crowded quarters (eg, barracks) under conditions of fatigue or stress. Thus
adenovirus outbreaks are seen more often among military recruits or campers.
(Choice A) Caliciviruses of medical importance include hepatitis E virus (HEV) and
Norwalk virus. HEV produces hepatitis while Norwalk virus is responsible for
epidemic outbreaks of viral gastroenteritis.
(Choice B) Paramyxoviruses of medical importance include the parainfluenza virus
responsible for croup (laryngotracheobronchitis) in children the respiratory syncytial
virus (RSV) responsible for bronchiolitis in infants the measles virus and the mumps
virus.
(Choice C)The parvovirus of medical importance is B19. It causes aplastic crises in
patients with sickle cell anemia erythema infectiosum (fifth disease) in children and
hydrops fetalis in the fetus.
(Choice E) The arenavirus of medical importance is lymphocytic choriomeningitis
virus (LCV). LCV predominantly causes a febrile aseptic meningoencephalitis or a
mild systemic influenza-like illness in humans exposed to infected hamsters or mice.
Person-to-person transmission has not been demonstrated.
(Choice F) Poxviruses of medical importance include smallpox vaccinia, cowpox
monkeypox, and the molluscum contagiosum virus. Smallpox (variola) has been
largely eradicated through vaccination. More common in children than adults
molluscum contagiosum predominantly affects the skin and mucous membranes.
This virus produces flesh-colored pruritic papules with an umbilicated center that
typically contains white, curd like material. These lesions are often found in
anogenital areas or on the trunk.

Educational Objective:
Adenoviruses are the cause of a pharyngoconjunctival fever that classically occurs in
small outbreaks among individuals living together in close quarters (such as military
barracks or camp dormitories).

225
USMLE WORLD STEP 1 MICROBIOLOGY

Q NO 74: A 65-year-old male taking amoxicillin for sinusitis is hospitalized with


diarrhea and abdominal cramps. Complete blood count shows leukocytosis. A.
This toxin responsible for his current condition primarily damages which of the
following components of intestinal mucosal cells?
Mitochondrial energy production
B. Cytoskeleton integrin
C. Apical ion transport
D. Ribosomal protein synthesis
E. Cell membrane integrity

Explanation:
C. difficile colonizes the colonic mucosa and releases toxins that cause mucosal
inflammation, cell damage and ultimately cell death with mucosal necrosis. This
organism is present in 2-3% of healthy adults and in about 70% of healthy infants
as a part of the gut’s normal microbial flora. Disease results when the patient uses
antibiotics that kill a large portion of the commensal organisms in the gut (which
normally keep potentially pathogenic organisms such as C. difficile in check).
Pathogenic strains of C. difficile produce 2 distinct toxins: toxin A (enterotoxin) and
toxin B (cytotoxin). Both play a role in the pathogenesis of C. difficile colitis. The
toxins bind specific receptors on intestinal mucosal cells and are internalized where
they exert their intracellular effects. Toxin A acts as a neutrophil chemoattractant
leading to mucosal inflammation, loss of water into the gut lumen (producing
diarrhea), and mucosal death. Toxin B causes actin depolymerization, loss of cellular
cytoskeleton integrity, cell death and mucosal necrosis.
(Choice A) Mitochondrial energy production is the primary source of ATP for human
cells. Nucleoside reverse transcriptase inhibitors and cyanide are examples of drugs
with associated mitochondrial toxicity.
(Choice C) Apical ion transport is affected by the cholera toxin, the main exotoxin of
Vibrio cholerae. The A subunits of this AB exotoxin activate adenylate cyclase
through a G-protein mechanism leading to increased transport of sodium and
chloride out of the gut mucosal cell and decreased sodium and chloride reabsorption.
Remember that free water follows sodium, so this toxin also causes free water loss
into the gut lumen leading to watery diarrhea.
(Choice D) Ribosomal protein synthesis is inhibited by the shiga and the shiga-like
toxins. Shiga toxin is the main exotoxin released by Shigella species while shiga-like
toxin is produced by
Enterohemorrhagic E. coli and Enteroinvasive E. coil. These organisms are not part
of the normal flora of the gut, and transmission is by the fecal oral route. They are
not associated with antibiotic use.
(Choice E) Cell membrane integrity loss is a characteristic of the alpha toxin
lecithinase, one of the many exotoxins released by Clostridium perfringens. C.
perfringens can cause a transient watery diarrhea, but it is much more frequently
associated with clostridial myonecrosis, or gas gangrene, a rapidly progressive form
of fasciitis associated with penetrating injury by soil-contaminated objects.

Educational Objective:
C. duff/die Toxin A attracts neutrophils causing mucosal inflammation, loss of water
into the gut lumen and diarrhea, as well as mucosal death. Toxin B, the cytotoxin,
causes actin depolymerization, loss of cellular cytoskeleton integrity, cell death and
mucosal necrosis.

226
USMLE WORLD STEP 1 MICROBIOLOGY

Q NO 75: A pharyngeal exudate sample from a 20-year-old female is placed on an


enriched medium containing vancomycin, colistin and trimethoprim. The media
would favor growth of which of the following bacteria?
A. Escherichia coil
B. Streptococcus pyogenes
C. Streptococcus pneumoniae
D. Klebsiella pneumoniae
E. Neisseria gonorrhoeae
F. Corynebacterium diphtheriae

Explanation:
Pharyngeal exudates are expected to contain large numbers of both Gram-positive
and Gram-negative microorganisms. To isolate the fastidious Neisseria gonorrhoeae
requires an enriched medium that will inhibit the growth of other bacteria normally
present in the oral cavity. A selective chocolate agar-based medium containing
various antibiotics, the Thayer-Martin medium, is commonly used to isolate Neisseria
from clinical specimens. The commonly used antibiotics are vancomycin to inhibit
Gram-positive bacteria colistin (polymyxin) to inhibit Gram- negative bacteria
including the commensal Neisseria species but not N. meningitidis and N.
gonorrhoeae, and nystatin to inhibit yeast.
(Choice A) Escherichia coil is a Gram-negative rod and would be inhibited by colistin
(polymyxin) and trimethoprim. It is not an important cause of pharyngitis. It is the
most common cause of Gram-negative sepsis.
(Choice B) S. pyogenes, or Group A Streptococci, are an important pharyngeal
pathogen as they can cause a pharyngitis which, if left untreated, can lead to
rheumatic fever. Streptococcus pyogenes would be inhibited by vancomycin.
(Choice C) Streptococcus pneumoniae is a Gram-positive coccus and would be
inhibited by vancomycin. Itis not an important cause of pharyngitis, but it is the
most common cause of meningitis and communi-acquired pneumonia in adults.
(Choice D) Klebsiella pneumoniae is a Gram-negative rod and would be inhibited by
colistin (polymyxin) and trimethoprim. It is not associated with pharyngitis, but is
does classically cause severe pneumonia in alcoholic or otherwise severely disabled
patients as well as nosocomial sepsis and urinary tract infections. Itis part of the or
pharyngeal normal flora and characteristically causes pulmonary infection following
aspiration.
(Choice F) Corynebacterium diphtheriae is a historically important cause of
pharyngitis in the US prior to the introduction of the DPT (Diphtheria-Pertussis-
Tetanus) vaccine. Children present clinically with fever and sore throat with an
evident pseudomembrane. It is a Gram-positive organism and would be inhibited by
vancomycin.

Educational Objective:
The Thayer-Martin selective medium is used to isolate Neisseria species from clinical
cultures. It is a chocolate (heated blood) agar that contains vancomycin to inhibit
the growth of Gram-positive organisms, colistin (polymyxin) and trimethoprim to
inhibit the growth of Gram-negative organisms other than Neisseria, and nystatin to
inhibit the growth of fungi.

227
USMLE WORLD STEP 1 MICROBIOLOGY

Q NO 76: A 55-year-old male is being treated with combination chemotherapy for


acute myeloid leukemia. His last complete blood count shows a white blood cell
count of 800/cmm and a platelet count of 89000!cmm. He is now admitted with
fever, chills and shortness of breath. His pulse is 120/min and BP is 90/40 mm
Hg. Physical examination shows the following skin findings.

Based on the clinical picture, which of the following bacteria is most likely
responsible for this patient’s symptoms?

A. Escherichia coli
B. Kelsiella pneumoniae
C. Streptococcus pyogenes
D. Streptococcus pneumoniae
E. Haemophilus influenzae
F. Pseudomonas aeruginosa

Explanation:
The question stem describes an immunosuppressed (neutropenic) male who is
experiencing symptoms consistent with bacterial sepsis. In cases of impaired
humoral immunity, there is increased susceptibility to infections with P. aeruginosa
as well as other pathogens. The lesions illustrated in the slide are consistent with
ecthyma gangrenosum, a skin finding that is strongly associated with bacteremia by
P. aeruginosa. These lesions result from perivascular bacterial invasion of arteries
and veins in the dermis and subcutaneous tissue with subsequent release of
exotoxins that are destructive to human tissue. Enzymes produced by P. aeruginosa
such as Exotoxin A (protein synthesis inhibition) Elastase (degrades elastin —
important for blood vessel destruction) Phospholipase C (degrades cellular
membranes) and pyocyanin (generates reactive oxygen species) are recognized as
important virulence factors and play a role in causing the vascular destruction and
cutaneous necrosis known as ecthyma gangrenosum.
(Choice A) Escherichia coil is not associated with this type of necrotic skin lesion. E.
coli is the most common cause of gram-negative sepsis, usually following UTI.
(Choice B) Klebsiella is most commonly associated with necrotizing pneumonia in
elderly or immunocompromised patients.

228
USMLE WORLD STEP 1 MICROBIOLOGY

(Choice C) Streptococcus pyogenes (Group A Streptococcus) frequently causes


cutaneous infections such as impetigo, erysipelas and abscess. Impetigo classically
occurs as a superinfection of a preexisting skin lesion such as an eczematous or
minor traumatic lesion. It forms a “golden crust” most characteristically. Erysipelas
is a unique form of skin infection that causes lesions involving the superficial layers
of the skin resulting in a very painful erythematous plaque with raised and well
demarcated borders.
(Choice D) Streptococcus pneumoniae can be isolated from blood, CSFI or
respiratory sources as it can cause disease in those locations. It is not associated
with ecthyma gangrenosum.
(Choice E) Haemophilus influenzae can be isolated from blood, CSFI or respiratory
sources. It is not associated with ecthyma gangrenosum.

Educational Objective:
Ecthyma gangrenosum is a cutaneous necrotic disease with a strong association with
Pseudomonas aeruginosa bacteremia end septicemia. It occurs after P. aeruginosa
invades perivascular and releases tissue destructive exotoxins causing vascular
destruction and resultant insufficiency of blood flow to patches of skin which become
edematous and subsequently necrose. Pseudomonas infections are common in
neutropenic patients, hospitalized patients, patients with burns and chronic
indwelling catheters.

229
USMLE WORLD STEP 1 MICROBIOLOGY

Q NO 77: Gram-positive cocci isolated from the blood of a patient with bacteremia
synthesize dextrans from glucose. The bacteria most likely contribute to which of
the following pathological states?
A. Glomerulonephritis
B. Sarcoidosis
C. Erythema nodosum
D. Migratory polyarthritis
E. Anterior uveitis
F. Dental caries

Explanation:
Viridans streptococci notably S. mutans and S. sanguis, are normally present in the
human mouth and are major contributors to tooth decay and the initiation of dental
caries. The organisms also cause bacterial endocarditis deep wound infections,
abdominal abscesses, and septicemia. Viridans streptococci are able to adhere to the
surface of tooth enamel and heart valves and multiply in those locations due to their
ability to produce insoluble extracellular polysaccharides (dextrans) using sucrose as
a substrate. The production of extracellular polysaccharides is thought to contribute
to their ability to cause infection by aiding colonization of host surfaces.
(Choice A) Poststreptococcal glomerulonephritis is a Type lll (immune complex
mediated) hypersensitivity reaction where immune complexes deposit on the
glomerular membrane and fix complement leading to inflammation, renal
dysfunction hematuria and nephritic range proteinuria.
(Choice B) Sarcoidosis is a poorly understood disease, and no infectious agent has
been linked to sarcoidosis in anyway at present.
(Choice C) Erythema nodosum is a skin condition characterized by erythematous to
violaceous subcutaneous nodules usually appearing on the legs. EN is an extremely
nonspecific finding on physical exam. It is associated with multiple bacterial
infections most commonly Streptococcus pyogenes pharyngitis, but also
Staphylococcus aureus, fungi (coccidioidomycosis, histoplasmosis and
blastomycosis), chlamydia and others. Additionally EN is associated with Crohn’s
disease sarcoidosis and many other inflammatory conditions.
(Choice D) Migratory polyarthritis is a characteristic of rheumatic fever (RE).
Rheumatic fever is an immune- mediated disease that can follow untreated Group A
Streptococcal (S. pyogenes) pharyngitis.
(Choice E) Anterior uveitis is an inflammation of the iris and can be caused by
infectious processes such as Herpes viruses syphilis and Lyme disease as well as
inflammatory conditions such as the HLA-B27-related diseases and sarcoid. It is not
associated with viridans streptococcal infections.

Educational Objective:
Viridans streptococci produce dextrans from glucose that aid these organisms in
colonizing host surfaces such as dental enamel and heart valves. These organisms
cause subacute bacterial endocarditis most classically in patients with preexisting
cardiac valvular defects after dental manipulation. This is why antibiotic prophylaxis
is used prior to dental work in patients with valvular abnormalities.

230
USMLE WORLD STEP 1 MICROBIOLOGY

Q NO 78: A 34-year-old Caucasian nurse with recent onset of malaise is found to


have the following set of laboratory findings:
Anti-HAV IgM Positive
Anti-HAV lgG Negative
HBsAg Negative
HBeAg Negative
Anti-HBsAg Positive
Anti-HBcAg Negative
Anti-HBeAg Negative
Anti-HCV Negative
This patient most likely:

A. Had oysters at a local restaurant


B. Consumed boiled eggs
C. Had a recent tattoo
D. Had an accidental needlestick
E. Had an unprotected sexual intercourse

Explanation:
The pathogen responsible for hepatitis A is an RNA picornavirus with an average
incubation period of 30 days. Transmission occurs through the fecal-oral route and is
common in areas with overcrowding and poor sanitation. Outbreaks frequently result
from contaminated water or food with raw or steamed shellfish the typical culprit in
the United States. Onset is acute, and symptoms can include malaise, fatigue,
anorexia, nausea, vomiting, mild abdominal pain, and an aversion to smoking.
Hepatomegaly is commonly seen. AST and ALT spike early in the illness, followed by
increases in bilirubin and alkaline phosphatase. Anti-HAV IgM is positive on the
hepatitis panel. Fortunately, hepatitis A infection is a self-limiting disease and does
not progress to chronic hepatitis, cirrhosis, or hepatocellular carcinoma. The
mortality rate is less than 0.2%, although a significantly prolonged prothrombin time
correlates with increased mortality.
Treatment of hepatitis A infection is largely supportive, with complete recovers/
expected in 3-6 weeks. Close contacts of individuals with hepatitis A should promptly
be given immune globulin. People considered at high risk (eg, those living in or
traveling to endemic areas, those with chronic liver disease or clotting-factor
disorders, men who have sex with men) should be given the hepatitis A vaccine as
prophylaxis.
(Choice B) The consumption of boiled eggs is not commonly associated with hepatitis
A infection. Were the boiled eggs insufficiently cooked, Salmonella infection would be
a risk.
(Choices C, D and E) Transmission of hepatitis A virus is not associated with
unsanitary tattooing (eg, with contaminated needles), accidental needlesticks, or
unprotected sexual intercourse. This virus is only transiently present in blood and is
not shed in saliva, urine, or semen.

Educational Objective:
Transmission of the hepatitis A virus occurs through the fecal-oral route and is
common in areas with overcrowding and poor sanitation. Outbreaks frequently result
from contaminated water or food, with raw or steamed shellfish a common culprit in
the United States.

231
USMLE WORLD STEP 1 MICROBIOLOGY

Q NO 79: Neurotransmitter release from certain spinal cord inhibitory interneurons


inhibits polysynaptic spread of impulses and insures proper functioning of agonist
and antagonist skeletal muscles. Absence of agonist/antagonist muscle
coordination and sustained impulse spread leading to spasm can result from
inadequate release of:

A. GABA due to C. difficile toxin


B. Acetylcholine due to C. botulinum toxin
C. Glycine due to C. tetani toxin
D. Serotonin due to P. aeruginosa toxin
E. Norepinephrine due to S. aureus toxin

Explanation:
Clostridium tetrad produces disease by the production of a potent protein exotoxin,
not by bacterial invasion of tissue. Even small amounts of tetanus toxin can be
deadly. At first, the toxin binds to receptors on the presynaptic membranes of the
motor neurons. From there the toxin migrates by the retrograde axonal transport
system to the cell bodies of these neurons and next to the spinal cord and brain
stem. Release of the inhibitory neurotransmitters glycine and gamma-aminobutyric
acid (GABA) from these inhibitory neurons is blocked. The suppression of inhibitory
nerve function results in an increased activation of nerves innervating muscles,
causing muscle spasms, spastic paralysis and hyperreflexia. The muscle spasms
involve both flexor and extensor muscles. Patients with tetanus have spastic muscle
contractions, difficulty opening the jaw (known as lockjaw or ”trismus”), a
characteristic smile called “risus sardonicus” and contractions of back muscles,
resulting in backward arching known as opisthotonos. Patients are extremely
irritable, and develop tetanic seizures, brought about by violent, painful muscle
contractions following minor stimuli such as a noise.
(Choice A) GABA is not involved with diseases caused by C. difficile such as
pseudomembranous colitis.
(Choice B) Acetylcholine release is inhibited by the botulinum toxin released by C.
botulinum, but this has the effect of causing the flaccid paralysis seen in clinical
botulism.
(Choice D) Serotonin release is not affected in any major way directly by infections.
(Choice E) Norepinephrine could not be responsible for the changes mentioned in the
question stem.

Educational Objective:
Tetanospasmin is the neurotoxin released by C. tetani It causes an inhibition of the
inhibitory interneurons in the spinal cord that regulate firing of primary motor
neurons. These inhibitory interneurons use GABA and glycine as their
neurotransmitters, but tetanospasmin prevents the release of these
neurotransmitters.

232
USMLE WORLD STEP 1 MICROBIOLOGY

Q NO 80: A 20-year-old college student is hospitalized with recent onset insomnia,


headaches, periodic agitation and dysphagia. Four weeks ago, he went on a field
trip to explore caves. His past medical history is insignificant. He does not
consume alcohol and has never used illicit drugs. Despite hospitalization and
treatment, the patient becomes comatose and dies. Which of the following
interventions would have most likely prevented this student’s illness?

A. Live attenuated vaccine


B. Viral component vaccine
C. Killed vaccine
D. Toxoid vaccine
E. Viral enzyme inhibitor drug
F. Cellular receptor inhibitor drug
G. Interferons

Explanation:
This clinical presentation of restlessness, agitation, and dysphagia progressing to
coma 30 to 50 days following an exposure to cave bats is strongly suggestive of
rabies encephalitis. Common manifestations of rabies include a nonspecific, flu-like
prodrome (malaise, anorexia, mild fever, headache, nausea, vomiting) and a
subsequent acute neurologic syndrome that includes agitation, persistent fever
variable consciousness, and painful spasms with swallowing or inspiration. The latter
is responsible for the dysphagia in rabies encephalitis. Generalized flaccid paralysis
and coma follow the acute neurologic phase, with most patients dying within two
weeks of becoming comatose.
Once deposited in a wound, the rabies virus travels retrograde via peripheral nerves
to the dorsal root ganglia and then to the brain. Massive replication occurs within the
central nervous system and the rabies virus spreads to other organs through neural
pathways; it is thought that at this point, postexposure prophylaxis is no longer
effective.
The CDC currently recommends prophylactic vaccination for individuals at high risk
of exposure to the rabies virus (eg, veterinarians, animal handlers, cave explorers,
laboratory workers handling infected tissues, and individuals who stay for more than
30 days in developing countries where rabid dogs are prevalent). The FDA-approved
rabies vaccine consists of various rhabdovirus strains grown in tissue cell culture and
inactivated (killed) by betapropiolactone.
(Choice A) No live attenuated rabies vaccine is currently recommended by the CDC.
In general, using a live vaccine to prevent a potentially fatal disease is a risky
endeavor.
(Choice B) The FDA-approved rabies vaccine consists of various rhabdovirus strains
grown in tissue cell culture and inactivated (killed) by beta-propiolactone. The
vaccine contains entire virus particles, rather than viral components.
(Choice D) Toxoid may be used to vaccinate against the injurious effects of a
microbial toxin, and is prepared by treatment of the toxin with formalin or heat. The
toxoid can then induce humoral immunity against the toxin without producing
serious toxic effects in the vaccinee. However, the pathogenesis of rabies infection
does not appear predominantly mediated by a viral toxin.
(Choices E, F and G) Like most viruses, rabies obtains access to host cells via
receptor-mediated endoq4osis. The rabies virus contains a single-stranded,
negatively charged RNA genome with a viral encoded RNA dependent RNA
polymerase. Unfortunately, there are no currently approved drugs available for the
treatment of rabies.

233
USMLE WORLD STEP 1 MICROBIOLOGY

Educational Objective:
The clinical presentation of restlessness, agitation, and dysphagia progressing to
coma 30 to 50 days following an exposure to cave bats is strongly suggestive of
rabies encephalitis. Prophylactic vaccination is recommended by the CDC for
individuals at high risk of exposure to rabid animals or their tissues. The FDA
approved rabies vaccine consists of various rhabdovirus strains grown in tissue cell
culture and then inactivated to produce killed virus vaccine.

234
USMLE WORLD STEP 1 MICROBIOLOGY

Q NO 81: A 34-year-old male presents to your office with a painless penile ulcer
several weeks after an episode of unprotected sexual intercourse. Which of the A.
following is most useful in diagnosing Treponema pallidum infection in this
patient?
Ulcer exudate microscopy after Gram staining
B. Ulcer exudate microscopy with darkfield illumination
C. Ulcer exudate cultures on enrichment media
D. Blood cultures on differential media
E. Antitoxin detection in the serum

Explanation:
Treponema pallidum is the causative agent of syphilis. Like other spirochetes T
pallidum is spiral-shaped and has a Gram-negative cell wall. Because the organism
is so thin, however, it cannot be visualized with standard Gram stain and
microscopy. Classically, darkfield microscopy of material scraped from the surface of
the cutaneous syphilitic lesion must be employed to visualize T. pallidum. In positive
cases, T. pallidum appears as a motile helical organism on darkfield examination.
Because spirochetal motility must be observed to differentiate T pallidum from other
spirochetes the microscopy should be performed immediately after specimen
collection (which is difficult for most clinical laboratories to arrange). As an
alternative, immunofluorescence can be used to demonstrate the presence of T
pallidum on microscopy. More commonly, the diagnosis of syphilis is confirmed with
serologic testing. The serologic tests can be divided into two groups: nontreponemal
tests and treponemal tests. Nontreponemal tests (eg, VDRLI RPR) evaluate for the
presence of cardiolipin, a byproduct of treponemal infection. The nontreponemal
tests are best used for screening and have a sensitivity of 10-99%. Because
nontreponemal tests are affected by antitreponemal therapy, they can be used to
follow disease progression and therapeutic response. The treponemal tests (eg, ETA-
ABS, MHA-TP) detect specific treponemal antigens and are typically used for
confirmation of a positive nontreponemal test or when clinical suspicion remains high
despite a negative nontreponemal test. The treponemal tests are not affected by
antitreponemal therapy and will remain positive for life.
(Choice A) Gram stain and microscopy cannot be used to visualize spirochetes
because they are too thin (despite being categorized as Gram-negative bacteria).
(Choice C) Cultures of syphilitic ulcer exudate on an enrichment medium would not
grow T pallidum because this organism is notoriously difficult to culture. Even when
incubated on a cellular medium, only a few generations of organisms are yielded if
any. However, an enriched medium is useful in the culture of Haemophilus ducreyi.
This microorganism is responsible for “chancroid” a similar primary lesion in the
genitalia characterized by a shallow painful ulceration on the genitals in conjunction
with swollen regional lymph nodes. H. ducreyi requires hematin (X factor) for growth
in culture much like its better known relative H. influenzae.
(Choice D) Blood cultures incubated on differential media are similar to exudate
cultures incubated on enrichment media in that they would not yield T pallidum
organisms. Of greater value are the nontreponemal and treponemal serologic tests
for syphilis that can be performed on blood.
(Choice E) Antitoxin detection in the serum is not used to test for T. pallidum
infection because this spirochete does not synthesize or release any toxins.
Educational Objective: The visualization of spirochetes (such as T. pallidum, the
causative organism in syphilis) can be accomplished with darkfield microscopy,
which reveals helical motile organisms.

235
USMLE WORLD STEP 1 MICROBIOLOGY

Q NO 82: A 34-year-old Caucasian male with advanced HIV infection complains of


headaches and fever. India ink staining of his cerebrospinal fluid reveals spherical
yeast forms with thick capsules. Which of the following is the most likely primary
focus of this patient’s infection?
A. Nasopharynx
B. Lungs
C. Intestine
D. Skin
E. Urinary tract

Explanation:
Cryptococcus neoformans is a fungus strongly associated with CNS infection in
immunocompromised individuals. It most commonly affects HIV (+) patients,
although those with hematologic malignancies and patients on high-dose steroid
therapy are also susceptible.
C. neoformans exists in the yeast form (single cells) only. It has a thick
polysaccharide capsule with antiphagocytic properties. This fungus is present in soil
and bird (especially pigeon) droppings. It infects humans via the respiratory tract
and enters the lungs. Lung infection by Cryptococcus is usually asymptomatic; from
the lungs the infection may disseminate to other organs.
C. neoformans has a predilection for the CNS. Cerebrospinal fluid does not contain
the complement components of the alternative pathway. Thus, there is impaired
phagocytosis of Cryptococcus in the CSF. The symptoms of cryptococcal
meningoencephalitis are non-specific. Patients have headache, fever, stiff neck,
altered mental status, nausea and vomiting. The presence of HIV infection or
another immune-suppressing illness is central to the diagnosis. C. neoformans is
seen on India ink-stained CSF as round yeast with peripheral clearing. (This light
peripheral zone corresponds to the polysaccharide capsule that does not stain with
India ink.)

Educational Objective:
Cryptococcus neoformans is present in soil and pigeon droppings. The yeast is
transmitted via the respiratory route, with the lungs as the primary site of entry. In
immunocompromised patients, lung infection may be followed by dissemination of C.
neoformans, often into the CNS.

236
USMLE WORLD STEP 1 MICROBIOLOGY

Q NO 83: Bacteria isolated from the lung tissue of a 32-year-old Caucasian male fail
to decolorize with hydrochloric acid and alcohol after staining carbolfuchsin. Which
of the following cell wall components is most likely responsible for this staining
phenomenon?

A. N-acetylmuramic acid
B. Teichoic acid
C. Lipopolysaccharide
D. Mycolic acid
E. Ergosterol

Explanation:
The procedure described above is the acid-fast stain, which is used in the detection
of a select few pathogenic organisms (eg, Mycobacteria and some Nocard/a species).
Although it is less sensitive than culture, the acid-fast stained smear allows for
immediate microscopic evaluation.
In the acid-fast stain for mycobacteria, the smear is first treated with an aniline dye
(eg, carbolfuchsin). The dye (red color) penetrates the bacterial cell wall, where it
binds with mycolic acids. The slide is then treated with hydrochloric acid and alcohol.
This acid alcohol dissolves the outer cell membranes of nontuberculous bacteria but
the presence of mycolic acids prevents decolorization of mycobacteria. A
counterstain (eg, methylene blue) is then applied and taken up by decolorized
bacteria. As a result, the carbolfuchsin acid-fast stain produces red mycobacteria
(initial stain) and blue non-acid fast bacteria.
The cell membrane and cell wall of mycobacteria are most similar to those in Gram-
positive organisms, causing mycobacteria to appear weakly positive on Gram stain.
However, the mycobacterial cell wall differs from that of the typical Gram-positive
organism in that they are encapsulated with mycolic acid, a waxy, long-chain fatty
acid that is covalently bound to the sugars within the cell wall. Another organism
that will also stain positive with the acid-fast technique is Nocardia. Nocardia is a
Gram-positive rod that contains mycolic acid in its cell wall. Because Nocardia
possesses less mycolic acid than do mycobacteria, Nocardia is more weakly acid
fast.
(Choice A) N-acetylmuramic acid and N-Acel glucosamine are the saccharides that
combine with an amino acid chain to form the peptidoglycan layer in both Gram-
positive and Gram-negative cell walls.
(Choice B) Teichoic acid is a molecule linked to the peptidoglycan cell wall of Gram-
positive bacteria (but not Gram- negative bacteria). Teichoic acid serves as an
antigenic determinant for organism identification in the laboratory and an antigenic
target for the human immune system.
(Choice C) Lipopolysaccharide (LPS) is a component of the outer cell envelope of
Gram-negative bacteria.
(Choice E) Unique to fungi ergosterol is the sterol component of fungal cell
membranes. This molecule is not found in human cell membranes, as human cells
have cholesterol in their cell membranes instead.

Educational Objective:
The acid-fact stain identifies organisms that have mycolic acid present in their cell
walls, including Mycobacterium and some Nocardia species. Acid-fast staining is
carried out by applying an aniline dye (eg, carbolfuchsin) to a smear and then
decolorizing with acid alcohol to reveal whether the organisms present are “acid
fast.”

237
USMLE WORLD STEP 1 MICROBIOLOGY

Q NO 84: A group of physicians in a developing country is advocating the early


empiric use of penicillin in the treatment of clinically diagnosed bacterial A.
pharyngitis, a condition that is quite common in children of that nation. What
would you expect to decrease as a result of their policy in the long term?
Expensive antibiotic requirement
B. Nephritic disease
C. Deaths caused by diarrhea
D. Bronchial asthma hospitalizations
E. Cardiac surgeries
F. Vaccination needs

Explanation:
Early antibiotic treatment of streptococcal pharyngitis can prevent many cases of
rheumatic fever (RE). RE and RE- induced valvular heart disease is still a major
problem in some developing nations. Due to poor living conditions, crowding and
little access to health care, these patients acquire heart disease more rapidly and in
more severe forms. The mechanism of rheumatic valvular damage is repeated
stimulus of E-lymphocytes to mount an antibody response against antigens on
Group A Streptococci (GAS) that are homologous to self antigens in the heart and
CNS. Therefore, recurrent untreated GAS pharyngitis will lead to a more rapid onset
and increased severity of rheumatic valvular disease.
RE involves three organ systems: joints, heart and brain. Rheumatic (migratory)
polyarthritis causes joint pain and inflammation that moves from one joint to
another and typically affects children ages 5-15. The inflammation of the heart
(rheumatic carditis) involves all layers of the heart from the pericardium to the
endocardium and can cause chest pain in the case of pericarditis. A characteristic
murmur of valvular incompetence usually involves the mitral valve and/or the aortic
valve. The mitral valve dilates and chordae elongate causing the valve leaflets to
incompletely close resulting in mitral insufficiency or regurgitation requiring
excessive treatment and frequently surgery (Choice E).
Eradication of infective streptococci from the pharynx prevents subsequent episodes
of RE. The drug of choice is penicillin. No penicillin resistant strains of S. pyogenes
have been detected as yet.
(Choice A) Penicillin treatment is not expensive. Prolonged rather than high-dose
penicillin treatment is required for the effective eradication of streptococci from the
pharynx. Frequently, this is not possible due to poor patient compliance.
(Choice B) Acute Post-Streptococcal Glomerulonephritis (APSGN) is one of the most
common nephritic syndromes and can follow pharyngitis or skin infections caused
only by “nephritogenic” strains of S. pyogenes (Group A Strep). This is an immune
complex deposition disease (Type Ill hypersensitivity) and will occur regardless of
treatment of the offending Streptococcal infection.
(Choice C) Deaths caused by diarrheal dehydration are most common in
underdeveloped countries. A viral etiology is the most common cause of diarrhea in
children, so widespread antibiotic use would not address this problem. Antibiotic use
would likely predispose to increased diarrhea from causes such as Clostridium
difficile colitis as well as antibiotic effect on the gut flora leading to loose stools.
(Choice D) Antibiotics do not play a role in the treatment of asthma unless a
bronchitis or pneumonia of a bacterial etiology is suspected.
(Choice F) There is no vaccine to prevent Streptococcus pyogenes infection since it is
yew difficult to separate antigenic determinants that elicit type-specific protective
immunity from closely related moieties that share anti genic determinants with
human heart tissue.

238
USMLE WORLD STEP 1 MICROBIOLOGY

Educational Objective:
Rheumatic fever (RE) can be reduced, as it has been in industrialized nations, by
prompt treatment of streptococcal pharyngitis and eradication of infective
streptococci with prolonged penicillin treatment. RE follows untreated Group A
Streptococcal (GAS) pharyngitis while acute post-streptococcal glomerulonephritis
can follow either pharyngitis or skin infection by GAS regardless of treatment.

239
USMLE WORLD STEP 1 MICROBIOLOGY

Q NO 85: A 65-year-old male presents to the ER with a sore on his right hand. He is
involved in the wool business and has recently returned from a tour of wool
processing plants in a series of Asian countries. Physical examination reveals an
ulcer with black eschar and significant surrounding edema. The toxin that caused
edema around the ulcer in this patient has an action most similar to that of a toxin
produced by:

A. Streptococcus pyogenes
B. Bordetella pertussis
C. Shigella dysenteriae
D. Clostridium botulinum
E. Clostridium difficile

Explanation:
There are two main virulence factors of B. anthracis: 1. the anthrax exotoxin
composed of edema factor, lethal factor, and protective antigen that are required for
either of the aforementioned toxins to function and 2. the antiphagocytic poly-
gamma-D-glutamic acid capsule. Edema factor is a calmodulin dependent adenylate
cyclase that causes an increase in the cAMP concentration inside host cells. Edema
factor is though to play an important role in suppression of neutrophil function and
accumulation of fluid within and between cells leading to edema.
Bordetella pertussis produces an edema factor-like exotoxin simply called
“extracellular adenylate cyclase.” The Bordetella pertussis adenylate cyclase toxin is
very important for successful colonization of the organism in the respiratory tract.
The extra cellular adenylate cyclase secreted by B. pertussis is able to enter
eukaryotic cells and cause a dramatic increase in cAMP levels leading to neutrophil
dysfunction just as edema factor in B. anthracis does.
(Choice A) Streptococcus pyogenes does not produce edema factor exotoxin ora
similar toxin, but it does cause disease by the production of toxins such as pyrogenic
toxin and toxic shock syndrome toxin (TSST). Pyrogenic toxin is only present in
strains of S. pyogenes that have been lysogeniz with a bacteriophage and it is the
causative agent in streptococcal scarlet fever a disease characterized by fever and a
fine erythematous rash. TSST causes toxic shock by widespread and nonspecific
activation of T- lymphocytes by a superantigen mechanism.
(Choice C) Shigella dysenteriae causes disease through the production of shiga-
toxin. Shiga toxin causes death of intestinal mucosal epithelial cells by entering the
cells and disabling the 60s ribosomal subunit thereby halting protein production.
(Choice D) Clostridium botulinum causes disease through the production of a potent
polypeptide neurotoxin called botulinum toxin. Botulinum toxin prevents the fusion
of presynaptic acetylcholine-containing vesicles with the neuronal membrane at the
neuromuscular junction.
(Choice E) Clostridium difficile causes disease through the production of several
toxins in the intestinal lumen. The best characterized are toxins A (enterotoxin) and
B (cytotoxin). Toxin A causes diarrhea by acting as a chemoattractant for
neutrophils which release cytokines leading to mucosal inflammation and GI fluid
loss. Toxin B depolymerizes actin filaments leading to mucosal cell death and bowel
wall necrosis with subsequent formation of a pseudomembrane.
Educational Objective: Bacillus anthracis edema factor is an adenylate cyclase that
causes massive increases in intracellular cAMP leading to neutrophil dysfunction and
collection of fluid within and between cells with resultant edema. This mechanism of
action is similar to that of a toxin produced by Bordetella pertussis simply called
“extracellular adenylate cyclase.”

240
USMLE WORLD STEP 1 MICROBIOLOGY

Q NO 86: E. coli strains isolated from a 4-year-old Caucasian female with bloody
diarrhea produce a substance that inhibits protein synthesis in human cells. The
substance shares many properties with the toxin produced by:
A. Shigella dysenteriae
B. Salmonella typhi
C. Pseudomonas aeruginosa
D. Proteus mirabilis
E. Vibrio cholerae
F. Clostridium difficile

Explanation:
Shiga-like toxins (Stx) or Vero cytotoxins (VT) are produced by Enterohemorrhagic
E. coli (EHEC). Shiga-like toxins (Stx) or Vero cytotoxins (VT) are synonymous; the
name Vero cytotoxin results from this toxin’s ability to kill Vero cells in cell cultures.
Shiga-like toxin I (Stx-l) or Vero cytotoxin (VT1) is the substance mentioned in the
question stem that is able to inhibit protein synthesis in human cells, and it is closely
related to the Shiga toxin produced by Shigella dysenteriae. Shiga-like toxin is an AB
toxin that consists of an active A subunit and five binding B subunits. The plasmid
coding for this toxin is transmitted to E. coli by a temperate bacteriophage. The B
subunits form pentamers with a central pore. The B subunits recognize the specific
receptors on the target cells and induce receptor-mediated endocytosis and
internalization of the toxin. Subsequently, the enzymatically active A subunit is
released. The A subunit then catalyses the removal of a single specific adenine
residue there by preventing binding of tRNA to the 60S ribosomal subunit and
inhibiting protein synthesis. This leads to intestinal mucosal cell death as well as
direct toxicity to renal endothelial cells.
(Choice B) Salmonella typhi causes typhoid, or enteric, fever. S. typhi is a facultative
intracellular parasite that enters the human digestive tract orally, penetrates the
intestinal mucosa, and travels to the mesenteric lymph nodes where it multiplies and
is phagocytosed by macrophages within which it can survive. Typhoid fever is
characterized by fever, abdominal pain, diarrhea and faintly erythematous macules
on the abdomen called “rose spots.”
(Choice C) Pseudomonas aeruginosa produces exotoxin A, an exotoxin that is very
similar in function to diphtheria toxin in that it halts human cellular protein synthesis
by inhibiting elongation factor-2. This results in arrest of translation as the growing
protein chain is unable to change positions in the ribosome to allow binding of a new
tRNA.
(Choice D) Proteus mirabilis does not produce toxins.
(Choice E) Vibrio cholerae is the causative agent of cholera and produces the
enterotoxin choleragen. Choleragen acts similarly to the heat labile toxin of ETEC in
that it causes increased intracellular cAMP in intestinal mucosal cells leading to
decreased absorption and increased secretion of sodium, chloride and water.
(Choice F) Clostridium difficile produces toxin A (enterotoxin) and toxin B (cytotoxin)
and causes pseudomembranous colitis most commonly in patients recently treated
with antibiotics for other reasons. Toxin B depolymerizes actin filaments causing
gastrointestinal mucosal cell death. Neither of these toxins resemble Shiga like
toxin.

241
USMLE WORLD STEP 1 MICROBIOLOGY

Educational Objective:
Shiga-like toxins (a.k.a. Vero cytotoxins) are produced by Enterohemorrhagic E. coil
(EHEC). As their name implies, these toxins are nearly identical to the Shiga toxin
produced by Shigella dysenteriae. These toxins function to inhibit the 60s ribosomal
subunit in human cells thereby blocking protein synthesis by preventing binding of
tRNA. This mechanism differs from that of diphtheria toxin and exotoxin A of
Pseudomonas in that the latter toxins act on EF-2, not the 60S ribosomal subunit.

242
USMLE WORLD STEP 1 MICROBIOLOGY

Q NO 87: A 23-year-old pregnant woman is found to have chronic viral hepatitis B


on liver biopsy. The presence of which of the following serologic markers is most
likely to increase the risk of vertical transmission of the virus?
A. HBcAg
B. HBsAg
C. HBeAg
D. Anti- HBcAg IgG
E. Anti- HBeAg IgM
F. Anti-HBsAg IgG

Explanation:
Vertical transmission of hepatitis B from pregnant females to the unborn child can
occur with active hepatitis B infection. Typically, such transmission takes place
during the passage of the fetus through the birth canal, but transplacental infection
can also occur. This is especially common in those women who developed acute
hepatitis B infection in the third trimester.
The presence of HBeAg (a soluble protein that is a marker of viral replication and
increased infectivity) in the mother significantly increases the risk of vertical
transmission of the virus. Were this woman HBeAg negative, her neonate’s risk of
infection would be 20%. If she were HBeAg positive, however, her neonate’s risk of
infection would be 95%. Moreover should the infant become infected, his chance of
progression to chronic hepatitis is 90%. Because of this concern, the newborns of all
mothers with active hepatitis B are passively immunized at birth with hepatitis B
immune globulin (HBIG)I followed by active immunization with recombinant HBV
vaccine. (Choice A) HBcAg is not detectable in serum as it is normally sequestrated
within the HBsAg coat.
(Choice B) HBsAg is the first virological marker detected in the serum after
inoculation, preceding both the elevation of serum aminotransferases and the onset
of clinical symptoms. It remains detectable during the entire symptomatic phase of
acute hepatitis B and suggests infectivity. It is not the most ominous marker in the
pregnant woman, however.
(Choice D) Anti-HBc appears in the serum shortly after the emergence of HBsAg and
remains detectable long after the patient recovers. The 1gM fraction signals the
acute phase of disease, whereas the lgG fraction signals recovery from the disease.
Because IgM anti-HBc is present in the “window period,” it is an important tool for
diagnosis when HBsAg has been cleared and anti-HBs is not yet detectable. Thus
1gM anti-HBc is the most specific marker for diagnosis of acute hepatitis B.
(Choice E) Anti-HBe suggests the cessation of active viral replication and low
infectivity.
(Choice F) Anti-HBsAg IgS appears in the serum after either successful HBV
vaccination or the clearance of HBsAg and remains detectable for life. It serves as an
indicator of non infectivity and immunity. However, there is a time lag between the
disappearance of HBsAg and the appearance of anti-HBs in the serum, which is
termed the “window period.”
Educational Objective: Vertical transmission of hepatitis B from pregnant females to
the unborn child can occur in women with active hepatitis B infection. The presence
of HBeAg (a marker of viral replication and increased infectivity) in the mother
greatly increases the risk of vertical transmission of the virus. Because of this
concern, the newborns of all mothers with active hepatitis B are passively
immunized at birth with hepatitis B immune globulin (HBIG) followed by active
immunization with recombinant HBV vaccine.

243
USMLE WORLD STEP 1 MICROBIOLOGY

Q NO 88: A 35-year-old female presents to the ER with a diffuse pruritic


papulopustular rash. Her friend was brought to the ER with similar A. Human
symptoms two days ago. Pustular fluid demonstrates oxidase-positive contact
gram-negative rods that produce pigment on culture medium. Which of B. Insects
the following is the most likely source of this patient’s infection? C. Food
D. Water
E. Soil
F. Pets

Explanation:
The clinical syndrome described is consistent with “hot tub folliculitis,” a superficial
Pseudomonal infection of the hair follicle. The presentation of this condition is most
commonly in outbreaks from public or hotel swimming pools or hot tubs where the
chemicals in the pool water have not been maintained at the appropriate
concentrations thereby allowing P. aeruginosa proliferation in the pool water. Many
infections by P. aeruginosa often begin with exposure to a water source or creation
of a moist environment (i.e. swimmer’s ear, hot tub folliculitis, burn wound).
Pseudomonas aeruginosa, a gram-negative rod, is ubiquitous in nature and is
commonly isolated from water sources. Evidence indicating that the organism
responsible for this patient’s illness is Pseudomonas aeruginosa includes oxidase
positivity which is indicative of the genus Pseudomonas, production of pigment
during culture (pyocyanin, properdin), and the characteristic clinical picture of “hot
tub folliculitis, an illness almost exclusively caused by P. aeruginosa.

Educational Objective:
“Hot tub folliculitis” is a superficial and self-limited P. aeruginosa infection of the hair
follicles that tends to occur in minor outbreaks following exposure to a pool or spa
where the chemicals have not been maintained at appropriate levels. The culture of
a pustule will reveal Gram-negative, oxidase positive, non-lactose fermenting, motile
rods that produce pigment.

244
USMLE WORLD STEP 1 MICROBIOLOGY

Q NO 89: A group of investigators has developed monoclonal antibodies to the


meningococcal pilus. They believe that these antibodies may decrease the risk
of infection by meningococci. If effective, these antibodies would directly A.
interfere with which of the following processes?
Intestinal attachment
B. Endothelial adhesion
C. Pharyngeal colonization
D. Incomplete phagocytosis
E. Hematogenous dissemination

Explanation:
Meningococci are commonly isolated from the oropharynx and nasopharynx of
asymptomatic carriers. Colonization with N. meningitidis may persist for several
weeks to months and occurs in 5%to 15% of individuals. Transmission of
meningococci usually occurs by exposure to respiratory droplets or direct contact
with respiratory secretions. The most likely route by which meningococci enter the
blood from the pharynx is via pilus-mediated adherence to, and penetration of the
mucosal epithelium subsequently gaining access to the circulation. N. meningitidis
produces an IgA protease that facilitates survival of the organism in the mucosa by
destroying antibodies that could potentially prevent the attachment and penetration
of the bacteria.
(Choice A) Antibodies that prevent intestinal attachment would be useful for
organisms whose route of entry involves ingestion such as Salmonella, Shigella,
Enteroinvasive E. coli and Entamoeba histolytica.
(Choice B) The use of antibodies to prevent endothelial adhesion would be too late in
the disease process to be effective, as the meningococcus would have entered the
bloodstream by that time and either colonized the meninges or released lipo
oligosaccharide (endotoxin), resulting in sepsis, purpuric or hemorrhagic cutaneous
lesions, circulatory collapse and death.
(Choice D) Antibodies could not be used to accomplish incomplete phagocytosis, a
process where foreign materials are ingested by phagocytes but are not destroyed.
This is the normal state for the Neisseria species, a facultative intracellular organism
that evades destruction due to its polysaccharide capsule.
(Choice E) While the use of antibodies to prevent hematogenous dissemination of N.
meningitidis would be advantageous to protect the patient from the morbidity and
mortality associated with this organism, it would be too late in the disease process
to provide the necessary high level of effectiveness. Further, the pilus mediates
mucosal adherence and penetration; survival in the circulation is dependent on the
antiphagocytic properties of the polysaccharide capsule that envelops the
meningococcus.

Educational Objective:
Meningococcal pili are responsible for epithelial attachment to nasopharynx.
Antibody coating these pili would prevent pilus-mediated attachment of the
meningococcus to the mucosal epithelium of the nasopharynx, thereby preventing
invasion and disease.

245
USMLE WORLD STEP 1 MICROBIOLOGY

Q NO 90: A 31-year-old female has the following findings on oral examination:


She has no significant past medical history and no other complaints. Which of the
following components of her medical history is most likely to reveal the cause
underlying this condition?

A. Personal allergies
B. Alcohol consumption
C. Recent travel
D. Animal contact
E. Sexual practices
F. Cave exploration

Explanation:
White plaques on the oral mucosa as shown in the image above are characteristic of
oral thrush, a localized manifestation of Candida albicans infection. The other types
of superficial disease associated with this fungus include cutaneous candidiasis
(“diaper rash” in infants) and Candida vulvovaginitis.
Oral thrush occurs in denture wearers, diabetics immunosuppressed patients, and
patients receiving steroids, antibiotics or chemotherapy. Oral thrush in an otherwise
healthy individual is suggestive of immunosuppression and is a common
presentation of HIV infection. Vulvovaginal candidiasis is associated with antibiotic
and contraceptive use, pregnancy, diabetes mellitus and HIV. Cutaneous candidiasis
occurs in areas exposed to heat and high humidity (such as the groin and perianal
area in infants).
Oral thrush most commonly presents with white patches on the oral mucosa
(membranous candidiasis). The patches can be scraped oft, revealing an
erythematous mucosal surface. Microscopic examination of KOH-treated scrapings
shows Candida yeast and pseudohyphae.
(Choices A, B, and C) Candida albicans infection is not associated with allergies,
alcohol consumption, or recent travel.
(Choice D) Animal contact may lead to infection with Microsporum species. This
fungus affects the superficial keratinized skin layer, causing dermatophytosis. There
is no association with oral thrush.
(Choice F) Histoplasma capsulatum is endemic to the Mississippi and Ohio River
basins. It is present in bird and bat droppings. Patients with histoplasmosis often
have a history of cleaning bird coops or exploring caves.

Educational Objective:
Oral thrush is caused by Candida albicans infection. It is associated with wearing
dentures, diabetes mellitus and immunosuppression. Unexplained oral thrush in an
otherwise healthy person suggests the possibility of HIV infection.

246
USMLE WORLD STEP 1 MICROBIOLOGY

Q NO 91: A 54-year-old male who has been HI V-positive for more than five years
presents to clinic for evaluation. The viral strains isolated from blood samples
drawn at this visit demonstrate a significant increase in pol gene mutations over
the last year. Which of the following is this observed finding most likely related to?
A. Evasion of humoral immune response
B. Prevention of CD4+ cell apoptosis
C. Reduced viral replication
D. Ineffective CD8+ cytotoxicity
E. Anti retroviral chemotherapy

Explanation:
Drug resistance is associated with the prolonged use of highly active anti retroviral
therapy (HAART) a regimen that includes inhibitors of HIV reverse transcriptase and
protease. The development of such drug resistance has been attributed to the high
mutation rate of the HIV genome and the selective pressure exerted by anti
retroviral drugs. Pol gene mutations are responsible for the emergence of HIV
protease variants that are resistant to standard protease inhibitors. The pol gene
mutations are also responsible for structural changes in HIV-1 reverse transcriptase
that render the enzyme resistant to standard nucleoside and non-nucleoside reverse
transcriptase inhibitors.
(Choice A) The humoral immune response against HIV is comprised of neutralizing
antibodies directed against the epitopes of folded viral envelope glycoproteins.
Frequently emerging after the primary infection are newHlV-1 “escape mutants” that
are no longer susceptible to host antibody neutralization. Because structural
glycoproteins are encoded for by the HIV-1 env gene (rather than the pol gene) viral
evasion of humoral immunity is more likely to occur secondary to a mutation of the
env gene.
(Choice B) Activation-induced apoptosis can occur in uninfected CD4+ cells as they
respond to MHC class II- associated HIV peptides on the surfaces of infected cells.
Abnormal intracellular signals transduced by HIV might also prime the infected
CD4+ T cells for apoptosis. Hypothetically, a reduction in CD4+ T cell apoptosis
could result if mutations in the HIV genome either changed the MHC class II
antigenic peptides on the infected cells or reduced the host intracellular caspase
activation. However, itis unclear to what extent these alterations actually occur.
Moreover, no direct association between these alterations and pol gene mutations
has been established.
(Choice C) Reduced viral replication is not likely to be associated with a high rate of
genomic mutations in HIVI as the replication rate is generally positively correlated
with the mutation rate.
(Choice D) HIV-1 possesses the intrinsic ability to mutate rapidly. During both initial
and chronic infection, mutations can develop in the HIV-1 epitopes recognized by
CD8+ cytotoxic T cells. As a result, cell-mediated immune (CMI) responses may
quickly become inadequate to control HIV. However, a specific linkage between such
CMI epitope evolution and pol gene mutations has not been established.
Educational Objective:
The high mutability of HIV-1 allows for the evasion of host humoral and cellular
immune responses and the development of resistance to anti-retroviral drugs. Pol
gene mutations are responsible for acquired resistance to HIV reverse transcriptase
inhibitors and HIV protease inhibitors. Env gene mutations enable escape from host
neutralizing antibodies.

247
USMLE WORLD STEP 1 MICROBIOLOGY

Q NO 92: A 52-year-old Asian male presents to your office with cough, night sweats
and occasional hemoptysis. Sputum cultures placed on a selective medium grow
mycobacteria microscopically observed to grow in parallel chains (“serpentine
cords”). This observed bacterial growth pattern most strongly correlates with:

A. Acid-fastness
B. Growth rates
C. Virulence
D. Pigmentation
E. Survival in macrophages

Explanation:
This patient’s clinical presentation is suggestive of pulmonary infection with
Mycobacterium tuberculosis. The growth of thick ropelike cords of mycobacterial
organisms in a twisted, “serpentine” pattern is consistent with the presence of cord
factor. Cord factor is a mycoside, meaning it is composed of two mycolic acid
molecules bound to the disaccharide trehalose. The presence of cord factor
correlates with virulence; mycobacteria that do not possess cord factor are not able
to cause disease. More specifically, cord factor is responsible for inactivating
neutrophils, damaging mitochondria, and inducing release of tumor necrosis factor.
(Choice A) Acid-fastness is a property inherent to all mycolic acid-containing
bacterial organisms, including Mycobacterium and Nocardia.
(Choice B) Growth rates are not responsible for the formation of cords by
mycobacteria. Mycobacteria grow at varying rates, and the clinically significant
mycobacteria are often very slow growing.
(Choice D) Pigment production is characteristic of some mycobacteria, but is not
associated with cord formation.
(Choice E) Sulfatides inhibit the fusion of lysosomes to phagosomes, permitting
mycobacteria to persist as facultative intracellular organisms within phagocytes.
Sulfatides, though considered a virulence factor, do not cause cord formation.

Educational Objective:
Virulent mycobacteria will grow as “serpentine” cords on enriched media secondary
to the presence of cord factor, a mycoside. Cord factor establishes virulence through
neutrophil inhibition, mitochondrial destruction, and the induced release of tumor
necrosis factor.

248
USMLE WORLD STEP 1 MICROBIOLOGY

Q NO 93: During the first trimester of pregnancy a 22-year-old immigrant from the
Middle East develops a low-grade fever and maculopapular rash. Over 48 hours the
rash spreads from her face and chest to her trunk and extremities. Physical
examination also detects post auricular lymphadenopathy. The mother is at risk for
which of the following immediate complications (from the first column) and the
fetus is at risk for which of the following congenital defects (from the second
column)?

Mother Infant
A. Polyarthralgia deafness
B. Deafness bowlegs
C. Polyarthralgia macrocephaly
D. Meningitis malformed teeth
E. Pneumonia cataracts

Explanation:
This patient’s immigration history suggests that she may not have been vaccinated
against measles, mumps, and rubella (MMR) at 12-15 months of age. Measles
(rubeola) and German measles (rubella) are two acute viral exanthems whose
maculopapular rashes begin on the head and neck and spread downward.
Generalized lymphadenopathy, particularly postauricular and occipital, is typically
more prominent in rubella. Most adult women with rubella develop polyarthritis and
polyarthralgia as sequelae.
Fetal infection with the rubella virus during the first trimester can cause sensory
neural deafness cataracts and cardiac malformations like a patent ductus arteriosus.
Maternal deafness, meningitis, or pneumonia would be yew unlikely.
(Choice D) Malformed teeth such as Hutchinson’s incisors and mulberry molars are
typical late manifestations of congenital syphilis.

Educational Objective:
Maternal rubella infection produces a low-grade fever, a maculopapular rash with
cephalocaudal progression, and posterior auricular and suboccipital
lymphadenopathy. Most adult women patients develop polyarthritis and
polyarthralgia as sequelae. Congenital rubella syndrome is associated with sensory
neural deafness, cataracts, and cardiac malformations (PDA).

249
USMLE WORLD STEP 1 MICROBIOLOGY

Q NO 94: There is a specific bacterial product that, when injected locally into the
muscles of patients with relentless focal dystonias such as torticollis, produces a
dramatic but temporary relief of symptoms. This substance is produced by
bacteria that demonstrate:
A. Anti phagocytic capsule
B. Hypervariable pili
C. IgG-binding outer membrane protein
D. Intracellular polyphosphate granules
E. Subterminal spore formation

Explanation:
Focal dystonia is a localized uncontrollable muscle contraction causing pain or
discomfort as well as physical deformity in some cases. A classic example is cervical
dystonia of the sternocleidomastoid muscle, or torticollis. Local injection of
botulinum toxin type B into the dystonic sternocleidomastoid muscle results in
muscular relaxation because the toxin prevents presynaptic release of acetylcholine,
the neurotransmitter responsible for muscle contraction, from the nerve terminal at
the neuro muscular junction. This effect is temporary however, because
regeneration of the nerve terminal eventually occurs. (This process takes
approximately three months.) For this reason, therapeutic botulinum toxin injections
must be repeated when the effects begin to diminish.
Botulinum toxin can also be used cosmetically to reduce the appearance of glabellar
and other facial wrinkles. Itis also used to relax the lower esophageal sphincter in
esophageal achalasia1totreatthe muscle spasms of multiple sclerosis and Parkinson’s
disease, and for other conditions resulting from involuntary muscle contraction.
Clostridium botulinum is a Gram positive spore-forming anaerobic bacillus that
synthesizes its potent neurotoxins intracellularly and releases them by autolysis.
(Choice A) The antiphagocytic capsule is a primary virulence factor for Streptococcus
pneumoniae, Haemophilus influenzae and Neisseria bacteria.
(Choice B) Hypervariable pili are characteristic of Neisseria meningitidis and N.
gonorrhoeae.
(Choice C) Staphylococcus aureus has an lgG-binding outer membrane protein, the
Protein A virulence factor. Protein A binds the Ec portions of lgG molecules thereby
preventing opsonization, phagocytosis and complement fixation.
(Choice D) Intracellular polyphosphate granules are a characteristic of
Corynebacterium diphtheriae. Granules within the cytoplasm are evident with
methylene blue staining.

Educational Objective:
Clostridia are Gram positive spore1orming anaerobic rods. C. botulinum is the
bacteria responsible for botulism, a toxin-mediated disease. Local injections of
botulinum toxin into muscle are used to treat focal dystonias, achalasia and spasms.

250
USMLE WORLD STEP 1 MICROBIOLOGY

Q NO 95: Health authorities are investigating an outbreak of respiratory infections


among a group of military recruits. Fifteen recruits have reported persistent
cough, low-grade fever and malaise. Each patient’s chest X-ray shows pulmonary
infiltrates more severe in appearance than would be expected from the patient’s
clinical status. Sputum cultures require a complex acellular medium enriched with
cholesterol in order to grow. Which of the following organisms is the most likely
cause of the outbreak?
A. Streptococcus pneumoniae
B. Klebsiella pneumoniae
C. Haemophilus influenzae
D. Legionella pneumophila
E. Coxiella burnetii
F. Mycoplasma pneumoniae
G. Coccidioides immitis
H. Histoplasma capsulatum
I. Pneumocystis jiroveci

Explanation:
These patients are suffering from “walking pneumonia” a condition generally caused
by Mycoplasma pneumoniae. Patients experience a chronic dry nagging cough, low-
grade fever and malaise. The tell-tale sign is a chest X-ray that looks much worse
than the patient appears clinically. Another clue that M. pneumoniae is responsible is
the finding that the organisms required cholesterol to grow. Only organisms of the
Mycoplasma genus require cholesterol for growth on artificial media, because their
cell membrane is composed of a single cholesterol- rich phospholipid bilayer. These
organisms completely lack a peptidoglycan cell wall, cell envelope or capsule. (Fungi
also incorporate sterols into their cell membranes, but fungi use the sterol
ergosterol. This unique sterol is used as a target for antifungal therapy.)
(Choice A) Streptococcus pneumoniae would cause lobar consolidation on chest X-
ray. This organism can grow well on standard unenriched blood agar, but it cannot
grow in the presence of optochin or bile.
(Choice B) Klebsiella pneumoniae is a common cause of pneumonia in debilitated,
hospitalized, and alcoholic individuals. The classic scenario presented on exams is
pneumonia in an alcoholic who is coughing up red jelly-like sputum (“currant jelly
sputum”). K. pneumoniae can be grown on standard agar but often MacConkey agar
is preferred because it contains bile which will inhibit the growth of contaminant
organisms.
(Choice C) H. influenzae requires chocolate agar (heat lysed blood agar)
supplemented with factor X (hematin) and factor V (NAD+) in order to grow.
(Choice D) L. pneumophila requires an L-cysteine-supplemented agar to grow.
(Choice E) Coxiella burnetii is the etiologic agent of Q-fever. Q-fever is a mild
pneumonia-like illness that results from inhalation of the C. burnetii spores that
commonly contaminate animal hides. An obligate intracellular parasite this organism
must be provided a cell culture in order to grow.
(Choices C and H) The fungus Coccidioides /mm/Us is the etiologic agent of San
Joaquin Valley fever (valley fever). It is most common in the southwest United
States and northern Mexico. Histoplasma capsulatum causes a similar illness, but is
typically localized to the Mississippi and Ohio River valleys. Both organisms can be
cultured on standard bacterial culture media.
(Choice I) Pneumocystis jiroveci (carinii is a protozoal organism that can cause a
severe pneumonia in HIV+ patients whose CD4 count has dropped below 200.
Patients often have characteristic “ground glass” infiltrates on chest X-ray.

251
USMLE WORLD STEP 1 MICROBIOLOGY

Pneumocystis carinii pneumonia (PCP) is diagnosed by silvers taining fluid collected


during bronchoscopy.

Educational Objective:
Mycoplasma pneumoniae is the causative agent of “walking pneumonia” a condition
characterized by a nagging nonproductive cough, low-grade fever and malaise.
Often the chest X-ray suggests a severe pneumonia even though the patient
appears relatively well. Mycoplasma species require cholesterol supplementation in
order to grow on artificial media.

252
USMLE WORLD STEP 1 MICROBIOLOGY

Q NO 96: A 30-year-old male is admitted to the intensive care unit after his wife
found him unconscious at home. Laboratory studies show a serum pH of 7.10
and positive serum ketones. His mucus membranes are extremely dry. The wife
notes that he has lost a significant amount of weight recently. Despite eating
and drinking the normal amounts. Treatment is initiated, and the patient’s pH
improves. However, he now complains of fever headache and eye pain.
Examination of the nasal cavity reveals black necrotic eschar adherent to the
inferior turbinate. Which of the following procedures would likely confirm the
diagnosis?
A. Lumbar puncture
B. Blood cultures
C. Mucosal biopsy
D. Serologic testing
E. Ophthalmoscopy
F. Skin hypersensitivity test

Explanation:
This patient’s clinical presentation is consistent with a diagnosis of diabetic
ketoacidosis. Facial pain and headache in a patient with diabetic ketoacidosis are
highly suggestive of mucormycosis (caused by Mucor, Rhizopus and Absidia).
Rhizopus has an affinity for ketones and high blood glucose because of its enzyme
ketone reductase. These fungi proliferate in blood vessel walls, causing necrosis of
the downstream tissue. Black necrotic eschar seen in the nasal cavity is a
characteristic finding. Mucormycosis can rapidly spread to the CNS, causing
confusion, neurological deficits and death. This is a condition that requires prompt
diagnosis and treatment.
Histologic examination of the affected tissue is necessary in order to make the
diagnosis of mucormycosis. These fungi appear as broad nonseptate hyphae with
right angle branching. Tissue invasion by hyphae is seen along blood vessels;
vascular thrombosis and tissue necrosis may also be visualized.
Mucormycosis must be differentiated from invasive aspergillosis, as Aspergillus
fumigatus can also affect the paranasal sinuses of immunosuppressed patients,
causing similar symptoms. On light microscopic evaluation of the affected tissue
Aspergillus is seen as septate hyphae with V-shaped branching (45° angle).
(Choice A) Lumbar puncture is necessary to diagnose cryptococcal meningitis. India
ink staining of cerebrospinal fluid reveals the classic encapsulated yeast of
Cryptococcus neoformans. This fungal infection usually affects AIDS patients.
(Choice B) Blood cultures are used for the diagnosis of disseminated mycoses (e.g.
Candida). They are usually negative in mucormycosis.
(Choices D and F) Serological testing is important in the diagnosis of allergic
bronchopulmonary aspergillosis (ABPA). Increased titers of IgE and antibodies to
Aspergillus fumigatus are diagnostic. Skin hypersensitivity testing is also useful.
(Choice E) Ophthalmoscopy is useful for the diagnosis of Candida endophthalmitis.

Educational Objective:
The findings of facial pain, headache, and black necrotic eschar in the nasal cavity in
a patient with diabetic ketoacidosis are highly suggestive of mucormycosis.
Histologic examination of the affected tissue is necessary to confirm the diagnosis.
These fungi show broad nonseptate hyphae with right angle branching. Treatment
consists of surgical debridement and amphotericin B.

253
USMLE WORLD STEP 1 MICROBIOLOGY

Q NO 97: Respiratory epithelial cells exposed to rhinovirus demonstrate interferon


expression that markedly exceeds the resting levels. The observed response
confers which of the following advantages to the host?
A. Rapid induction of virus-specific cell mediated immunity
B. Selective inhibition of viral protein synthesis
C. Downregulation of viral receptors on the cell surface
D. Inhibition of apoptosis in the infected cells
E. Inhibition of systemic inflammatory response

Explanation:
Interferon-y production is limited to I cells and NK cells, while interferon-a and
interferon-β are produced by a wide variety of eukaryotic cells in response to viral
infection. The latter interferons act as cytokines on neighboring cells, binding to
specific interferon receptors. Cells stimulated by interferon-a and interferon-43
cease permitting viral replication, and begin to synthesize antiviral proteins that
impair the translation of viral mRNA without impacting cellular mRNA.
(Choice A) interferons are a component of innate immunity which is host specific but
not virus specific. Interferon -y may be involved in the non-specific up regulation of
cell-mediated immunity, which has the potential to increase the activation and
efficacy of virus-specific T killer cells. (For example, interferon-y can stimulate
macrophages to produce more IL-12, which promotes differentiation of more naive T
helper cells into Th 1 type helper cells, which then secrete more interferon-y, etc.)
In contrast interferons α and β have anti-viral properties that are neither virus
specific nor responsible for increased adaptive immunity against specific viruses.
(Choice C) Interferons do not directly affect extracellular viruses and do not enter
the cell. Instead they attach to cell surface receptors and send signals for the
synthesis of antiviral proteins.
(Choice D) All interferons are capable of stimulating natural killer (NK) cells, which
can kill virally infected cells by inducing apoptosis. Therefore, interferons would be
expected to stimulate (not inhibit) the apoptosis of infected cells.
(Choice E) The primary effect of interferons is to decrease viral replication in the
host cells that surround an infected host cell. As a consequence interferons might
slightly decrease the local and systemic inflammation associated with viral
cytopathic effects but this would have minimal impact. Moreover, inhibition of the
systemic inflammatory response in the setting of infection does not necessarily
confer a survival advantage on the host. To a certain degree, systemic inflammation
may improve the host’s ability to combat infection.

Educational Objective:
Interferons α and β are produced by a wide variety of eukaryotic cells in response to
viral infection. These interferons act as cytokines on neighboring cells, stimulating
them to synthesize antiviral proteins that impair viral protein synthesis.

254
USMLE WORLD STEP 1 MICROBIOLOGY

Q NO 98: A 24-year-old female presents to your office with burning during


urination, urine clouding and urinary frequency. She denies fever, chills
and flank pain. She had a similar episode before that was treated with A.
antibiotics. She has no other medical problems and does not use tobacco
alcohol or drugs. Her vital signs are stable. Physical examination shows
suprapubic tenderness. Which of the following bacteria is most likely to be
isolated from this patient’s urine?
Klebsiella pneumoniae
B. Salmonella typhi
C. Salmonella enteritidis
D. Campylobacter jejuni
E. Shigella dysenteriae
F. Proteus mirabilis
G. Yersinia enterocolitica
H. Haemophilus Influenzae
I. Vibrio cholerae
J. Streptococcus pyogenes
K. Streptococcus pneumoniae
L. Entamoeba histolytica
M. Escherichia coli

Explanation:
Usually, the urinary tract and urine are sterile. The presence of microorganisms in
the urine is due to either a urinary tract infection (UTI), contamination or both.
Contamination is highly suspected if the urine is polymicrobial or atypical of the
microbial flora commonly seen in UTIs. A UTI can be symptomatic (cystitis,
pyelonephritis) or asymptomatic (asymptomatic bacteriuria). It affects an estimated
20% of women at some time during their lifetimes. Most often the infection is due to
invasion by enteric coliform bacteria that inhabit the periurethral vaginal introitus
and ascend into the bladder via the urethra. The most common causative organism
is Escherichia coil, which accounts for 80% or more of cases. Most of the remainder
of cases in sexually active women are caused by Staphylococcus saprophy1/cus
other gram-negative enterics such as Proteus account for a smaller percentage.
Normally, symptomatic UTIs are treated with a three-day course of a fluoroquinolone
or TMP-SMX (trimethoprim sulfamethoxazole). Symptoms of fever chills and flank
pain should alert the physician to the possibility of pyelonephritis.

Educational Objective:
E. coil is the most common pathogen causing cystitis and acute pyelonephritis. The
second most common cause of
UTI in sexually active women is Staphylococcus saprophyticus. UTIs most commonly
affect women because of their very short urethra; recurrent UTIs in boys or men
require investigation for urinary tract obstruction or reflux.

255
USMLE WORLD STEP 1 MICROBIOLOGY

256
USMLE WORLD STEP 1 MICROBIOLOGY

Q NO 99: A 52-year-old Caucasian male treated for Hodgkin’s lymphoma presents


to the ER with a two-week history of headaches and fever. Lumbar puncture
shows a moderate increase in the CS protein concentration. Microscopic evaluation
of his CS reveals the following: The patient should be initially treated with:

A. Ceftriaxone
B. Ampicillin
C. Acyclovir
D. Amphotericin B
E. isoniazid
F. Pyrimethamine

Explanation:
This image shows the appearance of Cryptococcus neoformans with India ink
staining. The thick polysaccharide capsule of this encapsulated yeast does not stain
with India ink, and is seen as a peripheral clearing around the dark central part of
the yeast. India ink staining of CSF is used for the diagnosis of cryptococcal
meningitis. A latex particle agglutination test to detect cryptococcal capsular antigen
in CSF can also be used to help make the diagnosis. Cryptococcal capsular antigen
detection is more specific than India ink staining of CSF. Culture of the fungus on
Sabouraud’s agar is necessary to confirm the diagnosis.
The typical presentation of cryptococcal meningitis is headache, fever and lethargy
in an HIV (+) patient. Nausea, vomiting, stiff neck, and neurologic deficits may also
be seen. The CSF changes in cryptococcal meningitis are similar to those seen in
other fungal infections of the CNSI and include low glucose, increased protein, and
increased cell count. Lymphocytes predominate, although some neutrophils may also
be seen.
Cryptococcal meningitis is treated with amphotericin B. Flucytosine is added in HIV
(+) patients. HIV (-i-) patients also receive maintenance therapy with fluconazole
for life.
(Choice A) Ceftriaxone is used for pneumococcal and Hemophilus influenza
meningitis. Fungal cell walls are not made of peptidoglycan, so 3-lactam antibiotics
are not effective.
(Choice B) Ampicillin is used to treat Listeria meningitis. Listeria is a gram-positive
rod with characteristic tumbling motility on light microscopy.
(Choice C) Acyclovir is an antiviral medication used to treat Herpes simplex
meningitis. HSV is diagnosed using PCR.
(Choice E) Isoniazid is an anti-tubercular drug that inhibits mycolic acid synthesis. It
is not active against fungi.
(Choice F) Pyrimethamine is an antifolate agent used in the treatment of malaria and
toxoplasmosis. It is not used to treat cryptococcal infections.

Educational Objective:
Cryptococcal meningitis is diagnosed via India ink staining of the CSF. The main
treatment for this infection is Amphotericin B.

257
USMLE WORLD STEP 1 MICROBIOLOGY

Q NO 100: A developing country is experiencing a rise in the incidence of neonatal


tetanus (first 28 days after the birth). Which of the following measures would be
the most appropriate prevention measure to control the problem?

A. Routine vaccination at birth


B. Peripartal antibiotics
C. Vaccination of young adults
D. Early postpartum breast feeding
E. Extended postpartum hospital stay

Explanation:
In developing countries, most cases of tetanus occur in mothers with incompletely
removed placentas and in newborns with unclean and infected umbilical cord
stumps.
Tetanus vaccine has been available since 1925. Immunity to Clostridium tetani is
produced by vaccination with a formalin-inactivated toxin, also known as toxoid. The
first vaccine dose is started approximately 2 months after birth. After the initial
childhood vaccine series at 2 months 4 months, 6 months 15 to 18 months and 10
to 12 years booster immunizations are required every ten years to maintain a
protective level of antibody. Since lgG can cross the placenta, a newborn will achieve
protection from maternally derived antibody if the mother has immunity.
(Choice A) Tetanus vaccination is not given at birth. The first dose of the Diphtheria-
Pertussis-Tetanus series is commonly administered at 2 months old. In areas of
endemic tetanus disease this vaccine can be given as early as 4- 6 weeks of age, but
not earlier. At birth the infant’s immune system is immature and unable to mount a
proper memory immune response against the tetanus toxoid. Additionally, in areas
where maternal immunity is high from proper vaccination, circulating maternal lgG
in the newborn would bind the toxoid and prevent it from initiating an immune
response in the newborn thus making the vaccine useless.
(Choice B) Peripartal antibiotics are not routinely provided to newborns for tetanus
prophylaxis. It is used in cases where group B streptococcal infection or colonization
has been identified in the mother or in cases of HIV-infected mothers where
antiretroviral therapy is initiated peripartally and continued in the newborn to help
prevent vertical spread of the virus to the infant.
(Choice D) While early postpartum breast feeding is generally very important in the
reduction of infectious disease in the newborn it will not reduce the incidence of
neonatal tetanus because breast feeding provides primarily IgA mucosal immune
support, and for a mother to be able to provide immunoglobulin specific to a
particular infectious agents she herself must be immune to that disease. A non-
tetanus immune mother would not be able to provide passive immunity to tetanus to
a newborn by placental lgG or by IgA acquired from breastmilk.
(Choice E) Extension of postpartum hospital stay may help in providing more rapid
medical treatment to an infant experiencing neonatal tetanus, but it would not
reduce the incidence of neonatal tetanus.

Educational Objective:
Illness caused by Clostridium tetani (tetanus) can be prevented by proper
immunization with a childhood series and a booster immunization every ten years
thereafter in adulthood. An immunized mother will be able to pass lgG through the
placenta to the fetus and provide passive immunity against neonatal tetanus until
the child receives its first tetanus vaccination at two months of age. Neonatal
tetanus usually results from C. tetani colonization of the umbilical stump.

258
USMLE WORLD STEP 1 MICROBIOLOGY

Q NO 101: A 24-year-old African-American female presents to the emergency room


with fever, malaise, and intense pain over her right thigh. She has a long history of
frequent presentation to the hospital with painful crises. Usually she is admitted and
treated with supplemental oxygen and intravenous narcotics. This episode of pain is
very different because she is febrile (103.0F) with exquisite tenderness over the
right thigh, as well as local warmth. You send her for an MRI, and based on those
results you wish to begin antibiotics promptly. Which of the following organisms is
most likely causing her clinical presentation?

A. Salmonella
B. Escherichia
C. Klebsiella
D. Staphylococcus
E. Shigella

Explanation:
Many patients with sickle cell disease come to the ER frequently with painful,
vasoocclusive crises; these crises usually respond well to oxygen, IV fluids, and
high-dose narcotics. The above presentation is different from a run-of-the-mill crisis
because the patient is febrile, which indicates infection.
The vaso-occlusion that is associated with sickle cell anemia not only causes the
painful “sickle cell crises,” it also causes a relative immune deficiency because the
spleen suffers widespread infarction. This functional asplenia puts sickle cell patients
at an increased risk of infection by encapsulated organisms such as Neisseria,
Haemophilus, Streptococcus pneumoniae, and Salmonella species. Certain
vaccinations are routinely given to patients with sickle cell disease and patients who
are asplenic for other reasons (i.e. trauma). These vaccinations are: the Pneumovax
for S. pneumoniae, Hib for H. influenzae type b, and the Meningitis polysaccharide
capsular vaccine for N. meningitidis. Salmonella possesses a special capsule called
the “Vi antigen” (Vi stands for virulence) which protects the bacterium from
opsonization and phagocytosis.
Salmonella is the most common cause of osteomyelitis in patients with sickle cell
anemia (choice A), followed by E. coli (K antigen capsule), then S. aureus. An
increased risk of osteomyelitis is present because vaso-occlusive crises cause focal
areas of bone necrosis; within which bacteria can easily establish infection.
(Choice C) Klebsiella can cause nosocomial pneumonia, UTI, and sepsis. Klebsiella is
most notorious for causing aspiration pneumonia, such as in a hospital setting or in
an alcoholic patient. Pneumonia caused by Klebsiella causes hemoptysis and is
classically associated with “currant jelly” sputum.
(Choice D) In normal, healthy adults and children, Staphylococcus aureus, is the
number one cause of osteomyelitis; however, it causes less than one-quarter of
osteomyelitis in sickle-cell patients.
(Choice E) Shigella has four pathogenic species which can all cause a form of
bacterial dysentery called shigellosis, which is associated with severe abdominal
cramping and tenesmus. Shigella is not a common cause of osteomyelitis.

Educational Objective:
Salmonella, followed by F. coil, is the most common cause of osteomyelitis in
patients with sickle cell anemia.
Patients with sickle cell disease have functional asplenia secondary to multiple
infarctions of the spleen, which makes these patients more prone to infection by
encapsulated organisms.

1
USMLE WORLD STEP 1 MICROBIOLOGY

Q NO 102: A 2-year-old male is brought to clinic with fever irritability, and


decreased oral intake. Physical examination reveals swollen gums with A.
ulcerative lesions and enlarged, tender cervical lymph nodes. Oral lesion
scrapings demonstrate cells with intranuclear inclusions. Which of the
following is most likely responsible for this patient’s disease?
Enveloped double-stranded DNA virus
B. Non-enveloped double-stranded DNA virus
C. Non-enveloped single-stranded DNA virus
D. Non-enveloped positive-sense RNA virus
E. Enveloped positive-sense RNA virus
F. Enveloped negative-sense RNA virus

Explanation:
This child presents with gingivostomatitis, fever, and lymphadenopathy in a scenario
highly suggestive of a primary type 1 herpesvirus (HSV-1) infection. Acute
gingivostomatitis in children aged 1 to 3 years is the most common clinical
manifestation of primary infection with HSV-1. There are few, if any, other illnesses
associated with this specific clinical presentation. Intranuclear inclusions are
characteristic of herpesviruses, which replicate predominantly within the host cell
nucleus. Parenthetically, measles infection is occasionally associated with
intranuclear inclusions, but also tends to produce bluish-gray Koplik spots on the
buccal mucosa. HSV-1 and other herpesviruses are enveloped and possess double-
stranded DNA genomes.
(Choices B, C, D, E, and F) Because HSV-1 is an enveloped, double-stranded DNA
virus, it would be incorrect to describe it as non-enveloped, single-stranded, or
containing an RNA genome.

Educational Objective:
Children aged one to three years who develop gingivostomatitis characterized by
ulcerative lesions with intranuclear inclusions are likely experiencing primary
infection with herpes simplexvirus type 1 (HSV-1). HSV-1 and other herpesviruses
are enveloped and possess double-stranded DNA genomes.

2
USMLE WORLD STEP 1 MICROBIOLOGY

Q NO 103: A new medication with a wide spectrum of antibacterial activity has been
synthesized. It is an analog of D-alanine-D-alanine that blocks bacterial
peptidoglycan cross-linking. It is also resistant to degradation by bacterial enzymes.
Which of the following bacteria is most likely to be resistant to this new drug?

A. Pasteurella multocida
B. Borrelia burgdorferi
C. Treponeme pallidum
D. Helicobacter pylori
E. Actinomyces israelii
F. Mycoplasma hominis

Explanation:
This new antibiotic inhibits synthesis of the peptidoglycan cell wall found in both
Gram positive and Gram negative organisms. This cell wall is essential for the
survival of these organisms because it acts as a permeability barrier and protects
them from destruction by osmotic stressors.
Organisms in the Mycoplasma genus, including Ureaplasma urealyticum, lack
peptidoglycan cell walls. Thus, cell wall synthesis inhibitors such as penicillins,
cephalosporins, carbapenems, vancomycin, and this new drug would be ineffective
(Choice F) against these organisms. Mycoplasma are very small organisms that have
only a single phospholipid bilayer membrane separating them from the environment.
Their cell membrane contains cholesterol just as human cell membranes do. Drugs
that are effective against the Mycoplasma genus include anti-ribosomal agents like
the macrolides and tetracyclines.
(Choice A) Pasteurella multocida is a Gram negative organism well-known for
causing wound infections following cat bites. Pasteurella multocida can be effectively
treated with penicillin because penicillin disrupts the very thin but essential
peptidoglycan cell wall present in Gram negative organisms.
(Choices B and C) Borrelia burgdorferi and Treponema pallidum are disease-causing
spirochetes that can be effectively treated with penicillin because they contain a thin
peptidoglycan cell wall.
(Choice D) Helicobacter pylori is a Gram negative helical organism that is closely
related to the Campylobacter genus. Multiple agents are required for complete
eradication of H. pylon infection. The recommended regimen includes a proton pump
inhibitor (e.g. omeprazole) combined with clarithromycin and amoxicillin with or
without bismuth (triple / quadruple therapy).
(Choice E) Actinomyces israelii is a Gram positive, fungus-like bacterium that has a
thick peptidoglycan cell wall and is effectively treated with penicillin.

Educational Objective:
All organisms in the Mycoplasma genus, including Ureaplasma, lack peptidoglycan
cell walls and are therefore resistant to agents that attack the peptidoglycan cell wall
such as penicillins, cephalosporins, carbapenems and vancomycin. Mycoplasma
infections can be treated with anti-ribosomal agents like tetracycline and
erythromycin.

3
USMLE WORLD STEP 1 MICROBIOLOGY

Q NO 104: Gram stain of a cerebrospinal fluid (CSF) sample from a 21-year-old


Caucasian male army recruit with fever and headache demonstrates bean-
shaped Gram negative cocci in pairs. Which of the following is the most likely A.
route of meningeal contamination in this patient?
Pharynx — lymphatics — meninges
B. Middle ear —p contiguous tissues — meninges
C. Traumatic wound — leaking CSF — meninges
D. Pharynx —p blood — choroid plexus — meninges
E. Primary lung focus — blood — meninges

Explanation:
Neisseria meningitidis is the second most common cause of acute bacterial
meningitis in adults in the U.S. When the organism proliferates, it enters the blood
stream and subsequently colonizes the choroid plexus and enters the meninges,
causing meningitis.
Neisseria meningitidis is transmitted from person to person by respiratory droplets
usually from asymptomatic carriers with nasopharyngeal colonization. Anyone who
comes in contact with the organism may become a carrier. The carrier rate in the
general population is approximately 5% but may be higher where individuals are
living in close quarters (e.g. new military recruits, college freshmen). The carrier
state is asymptomatic; less than 1 in 500- 1000 carriers develop hematogenous
dissemination resulting in septicemia or meningitis.
In meningococcemia, there may be signs and symptoms of sepsis and characteristic
petechial skin lesions. In fulminant meningococcemia, also known as Waterhouse-
Friderichsen syndrome, bilateral adrenal hemorrhage causes adrenal insufficiency,
brisk hypotension, and death.
(Choice A) Pharynx — lymphatics — meninges: This is the mechanism of spread of
Haemophilus influenzae meningitis. Infants and young children are most commonly
affected.
(Choice B) Middle ear — contiguous tissues — meninges: This is an unusual
mechanism by which S. pneumoniae may gain access to the CNS during an acute
infection of the middle ear.
(Choice C) Traumatic wound — leaking CSF — meninges: This is the most common
cause of S. aureus meningitis or CNS abscess. Following penetrating skull trauma or
neurosurgery, S. aureus from the skin is introduced into the CNS.
(Choice E) Primary lung focus — blood — meninges: This can be the mechanism of
dissemination of Mycobacterium tuberculosis meningitis or S. peroneae meningitis.
M. tuberculosis meningitis is a chronic meningitis, characterized by monocytes and
lymphocytes in the CSF, primarily affecting the basal meninges.

Educational Objective:
Neisseria meningitidis gains access to the CNS by first colonizing the nasopharynx
and subsequently invading the mucosal epithelium and gaining access to the
bloodstream. Through the blood, it spreads to the choroid plexus, gains access to
the CNS through the blood-brain barrier, and initiates an inflammatory process.

4
USMLE WORLD STEP 1 MICROBIOLOGY

Q NO 105: A 32-year-old male presents to the emergency room with progressive


difficulty walking over the last two days accompanied by tingling in his feet.
Physical examination reveals decreased muscular strength in the bilateral lower
extremities with absent deep tendon reflexes. His review of systems and past
medical history are unremarkable except for a bout of diarrhea two weeks ago
that resolved spontaneously. Which of the following agents is most likely
associated with this patient’s condition?
A. Vibrio
B. Campylobacter
C. Salmonella
D. Shigella
E. Escherichia
F. Clostridium

Explanation:
The patient in the vignette above likely has Guillain-Barré syndrome (GBS)I a
demyelinating syndrome of the peripheral nerves characterized by ascending muscle
weakness and paralysis. GBS is attributable in 10-30% of instances to C. jejuni
infection. Campylobacter jejuni is one of the most common causes of infectious
diarrhea.
(Choices A, C, D, E, and F) These organisms are not associated with GBS.

Educational Objective:
Campylobacter jejuni infection can lead to Guillain-Barre syndrome (GBS) in rare
instances. GBS is a demyelinating syndrome of the peripheral nerves characterized
by ascending muscle weakness and paralysis.

5
USMLE WORLD STEP 1 MICROBIOLOGY

Q NO 106: A 32-year-old male presents to the ER complaining of fever malaise, and


a maculopapular rash that includes his palms and soles. When the patients serum is
added to a mixture of cardiolipin, lecithin and cholesterol, extensive flocculation is
observed. The next step in management of his condition would require evaluation
for

A. Rheumatoid factor
B. Cold agglutinins
C. Spirochetal antibodies
D. Fungal antibodies
E. Bacterial toxin

Explanation:
The laboratory test described in this case is a nonspecific treponemal serologic test
known as the rapid plasma reagin (RPR) test. In this test, the patient’s serum is
mixed with a solution of cardiolipin, cholesterol and lecithin. Aggregation or
“flocculation” of the sample demonstrates the presence of cardiolipin antibodies in
the patient’s serum. This test is considered a nontreponemal serologic test because
it does not detect treponemal organisms or antibodies directed against treponemal
organisms. Instead, it detects antibodies to human cellular lipids released into the
bloodstream after cell destruction by T pallidum. These serologic tests will be
positive in cases of syphilis, yaws, pinta or bejel as all of these illnesses are caused
by subspecies of T pallidum.
Because the nontreponemal RPR and VDRL tests are easy and inexpensive, they are
typically performed first in patients with suspected Treponemal infection (as was
done in this case). If a nontreponemal test is positive, confirmation is established
with a treponemal test like ETA-ABS. This specific confirmatory test detects
antibodies directed against Treponema, and is carried out through indirect
immunofluorescence of patient serum mixed with whole killed T pallidum. A positive
ETA-ABS confirms infection with T pallidum.
(Choice A) Rheumatoid factor (RE) is an anti-Fc lgG antibody commonly used in the
evaluation of patients with autoimmune disease, especially rheumatoid arthritis
(RA). It is positive in most cases of rheumatoid arthritis, but the presence of RE
does not definitively establish the diagnosis of RA and the absence of RE does not
definitively exclude the diagnosis of RA.
(Choice B) Cold agglutinins are characteristic of Mycoplasma infection. The presence
of cold agglutinins can be tested for when patient blood is added to a tube
pretreated with an anticoagulant (eg, EDTA) and the tube is then placed in a cup of
ice. The RBCs agglutinate at the low temperature and return into suspension when
warmed.
(Choice D) Fungal antibodies can be used as a method of diagnosing acute infection
with Histoplasma.
(Choice E) The detection of bacterial toxin is not typically associated with
Treponemal infections because these spirochetes do not produce exotoxins.

Educational Objective:
The more common means of diagnosing infection with Treponemal pallidum involves
both screening and confirmatory serologic tests. The screening tests are known as
the RPR or VDRL, and the confirmatory test is known as the ETAABS.

6
USMLE WORLD STEP 1 MICROBIOLOGY

Q NO 107: A small urinary tract infection outbreak is reported among children in a


daycare center. Several boys develop acute onset dysuria and hematuria, both of
which resolve spontaneously with in three days. Polymerase chain reaction (PCR)
performed on the urine samples of these boys is most likely to detect genomic
fragments of which of the following viruses?
A. Human papilloma virus
B. Adenovirus
C. Parvovirus B19
D. Norwalk agent
E. Rotavirus
F. Mumps virus

Explanation:
A urinary tract infection characterized by dysuria and hematuria is most likely
hemorrhagic cystitis. Acute hemorrhagic cystitis in children maybe caused by
bacteria or viruses, though in a majority of cases no infectious agent can be cultured
from the urine. The most common known viral cause of acute hemorrhagic cystitis in
children (and males in particular) is adenovirus; serotypes 11 and 21 of adenovirus
subgroup B have been prebacillary implicated in this illness.
(Choice A) Human papilloma virus (a papovavirus) gives rise to warts cervical
intraepithelial neoplasia, and carcinoma of the cervix. It is not associated with
hemorrhagic cystitis.
(Choice C) Parvovirus B19 is predominantly responsible for aplastic crises in patients
with chronic hemolytic disorders (eg, sickle cell anemia), the childhood viral
exanthem of childhood termed erythema infectiosum (fifth disease), and hydrops
fetalis.
(Choice D) The Norwalk agent (which was isolated during a public elementary school
outbreak in Norwalk, Ohio) is a calicivirus responsible for epidemics of viral
gastroenteritis worldwide.
(Choice E) Rotavirus, a reovirus, is the predominant global cause of infantile
gastroenteritis and the most common cause of fatal diarrhea in children. Rotavirus is
a major cause of acute diarrheal illness during the winter months in the United
States.
(Choice F) Mumps predominantly causes parotitis, orchitis, and (rarely) aseptic
meningitis.

Educational Objective:
A urinary tract infection characterized by dysuria and hematuria is most likely acute
hemorrhagic cystitis. Adenovirus is the most common known viral cause of acute
hemorrhagic cystitis outbreaks in children.

7
USMLE WORLD STEP 1 MICROBIOLOGY

Q NO 108: A 33-year-old female presents to a fertility clinic for assessment. She


has been attempting to conceive for one year without success and her
monogamous male partner has been found to have an adequate sperm count
and sperm motility. Her past medical history is significant for 15 years of oral
contraceptive use prior to two years ago purulent urethritis 5 years ago treated
promptly with ceftriaxone, and hypertension controlled with methyldopa. Which
of the following is the most likely major contributing factor in her inability to
conceive?
A. Methyldopa oocyte toxicity
B. Inadequate time of attempted conception
C. Inappropriate antibiotic therapy
D. Oral contraceptive-induced ovarian failure
E. Advanced maternal age

Explanation:
Tubal disease causes approximately 20% of infertility due to female causes. The
most common cause of tubal-factor infertility is pelvic inflammatory disease (PID).
PID is most frequently caused by Neisseria gonorrhoeae and Chlamydia trachomatis.
Infection by either or both of these organisms can often be asymptomatic, but if
symptomatic they will initially cause a purulent urethritis followed by ascension to
the cervix where infection can further spread to cause purulent infection and
inflammation in the endometrium, fallopian tubes and peritoneal cavity. Conditions
associated with this ascension of infection into the female genital tract and
peritoneum include: 1) PID, which is manifest clinically as purulent cervical
discharge and cervical motion tenderness, 2) Salpingitis and tubo-ovarian abscess,
and 3) Peritoneal inflammation including the Fitz-Hugh-Curtis syndrome from
inflammation of the hepatic capsule. Treatment of gonococcal PID must also always
include treatment for C. trachomatis as Chlamydia will often co-infect with Neisseria
gonorrhoeae. A third-generation cephalosporin will treat the gonococcal infection and
further treatment with azithromycin or doxycycline is required to treat the
Chlamydia, which is not sensitive to the Beta lactams. In this case the patient was
only treated with a cephalosporin (Choice C), and Chlamydia was allowed to persist
and cause an asymptomatic infection leading to fallopian tube scarring and
subsequent infertility. (Choice A) Methyldopa is not known to cause oocyte toxicity
or problems in pregnancy. In fact, it is the first-line drug in treating hypertension in
pregnancy. (Choice B) The couple mentioned in the question stem has been
attempting to conceive for one year. Studies have shown that by one year over 80%
of women attempting to conceive will be pregnant. People attempting to conceive
are asked to do so for at least one year before seeking medical intervention for
fertility treatment. (Choice D) Oral contraceptive use is not associated with ovarian
failure. Ovarian failure can occur approximately 2 years earlier in women who are
smokers, and treatments such as radiation and chemotherapy are known to be toxic
to the ovaries. (Choice E) Advanced maternal age becomes a factor in the ability to
conceive after the age of 35 and as women approach menopause, which
characteristically occurs at approximately 51 years old. The decrease in fecundity
after the age of 35 is primarily due to aging of the oocytes and decreased ovarian
reserve of oocytes.
Educational Objective: Pelvic inflammatory disease is most frequently caused by N.
gonorrhoeae and C. trachomatis. If Neisserial or Chlamydial urethritis, cervicitis or
PID is not treated, they can lead to scarring of the fallopian tubes and infertility or
ectopic pregnancy.

8
USMLE WORLD STEP 1 MICROBIOLOGY

Q NO 109: An infant born to a 25-year-old intravenous drug abuser develops


prominent oral thrush, interstitial pneumonia, and severe lymphopenia during
the first year of life. The mother received no prenatal care. She refuses to A.
undergo any kind of testing and demands “quick relief” for her child. Which of Live
the following prenatal interventions would have most likely prevented this
infant’s condition?
attenuated vaccine
B. Viral component vaccine
C. Killed vaccine
D. Toxoid vaccine
E. Viral enzyme inhibitor drug
F. Cellular receptor inhibitor drug
G. Interferons
H. Prenatal vitamins

Explanation:
Oral thrush, interstitial pneumonia and severe lymphopenia during the first year of
life are consistent with the vertical transmission of HIV-1 from mother to fetus. The
risk of HIV infection occurring in an infant born to an HIV-positive mother who
received no prenatal antiretroviral therapy is estimated to be 1S-39%. Studies have
shown that maternal prophylaxis during pregnancy with the nucleoside analog
zidovudine (ZDVI AZT)I a retroviral reverse transcriptase inhibitor, reduces the risk
of perinatal transmission by about two-thirds in HIV-positive women who have not
previously received antiretroviral therapy. The CDC currently recommends a three-
part regimen of zidovudine (ZDV). Antiretroviral treatment ideally should be initiated
at 14 weeks of gestation and continued throughout pregnancy followed by
intravenous ZDV during labor, and oral administration of ZDV to the infant for six
weeks postpartum.
(Choices A, B, C, and D) Ongoing trials of preventive HIV vaccines are currently no
more advanced than Phase III.
(Choice F) HIV viruses attach to their major target host cells (CD4+ T cells)
primarily via the binding of viral envelope glycoprotein gpl2O to the CD4
transmembrane glycoprotein. Enfuvirtide currently is the only HIV fusion inhibitor
commercially available. This drug is thought to interfere with the entry of HIV-1 into
target cells by blocking conformational changes in the gp41 subunit of the HIV-1
glycoprotein required for fusion of the virus to the membrane CD4 receptor of the
host cell. This is currently an FDA pregnancy class B drug, meaning that there is no
evidence of fetal risk in humans. Studies concerning efficacy in prevention of
perinatal transmission are lacking.
(Choice G) Leukocyte IFN-a is currently approved for the treatment of hepatitis B
and C virus infection hairy cell leukemia condyloma acuminatum, and Kaposi’s
sarcoma.

Educational Objective:
Studies have shown that maternal prophylaxis during pregnancy with the nucleoside
analog zidovudine (ZDV, AZT), a retroviral reverse transcriptase inhibitor, reduces
the risk of perinatal transmission by about two-thirds in HI V-positive women who
have not previously received anti retroviral therapy.

9
USMLE WORLD STEP 1 MICROBIOLOGY

Q NO 110: A 52-year-old male is having low-grade fevers after recent replacement


of his aortic valve. Repeated blood cultures grow gram-positive cocci. Which of the
following characteristics is most consistent with Staphylococcus epidermidis as the
cause of this patient’s symptoms?

A. Catalase negative
B. Coagulase negative
C. Mannitol fermenting
D. Novobiocin resistance
E. Yellow pigment production

Explanation:
The scenario emphasizes the differentiating characteristics of staphylococci. The
Staphylococci are Gram-positive cocci that most commonly form clusters, but can
also form pairs and rarely short chains. The catalase test (with 3% hydrogen
peroxide) differentiates Streptococci (catalase-negative) from Staphylococci
(catalase-positive). The ability to clot blood plasma (slide and tube coagulase tests)
separates Staphylococci into two groups: the coagulase positive Staphylococci
species, which consists of the most pathogenic species Staphylococcus aureus, and
coagulase-negative staphylococci (CNS) which includes S. epidermidis, S.
haemolyticus, S. saprophyticus, and 30+ other species. Among coagulase-negative
staphylococci, S. epidermidis is a ubiquitous skin contaminant.
S. epidermidis is capable of causing an opportunistic infection associated with
foreign bodies: it is the most common cause of endocarditis in patients with
prosthetic valves and septic arthritis in patients with prosthetic joints.
(Choice A) All Staphylococci are catalase-positive and streptococci are catalase-
negative.
(Choice C) Mannitol fermentation is a property of S. aureus. The other Staphylococci
are not able to ferment mannitol. (Choice D) Novobiocin resistance is a property of
Staphylococcus saprophyticus; this is the property of S. saprophyticus that allows it
to be differentiated from the other coagulase-negative Staphylococci.
(Choice E) Yellow pigment is produced by Staphylococcus aureus (which accounts for
its name: aureus = gold). Staphylococcus aureus typically causes infective
endocarditis in intravenous drug abusers. It is also the most common cause of
osteomyelitis.

Educational Objective:
Staphylococcus epidermidis and S. saprophyticus are both coagulase-negative
staphylococci species. Staphylococcus epidermidis is the most common cause of
infective endocarditis in patients with prosthetic valves and septic arthritis with
prosthetic joints. It is susceptible to novobiocin.

10
USMLE WORLD STEP 1 MICROBIOLOGY

Q NO 111: A 72-year-old man is brought to the emergency department from a


skilled nursing facility because of altered mental status, skin pushing, and a
high fever. His blood pressure is 60/20 mmHg, and heart rate is 110/mm. If A.
blood cultures are positive for E. coli, which of the following is the most likely
source of bacteremia in this patient?
Small intestine
B. Colon
C. Urinary tract
D. Biliary tree
E. Lungs
F. Oropharynx
G. Skin

Explanation:
This man is suffering from septic shock. The most common source of E. coli
bacteremia is the urinary tract. Some common predisposing factors to urosepsis are
urinary obstruction (i.e. BPH) fecal incontinence, a neurogenic bladder secondary to
diabetes, and frequent or indwelling catheterization.
Gram-negative sepsis or septic shock results from the body’s systemic reaction to
lipopolysaccharide endotoxin (a component in the membranes of some bacteria).
Signs of sepsis include hyper or hypothermia tachycardia, an elevated respiratory
rate, and an increased white blood cell count. Hyperventilation can lead to a
respiratory alkalosis.
Severe sepsis is marked by organ dysfunction from poor blood flow. Oliguria from
poor renal perfusion and altered mental status from poor cerebral perfusion are
often present. Late-stage sepsis will also manifest as metabolic lactic acidosis as a
result of poor tissue perfusion. Treatment at this time is focused on maintaining
blood pressure and organ perfusion with intravenous fluids, administering antibiotics
specific for the offending organism, and maintaining proper ventilation and
oxygenation.
(Choice A) Many microorganisms are inhabitants of the human GI tract. Under
normal circumstances, they do not cause systemic disease.
(Choice B) The normal flora of the colon is dense, primarily anaerobic, and
infrequently leads to infection. Exceptions include bowel perforation.
(Choice D) The biliary tree exhibits a number of protective mechanisms against
bacterial colonization and infection. The sphincter of Oddi prevents the reflux of non
sterile material (i.e. contents of the duodenum) into the biliary tree, bile salts inhibit
the growth of many bacteria, the downstream flow of bile helps to wash organisms
away from the biliary tree, and the secretory IgA in bile helps to prevent bacterial
adherence. Despite these mechanisms, bacteria can occasionally cause biliary tract
infection leading to cholangitis. Organisms most commonly responsible for this are
E. coli, Klebsiella, Enterobacter, and Enterococci. These organisms can cause
systemic illness and sepsis if cholangitis is left untreated. However, this process is a
much less frequent cause of bacteremia than UTI.
(Choice E) E. coli and other Gram-negative enteric rods (Pseudomonas,
Acinetobacter, and Klebsiella) are the most common causes of health care facility-
acquired pneumonia. From the lungs, these organisms can easily gain access to the
bloodstream and cause sepsis; however, sepsis from E. coli is less frequently caused
by pneumonia than by UTI.

(Choice F) Bacterial causes of pharyngitis and sinusitis do not tend to cause systemic
illness.

11
USMLE WORLD STEP 1 MICROBIOLOGY

(Choice G) The normal skin flora includes mostly Gram-positive organisms such as
Staphylococci and Streptococci.

Educational Objective:
E. coli is the most common cause of urinary tract infection in both healthy adults and
elderly patients. E. coli is a part of the normal bowel flora and special adhesive
proteins allow some strains to colonize and ascend the urinary tract. This can result
in pyelonephritis or bacteremia and sepsis from access to the bloodstream. The most
common cause of E. coli bacteremia is a urinary tract infection.

12
USMLE WORLD STEP 1 MICROBIOLOGY

Q NO 112: Naked RNA molecules are isolated from viral particles in the nasal
exudate of a 23-year-old male. After introduction into human cells, the naked
RNA molecules induce viral protein synthesis and viral genome replication. A.
The patient is most likely infected with:
Influenza virus type A
B. Respiratory syncytial virus
C. Rotavirus
D. HIV
E. Rhinovirus

Explanation:
For a naked (nonenveloped) RNA molecule to induce viral protein synthesis in the
host cell, it must act as mRNA capable of using the host’s intracellular machinery for
translation. In other words the RNA molecule must be single- stranded and positive
sense (SS+). Among the viruses listed, only rhinovirus (a picornavirus) contains
SS+ RNA. Generally speaking, only naked 33+ RNA molecules are infectious,
whereas the naked single-stranded negative sense (SS—) RNA molecules and the
naked double-stranded RNA molecules are not.
Other nonenveloped RNA viruses include enteroviruses (eg, poliovirus,
coxsackievirus, echovirus, hepatitis A virus), reoviruses (eg, rotavirus), and Norwalk
virus.
(Choice A) Influenza A is an orthomyxovirus that contains SS—RNA. In order for this
virus to replicate in a host cell an RNA-dependent RNA polymerase within the intact
virion must also gain entry into the host cell.
(Choice B) Respiratory syncytial virus (RSV) is a paramyxovirus that contains SS—
RNA. In order for this virus to replicate in a host cell an RNA-dependent RNA
polymerase within the intact virion must also gain entry into the host cell.
(Choice D) HIV is a retrovirus that contains 33+ RNA packaged with reverse
transcriptase, an RNA-dependent DNA polymerase.
(Choice C) Rotavirus is a reovirus containing double-stranded RNA. Its naked RNA is
therefore incapable of inducing viral protein synthesis in a host cell and is non-
infectious. For this virus to replicate in a host cell a specific viral RNA polymerase
present in the intact virion must also gain entry into the host cell.

Educational Objective:
For a naked RNA molecule to induce viral protein synthesis in a host cell it must be
able to act directly as mRNA using the host’s intracellular machinery for translation.
Thus naked viruses containing single-stranded positive-sense RNA can be infectious
whereas naked viruses containing single- or double-stranded negative sense RNA
are not infectious.

13
USMLE WORLD STEP 1 MICROBIOLOGY

Q NO 113: A 23-year-old intravenous drug user is hospitalized with fever and


fatigue. You obtain the following dynamics of serologic markers in this patient
(see the graph below).

This patient most likely:

A. Had acute hepatitis B and successfully recovered


B. Had an exacerbation of preexisting hepatitis B
C. Had acute hepatitis B that progressed to chronic hepatitis with low infectivity
D. Had acute hepatitis B that progressed to chronic hepatitis with high infectivity
E. Was vaccinated against HBV during the hospitalization

Explanation:
The serologic markers for hepatitis B infection appear on a predictable timeline with
a long asymptomatic incubation period (lasting an average of 6-8 weeks) followed
by acute disease lasting several weeks to months. HBsAg first appears before the
onset of symptoms, peaks when the patient is most ill, and then becomes
undetectable in 3-6 months. Shortly after HBsAg first appears HBeAg and HBV DNA
can be detected in the serum and are markers of active viral replication. Anti-HBcAg
1gM and elevated serum transaminases appear shortly before symptom onset. Over
the next few months, the anti-HBcAg 1gM component is replaced with an lgG
component. Anti-HBe appears shortly after HBeAg vanishes and suggests subsiding
viral activity. Anti-HBs lgG arises once the acute disease has resolved, and is
typically not detectable until weeks or months after HBsAg has disappeared. This
antibody may persist indefinitely, conferring lifelong immunity.
Given this serologic marker graph, then, it appears that this patient now has
undetectable levels of HBsAg and HBeAg with moderate to high levels of anti-HBcAg
and anti-HBsAg. These findings are suggestive of an acute hepatitis B infection that
has completely resolved.
(Choice B) An exacerbation of preexisting hepatitis B would be characterized by a
decline followed by a spike in antigen levels.
(Choice C) An acute hepatitis B that progressed to chronic hepatitis with low
infectivity would be indicated by persistent levels of HBsAg with low or undetectable
levels of HBeAg (the marker for high infectivity) accompanied by low to moderate
levels of anti-HBcAg and anti-HBeAg.

14
USMLE WORLD STEP 1 MICROBIOLOGY

(Choice D) An acute hepatitis B that progressed to chronic hepatitis with high


infectivity would be indicated by persistent levels of HBsAg, HBeAg, and HBV DNA
with low to moderate levels of anti-HBcAg, anti-HBsAg and anti HBeAg.
(Choice E) Vaccination against HBV would be identified by a surge in anti-HBsAg
with no anti-HBcAg, anti-HBeAg, or viral antigens detected.

Educational Objective:
A resolved hepatitis B infection is suggested by moderate to high levels of anti-
HBcAg and anti-HBsAg with no detectable viral antigens.

15
USMLE WORLD STEP 1 MICROBIOLOGY

Q NO 114: A 32-year-old migrant ranch worker is brought to the emergency room


with painful paroxysmal involuntary muscle contractions involving mainly the jaw,
neck and trunk. The causative substance has most likely traveled by which of the
following routes?
A. Wound — neuron axons — salivary glands
B. Wound — motor neuron axons — spinal cord
C. Fibrinous exudate —a systemic circulation — cortical neurons
D. Food — systemic circulation — meninges
E. Food —a systemic circulation —a peripheral nerves

Explanation:
Tetanospasmin is a potent neurotoxin produced by the vegetative cells of
Clostridium tetani. C. tetani spores are commonly found in soil and may contaminate
injured tissue, classically puncture wounds from soil-contaminated objects. The low
oxygen tension and high availability of nutrients in a puncture wound provides a
localized anaerobic environment for the germination of spores growth of vegetative
cells and production of toxin within bacterial cells. Upon bacterial cell autolysis, the
toxin is released and gains access to the motor neuron at the axon terminal. The
toxin then travels up the motor neuron axon by retrograde transport to the spinal
cord and medulla. The organism itself does not travel, but rather remains at the
local wound site.
Tetanus toxin inhibits the inhibitory interneurons that normally regulate the motor
neuron, leading to unregulated action of the motor neuron and tetany. Classic
tetanus findings include jaw stiffness due to spasmodic contraction of the masseter
muscle (trismus) and sustained contraction of the facial muscles producing a bizarre
“smiling” appearance (risus sardonicus). Spasmodic contraction further extends to
the back and neck leading to opisthotonos, and respiratory muscle involvement can
lead to respiratory failure.
(Choice A) Wound — neuron axons — salivary glands: This is the route that the
rabies virus takes. The virus is inoculated into a bite wound from the saliva of a
rabid animal. The virus is taken up at the motor neuron terminal and travels by
retrograde axonal transport into the nerves of the spinal cord and into the
cerebellum, brainstem and hippocampus. From there it travels within nerve axons to
the tissues of the salivary gland.
(Choice C) Fibrinous exudate — systemic circulation — cortical neurons: This is the
route that diphtheria toxin takes from the pseudomembranous exudate it forms in
the pharynx to the bloodstream and subsequently to cardiac and cerebral cortical
tissues.
(Choice D) Food — systemic circulation — meninges: Listens monocytogenes is a
Gram positive food-borne pathogen that causes meningitis in neonates and
immunocompromised patients through the path above.
(Choice E) Food — systemic circulation —a peripheral nerve: This is the route that
botulinum toxin takes in causing adult food-borne botulism.

Educational Objective:
Clostridium tetani is responsible for tetanus, a toxin-mediated disease that causes
uncontrolled muscle spasms and respiratory failure. Toxin travels within the motor
neuron by retrograde transport into the spinal cord where it causes inhibition of
inhibitory interneurons and unregulated firing of primary motor neurons.

16
USMLE WORLD STEP 1 MICROBIOLOGY

Q NO 115: A 63-year-old male is brought to the ER with recent onset of high fever,
confusion, headache watery diarrhea and productive cough. He has been smoking
two packs of cigarettes daily for more than 30 years and has been diagnosed with
chronic bronchitis. Otherwise his past medical history is not significant. His blood
pressure is 100170 mmHg, pulse is 90/mm, and respirations are 34/mm and
temperature is4O.5C (104.9F). Sputum microscopy after Gram staining reveals
numerous neutrophils but no bacteria. Which of the following is the most likely
cause of this patient’s disease?

A. Mycoplasma pneumoniae
B. Klebsiella pneumoniae
C. Streptococcus pyogenes
D. Legionella pneumophila
E. Mycobacterium kansasii
F. Coccidioides immitis

Explanation:
The classic presentation of Legionella infection leading to Legionnaires disease
includes very high fever in a smoker accompanied by diarrhea, confusion, and cough
causing chest pain. Legionella pneumophila is one of the most common causes of
community acquired pneumonia. Diagnosis can be difficult because the signs and
symptoms are not specific, but Legionella should be suspected in a patient with
radiographic evidence of pneumonia with a high fever and accompanying
gastrointestinal symptoms such as diarrhea. Acquiring sputum that is diagnostic is
difficult and unreliable frequently with no bacteria on sputum staining. Diagnosis is
most commonly made by testing for Legionella antigen in the urine. Legionnaires
disease can cause a life threatening pneumonia if not recognized and treated
properly. The treatment of choice for this organism is erythromycin.
Legionella is a common contaminant of water and can be spread by inhalation of
aerosolized water from natural water sources, air conditioners and other water-
based cooling systems, as well as tap water used in the healthcare setting.
(Choice A) Mycoplasma pneumoniae is the etiologic agent in walking pneumonia.
This is an illness that predominantly affects young adults causing a tracheobronchitis
or pneumonia. Patients often have a mild fever and mild transient anemia as well as
a cough. Chest radiography will show diffuse bilateral infiltrates and give the
impression of a pneumonia that is much more severe than the patient appears
clinically. Mycoplasma pneumoniae would also not reveal organisms on a Gram-stain
of the sputum because these organisms do not have a peptidoglycan cell wall.
(Choice B) Klebsiella pneumoniae is a Gram-negative rod and a major cause of
nosocomial pneumonia as well as pneumonia in alcoholic or otherwise debilitated
patients. It would reveal Gram-negative rods on sputum microscopy.
(Choice C) S. pyogenes is the etiologic agent of streptococcal pharyngitis, impetigo
and abscess.
(Choice E) Mycobacterium kansasii is an atypical mycobacterium that can cause
disease similar to that caused by M. tuberculosis. M. kansasii is an acid-fast rod that
is similar to L. pneumophila in that it is a water contaminant, but is frequently
contaminates municipal drinking water systems.
(Choice F) Coccidioides immitis is a fungal organism that is the etiologic agent of
“San Joaquin Valley Fever.” Coccidioidomycosis is frequently an asymptomatic
infection, but it can also cause severe disseminated disease and fungemia in
immunocompromised patients.

17
USMLE WORLD STEP 1 MICROBIOLOGY

Educational Objective:
Legionella pneumophila causes Legionnaires disease. Legionnaire’s disease is
characterized by a propensity to cause disease in smokers consisting of very high
fever, diarrhea, headache and confusion. Labs show hyponatremia. L. pneumophila
is a Gram-negative rod.

18
USMLE WORLD STEP 1 MICROBIOLOGY

Q NO 116: In a hypothetical population that is universally vaccinated with


recombinant HBsAg, which of the following diseases would almost entirely
disappear? A.
Hepatitis C infection
B. Delta agent infection
C. Hepatocellular carcinoma
D. Membranous glomerulopathy
E. Mixed cryoglobulinemia
F. Fulminant viral hepatitis

Explanation:
Often referred to as the delta agent or the hepatitis delta virus, hepatitis D virus is a
replication-defective RNA virus that is only capable of causing infection when
encapsulated with HBsAg. Therefore, HDV infection can arise either as an acute
coinfection with hepatitis B virus (with the HBV established first to provide the
HBsAg for the HDV) or as a superinfection of a chronic HBV carrier. In a population
that is universally vaccinated with recombinant HBsAg, then the hepatitis D virus
would not be able to replicate and would cease to be a significant threat.
(Choice A) Hepatitis C infection is not associated with HBsAg or hepatitis B infection;
therefore, vaccination against HBV would not have an impact on HCV incidence.
(Choice C) Hepatocellular carcinoma can arise secondary to numerous causes,
including both hepatitis B infection and hepatitis C infection. A population that is
universally vaccinated against hepatitis B infection would have a lower incidence of
hepatocellular carcinoma but this cancer would not disappear altogether.
(Choices D and E) Chronic hepatitis C infection has been associated with
extrahepatic disease in as many as 38% of infected patients. Membranous
glomerulopathy and mixed cryoglobulinemia are two examples of common
extrahepatic disease manifestations in HCV-positive patients. Vaccination against
HBV would therefore not have an impact on the incidence of these diseases.
(Choice F) Fulminant viral hepatitis while rare can arise with any of the hepatitis
viruses (A. B, C, D, or E). Therefore vaccination against only HBV will not eliminate
the incidence of fulminant viral hepatitis.

Educational Objective:
Often referred to as the delta agent or the hepatitis delta virus hepatitis D virus is a
replication-defective RNA virus that is only capable of causing infection when
encapsulated with HBsAg.

19
USMLE WORLD STEP 1 MICROBIOLOGY

Q NO 117: A 44-year-old male presents to the emergency room complaining of


severe headache, nausea, and fever. Lumbar puncture and cerebrospinal fluid
(CSF) examination reveal the following:
Opening pressure 300 cm H20
Glucose 20 mg/dL
Protein 200 mg/dL
RBC 4/mm3
WBC 360/mm3
Neutrophils 90%
Lymphocytes 10%
CSF gram stain would most likely reveal:

A. Bean-shaped Gram negative cocci in pairs


B. Lancet-shaped Gram positive cocci in pairs
C. Gram positive cocci in clusters
D. Non-motile Gram negative coccobacilli
E. Motile Gram positive rods

Explanation:
A patient presenting with headache, fever and nuchal rigidity should immediately
raise suspicion for meningitis. In bacterial meningitis CSF analysis typically shows
elevated neutrophils, decreased glucose, and elevated protein. The morphology of
any observed bacteria on CS Gram stain provides an excellent preliminary
identification of the pathogen (whereas CSF cultures take time to grow).
Streptococcus pneumoniae is the most common cause of bacterial meningitis in
adults. On CSF Gram stain, one would see lancet-shaped Gram positive cocci in
pairs. S. pneumoniae meningitis often follows a pulmonary infection or mild upper
respiratory infection. Alcoholics, sickle cell anemia patients, asplenic individuals, or
those in generally poor health are at risk for meningitis caused by S. pneumoniae.
(Choice A) Bean-shaped Gram negative cocci in pairs are the typical morphology
observed with Neisseria meningitidis. N. meningitidis is the second most common
cause of meningitis in patients less than 60 years of age after S. pneumoniae. N.
meningitidis tends to occur in outbreaks where many individuals are living in close
quarters, for example, army barracks and college dormitories. It can be part of the
normal nasopharyngeal flora, with a carrier rate of approximately 5%.
(Choice C) Gram positive cocci in clusters are the typical morphology observed with
Staphylococcus species. Staphylococci are an unusual cause of meningitis.
Staphylococci meningitis occurs most often in neurosurgical patients. (It is thought
that bacteria gain access to the meninges by direct inoculation during the procedure
or postoperatively via the wound.)
(Choice D) Haemophilus influenzae is a Gram negative coccobacillus. H. influenzae
infection is becoming less and less common now that vaccination with the Hib
capsule vaccine, a protein-capsule polysaccharide conjugate vaccine, has become
widespread. Previously, H influenzae was a major cause of meningitis in very young
children, with a mortality rate of approximately 5% and frequent sequelae including
learning delays and mental retardation. It is also the etiologic agent for acute
epiglottitis.
(Choice E) Listeria monocytogenes has the morphologic appearance of motile Gram
positive rods. It is a facultatively intracellular bacterium, the only Gram positive
bacterium to produce lipopolysaccharide endotoxin. Listeria is the third most
common cause of meningitis in neonates, after Group B Streptococci and E coli.
Listeria also causes meningitis in immunosuppressed patients.

20
USMLE WORLD STEP 1 MICROBIOLOGY

Educational Objective:
Bacterial meningitis causes an increase in CSF neutrophil count and protein
concentration as well as a decrease in CS glucose. Streptococcus pneumoniae is a
leading cause of community acquired pneumonia otitis media, and meningitis in
adults. S. pneumoniae appears as lancet-shaped Gram positive cocci in pairs on
Gram stain.

21
USMLE WORLD STEP 1 MICROBIOLOGY

Q NO 118: Gram-negative coccobacilli isolated from the CSF of an infant with


meningitis demonstrate little growth on sheep agar but grow well once
incubated on the same media with Staphylococcus aureus. Staphylococci
promote the growth of the bacteria by supplementing:

A. ATP
B. Catalase
C. NAD+
D. Pyruvate
E. Iron

Explanation:
Subject: Haemophilus influenzae growing around streak of Staphylococcus aureus

Subject: Haemophilus influenzae on chocolate agar plate


Haemophilus influenzae is a blood loving organism and requires both X factor
(exogenous hematin) and V factor (NAD+) to support growth. Furthermore, 5%
sheep blood agar plates not only lack sufficient nutrients to support the growth of
Haemophilus species, but they also do not allow the growth of Haemophilus species
because of the presence of V factor inactivating enzymes found in the media. Growth
of Haemophilus species can be achieved on 5% sheep blood agar by cross streaking
the medium with Staphylococcus aureus. Colonies of H. influenzae will grow around
the hemolytic S. aureus coon resulting in the characteristic “Satellite” phenomenon.
When the enzymes of beta hemolytic S. aureus lyse the red blood cells in the
medium X factor (hematin) is released, and V factor (NAD+, choice C) is actively
secreted by staphylococci into the growth medium. S. aureus thereby provides the X
and V factors necessary to support the growth of Haemophilus species in sheep
blood agar.
Because Haemophilus species are part of the normal flora of the upper respiratory
tract, itis critical to differentiate it from other bacteria through biochemical means.
Part of the identification criteria is to demonstrate that colonies suspected of being
Haemophilus species require X and V factor for growth.
(Choice B) Catalase is an enzyme present in the Staphylococci and the presence or
absence of this enzyme is used to differentiate the Staphylococci from the
Streptococci.

22
USMLE WORLD STEP 1 MICROBIOLOGY

(Choice D) Pyruvate is the end product of glycolysis and is a starting substrate for
synthesis of glucose, lactate, fatty acids, amino acids and nucleic acids. It is used in
all pathogenic bacteria in the metabolism of sugars for energy in the form of AlP.
(Choice E) Iron is an essential growth factor for many bacteria and is provided in all
blood agars from the iron within the erythrocytes.
(Choice F) Lactose fermentation as a source of energy is best known in E. coli with
the well studied lac-operon. Lac-operon is an inducible and repressible genetic
sequence in the E. coli genome that codes for the enzymes necessary for the
fermentation of lactose in the absence of glucose. Itis activated by a sensed glucose
deficit and repressed when glucose is again available.

Educational Objective:
H. influenza is a “blood loving” organism. Part of the laboratory identification process
of H. influenzae is demonstration of the requirement of X(hematin) and V (NAD+)
factors for growth. This can be accomplished by growing H. influenzae in the
presence of S. aureus and demonstrating the “satellite phenomenon” where H.
influenzae grow only near the beta-hemocytic S. aureus colonies because they
produce the needed X and V factors.

23
USMLE WORLD STEP 1 MICROBIOLOGY

Q NO 119: A 34-year-old HIV-positive male presents with sudden-onset fever,


chills, cough, and left-sided chest pain that worsens with deep breathing.
Physical examination reveals bronchial breath sounds over the left lower lung. A.
His most recent CD4 count was 800 cells/mm3. Which of the following
organisms is most likely responsible for this patient’s symptoms?
Streptococcus pneumoniae
B. Mycoplasma pneumoniae
C. Pneumocystis jiroveci
D. Staphylococcus aureus
E. Legionella pneumophila
F. Moraxella catarrhalis
G. Mycobacterium tuberculosis

Explanation:
This patient presents with signs and symptoms of a focal lobar communist acquired
pneumonia. Because he is HIV+, one might be tempted to think that the cause is an
AIDS related opportunistic pathogen. However the normal CD4+ lymphocyte count
for adults ranges between 400-1400 cells/mm3 counts <200 cells/mm3 are
associated with a significantly increased risk of opportunistic infection. This patient is
not severely immunocompromised at this stage of his HIV infection. In an
immunocompetent individual, pneumococcus is the most common cause of
community acquired pneumonia, accounting for around 7O% of cases.

Educational Objective:
The most common cause of community acquired pneumonia in immunocompetent
hosts (which would include an HIV+ patient whose CD4 count is >200 cells/mm3) is
Streptococcus pneumoniae.

24
USMLE WORLD STEP 1 MICROBIOLOGY

Q NO 120: A community hospital is experiencing an increased incidence of


nosocomial pneumonias. Most cases occur in patients with long
hospitalizations. The microorganism isolated in several cases is visualized A.
with silver stains and demonstrates slow growth on complex media such
as charcoal-yeast extract supplemented with cysteine. Which of the
following is most likely implicated in the increased incidence of nosocomial
pneumonias in the hospital?
Infection carriers among the hospital staff
B. Poor isolation of the infected patients
C. Failed sterilization of mechanical ventilators
D. Colonization of the hospital water system
E. Widespread use of antimicrobial agents
F. Widespread use of intravascular devices

Explanation:
The organism described in the question stem is Legionella pneumophila. L.
pneumophila requires special culture media to be grown in vitro. L. pneumophila is
traditionally cultured on media supplemented with L-cysteine, and the agar used is
buffered charcoal yeast extract agar. Outbreaks of L. pneumophila have been
associated with water colonized by the organism being aerosolized and subsequently
inhaled. Contaminated water used to humidify air either in a commercial or
residential setting or in the hospital setting can lead to infection. Tap water used in
patient care or to humidify air during mechanical ventilation can be a method of
transmission and water-based cooling systems are used in large buildings such as
hospitals, and this too could be contaminated with L pneumophila.
(Choice A) Infection carriers among the hospital staff is a frequent cause of
methicillin-resistant Staphylococcus aureus. Health care workers can be
asymptomatic nasal carriers of this organism and unwittingly pass this organism to
their patients resulting in wound infections and intravenous catheter-related
infections.
(Choice B) Poor isolation of the infected patients can result in the spread of many
infectious diseases in the hospital (C.difficile, MRSA and VRE).
(Choice C) Failed sterilization of mechanical ventilators can lead to nosocomial
pneumonia with E coli Klebsiella pneumoniae, Acinetobacter and Pseudomonas. A
large portion of nosocomial pneumonias occur in intubated patients.
(Choice E) Widespread use of antimicrobial agents leads to multi-drug resistant
organisms but does not predispose to the development of pneumonia itself.
(Choice F) Widespread use of intravascular devices does convey increased risk of
intravenous catheter-related bacteremia and sepsis but not pneumonia with
Legionella.

Educational Objective:
L. pneumophila commonly contaminates natural bodies of water municipal water
humidification systems and air conditioning and water-based cooling systems as are
used in many commercial applications. The organism is inhaled in aerosolized water
and establishes infection by the pulmonary route. It requires special enriched media
for growth: specifically it requires media supplemented with L-cysteine and culture
on charcoal-yeast extract agar.

25
USMLE WORLD STEP 1 MICROBIOLOGY

Q NO 121: A46-year-old male who just returned from a mission trip to Latin
America presents to your office complaining of fever myalgias, dull
abdominal pain, and a week-long history of watery diarrhea that has A.
recently become bloody. Physical examination reveals a fever of 38.9°C
(102.0°F), hepatosplenomegaly, and rose-colored spots at the periumbilical
area. Which of the following is the most likely cause of this patient’s
symptoms?
Escherichia coil
B. Klebsiella pneumoniae
C. Salmonella typhi
D. Salmonella arteritides
E. Campylobacter fetus
F. Shigella flexneri
G. Proteus mirabilis
H. Yersinia enterocolitica
I. Haemophilus influenzae
J. Helicobacter pylori
K. Vibrio cholerae
L. Streptococcus pyogenes
M. Streptococcus pneumoniae

Explanation:
Typhoid fever is common in most parts of the world except in industrialized regions
such as the United States, Canada, Western Europe, Australia, and Japan. In
developed nations typhoid fever is generally not suspected unless there is a history
of recent travel to areas where the disease is endemic. Over the past several years,
travelers to Asia, Africa, and Latin America have been especially at risk. Typhoid
fever (also referred to as “enteric fever”) is a life-threatening illness caused by the
bacterium Salmonella typhi or Salmonella paratyphi. Other species of Salmonella are
not associated with typhoid fever. Typhoid fever is a fecal-oral disease that begins
after ingestion of S. typhi or paratyphi.
These organisms penetrate the gut mucosa both via transporters on enterocytes and
via phagocytosis by M cells in Peyer’s patches. The organisms are then
phagocytosed by macrophages, within which Salmonella (para) typhi are specially
adapted to survive. Macrophages carry the infective organisms to the liver, spleen,
and bone marrow. Hepatosplenomegaly from organism growth ensues. From here,
these species are able to cause bacteremia and sepsis. Salmonella typhi and
paratyphi also colonize the gallbladder, which allows access to the gut lumen on a
virtually limitless basis.
Within the gut lumen, S. typhi and paratyphi do more than disseminate—they can
cause drastic inflammation within Peyer’s patches, causing intestinal hemorrhage as
well as potential gut perforation which can cause polymicrobial peritonitis and sepsis
the mechanisms by which typhoid fever can cause death. Patients who do not
experience fulminant disease are at risk for becoming chronic carriers of the
bacterium (such as Typhoid Mary) and can unknowingly affect dozens of other
people. S. typhi and paratyphi are shed from the bile into the stool. Contaminated
water and the handling of food with unwashed hands then allow the bacteria to be
ingested. There is no animal reservoir for this disease.
The common clinical picture of this disease is mild abdominal cramping with a low
fever and diarrhea OR constipation initially. Subsequently the patient can develop
salmon-colored “rose spots” on the abdomen develop hepatosplenomegaly and

26
USMLE WORLD STEP 1 MICROBIOLOGY

recolonization of the gut leading to hemorrhagic diarrhea and sepsis. None of the
other answer choices cause the clinical scenario mentioned above.

Educational Objective:
Cases of typhoid fever in the United States usually occur after a patient has traveled
to a location where the disease is endemic. Typhoid fever is caused by Salmonella
typhi and paratyphi and causes a clinical presentation of escalating fever with initial
diarrhea or constipation followed by hepatosplenomegaly, the formation of “rose
spots” on the abdomen and possible hemorrhagic enteritis with bowel perforation.

27
USMLE WORLD STEP 1 MICROBIOLOGY

Q NO 122: Healthy volunteers orally inoculated with pathogenic strains of


Clostridium difficile do not develop signs of infection. Which of the
following protective mechanisms is most likely responsible? A. Cell-
mediated immunity
B. Mucosal IgA antibodies
C. Gastric acidity
D. Intestinal biomass
E. Rapid gastrointestinal transit

Explanation:
Over 400 types of bacteria inhabit the healthy human gastrointestinal tract as part
of the normal gut flora. In healthy humans, the gastrointestinal tract has very few
aerobes (i.e. Pseudomonas), approximately 10% facultative anaerobes (Escherichia
coli, Klebsiella, Streptococcus Lactobacillus, Staphylococcus, and Bacillus), and a
majority of strict anaerobes (Bifidobacterium, Fusobacterium, Clostridium,
Eubacterium Peptococcus, and Peptostreptococcus). These intestinal bacteria
effectively suppress overgrowth of Clostridium difficile and many other potentially
pathogenic bacteria by competing for nutrients and adhesion sites within the gut.
Treatment with antibiotics can alter the intestinal balance of bacteria leading to a
potential overgrowth of pathogenic strains and clinical disease. C. difficile requires
prior antibiotic use to establish infection. It causes disease by releasing two toxins
that damage the mucosal lining of the large intestine leading to diarrhea (Toxin A)
and necrosis (Toxin B) with pseudomembrane formation.
(Choice A) Cell-mediated immunity is not essential for the prevention of C. difficile
infection, but is required to eradicate infections with organisms like Legionella,
Neisseria gonorrhoeae, Listeria monocytogenes, viruses, and protozoans like
Leishmania, which reside within cells, thus avoiding the humoral (antibody) immune
response.
(Choice B) Mucosal IgA antibodies prevent microbial attachment to host mucosal
surfaces by binding and interfering with the adhesion molecules on the
microorganism. Organisms like N. gonorrhoeae evade IgA by secreting an IgA
protease virulence factor that degrades IgA in mucosal secretions.
(Choice C) Gastric acidity degrades and for destroys many pathogens, but clostridial
spores can survive this environment and go onto germinate in the anaerobic
environment of the gut where their growth is kept in check by the normal flora.
(Choice E) Rapid gastrointestinal transit is rarely effective at “washing away”
pathogenic diarrhea-causing organisms, because these organisms are well-adapted
to adhere to the gut mucosa in the setting of voluminous diarrhea.

Educational Objective:
In absence of the normal intestinal microbial flora (as may be the case after a
course of antibiotics) Clostridium difficile can overgrow and produce enterotoxin
(toxin A) and cytotoxin (toxin B). Clinical disease resulting from C. difficile
overgrowth can range from transient diarrhea to severe pseudomembranous colitis.

28
USMLE WORLD STEP 1 MICROBIOLOGY

Q NO 123: Transient bacteremia is demonstrated in a healthy 22-year-old male


after a dental procedure. The Gram-positive bacteria isolated from the blood A.
synthesize dextrans from sucrose. Which of the following would be the most
likely adherence site for these bacteria?
Endothelial surface glycoproteins
B. Subendothelial collagen
C. Subendothelial glycosaminoglycans
D. Circulating heparins
E. Fibrin-platelet aggregates

Explanation:
Viridans streptococci are normal inhabitants of the oral cavity and cause transient
bacteremia after dental procedures. These organisms are capable of producing
extracellular polysaccharides (dextrans) using sucrose as a substrate. Dextrans
facilitate streptococcal adherence to fibrin. Fibrin and platelets are deposited at sites
of endothelial trauma, providing a site for bacterial adherence and colonization
during bacteremia leading to the formation of avalvular vegetation. Without
preexisting endothelial damage leading to fibrin and platelet deposition at the cusps
of valve leaflets the viridans streptococci are unable to adhere to the valve and
establish an infection leading to endocarditis. S. aureus, on the other hand, is a
more aggressive organism and can adhere to intact endothelium without a
preexisting lesion.
(Choice A) Endothelial surface glycoproteins mediate the binding of immune cells to
the endothelium. This is accomplished by the expression of cell adhesion molecules
on the surface of inflamed endothelium. Neutrophils are then able to bind and
migrate between the endothelial cells and are followed by monocytes, and
lymphocytes that bind the endothelium in a similar fashion.
(Choices B and C) Together with subendothelial collagen subendothelial
glycosaminoglycans form the subendothelial fibrous cap over the central core of an
atherosclerotic plaque. These substances are potent activators of platelets and when
exposed to the bloodstream by the denuding of the endothelium in an
atherosclerotic plaque they contribute to rapid thrombus formation that in a
coronary artery, can lead to myocardial infarction.
(Choice D) Circulating heparins prolong the partial thromboplastin time by activating
antithrombin III, thereby decreasing the activity of thrombin, clotting factors IX, X,
XI, and XII and ultimately preventing the conversion of fibrinogen to fibrin. The net
effect is the prevention of fibrin deposition and clot formation. Circulating heparins
would not be a binding site for bacteria; if any thing they would prevent the
deposition of fibrin required by some bacteria such as the viridans streptococci for
adherence to human tissue.
Educational Objective:

Viridans streptococci are normal inhabitants of the oral cavity and are a cause of
transient bacteremia after dental procedures in healthy and diseased individuals. In
order to cause subacute bacterial endocarditis they require preexisting valve damage
leading to the local deposition of fibrin required for them to adhere.

29
USMLE WORLD STEP 1 MICROBIOLOGY

Q NO 124: An infant born to a 27-year-old immigrant is evaluated for hearing loss.


Physical examination demonstrates white pupils and a continuous machinery
murmur over the left infraclavicular area. Which of the following prenatal
interventions would have most likely prevented this infant’s condition?

A. Live attenuated vaccine


B. Viral component vaccine
C. Killed vaccine
D. Toxoid vaccine
E. Viral enzyme inhibitor drug
F. Cellular receptor inhibitor drug
G. Interferons
H. Prenatal vitamins

Explanation:
The congenital rubella syndrome is predominantly characterized by neonatal defects
of the head (microcephaly, mental retardation), eyes (cataracts), ears (deafness),
and heart cardiovascular system (patent ductus arteriosus, peripheral pulmonic
stenosis). The most classic clinical triad of congenital rubella includes congenital
cataracts (white pupils) sensory-neural deafness, and patent ductus arteriosus. Live
attenuated rubella virus vaccine is currently recommended not only for children at
12-15 months and again atl-6 years of age but also in non-pregnant women of
childbearing age who lack serum antibody against rubella. At the time of vaccination
women are strongly advised to avoid pregnancy for the next four weeks. This
vaccination policy has markedly decreased the incidence of congenital rubella.
(Choices B and C) A viral component or killed vaccine is frequently less effective
than a live attenuated vaccine. The viral component or killed vaccines tend to induce
only humoral immunity, whereas the live attenuated vaccines induce both humoral
and cell-mediated immunity. One risk with the live vaccines is the potential for
reversion to virulence. Hepatitis A and rabies vaccines are examples of killed
vaccines, whereas measles, mumps, and rubella vaccines are examples of live
vaccines.
(Choice D) A toxoid maybe prepared by treatment of the toxin with formalin or heat.
The toxoid can then induce humoral immunity against the toxin without producing
serious toxic effects in the vaccinee. Because the pathogenesis of rubella infection is
not predominantly mediated by a viral toxin, toxoid is ineffective for this disease.
(Choice E) Reverse transcriptase and protease inhibitors are used for HIV infection.
(Choice F) Enfuvirtide is a fusion inhibitor approved for HIV infection. No drugs are
available for the treatment of rubella at this time.
(Choice G) Leukocyte EN-a is a glycoprotein that inhibits various stages of viral RNA
and DNA synthesis. It is currently approved for the treatment of hepatitis B and C
virus infection, hairy cell leukemia, condyloma acuminatum, and Kaposi’s sarcoma.
It is not effective in the treatment of rubella infection.

Educational Objective:
The congenital rubella syndrome is predominantly characterized by neonatal defects
of the head (microcephaly, mental retardation), eyes (cataracts), ears (deafness),
and heart/cardiovascular system (patent ductus arteriosus, peripheral pulmonic
stenosis). To decrease the incidence of this syndrome, the CDC currently
recommends the vaccination of children and non-pregnant females of childbearing
age with live, attenuated rubella virus vaccine.

30
USMLE WORLD STEP 1 MICROBIOLOGY

Q NO 125: A 24-year-old male presents to your ER with severe nausea and


vomiting two hours after consuming a mayonnaise- containing salad at an
outdoor picnic. His condition improved significantly in several hours. You A.
inform the patient that his symptoms were caused by which of the
following?
Exotoxin formed prior to ingestion
B. Exotoxin formed after ingestion
C. Endotoxin
D. Bacterial invasion of gut mucosa
E. Parasitic Infection

Explanation:
Staphylococcus aureus is capable of producing a highly heat-stable protein toxin
called enterotoxin that causes staphylo enterotoxemia, a syndrome characterized by
nausea, vomiting and abdominal cramps following ingestion of preformed exotoxin.
S. aureus contaminates food by direct inoculation from food handlers who are
frequently asymptomatic carriers. Subsequently the food is allowed to sit at room
temperature for an extended period of time (i. e. at a picnic / pot luck), and S.
aureus is able to multiply and produce exotoxin. This preformed exotoxin is then
ingested by the patient resulting in symptoms. Because this is a preformed exotoxin,
there is no person-to-person transmission, but outbreaks can occur with many
people eating the same contaminated food. It is also the reason for the rapid onset
of symptoms (usually less than 6 hours).
Poultry and egg products, meat and meat products, egg, tuna, chicken, potato and
macaroni salads, cream-filled pastries, and milk and dairy products are the foods
that are frequently incriminated in staphylococcal food poisoning. The most
frequently tested food item is a mayonnaise-containing food like potato or macaroni
salad.
A similar clinical picture of rapid-onset nausea, vomiting, abdominal cramping and
retching can be seen in food poisoning caused by Bacillus cereus which also
contaminates food with a preformed heat-stable exotoxin. B. cereus
characteristically contaminates starchy foods such as rice (reheated fried rice).
(Choice B) Examples of food poisoning resulting from exotoxin formed after
ingestion of the contaminated food include Enterotoxigenic E. coli (ETEC) and V/brio
cholera that cause watery diarrhea, and Enterohemorrhagic E. coli (EHEC) and
Shigella, which cause inflammatory diarrhea.
(Choice C) All of the Enterobacteriaceae contain the endotoxin Lipid A, a component
of their lipopolysaccharide outer membranes. It is released in small amounts by
normally dividing Gram-negative bacteria, but it can be released in large amounts
during widespread bacteriolysis as with the initiation of antibiotic therapy in a
patient with Gram- negative sepsis.
(Choice D) Examples of bacteria that invade the gut mucosa are Salmonella species,
Shigella Yersinia enterocolitica, and Enteroinvasive E. coli (EIEC) and Campylobacter
jejuni. These organisms can cause bloody and/or inflammatory diarrhea as well as
systemic illness as in the case of Salmonella typhi and Yersinia enterocolitica.
(Choice E) Examples of parasites causing enteritis include Entamoeba histolytica
(bloody diarrhea and liver abscesses), Giardia lamblia (a protozoan that causes non-
bloody, putrid-smelling, frothy diarrhea and bloating most commonly contracted
after drinking pond or stream water on a camping trip), and Cryptosporidium
(massive watery diarrhea in HIV patients).

31
USMLE WORLD STEP 1 MICROBIOLOGY

Educational Objective:
Diseases caused by “exotoxin” release by S. aureus are:
1. Toxic shock syndrome
2. Staphylococcal scalded syndrome
3. Gastroenteritis
Staphylococcal food poisoning occurs after a food handler inoculates food (usually a
mayonnaise containing product on Step tests) with S. aureus that is allowed to
incubate at room temperature producing heat-stable exotoxin that causes rapid-
onset nausea, vomiting and abdominal cramping.

32
USMLE WORLD STEP 1 MICROBIOLOGY

Q NO 126: A 23-year-old heterosexual male with two month history of fatigue and
malaise is found to have tender hepatomegaly. His past medical history is
insignificant. He denies any contact with individuals having similar symptoms or
any recent travel. He smokes 2 packs a day and consumes 1-2 bottles of beer on
weekends. He does not use illicit drugs. He admits to having several episodes of
unprotected sex during the past year. Based on the risk factor profile which of the
following is most likely to be present in this patient?

A. HAV in the stool


B. Serum anti-HAV lgM
C. Serum HBsAg
D. Serum Anti-HBsAg lgG
E. HCV DNA in liver biopsy tissue
F. Serum anti-HDV IgG

Explanation:
Hepatitis B virus can be detected in all bodily fluids with the exception of stool. While
blood is the primary means of HBV transmission, the virus can also be spread by
exposure to semen, saliva, sweat, tears, breast milk, and pathologic effusions.
Individuals at highest risk for contracting hepatitis B virus include intravenous drug
abusers, men who have sex with men, health-care workers subject to needle-stick
accidents patients on dialysis and blood product recipients. Therefore, although this
heterosexual patient would not typically be considered “high risk.” his clinical
presentation suggests an acute hepatitis B infection contracted during an episode of
unprotected sex.
(Choices A and B) As hepatitis A virus is transmitted via the fecal-oral route HAV in
the stool or serum anti-HAV 1gM would be expected in an individual from an
endemic area (eg, Mexico) or who consumed tainted insufficiently heated food (eg,
oysters).
(Choice D) Serum anti-HBsAg lgG appears after either successful HBV vaccination or
the clearance of HBsAg. This marker remains detectable for life and serves as an
indicator of non infectivity and immunity. Since this patient is symptomatic he has
obviously not cleared the virus from his system and would therefore not have anti-
HBsAg present yet in circulation.
(Choice E) HCV DNA would be expected in the liver biopsy tissue of an individual
with classic exposure history (eg, intravenous drug abuse). The primary routes of
HCV transmission include inoculations and blood transfusions with sexual
transmission far less common.
(Choice F) Transmission of the hepatitis D virus occurs primarily through inoculations
and blood transfusions. with sexual transmission far less common. Moreover for an
individual to become infected with HDV, co-infection with HBV or superinfection of
chronic HBV is necessary.

Educational Objective:
Blood is the primary means of HBV transmission but the virus can also be spread by
exposure to semen saliva sweat, tears breast milk and pathologic effusions.

33
USMLE WORLD STEP 1 MICROBIOLOGY

Q NO 127: A 23-year-old male is being treated for presumed Mycoplasma


pneumonia with erythromycin. He demonstrates mild anemia and
elevated serum LDH level. Two months later the patient has no anemia. A.
The absence of anemia in this patient at this point is best explained by:
Elimination of intracellular bacteria
B. Replenished intrinsic factor
C. Faded immune response
D. Replenished intracellular enzyme stores
E. Replenished body iron stores

Explanation:
Clinical sequelae of M. pneumonia infection can go beyond the airways. M.
Pneumoniae infection can cause hemolysis due to antigenic similarity between
antigens in the cell membrane of M. pneumoniae and in the cell membrane of
erythrocytes. When the immune system mounts a response against these M.
pneumoniae antigens it also destroys some RBCs resulting in a mild anemia. These
antibodies that cross-react between M. pneumoniae and RBCs are called “cold-
agglutinins” because they are able to agglutinate RBCs in vitro at low temperatures.
After the infection has been eliminated and the immune system is no longer
activated against M. pneumoniae, the concentration of these antibodies decreases
and the anemia spontaneously resolves. M. pneumoniae can also cause the Stevens-
Johnson syndrome and joint pains among other rare sequelae.
(Choice A) Elimination of intracellular bacteria would have no effect on M.
pneumoniae as this is an extracellular organism. Functional cell-mediated immunity
is required for the elimination of intracellular organisms. Notable intracellular
organisms include Salmonella, Neisseria gonorrhoeae, Chlamydia (obligate), and
Rickettsia (obligate).
(Choice B) Intrinsic factor deficiency is not seen with Mycoplasma infections.
(Choice D) Enzyme deficiencies that cause anemia include glucose-6-phosphate
Dehydrogenase (G6PD) deficiency and pyruvate kinase deficiency. Drugs can induce
hemolysis in patients with a G6PD deficiency if they use drugs that cause oxidative
stress such as sulfonamides and anti-malarial agents (as antimicrobial examples of
drugs causing oxidative stress).
(Choice E) Replenished body iron stores would cause a resolution of anemia if the
anemia was due to pure iron deficiency. The most common cause of iron deficiency
anemia in adults is blood loss.

Educational Objective:
Mycoplasma pneumoniae is the causative agent in walking pneumonia and many
cases of tracheobronchitis. It is an organism with no peptidoglycan cell wall; it only
has a phospholipid bilayer cell membrane. It shares antigens with human
erythrocytes, and when the body mounts a response against these antigens it also
lyses red blood cells leading to anemia. The antibodies causing this RBC destruction
are referred to as cold agglutinins.

34
USMLE WORLD STEP 1 MICROBIOLOGY

Q NO 128: Epidemiological analyses show a significant increase in staphylococcal


infections (bacteremia and sepsis) in several large city hospitals over the last 20
years. This increase in the infection incidence most closely correlates with the
increased use of:
A. Prophylactic antibiotics
B. Cyclosporine immunosuppression
C. Intravascular devices
D. Laparoscopic surgeries
E. Blood transfusions

Explanation:
Intravascular catheters are indispensable in modern-day medical practice. Although
these devices provide necessary vascular access, their use puts patients at risk for
local and systemic infections. A total of 50000 to 20000 central venous catheter-
associated bloodstream infections have been estimated to occur annually in the US.
The most common pathogens isolated from hospital-acquired bloodstream infections
are: coagulase-negative Staphylococci, S. aureus, Enterococcus, Escherichia co/i,
Enterobacter, Pseudomonas aeruginosa, Klebsiella pneumoniae, and Candida.
(Choice A) Prophylactic antibiotics administered prior to surgical procedures
decrease the incidence of wound infections. The prophylaxis augments host defense
mechanisms at the time of bacterial invasion by decreasing the size of the inoculum.
Traditionally, a first-generation cephalosporin is used within 2 hours of the first
incision; this will provide good coverage for much of the normal skin flora.
(Choice B) Cyclosporine immunosuppression is frequently used in patients with solid
organ transplants. Cyclosporine is a calcineurin inhibitor and exerts its
immunosuppressive effect primarily by preventing activation of T-helper cells.
Though infection is a frequent side effect of immunosuppression in transplant
recipients, appropriate antimicrobial agents can usually control the infections. The
relative contribution of cyclosporine use to increased bacteremia is significantly less
than that of intravenous catheters.
(Choice D) Infection is a common complication of any surgical procedure, but the
rate of infection is very low in laparoscopic surgery. Additionally, preoperative
antibiotics reduce the incidence of operative wound infections.
(Choice E) Blood and blood components can be contaminated, but the incidence of
this is extremely low as these products are thoroughly screened and prepared prior
to reaching the blood bank. This is certainly not the reason for increased bacteremia.
The risk in the United States of contracting HIV from a blood transfusion is 1 in
1,000,000 units transfused, and the risk of contracting Hepatitis C is 1 in 100,000
units transfused.

Educational Objective:
A total of 50,000 to 120,000 central venous catheters-associated bloodstream
infections have been estimated to occur annually in the US. Indwelling central
catheters predispose to bacteremia and sepsis and should be monitored regularly for
signs and symptoms of infection and suspected in hospitalized patients with new-
onset fever or bacteremia.

35
USMLE WORLD STEP 1 MICROBIOLOGY

Q NO 129: Human influenza virus is known to participate in genetic reassortment


when exposed to animal orthomyxoviruses. Which of the following viruses is most
likely to undergo a similar process?
A. Poliovirus
B. Human papilloma virus
C. Rotavirus
D. Measles virus
E. Hepatitis C virus

Explanation:
Genetic shift in the influenza A virus involves reassortment of genomic segments of
the human strain with genomic segments of the animal (particularly avian) strain
when both viruses coinfect pigs. The resulting new strain can cause a new influenza
pandemic if it demonstrates dramatically altered viral surface glycoproteins (eg,
hemagglutinin). The key structural feature that enables influenza viruses to undergo
such relatively rapid genetic shifts through reassortment is the segmented nature of
their genomes. Members of another virus family with a segmented genome could
also undergo such reassortment—as opposed to the less radical, slower genetic drift
that occurs in all viral families via point mutations. Among the viruses listed above,
only rotavirus also has a segmented genome.
(Choices A, B, D, and E) Poliovirus, human papilloma virus, measles virus, and
hepatitis C virus all lack the segmented genome necessary to undergo reassortment.

Educational Objective:
Viruses with segmented genomes (eg, orthomyxoviruses and rotaviruses) are
capable of genetic shifts through reassortment. Reassortment involves exchange of
entire genome segments, a far more dramatic process than the point mutations
responsible for genetic drift.

36
USMLE WORLD STEP 1 MICROBIOLOGY

Q NO 130: A 32-year-old Caucasian male undergoing treatment for Hodgkin’s


lymphoma experiences fever, dyspnea and malaise. Blood cultures incubated
for 3 hours at 37°C reveal the following findings:

Which of the following is the most likely cause of this patient’s condition?

A. Rhizopus species
B. Aspergillus fumigatus
C. Candida albicans
D. Cryptococcus neoformans
E. Histoplasma capsulatum
F. Coccidioides immitis

Explanation:
Candida fungi exist in yeast (single cell) form. On light microscopy, they appear as
budding yeast with pseudohyphae. The germ tube test is diagnostic of Candida
albicans and allows it to be distinguished from other members of the Candida family
(such as C. tropicalis and C. glabrata). Inoculation of Candida albicans into serum at
37°Cfor3 hours leads to the formation of true hyphae from the yeast as shown in the
microscope image above. These growing hyphae are called “germ tubes.”
Candida species are a part of the normal human skin and mucous membrane flora.
Candida can cause localized disease in immunocompetent people with decreased
local defenses. Potential manifestations include:
1. Oral thrush. White patches form on the oral mucosa in patients who wear
dentures, frequently use antibiotics or inhaled steroids, or have diabetes mellitus.
They may be asymptomatic or experience burning pain.
2. Vulvovaginal candidiasis. Patients complain of vulvar pruritus and vaginal
discharge. Physical examination reveals vulvar erythema and white thick vaginal
discharge. This condition is associated with antibiotic use diabetes mellitus and
contraceptives.
3. Cutaneous candidiasis can occur in areas exposed to heat or high humidity (dish
washers hands infants groins). Signs include erythema, a vesiculopapular rash
maceration and fissuring.
In immunocompromised patients Candida can also cause disseminated disease that
may affect any organ system. Potential manifestations include pneumonia
esophagitis right-sided endocarditis, abscesses and candidemia (sepsis).
(Choices A and B) Rhizopus and Aspergillus exist only in mold form (true hyphae).
Germ tubes are not seen.

37
USMLE WORLD STEP 1 MICROBIOLOGY

(Choice E) Histoplasma capsulatum is a dimorphic fungus that exists in yeast form at


body temperature. Microscopic examination of tissue specimens would reveal oval
yeast cells within macrophages. Histoplasma does not form germ tubes.
(Choice F) Coccidioides immitis is a dimorphic fungus that forms molds at 25°C but
exists as spherules in human tissues. Spherules are covered by a capsule and
contain endospores. Germ tubes are not formed.

Educational Objective:
Candida albicans is the most common cause of opportunistic mycosis. It can affect
any organ and cause generalized candidemia. Yeasts and pseudohyphae on light
microscopy and a positive germ tube test are diagnostic of Candida albicans
infection.

38
USMLE WORLD STEP 1 MICROBIOLOGY

Q NO 131: Histopathology of an exudate from a patient with a localized skin


infection demonstrates gram-positive cocci in clusters. The defensive protein A.
bound to this organism’s peptidoglycan cell wall does which of the following?
Cleaves IgA
B. Binds the Fc portion of IgG
C. Interacts with MHC type II antigens
D. Causes hemolysis
E. Activates complement

Explanation:
Seeing gram positive cocci in clusters on gram stain indicates the presence of a
Staphylococcus species. Protein A is part of the outer peptidoglycan layer of
coagulase positive Staphylococcus species (i.e. Staphylococcus aureus). Protein A
binds non specifically with the Pc portion of lgG at the complement-binding site thus
preventing the activation of complement. This results in decreased production of C3b
leading to impaired opsonization.

(Choice A) Cleavage of IgA is accomplished by an IgA protease that is a virulence


factor for some isolates of Streptococcus pneumoniae. Streptococci appear
microscopically as gram positive cocci in pairs or chains.
(Choice C) Interaction with MHC type II on an antigen presenting cell is a proper of
both normally processed antigen as well as of superantigens. MHC presents antigens
to T-lymphocytes in order to initiate an immune response. Superantigens, however,
are different in that they are never internalized or processed by the antigen
presenting cell and they do not interact with the standard antigen binding sites on
the MHC II molecule or the T-cell receptor. Though superantigens ARE proteins
synthesized by Staphylococci, they ARE NOT bound to the peptidoglycan cell wall.
(Choice D) Hemolysin, secreted by Staphylococci is able to produce hemolysis as
well as the destruction of neutrophils, macrophages and platelets. This is a secreted
factor and is not bound to the cell wall.
Educational Objective: Protein A, characteristic for S. aureus, binds to the c portion
of lgG at the complement-binding site, thus preventing the activation of
complement. This results in decreased production of C3b leading to impaired
opsonization.

39
USMLE WORLD STEP 1 MICROBIOLOGY

Q NO 132: An outbreak of water-born gastroenteritis reported in Latin America is


caused by oxidase-positive, gram-negative, comma-shaped rods that can survive
on alkaline media. Stool microscopy in these patients is expected to
demonstrate:
A. Mucus and some epithelial cells
B. Many leukocytes, neutrophils predominant
C. Many leukocytes, eosinophils predominant
D. Many leukocytes, lymphocytes predominant
E. Many erythrocytes and some leukocytes

Explanation:
The clinical case describes infection with Vibrio cholerae. V. cholerae are oxidase-
positive, gram-negative comma- shaped rods that are able to grow on alkaline
enrichment medium that kills most organisms of the normal flora of the gut. This
morphology may be confused with infection by Campylobacter jejuni, one of the
most common causes of diarrhea worldwide which is a curved motile gram-negative
oxidase positive rod but this organism is unable to survive alkaline enrichment.
Diarrheal pathogens either produce disease through the production of enterotoxins
or through damage the intestinal epithelium. Cholera does not cause disease by
invading and does not enter the bloodstream. Cholera toxin increases levels of cAMP
by increasing the activity of adenylate cyclase in intestinal mucosal cells by a
mechanism identical to that of the heat labile toxin produced by Enterotoxigenic E.
coli (ETEC). Increased intracellular cAMP causes increased efflux of sodium and
chloride from the intestinal epithelial cells and decreased reabsorption of these ions.
This leads to massive water loss and watery diarrhea. Because V. cholerae does not
invade the mucosa or cause enterocyte cell death no leukocytes or erythrocytes will
be visualized on stool microscopy and only epithelial cells and mucous will be seen
(choice A).
(Choice B) Many leukocytes, neutrophils predominant would be typical of an
invading bacterium such as Salmonella species. Salmonella enteritidis causes
invasive diarrhea most frequently without hemorrhage.
(Choice C) Diarrhea with PERIPHERAL eosinophilia can be indicative of a number of
diseases. Infectious causes of this condition include many of the intestinal parasites
such as Strongyloides, Ancylostoma, Ascaris, Toxicara, Trichura, and Trichinella. It is
not commonly associated with Giardia or Entamoeba.
(Choice E) Many erythrocytes and some leukocytes would be typical of an invading
organism such as EIEC or Shigella. Erythrocytes in the stool without pus would
indicate a hemorrhagic organism such as E. coli 0157:H7 (EHEC) which does not
invade but does cause toxin-mediated enterocyte death and intestinal mucosal
hemorrhage and necrosis. Because this organism does not invade it does not elicit a
response from WBCs.

Educational Objective:
Vibrio cholerae along with Enterotoxigenic E. coli (ETEC) cause a purely toxin-
mediated watery diarrhea. The toxins secreted by these organisms do not cause cell
death; they modify electrolyte handling by enterocytes, so no blood or pus
(leukocytes) is noted on stool microscopy during infection by these organisms.

40
USMLE WORLD STEP 1 MICROBIOLOGY

Q NO 133: A 67-year-old male is hospitalized with low-grade fevers fatigue and a


diastolic murmur at the left sternal border. Blood cultures reveal Gram positive
cocci that are catalase-negative and able to grow in 6.5% saline. This patient’s
medical history is most likely to reveal which of the following procedures in the
past month?
A. Dental extraction
B. Skin biopsy
C. Sinus drainage
D. Nasal polypectomy
E. Cystoscopy

Explanation:
Enterococcal endocarditis usually occurs in elderly men who have recently
undergone manipulation of areas colonized by this organism, such as the GI or GU
tracts. In women enterococcal endocarditis can occur following obstetrical
procedures. Enterococci are Lancefield Group D streptococci, Gram positive cocci
that can be cultured in hypertonic (6.5%) saline and bile. These bacteria are
gamma-hemolytic.
The enterococci are resistant to many antibiotics and can be very difficult to treat.
Penicillin is often combined with an aminoglycoside for a synergistic effect, but
increasingly bacteria resistant to both aminoglycosides and β-lactams (including
penicilli nase-resistant types) are emerging. Additionally, vancomycin-resistant and
linezolid-resistant enterococci have emerged.
(Choice A) Dental extraction is associated with endocarditis caused by S. viridans, a
Gram positive coccus. In most cases, S. viridans causes subacute bacterial
endocarditis in already abnormal heart valves (e.g. congenital valvular abnormalities
valves damaged by rheumatic fever.)
(Choice B) Skin biopsy is a benign procedure that does not cause endocarditis. Skin
is colonized with staphylococcus (both S. aureus and S. epidermidis),
Corynebacterium species and Pseudomonas.
(Choice C) Sinus infections are usually caused by Streptococus pneumoniae (most
often) Haemophilus influenzae, or Moraxella catarrhalis. S. pneumoniae is a yew rare
cause of endocarditis and would not grow in hypertonic saline.
(Choice D) Removal of a nasal polyp can cause transient bacteremia. Organisms that
colonize the nose include Corynebacterium species Haemophilus species
staphylococci streptococci and Neisseria species. Of these staphylococci and
streptococci may cause endocarditis but the organism described in the question
stem displays characteristics of Enterococcus.

Educational Objective:
Enterococcus is a component of the normal colonic flora capable of growing in
hypertonic (6.5%) NaCl and bile. Urine in the bladder is usually sterile but can be
contaminated by instruments or catheters passed through the outer part of urethra.
Contaminating organisms include colonic flora (which are also frequently found on
the perineum and skin surrounding the genitals) such as enterococcus. Thus
genitourinary manipulation has been known to cause enterococcal endocarditis.

41
USMLE WORLD STEP 1 MICROBIOLOGY

42
USMLE WORLD STEP 1 MICROBIOLOGY

Q NO 134: A 2-day-old newborn develops lethargy and respiratory distress. Blood


cultures grow beta-hemocytic Gram-positive cocci in chains that are bacitracin-
resistant. Which of the following measures could have prevented this patient’s
condition most effectively?
A. Penicillin at 30 weeks gestation
B. Maternal vaccination
C. Intrapartum ampicillin
D. Postnatal immunoglobulin
E. Breastfeeding restriction

Explanation:
The finding of Gram-positive cocci in chains indicates Streptococcus as Staphylococci
classically form clusters. Group A Streptococci and Group B Streptococci are beta-
hemocytic, but bacitracin resistance excludes S. pyogenes (GAS) and indicates S.
agalactiae (GBS). The 2002 guidelines for perinatal group B strep prevention
recommend universal prenatal screening for group B streptococcal colonization by
maternal vaginal and rectal culture at 35-37 weeks gestation. In women who culture
positive for GBS or in women who have had an infant affected by GBS in the past,
intrapartum antibiotic prophylaxis is indicated to prevent neonatal GBS sepsis,
pneumonia and meningitis. The incidence of group B streptococcal disease in babies
less than a week old is declining due to these recommendations.
Penicillin remains the first line agent for intrapartum antibiotic prophylaxis, with
ampicillin an acceptable alternative.
(Choice A) Use of penicillin G at 30 weeks gestation would serve to eliminate GBS at
that time and likely for the next few weeks in the expecting mother. However, after
approximately4 weeks depending on the dose administered, she is again vulnerable
to colonization with GBS. Treatment with antibiotics this early in pregnancy is
ineffective for GBS prophylaxis.
(Choice B) Theoretically immunization of women during or before pregnancy could
prevent peripartum maternal disease and protect infants from prenatally acquired
infection by transplacental transfer of protective lgG antibodies. This would eliminate
the need for prenatal GBS screening and intrapartum antimicrobial prophylaxis in
women with a clear vaccination history. Serotype-specific antibodies to GBS capsular
polysaccharide have been shown to protect against disease, but as yet a vaccine is
not available against GBSI so this can not be the correct answer. Vaccines are
currently being developed against the capsular polysaccharide of GBS.
(Choice D) Administration of postnatal immunoglobulin in newborns suffering from
GBS infections has been studied with no true demonstration of efficacy. It is not
widely used in clinical practice.
(Choice E) Breast feeding should not be restricted under normal circumstances
because the human milk, especially the colostrum, provides some mucosal immune
protection to the newborn. Additionally, breastfeeding is recommended by the
American Academy of Pediatrics as the sole source of nutrition for all infants for the
first six months due to its superior nutritional content, its ability to promote the
proper development of the infant GI tract and the immune protection afforded by
agents such as IgA in human breast milk.
Educational Objective:
Universal prenatal screening for group B strep colonization by vaginal-rectal culture
at 35-37 weeks gestation is recommended to identify colonized women who require
INTRAPARTUM antibiotics, most frequently with penicillin or ampicillin, to prevent
neonatal GBS sepsis, pneumonia and meningitis.

43
USMLE WORLD STEP 1 MICROBIOLOGY

Q NO 135: A 58-year-old male presents to the ER with fever, headache and


vomiting. His past medical history is significant for a recent kidney
transplantation. Lumbar puncture reveals CSF pleocytosis and a normal CSF A.
glucose concentration. Gram positive rods with tumbling motility at room
temperature are seen on CSF microscopy. How this patient’s infection was
most likely transmitted?
Unprotected sex
B. Contaminated food
C. Respiratory droplets
D. Arthropod bite
E. Needle stick

Explanation:
The scenario described is classic for Listeria meningitis. An organ transplant
recipient, the patient in the vignette is almost certainly taking immunosuppressive
anti-rejection medications. In adults, listeriosis occurs almost exclusively in the
immunocompromised. The bacteria most commonly gain access to the bloodstream
following ingestion of contaminated food. Unpasteurized milk and milk products
undercooked meats, and unwashed raw vegetables are common culprits.
L monocytogenes is an opportunistic agent and a facultative intracellular parasite
that grows within macrophages in immunocompromised human hosts. Itis able to
multiply at4°C, a unique feature that laboratories exploit when culturing the
organism, a process called cold enrichment.
(Choice A) Unprotected sex is the mode of transmission of sexually transmitted
diseases such as HIV, Hepatitis B, gonorrhea chlamydia, syphilis, LGVI human
papilloma virus (the causative agent in cervical cancer) and others.
(Choice C) N. meningitidis, H. influenzae, respiratory syncytial virus Mycobacterium
tuberculosis, and many other organisms are transmitted via respiratory droplets.
Listeria infections are not transmitted in this manner, however.
(Choice D) Arthropod bites transmit: leishmaniasis (sandfly bite),
plasmodium/malaria (anopheles mosquito bite), and Chagas disease (tsetse fly bite)
Borrelia burgdorferi/Lyme disease (Ixodes tick bite).
(Choice E) Needle sticks can transmit hepatitis B, hepatitis C, HIVI as well as other
infections.

Educational Objective:
1. Listeriosis is most commonly transmitted through food ingestions and causes
sepsis and meningitis in immunocompromised adults. Listeria can also cause
neonatal meningitis, being transmitted transplacentally or via vaginal contact during
delivery.
2. Listeria grows well in cold temperatures (cold enrichment) and thus can
contaminate refrigerated food.
3. Listeria is a gram positive rod with V or L formations resembling corynebacterium,
but its tumbling motility is a unique feature.

44
USMLE WORLD STEP 1 MICROBIOLOGY

Q NO 136: Stool cultures of a 4-year-old boy with self limited diarrhea and
abdominal cramps grow non-lactose-fermenting gram negative rods on
MacConkey’s agar. Organisms do not produce gas on fermentation of A.
glucose and do not produce H25 on TSI agar. Which of the following is the
most important pathogenic mechanism for the infection caused by this
bacterium?
Exotoxin production
B. Mucosal invasion
C. Intestinal colonization
D. Bacteremia
E. Proliferation in lymph nodes

Explanation:
The biological details and the type of diarrhea present indicate that this child has
been infected by an organism of the Shigella species. Shigellosis is a hemorrhagic
diarrheal disease caused by organisms of the genus Shigella. The most common
species causing shigellosis in industrialized countries is Shigella sonnei, while the
most common strain in developing nations is Shigella flexneri. Shigella is transmitted
by the fecal-oral route and is never a component of the normal bacterial gut flora.
Shigellosis is primarily a pediatric disease, but also shows a predilection for male
homosexuals and adults in skilled nursing facilities.
Mucosal invasion (Choice B) is the essential pathogenic mechanism for Shigella
infection. Shigella invades the gastrointestinal mucosa specifically via M cells located
in Peyer’s patches. Shigella and EIEC both invade intestinal epithelial cells and
release, respectively shiga toxin and shiga-like toxin. Shiga toxin causes cell
destruction by inhibiting cellular protein synthesis. Shallow ulcers form where cells
have sloughed oft; these ulcerations can be detected by colonoscopy. The shiga
toxin is a classic bacterial AB exotoxin with a single A subunit bound to five B
subunits. The A subunit inactivates the 60S ribosome of the host, thereby halting
protein synthesis and causing cell death.
(Choice A) Although it is true that the exotoxin is what causes cell death in the
gastrointestinal mucosa, without the ability to invade the gastrointestinal mucosa,
Shigella would be unable to cause disease. Bacterial invasion is not necessary to
some other exotoxin-causing illnesses, such as S. aureus food poisoning, botulism,
and B. cereus food poisoning.
(Choice C) Intestinal colonization with Shigella does not occur. If Shigella is isolated
from a stool culture, an active infection is taking place.
(Choice D) Bacteremia is generally not caused by Shigella infections. Shigella is able
to invade the gastrointestinal mucosa and survive within the mucosal epithelial cells,
but Shigella is readily phagocytosed and destroyed by macrophages after entering
the bloodstream. Enteric bacteremia can be caused by the following species: E. coli
(sepsis), Salmonella typhl (sepsis, typhoid fever), Pseudomonas aeruginosa (sepsis,
endocarditis, and osteomyelitis), Klebsiella (sepsis), and Proteus (sepsis).
(Choice E) Proliferation in lymph nodes is not a characteristic of Shigella. In contrast,
both Salmonella typhl and Yersinia enterocolitica can penetrate the bowel mucosa,
gain access to the lymphatics, and proliferate in the mesenteric lymph nodes. When
Y. enterocolitica causes lymph node infection, the mesenteric nodes can enlarge and
become inflamed, leading to right lower quadrant pain that is often confused
clinically with appendicitis; hence, the term “pseudoappendicitis.”

45
USMLE WORLD STEP 1 MICROBIOLOGY

Educational Objective:
Mucosal invasion is an essential pathogenic mechanisms for Shigella infection.
Shigella gains access to the gut mucosal epithelium specifically by entering M cells in
Peyer’s patches. It then escapes the phagosome, spreads laterally to other epithelial
cells, and releases Shiga toxin. A subunit of this toxin disables the 60S ribosomal
subunit, which causes failure of protein synthesis in the host, as well as mucosal cell
death. Blood-tinged diarrhea that contains mucous and pus, as well as severe
cramping abdominal pain, follow.

46
USMLE WORLD STEP 1 MICROBIOLOGY

Q NO 137: Genital examination of a 31-year-old female reveals yellow discharge


from the cervical most that demonstrates abundant neutrophils on light
microscopy. The patient will most likely experience which of the following as a
sequela of this disease?
A. Endometrial hyperplasia
B. Hydatidiform mole
C. Ovarian cancer
D. Polycystic ovary syndrome
E. Infertility

Explanation:
An endocervical infection with N. gonorrhoeae, if left untreated. may ascend to
cause pelvic inflammatory disease (PID), which is manifested as endometritis,
salpingitis, tubo-ovarian abscesses, or pelvic peritonitis. Frequently, women with
gonococcal urethritis will be asymptomatic until PID or peritonitis occurs. The risk of
developing PID from untreated N. gonorrhoeae is between 8% and 10%. The most
serious complication of PID is permanent damage to the reproductive system
resulting in infertility. PID is also associated with other infections such as Chlamydia.
As a result of the inflammatory response to the infection, scarring and blockage of
the fallopian tubes can lead to infertility if sperm are unable to bypass the blockage,
and ectopic pregnancies if sperm can pass but a fertilized ovum can not. The finding
of mucopurulent cervicitis is typical for most women with PID. The occurrence of
infertility as a sequela of PID is estimated to be 15-20% for women who have a
single episode of PID and 50-80% for women with multiple episodes of PID.
(Choice A) Endometrial hyperplasia usually presents clinically as dysfunctional
uterine bleeding and is caused by elevated estrogen unopposed by progesterone
acting on the endometrial lining. The excess estrogen can be exogenous as in
hormone replacement therapy that does not contain progesterone, or it can be
endogenous such as with an estrogen-secreting ovarian tumor or with increased
aromatization of androgens to estrogens in the adipose tissue of obese women.
(Choice B) A hydatidiform mole usually presents early in pregnancy with vaginal
bleeding and fundal size greater than dates.
(Choice C) Infection is nota risk factor for ovarian cancer.
(Choice D) Polycystic ovary (Stein-Leventhal) syndrome is characterized by young
females with infertility, acne, hirsutism, insulin resistance and central obesity as well
as characteristic bilateral ovarian cysts.

Educational Objective:
Mucopurulent cervicitis with cervical motion tenderness is a frequent indicator of PID
caused by N. gonorrhoeae or Chlamydia trachomatis. PID can potentially lead to
ectopic pregnancy and infertility due to salpingitis leading to scarring of the fallopian
tubes if not treated appropriately.

47
USMLE WORLD STEP 1 MICROBIOLOGY

Q NO 138: Group A Streptococci demonstrate significant resistance to phagocytic


killing when placed in fresh human blood. This resistance can be most effectively
overcome by adding antibodies to which of the following?
A. Hyaluronate
B. Protein M
C. Streptolysin 0
D. DNase
E. Streptokinase
F. Teichoic acid

Explanation:
Protein M is the major virulence factor for Streptococcus pyogenes; it inhibits
phagocytosis and the activation of complement. The cell surface of Streptococcus
pyogenes is biochemically diverse: Lancefield Group A Streptococci are divided into
more than 80 serogroups based on M proteins. In addition to their antiphagocytic
and anticomplement properties M protein is also cytotoxic for neutrophils in the
serum and a mediator of bacterial attachment. M proteins are antigenic and
stimulate type-specific immunity.
(Choice A) Hyaluronate is present in the capsule (i.e. external to the cell wall) and
has antiphagocytic activity. Hyaluronic acid in bacteria cannot be distinguished from
the ground substance found in human connective tissue and, therefore, cannot be
immunogenic.
(Choice C) Streptolysin 0 is an oxygen-labile immunogenic extracellular protein
produced by most S. pyogenes strains. It lyses erythrocytes (S. pyogenes is beta-
hemolytic) and polymorphonuclear leukocytes. Streptolysin 0 is the antigen
responsible for the production of antibodies tested for with the ASO (Anti-
Streptolysin-O) titer. It does not convey resistance to phagocytosis.
(Choice D) DNase is an extracellular enzyme that depolymerizes viscous DNA in pus
and disintegrates polymorphonuclear leukocytes, allowing the streptococcus to move
more freely in the tissue.
(Choice E) Streptokinase is an extracellular enzyme that catalyzes plasminogen to
plasmin, resulting in fibrin digestion and facilitating the spread of streptococci in
infected tissue. Streptokinase is used pharmacologically as a thrombolytic agent.
(Choice F) Teichoic acid and lipoteichoic acid are substances that are integrated into
the peptidoglycan cell wall of some Gram-positive bacteria such as Staphylococcus
aureus. They have a minor lipopolysaccharide-like endotoxin effect.

Educational Objective:
Protein M is the major virulence factor for Streptococcus pyogenes. It inhibits
phagocytosis and complements activation, mediates bacterial adherence, and is the
target of type-specific humoral immunity to S. pyogenes.

48
USMLE WORLD STEP 1 MICROBIOLOGY

Q NO 139: A 6-day-old Caucasian infant born to a 22-year-old Caucasian female is


found to have positive HBsAg and HBeAg. This patient is most likely to have which
of the following combination of infection outcomes?

Risk of chronic Viral replication Histologic findings


Infection rate of liver injury
A. Low High Severe
B. Low Low Severe
C. Low Low Mild
D. High High Severe
E. High High Mild

Explanation:
Vertical transmission of hepatitis B from pregnant females to the unborn child can
occur with active hepatitis B infection. Typically, such transmission takes place
during the passage of the fetus through the birth canal, but transplacental infection
can also occur. This is especially common in those women who developed acute
hepatitis B infection in the third trimester.
In HBV-positive mothers who are negative for HBeAg (a soluble protein that is a
marker of viral replication and infectivity), the neonate’s risk of infection is 20%. In
mothers who are HBeAg positive, however, the neonate’s risk of infection is 95%.
Once the infant becomes infected, rapid viral replication occurs and the chance of
progression to chronic hepatitis is 90%. Only mild liver injury will be evident
histologically in these neonates because HBV is not inherently cytotoxic and the
neonatal immune system is still immature.
(Choices A, B, and C) The risk of chronic infection is actually quite high (95%) in a
neonate born to an HBsAg- and HBeAg-positive mother.
(Choice D) The histologic findings in the HBV-infected neonate are suggestive of mild
liver injury, not severe liver injury.

Educational Objective:
Neonates born to HBsAg- and HBeAg-positive mothers are at high risk of chronic
infection, experience fast HBV replication, and demonstrate mild hepatic inju
histologically.

49
USMLE WORLD STEP 1 MICROBIOLOGY

Q NO 140: After exposure to mutagens, several strains of enveloped RNA viruses


isolated from the nasal exudate of experimental animals acquire the ability to
infect human epithelial cells. Which of the following viral-encoded proteins is
most likely to have mutated in this circumstance?
A. Surface glycoprotein
B. Capsid protein
C. RNA polymerase
D. Endonuclease
E. Protease
Explanation:
A major determinant of viral tropism for the specific tissues of specific hosts is the
extent to which the viral surface proteins can bind to complementary host cell
plasmalemma receptors. In the case of an enveloped virus, whether or not the virus
can attach to a specific host cell generally depends on if a viral envelope
glycoprotein with a high binding affinity for a host cell surface glycoprotein is
present. A mutation in a viral-encoded envelope glycoprotein can therefore
dramatically affect the range of host cells that the virus can attach to or infect.
One such example would be a mutation in the hemagglutinin of an influenza A strain
that was previously confined to domestic livestock. If the mutation conferred a new
binding affinity for a neuraminic acid-containing glycoprotein on the surface of
human nasopharyngeal epithelial cells, then the virus would no longer be a threat
only to domestic livestock and humans would be vulnerable to infection.
(Choice B) With enveloped viruses, the nucleocapsid protein surrounding the viral
genome is not directly involved in viral attachment to host cell surface receptors.
(Choice C) Enveloped viruses with single-stranded, negative-sense RNA may contain
a viral encoded RNA dependent RNA polymerase. In some viral families (eg,
paramyxoviruses), the RNA polymerase may be able to transcribe the viral genome
with minimal dependence on specific host cell mRNA primers. Thus, a mutation in
this enzyme may not alter the host range of viral infectivity.
(Choice D) Enveloped viruses with single-stranded, negative-sense RNA utilize
different methods to initiate genomic transcription. Some (eg, influenza virus) have
a viral encoded endonuclease associated with the viral RNA-dependent RNA
polymerase. This endonuclease is thought to cleave short 5 RNA fragments that are
used to prime the viral RNA for transcription by the viral RNA polymerase (“cap
snatching”). The host range of infectivity of an orthomyxovirus could conceivably be
altered by an endonuclease mutation. However, other enveloped RNA viruses —
including those with negative sense RNA (eg, paramyxovirus) and those with
positive sense RNA (eg, hepatitis C virus) — can synthesize new viral genomic and
messenger RNA without endonuclease involvement. Therefore, the host range of
infectivity demonstrated by the enveloped RNA viruses is not likely to be affected by
a viral endonuclease mutation. (Choice E) Viral encoded protease is required for the
replication of single-stranded, positive sense RNA viruses that possess genomes
which function as viral polycistronic mRNA and produce a poly protein product. A
mutation in such a protease would be expected to nonspecifically affect viral
replication, independent of the characteristics of any given host cell.
Educational Objective: Changes in host range are most commonly caused by a
mutation in the viral encoded surface glycoprotein that mediates virion attachment
to target host cell plasmalemma receptors. Mutations in viral encoded capsid
proteins, RNA polymerases, endonucleases, or proteases could nonspecifically affect
viral replication in the standard host cell but would be less likely to significantly alter
the range of host cell types that an enveloped RNA virus could successfully infect.

50
USMLE WORLD STEP 1 MICROBIOLOGY

Q NO 141: A 22-year-old Caucasian male with fever and joint pain is found to have
atypical lymphocytes on his blood smear. This patient’s symptoms are caused by an
enveloped virus having partially double-stranded circular DNA. An enzyme packed
in its virion has RNA-dependent DNA-polymerase activity. This patient is most likely
infected with:

A. Epstein-Barr virus
B. Hepatitis B virus
C. Cytomegaly virus
D. Human immunodeficiency virus
E. Coxsackie virus
F. Parvovirus B 19

Explanation:
A member of the DNA-containing family Hepadnaviridae, the mature hepatitis B
virus is a spherical double-layered “Dane particle” that has a hexagonal core covered
with an outer surface envelope of protein, lipid, and carbohydrate. The HBV genome
is a partially double-stranded circular DNA molecule, and replication of this genome
is accomplished through a reverse transcriptase DNA polymerase that creates an
intermediate + single-stranded RNA template. The progeny of this process is double-
stranded DNA.
(Choices A and C)The Epstein-Barr virus and cytomegalovirus are members of the
herpesvirus family. All viruses within this family contain an icosahedral core
surrounded by a lipoprotein envelope and have double-stranded DNA with no
polymerase. These are the only viruses to acquire their envelopes by budding from
the nuclear membrane.
(Choice D) Human immuno deficiency virus is within the lentivirus subgroup of
retroviruses. It contains a bar-shaped core surrounded by a glycoprotein envelope
that includes gp12O and gp4l two glycoproteins specific to HIV. The genome is
diploid, consisting of two + single-stranded RNA molecules. Reverse transcriptase
uses the genome RNA as a template to synthesize DNA.
(Choice E) Coxsackie virus is within the picornavirus family and is composed of an
icosahedral nucleocapsid and a + single-stranded RNA genome. The RNA has a
protein on the 5 end that acts as a primer for transcription by RNA polymerase.
(Choice F) Parvovirus B19 is a small nonenveloped icosahedral virus with a-single-
stranded DNA genome. There is no virion polymerase.

Educational Objective:
The hepatitis B virus genome consists of partially double-stranded circular DNA.
Replication is accomplished through a reverse transcriptase DNA polymerase that
creates an intermediate + single-stranded RNA template and double stranded DNA
progeny.

51
USMLE WORLD STEP 1 MICROBIOLOGY

Q NO 142: An outbreak of hepatitis in New Delhi was characterized by high


incidence of fulminant hepatitis in pregnant women, who experienced a A. Is
mortality rate of 20%. The virus responsible for the outbreak: an
undeveloped RNA virus
B. Is transmitted parenterally
C. Is associated with chronic hepatitis
D. Infects only HBsAg-positive individuals
E. Has oncogenic properties

Explanation:
Hepatitis E virus is an underdeveloped, single-stranded RNA virus spread through
the fecal-oral route. Infection with HEV occurs primarily in young and middle-aged
adults living in Asia sub-Saharan Africa and Mexico, with an average incubation
period of six weeks. While the virus is shed in the stool during the acute illness the
disease is typically self- limited and not associated with either chronic liver disease
or a carrier state. HEV Ag or HFV RNA can be detected
in the stool or liver in the earliest stages of infection (when the patient is
asymptomatic). Later serum transaminases and 1gM anti-HEV titers rise in
association with clinical illness. The most concerning feature of hepatitis E is the high
mortality rate observed in infected pregnant women.
(Choice B) While hepatitis B, C, and D can be transmitted parenterally (with
intravenous drug abusers and hemophiliacs are particularly high risk) hepatitis A and
E are primarily transmitted via the fecal-oral route.
(Choice C) Hepatitis E causes an acute illness and is not associated with chronic
infection.
(Choice D) Unlike hepatitis D virus hepatitis E is not dependent on HBsAg for
infection and multiplication.
(Choice E) Because of their association with hepatocellular carcinoma hepatitis B and
C have been suggested to have oncogenic properties. Hepatitis E is not associated
with an increased risk of hepatocellular carcinoma however.

Educational Objective:
Hepatitis E virus is an undeveloped, single-stranded RNA virus spread through the
fecal-oral route. The most concerning feature of hepatitis E infection is the high
mortality rate observed in infected pregnant women.

52
USMLE WORLD STEP 1 MICROBIOLOGY

Q NO 143: Picornaviruses are small RNA-containing viruses. Which of the following


picornaviruses are the most acid-labile?

A. Coxsackie A
B. Echovirus
C. Poliovirus
D. Rhinovirus
E. Hepatitis A virus

Explanation:
The viruses listed above are all members of the Picornaviridae family, which includes
the rhinovirus and enterovirus genera. In general, viruses are stable between pH
values of 5.0 and 9.0. Once the pH drops below 5.0 (as in the stomach, which can
have a pH of 1 .0), the acid-labile rhinoviruses are inactivated. Enteroviruses, in
contrast, are relatively acid stable and can therefore pass through the stomach and
into the small intestine without being denatured or degraded by the acid
environment. This explains why enteroviruses can colonize or infect the Gl tract,
whereas rhinoviruses are limited to colonization or infection of the upper respiratory
tract.
(Choices A, B, C, and E) These are enteroviruses capable of infecting or colonizing
the lower 0 tract.

Educational Objective:
The Picornaviridae family includes the rhinovirus and enterovirus genera.
Rhinoviruses are acid-labile and therefore cannot colonize the GI tract or cause
gastroenteritis. Enteroviruses are relatively acid stable and can therefore pass
through the stomach to colonize or infect the 01 tract.

53
USMLE WORLD STEP 1 MICROBIOLOGY

Q NO 144: The vaccine that provides protection against H. influenzae contains


capsular polysaccharide of H. influenzae type b as well as diphtheria toxoid. A.
Diphtheria toxoid in this vaccine:
Improves safety
B. Increases solubility
C. Mounts diphtheria protection
D. Increases immunogenicity
E. Cross-reacts with human antigens

Explanation:
The Haemophilus influenzae type b (Hib) vaccine consists of PRP (poly ribose-ribitol-
phosphate) derived from the capsule of H. influenzae type b coupled with either
diphtheria or tetanus toxoid. This combination vaccine is now approved for use in
infants with the initial series of vaccinations at 2, 4, and 6 months of age and with a
booster dose atl5 months of age.
Infants possess protective maternally derived transplacentally acquired lgG against
Hib, but this protection fades as these immunoglobulins are degraded during the
first few months of life. Infants and young children are unable to mount an adequate
immune response against T-lymphocyte-independent antigens such as bacterial
capsular polysaccharide. When the polysaccharide antigen is conjugated with
diphtheria or tetanus protein toxoid the conjugate becomes a T-cell-dependent
antigen, and the immunogenicity of the vaccine is thereby increased (Choice
D). Infants and young children have been shown to be quite capable of mounting
protective immune responses to T cell dependent antigens. The additional protection
afforded by this protein-polysaccharide conjugate vaccine results from T-cell
dependent stimulation of B-lymphocytes to undergo class-switching from 1gM to lgG
and the production of memory B-lymphocytes that stand ready to fight Hib infection
should it occur. Because of the conjugated vaccine there has been a tremendous
decline in the incidence of invasive disease caused by H. influenzae type b.
(Choice A) Conjugation of the PIRP polysaccharide with protein antigens in the Hib
conjugate vaccine was not done for safety reasons. There are presently no
contraindications or precautions identified for administration of the Hib vaccine.
(Choice B) Protein is not conjugated to polysaccharide in the Hib vaccine in an effort
into increase solubility. Solubility is achieved in vaccine preparations through the use
of additives such as aluminum hydroxide or aluminum sulfate.
(Choice C) The conjugation of diphtheria toxoid or tetanus toxoid to the PRP
polysaccharide of H. influenzae type b does elicit a response to both the Hib
polysaccharide antigen and the toxoid used either tetanus or diphtheria. Though this
response is elicited, it is not the specific reason for conjugating PRP polysaccharide
to toxoid.
(Choice E) Neither the diphtheria toxoid nor the tetanus toxoid cross-react with
human antigens in the conjugated Hib vaccine. This would be an undesirable effect.

Educational Objective:
The Hib vaccine consists of PRP capsular polysaccharide conjugated with either
tetanus or diphtheria toxoid. Protein conjugation causes a T-cell mediated immune
response leading to immunoglobulin class switching and generation of memory B-
lymphocytes. This response would not occur with pure polysaccharide immunization.

54
USMLE WORLD STEP 1 MICROBIOLOGY

Q NO 145: A viral protein synthesized in the macrophages of a 22-year-old HI V-


positive male is glycosylated and cleaved into two smaller proteins in the
endoplasmic reticulum. These newly formed proteins are most likely responsible
for which of the following?
A. Viral DNA synthesis from RNA template
B. Viral poly protein cleavage
C. Viral DNA integration into the host genome
D. Virus-induced cell apoptosis
E. Virion absorption by the target cells

Explanation:
In the HIV replication cycle poly protein precursors are encoded by structural genes
(gag pol, and env). Only the poly protein product of the env gene is glycosylated to
become gpl6O. The gp16O is then proteolytically cleaved with in the endoplasmic
reticulum and Golgi apparatus to form gp120 and gp41. gp120 mediates viral
absorption by binding to the CD4 receptor of susceptible cells. Transmembrane gp4l
anchors gp120 through noncovalent interactions mediating the fusion process
between viruses and target cells.
(Choice A) HIV DNA synthesis from an RNA template is catalyzed by reverse
transcriptase (PT), which is not a glycoprotein. Moreover, once the poly protein
precursor of PT (Pr160gag-pol precursor) is translated from the pol gene, it is then
incorporated into an immature virion and proteolytically cleaved into various viral
enzymes during virion maturation.
(Choice B) HIV polyprotein cleavage is mediated by viral protease.
(Choice C) HIV DNA integration into the host genome is catalyzed by viral integrase.
(Choice D) HIV infection of CD4+Tcells is thought to primarily cause cell damage
through a direct cytopathic effect. Activation-induced apoptosis may occur in those
uninfected CD4+ cells that respond to the MHC class II- associated HIV peptides
displayed on the surface of infected cells.

Educational Objective:
In the HIV replication cycle, polyprotein precursors are encoded for by the structural
genes (gag pol, and env). Only the env gene polyprotein product is glycosylated to
gpl60 and proteolytically cleaved within the endoplasmic reticulum and Golgi
apparatus to form the envelope glycoproteins gpl20 and gp41.

55
USMLE WORLD STEP 1 MICROBIOLOGY

Q NO 146: E. coli isolated from a patient with diarrhea does not ferment sorbitol on
sorbitol containing MacConkey agar and does not produce glucuronidase. The toxin
produced by these bacteria would most likely:

A. Activate adenylate cyclase


B. Activate guanylate cyclase
C. Inactivate ribosomal subunits
D. Inactivate EF-2
E. Disrupt the cellular cytoskeleton

Explanation:
Most O157:H7 Shiga-like toxin producing E. coli strains do not ferment sorbitol
during overnight incubation in contrast to the approximately 80% of other E. coli
strains that ferment sorbitol rapidly. Sorbitol-containing MacConkey agar is used for
isolation of this strain. Additionally E. coli O157:H7 does not produce glucuronidase.
Glucuronidase production is a characteristic of other strains of F coil.
EHEC is recognized as a cause of hemorrhagic colitis most characteristically
associated with ingestion of inadequately cooked hamburger meat. EHEC strains
produce a toxin that is virtually identical to the Shiga toxin produced by Shigella
dysenteriae. This Shiga-like toxin is phage encoded, and its production is enhanced
by iron deficiency. The Shiga and the Shiga-like toxins inactivate the 60S ribosomal
subunit in human cells leading to an inhibition of protein synthesis and eventual cell
death. Infection with EHEC can also lead to the hemolytic-uremic syndrome (HUS)
characterized by thrombocytopenia, microangiopathic hemolytic anemia and renal
insufficiency (uremia).
EHEC do not invade the intestinal mucosa: this is a characteristic of Enteroinvasive
F. coil as well as other causes of hemorrhagic diarrhea. The organism does not
produce LT or ST; these toxins are produced by Enterotoxigenic F. coil (ETEC).
(Choice A) Activate adenylate cyclase: Bacteria that produce toxins capable of
activating adenylate cyclase include: B. pertussis (Pertussis toxin) B. anthracis
(Edema factor), ETEC (Heat labile toxin), Campylobacter jejuni (C. jejuni
enterotoxin), Bacilus cereus (Heat labile enterotoxin) and V. cholerae (Choleragen
toxin). Increased intracellular cAMP in intestinal mucosal cells leads to decreased
absorption and increased secretion of sodium, chloride and water.
(Choice B) Activate guanylate cyclase: Bacteria that produce toxins capable of
activating guanylate cyclase include: ETEC (Heat stable toxin) and Yersinia
enterocolitica (Y. enterocolitica enterotoxin). These toxins increase intracellular
cGMP leading to watery diarrhea and electrolyte loss.
(Choice D) Inactivate EF-2: Bacteria that produce toxins capable of inactivating
elongation factor-2 (EF-2) include: Corynebacterium diphtheriae (Diphtheria toxin)
and Pseudomonas aeruginosa (Exotoxin A). These exotoxins inhibit protein synthesis
in eukaryotic cells by catalyzing ADP-ribosylation of EF-2 causing this protein to be
inactivated. (Choice E) Disrupt cytoskeleton: Clostridium difficile produces an
exotoxin capable of disrupting the cytoskeleton (Toxin B, Cytotoxin). This toxin acts
by depolymerizing actin leading to cell death.
Educational Objective: The strain of Enterohemorrhagic E. coli (EHEC) O157:H7 is
thought to cause at least 80% of cases of hemolytic-uremic syndrome (HUS) in
North America and is recognized as a common cause of bloody diarrhea in developed
countries. It is associated with consumption of undercooked ground beef and
elaborates a Shiga-like toxin capable of inhibiting protein synthesis in colonic
mucosal cells and renal endothelial cells. This particular strain of E. coli is unable to
ferment sorbitol and does not produce a glucuronidase.

56
USMLE WORLD STEP 1 MICROBIOLOGY

Q NO 147: A 48-year-old Caucasian male presents to your office with a six-month


history of cough and fatigue. He also complains of malaise and weight loss. Chest
X-ray shows a pulmonary infiltrate in the lower lobe of the left lung. Sputum
potassium hydroxide preparation reveals the following: What is the most likely
cause of this patient’s condition?
A. Rhizopus species
B. Aspergillus fumigatus
C. Candida albicans
D. Cryptococcus neoformans
E. Blastomyces dermatitidis
F. Histoplasma capsulatum
G. Coccidioides immitis
H. Sporothrix schenckii

Explanation:
The image above shows encapsulated yeast with a single, broad-based bud. This is
the typical appearance of Blastomyces dermatitidis, a fungus endemic to the Great
Lakes, and Ohio and Mississippi River regions. It is present in soil and rotten organic
matter. Blastomycosis a dimorphic fungus meaning it assumes different forms at
different temperatures. The mold form (branching hyphae) predominates in the
environment with average temperatures of 25-30°C. In the human body (37-40°C),
it assumes the yeast form (single cells).
Infection occurs by inhalation of aerosolized fungus from the environment. In the
lungs Blastomyces assumes yeast form multiplies and induces a granulomatous
response. In about half of immunocompetent individuals blastomycosis remains
asymptomatic. In others it may present as a lung infection or cause a flu-like illness
(fever, chills myalgia, headache nonproductive cough) or pneumonia (fever, cough
pleuritic chest pain). Pulmonary blastomycosis is diagnosed by KOH preparation of
the sputum.
In immunocompromised patients blastomycosis can cause disseminated disease.
Patients experience systemic symptoms (fever weight loss, night sweats) lung
involvement (cough dyspnea) skin lesions (papules, pustules, ulcers verrucous
lesions) and bone pain (caused by lytic lesions).
(Choice B) Aspergillus fumigatus can cause lung disease in immunocompromised
patients. It has a mold form only; in tissue samples it is seen as septate hyphae that
branch at 45° angles.
(Choice D) Cryptococcus neoformans causes lung disease and meningitis in
immunocompromised patients. In contrast to blastomycosis, it forms narrow based
buds. Cryptococcus has a thick polysaccharide capsule that appears clear with India
ink staining and stains red with mucicarmine.
(Choice F) Histoplasma capsulatum can cause lung disease. Like Blastomyces, it is a
dimorphic fungus. Unlike Blastomyces, the yeast form of Histoplasma is found
intracellularly within macrophages.
(Choice G) Coccidioides immitis is also a dimorphic fungus. It is seen as spherules
(round encapsulated structures containing many endospores) in tissue samples.
Educational Objective:
Blastomyces dermatitidis is a dimorphic fungus that is seen in tissue as round yeasts
with doubly refractive walls and “broad based budding.” It is endemic to the Great
Lakes and the Ohio and Mississippi River regions. The lungs are the primary site of
involvement.

57
USMLE WORLD STEP 1 MICROBIOLOGY

Q NO 148: A 49-year-old Caucasian male with a long history of cough and weight
loss is found on chest x-ray to have a pulmonary infiltrate. His medical history is
otherwise unremarkable. He does not use tobacco, alcohol or drugs.
Bronchoscopy with transbronchial al biopsy reveals granulomatous inflammation.
Which of the following agents is most likely to cause chronic pneumonia in an
immunocompetent host?
A. Candida albicans
B. Blastomycetous dermatitidis
C. Pneumocystis jiroveci
D. Aspergillus fumigatus
E. Mucor species

Explanation:
Blastomycosis dermatitidis is a dimorphic fungus that can cause pulmonary
infections in immunocompetent people. A history of travel to an endemic area (Great
Lakes Mississippi and Ohio River basins) should raise suspicion for this infection. The
fungus exists in the environment in mold form found in soil, organic matter, and in
many animals (e.g. dogs horses). Blastomyces is transmitted by the respiratory
route entering the lungs and transforming into yeast form (thermal dimorphism).
B. dermatitidis may be asymptomatic in an immunocompetent person but it can also
cause a flu-like illness or cough with sputum production hemoptysis, dyspnea and
pleuritic chest pain. The infection may become
chronic. Pulmonary blastomycosis is characterized by granuloma formation.
Sputum stain with KOH is diagnostic. It reveals round yeast with thick doubly
refractive walls. Each yeast has a single broad-based bud. Itraconazole is the
preferred treatment for blastomycosis.
(Choice A) Candida albicans is an opportunistic pathogen. It may cause a superficial
infection in an immunocompetent host (e.g. vulvovaginitis). Systemic disease
however occurs only in the immunosuppressed (especially neutropenic patients).
(Choice C) Pneumocystis jiroveci (formerly known as Pneumocystis carinii) causes
pulmonary disease in immunocompromised patients. HIV (+) patients and transplant
recipients are susceptible to this infection. P. jiroveci does not cause symptoms in
immunocompetent hosts.
(Choice D) Aspergillus fumigatus is an opportunistic pathogen. It causes invasive
aspergillosis in immunocompromised patient’s aspergillomas in patients with old lung
cavities and allergic bronchopulmonary aspergillosis (ABPA) in people with asthma.
(Choice E) Mucor species infect the paranasal sinuses in patients with neutropenia
burns or diabetes mellitus. Mucormycosis is strongly associated with diabetic
ketoacidosis.

Educational Objective:
Blastomyces dermatitides can cause pulmonary disease in the immunocompetent
host. In immunocompromised individuals, it may lead to disseminated mycosis
(fever pulmonary symptoms skin and bone involvement).

58
USMLE WORLD STEP 1 MICROBIOLOGY

Q NO 149: A 17-year-old female presents to your office with dark urine and facial
puffiness. She had a minor skin infection several weeks ago that resolved
spontaneously. Microscopic examination of the urine sediment shows red blood
cell casts. The organism responsible for this patient’s symptoms would most likely
demonstrate:
A. Catalase positivity
B. Growth in 6.5% NaCl
C. Bile solubility
D. Bacitracin sensitivity
E. Optochin sensitivity

Explanation:
The minor skin infection mentioned in this question stem is impetigo, a skin infection
caused by Streptococcus pyogenes (Group A Strep). S. pyogenesis a catalase
negative beta-hemolytic, Gram-positive coccus that forms chains on microscopy. It
is unique among beta-hemolytic streptococci because it is sensitive to bacitracin.
Rheumatic fever and acute post-streptococcal glomerulonephritis (APSGN) are the
late sequelae that can follow an acute group A streptococcal infection. The patient in
this question stem is experiencing symptoms consistent with post-streptococcal
glomerulonephritis.
(Choice A) Streptococci are catalase-negative, staphylococci are catalase-positive.
(Choice B) Enterococci are the only members of the Streptococcus genus that can
grow in broth containing 6.5% NaCl.
(Choices C and E) Streptococcus pneumoniae is unable to grow in bile (bile soluble)
and optochin susceptible. These tests are used to differentiate S. pneumoniae from
other alpha-hemolytic streptococci such as those in the viridans group.

Educational Objective:
Streptococci are catalase-negative, staphylococci are catalase-positive. S. pyogenes
(Group A Streptococcus) is beta-hemolytic and bacitracin susceptible. S. agalactiae
(Group B Streptococcus) is beta-hemolytic and bacitracin resistant. Streptococcus
pneumoniae is bile soluble (unable to be cultured in bile) and cryptochid susceptible.

59
USMLE WORLD STEP 1 MICROBIOLOGY

60
USMLE WORLD STEP 1 MICROBIOLOGY

Q NO 150: A 65-year-old male hospitalized for community acquired pneumonia


develops fever and loose stools on his fourth hospital day. He recently traveled
to Mexico for vacation without taking any prophylactic antibiotics or receiving A.
any vaccinations. Sigmoidoscopy demonstrates white yellow membrane-like
plaques on the colonic mucosa and biopsy shows that these plaques are
composed of fibrin and inflammatory cells. The patient’s condition is most likely
associated with which of the following?
Recent travel to Mexico
B. Missed vaccination
C. Antibiotic therapy
D. Gastritis prophylaxis
E. Consumption of home-canned foods

Explanation:
Antibiotics disturb the normal intestinal microbial ora and can lead to overgrowth of
Clostridium difficile, an organism that is sometimes part of the normal gut flora or
that gains access to the patient after antibiotics have killed most of the commensal
bowel flora. C. difficile tends to survive the antibiotic therapy that kills most of the
other bacteria that compose the normal gut flora. These other organisms normally
are responsible for keeping potential pathogens like C. difficile in check.
In the absence of other intestinal inhabitants, the vegetative cells of C. duff/die
produce enterotoxin A, which causes watery diarrhea, and cytotoxin B, which causes
colonic epithelial cell necrosis and stimulation of fibrin deposition. C. difficile disease
may range in severity from mild transient watery diarrhea to severe
pseudomembranous colitis. Metronidazole is the treatment of choice for C. difficile
pseudomembranous colitis.
(Choice A) Recent travel to Mexico can be associated with traveler’s diarrhea.
Traveler’s diarrhea is most frequently bacterial in etiology, and the most common
causes are Enterotoxigenic E. coli Campylobacter, Salmonella and Shigella. Parasites
such as Entamoeba and Giardia Iamblia can also be responsible.
(Choice B) Hepatitis A and salmonella infection are vaccine-preventable causes of
diarrhea but they do not cause pseudomembrane formation.
(Choice D) ICU patients commonly receive prophylaxis against acute gastritis with
proton pump inhibitors. Loose stools are a frequent side effect of this class of drugs,
but pseudomembrane formation is not associated.
(Choice E) In the anaerobic environment of a vacuum-sealed can, C. botulinum can
mature from its spore form to a vegetative cell and begin synthesizing its potent
neurotoxin. This toxin inhibits acetylcholine release from the nerve terminals at
neuromuscular junctions.

Educational Objective:
Antibiotics disturb the normal intestinal flora and can lead to C. difficile overgrowth.
C. difficile causes colitis with severity ranging from transient watery diarrhea to
pseudomembranous colitis. Diarrhea may begin up to four weeks after antibiotic
therapy. Metronidazole is the treatment of choice.

61
USMLE WORLD STEP 1 MICROBIOLOGY

Q NO 151: The serum of a 23-year-old male contains a large amount of a specific


glycoprotein that forms spheres and tubules approximately 22 nm in diameter.
The glycoprotein turns out to be a product of viral synthesis that poorly correlates
with viral replication. The glycoprotein described is most likely:
A. Envelope component of HBV
B. Nucleocapsid component of HBV
C. Capsid component of HAV
D. Viral polymerase of HCV
E. Nonstructural protein of HCV

Explanation:
The HBV genome encodes numerous protein sequences, including three antigens, a
DNA polymerase, and a transcriptional transactivator from the X region.
HBcAg is a nucleocapsid core protein that resides within hepatocytes and assembles
virion.
HBeAg is a nucleocapsid core and precore protein that is a marker of high infectivity.
The core component resides within hepatocytes and assembles virion, while the
precore component directs secretion into blood.
HBsAg is a noninfective envelope glycoprotein that forms spheres and tubules 22 nm
in diameter. Infected hepatocytes may secrete enormous quantities of this
glycoprotein, often considerably exceeding the amount of HBcAg produced.
The DNA polymerase uses reverse transcriptase to replicate the genome and
produces an intermediate RNA template. HBx, the transcriptional transactivator of
viral genes from the X region, is necessary for viral replication. It is also thought to
function in the deregulation of hepatocyte replication and the subsequent
development of hepatocellular carcinoma in those individuals infected with HBV.
(Choice B) HBcAg and HBeAg are two nucleocapsid components of HBV that have
been found to correlate with viral replication.
(Choice C) Hepatitis A virus has a nonenveloped icosahedral nucleocapsid. This virus
is not associated with glycoproteins that form spheres or tubules or glycoproteins
that poorly correlate with viral replication.
(Choices D and E) Hepatitis C virus is enveloped and contains a positive, single-
stranded RNA genome with no virion polymerase. This virus is not associated with
glycoproteins that form spheres or tubules or glycoproteins that poorly correlate with
viral replication.

Educational Objective:
A component of the hepatitis B virus envelope, HBsAg is a noninfective glycoprotein
that forms spheres and tubules 22 nm in diameter. Infected hepatocytes may
secrete enormous quantities of HBsAg, often considerably exceeding the amount of
HBcAg produced.

62
USMLE WORLD STEP 1 MICROBIOLOGY

Q NO 152: A 59-year-old female is brought to the ER with fever skin flushing and
an altered level of consciousness. Her blood pressure is 50/20 mmHg, and her
heart rate is 120/mm. If blood cultures are positive for F. coli, which of the
following bacterial factors is most likely responsible for this patient’s current
condition?
A. Capsule
B. Heat-stable exotoxin
C. O antigen
D. Fimbrial antigen
E. Lipid A
F. Flagellar antigen

Explanation:
This woman is suffering from septic shock. Septic shock results form the release of
endotoxins into the bloodstream. Regardless of the bacterial source, most mammals,
including humans experience the same range of toxic biological effects as a result of
these endotoxins.
Endotoxins are found in the outer membrane of Gram-negative bacteria which is
composed of lipopolysaccharide (LPS). LPS is released during destruction of the
bacterial cell wall. It can also be released during cell division. LPS is a very long
heat-stable molecule arranged into three regions: 0 antigen core polysaccharide and
Lipid A. Lipid A is responsible for the toxic properties of LPS that lead to Gram-
negative sepsis and endotoxic septic shock.
Lipid A induces shock by activation of macrophages and granulocytes. This activation
results in the synthesis of endogenous pyrogens such as IL-i prostaglandins, and the
inflammatory mediators: tumor necrosis factor-alpha (TNF-alpha) and interferon.
These cytokines then induce a febrile response by the action of IL-1 on the
hypothalamus, as well as hypotension increased vascular permeability with third-
spacing of fluids diarrhea disseminated intravascular coagulation and death.
(Choice A) E. coli strains that cause neonatal meningitis synthesize K-1 capsular
antigens.
(Choice B) Heat-stable exotoxin is one of the enterotoxins produced by ETEC.
(Choice C) The 0 antigen is a cell wall outer membrane polysaccharide antigen used
to classify gram-negative bacteria.
(Choice D) Fimbriae or pili are virulence factors that allow bacteria to adhere to the
target tissue and establish infection. Examples of organisms that use pili are
Neisseria meningitidis, uropathogenic and diarrheogenic E. coli, Bordereau pertussis,
and Vibrio cholerae.
(Choice F) Flagellar (H) antigen is a heat-labile protein which is one component of
the serologic classification of the entero Bacteriaceae.

Educational Objective:
Gram-negative sepsis is caused by the release of LPS from bacterial cells during
division or by bacteriolysis; LPS is not actively secreted by bacteria. Lipid A is the
toxic component of LPS; it causes activation of macrophages leading to the
widespread release of IL-1 and TNF-alpha, which cause the signs and symptoms of
septic shock: fever hypotension diarrhea oliguria, vascular compromise and DIC.

63
USMLE WORLD STEP 1 MICROBIOLOGY

Q NO 153: A 43-year-old with advanced HIV infection presents to the emergency


room because of worsening shortness of breath, cough and fever. Chest X-ray
reveals diffuse interstitial infiltrates bilaterally. A lung biopsy specimen is shown
below:

Which of the following describes the organism most likely responsible for his current
condition?

A. Enveloped double-stranded DNA virus


B. Nonenveloped double-stranded DNA virus
C. Enveloped double-stranded RNA virus
D. Enveloped single-stranded RNA virus
E. Pneumocystis jiroveci
F. Filamentous Gram positive rod

Explanation:
This lung biopsy shows an enlarged, centrally located epithelial cell with intranuclear
and cytoplasmic inclusions, Hidings characteristic of cytomegalovirus (CMV). CMV is
an enveloped virus that contains a double-stranded DNA genome.

Educational Objective:
The finding of interstitial pneumonia in an AIDS patient with intranuclear and
cytoplasmic inclusion bodies histologically points to opportunistic infection with CMV.
CMV is an enveloped virus that contains a double-stranded DNA genome.

64
USMLE WORLD STEP 1 MICROBIOLOGY

Q NO 154: A 36-year-old immigrant from Peru presents to the physician’s office


because of difficulty swallowing liquids. He says he also has difficulty belching. A.
Physical examination is unremarkable. Barium swallow shows a dilated
esophagus, and manometry confirms absent peristalsis in the smooth muscle
portion of the esophagus. If this patient’s symptoms are caused by infection,
which of the following organisms is the most likely cause?
Cytomegalovirus
B. Babesia divergens
C. Toxoplasma gondii
D. Isospora belli
E. Brucella melitensis
F. Trypanosoma cruzi
G. Campylobacter fetus

Explanation:
Dysphagia for liquids and dull cults belching in association with a dilated esophagus
and absent peristalsis in the smooth muscle portion of the esophagus is diagnostic of
achalasia. Achalasia is most often a primary disorder (congenital). It is always
caused by dysfunction of ganglion cells of the myenteric plexus. When a patient from
Central or South America, presents with achalasia, however, suspect infection by
Trypanosoma cruzi.
Trypanosoma cruzi is a parasite transmitted by an insect that lives in the walls of
rural huts—the reduviid bug. T. cruzi produces a neurotoxin that destroys the
myenteric plexus and causes intramural, parasympathetic denervation of smooth
muscle. In the esophagus, this neurotoxin incapacitates the lower esophageal
sphincter, so that food gets “stuck” in the esophagus. Proximal to this obstruction,
the esophagus is markedly dilated. T. cruzi infection can cause similar changes in
the sigmoid colon and ureter, respectively causing megacolon and megaureter.
(Choice A) Cytomegalovirus causes “heterophil-negative” mononucleosis in
immunocompetent hosts. In immunocompromised patients, CMV causes various
systemic infections, such as pneumonia, hepatitis, retinitis, esophagitis, and
diarrhea.
(Choice B) Babesia divergences endemic on the northeastern coast of the United
States. It is transmitted by tick bites and causes babesiosis, a malaria-like illness
with a predilection for asplenic patients.
(Choice C) Toxoplasma infection causes a mononucleosis-like illness in
immunocompetent hosts and CNS involvement in immunocompromised patients.
(Choice D) Isospore belli causes chronic, watery, profuse diarrhea in
immunocompromised patients, especially those with AIDS.
(Choice E) Brucella melitensis is acquired by drinking infected milk or by direct
contact with infected sheep and goats. Fever, malaise, lymphadenopathy, and
hepatosplenomegaly characterize the resultant “brucellosis,” which is extremely rare
in the United States.
(Choice G) Campylobacter fetus (or intestinalis) is an opportunistic pathogen that
infects immunocompromised persons. Infection can cause septicemia in newborns,
women in the third trimester of pregnancy, and debilitated elderly patients.
Educational Objective:
Trypanosoma cruzi is endemic in rural areas of Central and South America. It causes
Chagas disease (American trypanosomiasis). This parasite can destroy the myenteric
plexi in the esophagus, intestines, and ureters, causing secondary achalasia,
megacolon, and megaureter, respectively.

65
USMLE WORLD STEP 1 MICROBIOLOGY

Q NO 155: A 5-year-old boy is found to have a high serum level of antibodies


against polyribitol ribose phosphate (PRP). The antibodies would most likely A.
carry protection against
Pyelonephritis
B. Miliary tuberculosis
C. Rheumatic fever
D. Osteomyelitis
E. Epiglottitis
F. Malignant pustule

Explanation:
Epiglottitis is almost exclusively caused by Herophilus influenzae type b and was
commonly seen in children between 2 and 7 years old prior to introduction of the
polysaccharide-protein conjugate Hib vaccine in the late 1980s. It was also the
common cause of meningitis prior to the vaccination.
Epiglottitis is usually an acute infection, with an abrupt onset of obstructive laryngeal
edema. Diagnosis can be made on symptoms of acute fever inspiratory stridor,
drooling odynophagia, and positive “thumb sign” on lateral cervical x-ray due to the
edematous epiglottis. The capsule of H. influenzae is immunogenic and is composed
of a linear polymer composed of ribose, ribitol, and phosphate referred to as
polyribosyl-ribitol-phosphate (PRP). Antibodies to PRP facilitate complement-
dependent phagocytosis and killing thru opsonization.
To effectively prevent infection with H. influenzae type b individuals must mount an
antibody response against the capsular material. In children less than 2 months old
humoral immunity to H. influenzae is received transplacentally from maternal lgG if
the mother is immune. The first dose of H. influenzae type b vaccine is given at 2
months of age.
(Choice A) Pyelonephritis is most commonly caused by F coil in both adults and
children and accounts for over 90% of cases in some studies.
(Choice B) Miliary tuberculosis is caused by Mycobacterium tuberculosis.
Mycobacterial cell wall contains several complex lipids (mycolic acids).
(Choice C) Rheumatic fever is a sequela of untreated group A Streptococcal
pharyngitis. Antistreptolysin C (ASO) are elevated with recent streptococcal
infection.
(Choice D) Osteomyelitis in children and adults with no other comorbidities is most
commonly caused by Staphylococcus aureus.
(Choice F) Malignant pustule (painless ulcer with black eschar and local edema) is
caused by B. anthracis. It also has a capsule but is composed of D-glutamate.

Educational Objective:
The Haemophilus influenzae type b (Hib) vaccine is composed of polyribosyl-ribitol-
phosphate (PRP) a component of the Hib capsule, conjugated with diphtheria or
tetanus toxoid. Immune activation with antibody production and memory B-
lymphocyte induction against PRP provides lasting immunity against Hib in children
as young as 2 months old. Epiglottitis is almost exclusively caused by Haemophilus
influenzae type b.

66
USMLE WORLD STEP 1 MICROBIOLOGY

Q NO 156: A 4-year-old Asian immigrant is brought to clinic with high fever nasal
discharge lacrimation and dry cough. Physical examination reveals several blue-
white spots on his buccal mucosa. Over the next several days, which of the
following is most likely to develop?
A. Bronchiolitis
B. Parotitis
C. Profuse diarrhea
D. Skin rash
E. Jaundice

Explanation:
Rubeola is caused by infection with the measles virus a member of the
paramyxovirus family. The measles virus is an enveloped, non-segmented, negative-
sense RNA virus. A classic clinical manifestation of measles is the presence of
numerous small blue-white spots on the buccal mucosa adjacent to the second
molars (“Koplik spots”). The Koplik spots are sometimes likened to “grains of sand”
on an erythematous base. When Koplik spots are accompanied by cough, coryza,
and conjunctivitis measles infection is the likely diagnosis. In fact a mnemonic for
the initial clinical presentation of typical measles is “CCCK” (cough, coryza, and
conjunctivitis and Koplik spots). In typical measles the appearance of ”CCCK”
heralds the development of a cutaneous maculopapular rash with in the next day or
two.
This patient’s recent immigration status also supports a diagnosis of measles as he
may not have been vaccinated against measles mumps and rubella (MMR) at 12-15
months of age.
(Choice A) Occasionally patients with rubeola will develop radiographically evident
pneumonia or laryngotracheobronchitis (croup): bronchiolitis is not typically seen.
Respiratory syncytial virus (RSV) can cause an upper respiratory infection that
progresses to bronchiolitis, but RSV is not associated with buccal lesions.
(Choice B) If this patient were not vaccinated against measles mumps and rubella
(MMR) at 12-15 months of age he is at risk for developing mumps parotitis. Measles
however is not commonly associated with parotitis.
(Choice C) Rotavirus often causes an upper respiratory infection followed by
diarrhea.
(Choice E) Hepatitis A virus can cause jaundice but is not typically associated with
cough rhinitis or buccal lesions.

Educational Objective:
The tetrad of cough coryza, conjunctivitis, and Koplik spots (CCCK) is diagnostic of
measles (rubeola) infection. Koplik spots are tiny white or blue-gray lesions on the
buccal mucosa that precede the maculopapular skin rash.

67
USMLE WORLD STEP 1 MICROBIOLOGY

Q NO 157: Microscopic examination of a sputum sample from a 34-year-old male


with fever and cough reveals Gram positive lancet-shaped cocci in pairs. These
bacteria are likely to be:
A. Catalase positive
B. Optochin resistant
C. Bile soluble
D. Capable of complete hemolysis
E. Bacitracin sensitive
F. Able to grow in 6.5% NaCl

Explanation:
The finding of lancet-shaped Gram positive diplococci in a patient with fever and
cough is suggestive of infection with S. pneumoniae. All streptococci are catalase
negative. In the laboratory, the optochin and bile solubility tests are used to
differentiate pneumococci from S. viridans and Group D Streptococci. Alpha-
hemolytic organisms sensitive to optochin are identified as S. pneumoniae while
alpha-hemolytic organisms that are optochin-resistant are identified as S. viridans.
S. pneumoniae cannot grow in the presence of bile and is considered bile soluble, in
contrast to the Group D Streptococci (i.e. Enterococci, S. bovis)which can grow in
the presence of bile (Choice C).
(Choice A) All Staphylococci are catalase positive.
(Choice B) Streptococcus viridans is an optochin-resistant organism that is a
component of the normal oropharyngeal flora. It can be an etiologic agent in dental
caries as well as in subacute bacterial endocarditis.
(Choice D) Complete (or beta) hemolysis is characteristic of Group A streptococcus
(S. pyogenes), Group B streptococcus (S. agalactiae), and Staphylococcus aureus.
S. pneumoniae and viridans group streptococci exhibit alpha-hemolysis, also called
partial or green hemolysis.
(Choice E) Bacitracin susceptibility testing is used to differentiate Streptococcus
pyogenes from the other betahemolytic streptococci. Streptococcus pyogenes is the
only beta-hemolytic streptococcus that is sensitive to bacitracin.
(Choice F) Group D streptococci can grow in the presence of bile. These organisms
are further divided into enterococci and non-enterococci (S. bovis) based on their
ability to grow in hypertonic saline (6.5% NaCl). Enterococci grow in hypertonic
saline while the non-enterococci cannot. S. bovis endocarditis and bacteremia are
associated with colon cancer.

Educational Objective:
S. pneumoniae are Gram-positive, alpha-hemolytic, optochin-sensitive, bile-sensitive
diplococci. S. viridans is also alpha-hemolytic, but it is optochin-resistant. S.
pyogenes (Group A Strep) appears as Gram-positive cocci in chains and can be
identified by its susceptibility to bacitracin.

68
USMLE WORLD STEP 1 MICROBIOLOGY

69
USMLE WORLD STEP 1 MICROBIOLOGY

Q NO 158: Viral particles isolated from the nasal exudate of a 10-year-old male are
shown to lose their infectively once exposed to ether. It can be concluded that the
viral particles are most likely:

A. Heat-stable
B. Defective
C. Enveloped
D. Icosahedral
E. Recombinant

Explanation:
Ether and other organic solvents can inactivate the “enveloped” viruses, which by
definition have an outer lipid bilayer coat acquired from the host cell plasma or
nuclear membranes. The solvent-induced disruption or dissolution of the envelope
lipid results in a loss of viral infectivity. Non-enveloped viruses are generally
resistant to the action of ether.
(Choice A) Virus infectivity is generally destroyed by heating to 50-60°C for 30
minutes with a few exceptions. The addition of a lipid bilayer envelope to the viral
nucleocapsid does not appear to confer greater heat stability.
(Choice B) Defective viruses are functionally deficient in some aspect of viral
replication, often lacking part of the normal viral genome. Ether exposure can inhibit
infectivity in non-defective, fully intact viruses.
(Choice D) The capsid symmetry of a virion is not directly related to its ether
sensitivity, which is determined primarily by the presence or absence of a lipid
bilayer envelope surrounding the nucleocapsid.
(Choice E) Viral recombination —which is defined as the exchange of genes between
two chromosomes (or chromosomal intermediates) by crossing over— does not
depend on the presence of a nucleocapsid envelope. Ether sensitivity, however,
depends upon the presence of a lipid bilayer envelope.

Educational Objective:
Ether and other organic solvents dissolve the lipid bilayer that makes up the outer
viral envelope. Loss of infectivity after ether exposure is a characteristic feature of
enveloped viruses.

70
USMLE WORLD STEP 1 MICROBIOLOGY

Q NO 159: A 5-year-old child who recently moved from Eastern Europe is brought
to the emergency room with difficulty breathing and a low-grade fever. Physical
examination reveals a grayish pharyngeal exudate, enlarged cervical lymph nodes
and partial soft palate paralysis. Bacteria isolated from the pharyngeal exudate
demonstrate exotoxin production in the laboratory. This exotoxin acts via the
following mechanism:
A. Intracellular protein ribosylation
B. Degradation of lecithin
C. Increased sensitivity to histamine
D. Inactivation of the 60s ribosomal subunit
E. Autoimmune activation by antigenic mimicry

Explanation:
This child has diphtheria, a disease caused by the C. diphtheriae AB-exotoxin. The A
(active) subunit of this exotoxin transfers a ribose residue from NAD to a histidine on
elongation factor-2 (EF-2). The EF-2 protein is essential for peptide chain
translocation on the ribosome in the process of translation. Ribosylation inactivates
EF-2, thus inhibiting protein synthesis and causing cell death.
(Choice B) Lecithinase (also known as phospholipase C or C. perfringens alpha toxin)
is the exotoxin released by Clostridium perfringens. This enzyme degrades
membrane phospholipids (including lecithin), leading to cell membrane destruction
and cell death. C. perfringens is the organism that causes gas gangrene.
(Choice C) Pertussis toxin is an AB exotoxin released by Bordetella pertussis that
stimulates intracellular G-proteins to increase cAMP production, leading to increased
insulin production lymphocyte and neutrophil dysfunction, and increased sensitivity
to histamine.
(Choice D) Shigellas shiga toxin and enterohemorrhagic E. coli (EHEC)’s shiga-like
toxin cause inactivation of the 60s ribosomal subunit. (This is in contrast to the
diphtheria toxin which inactivates elongation factor-2, not the ribosome itself.)
(Choice E) Group A streptococcal (GAS) pharyngitis can lead to rheumatic fever via
antigenic mimicry. Because GAS and cardiac myocytes have similar antigenic protein
sequences, when the immune system mounts a response against GAS it may also
cause myocarditis and valve destruction overtime.

Educational Objective:
Diphtheria toxin is an AB exotoxin that ribosylates and inactivates elongation factor-
2 (EF2). This inhibits protein synthesis and ultimately leads to cell death.

71
USMLE WORLD STEP 1 MICROBIOLOGY

Q NO 160: A 25-year-old male is hospitalized with fever, anorexia and fatigue. He


admits to IV drug abuse and many episodes of unprotected sex over the past few
years. You obtain the following dynamics of serologic markers in this patient (see
the graph below).

This patient most likely:

A. Had acute hepatitis B and successfully recovered


B. Had an exacerbation of preexisting hepatitis B
C. Had acute hepatitis B that progressed to chronic hepatitis with low infectivity
D. Had acute hepatitis B that progressed to chronic hepatitis with high infectivity
E. Was vaccinated against HBV during the hospitalization

Explanation:
The serologic markers for hepatitis B infection appear on a predictable timeline with
a long asymptomatic incubation period (lasting an average of 6-8 weeks) followed
by acute disease lasting several weeks to months. HBsAg first appears before the
onset of symptoms, peaks when the patient is most ill, and then becomes
undetectable in 3-6 months. Shortly after HBsAg first appears, HBeAg and HBV DNA
can be detected in the serum and are markers of active viral replication. Anti-HBcAg
1gM and elevated serum transaminases appear shortly before symptom onset. Over
the next few months, the anti-HBcAg 1gM component is replaced with an lgG
component. Anti-HBe appears shortly after HBeAg vanishes and suggests subsiding
viral activity. Anti-HBs lgG arises once the acute disease has resolved, and is
typically not detectable until weeks or months after HBsAg has disappeared. This
antibody may persist indefinitely, conferring lifelong immunity.
Given this serologic marker graph then it appears that this patient has a persistence
of HBsAg and HBeAg over a long period with low to moderate levels of anti-HBcAg
and anti-HBeAg. These findings are suggestive of an acute hepatitis B infection that
has not resolved but rather has progressed to a highly infectious chronic hepatitis B
(note the persistence of HBeAg despite the co-existence of anti-HBeAg).
(Choice A) Were this patient to have fully recovered from his bout of acute hepatitis
B, all antigens should have dropped to undetectable levels and he should have high
levels of anti-HBsAg lgG and anti-HBcAg lgG.
(Choice B) An exacerbation of preexisting hepatitis B would be characterized by a
decline followed by a spike in antigen levels.

72
USMLE WORLD STEP 1 MICROBIOLOGY

(Choice C) An acute hepatitis B that progressed to chronic hepatitis with low


infectivity would be indicated by a graph similar to the one shown for this patient
with low or undetectable levels of HBeAg (the marker for high infectivity).
(Choice E) Vaccination against HBV would be identified by a surge in anti-HBsAg
with no anti-HBcAg, anti-HBeAg, or viral antigens detected.

Educational Objective:
If HBeAg persists for several months and host antibodies remain at low or moderate
levels, suspect chronic hepatitis B infection with high infectivity.

73
USMLE WORLD STEP 1 MICROBIOLOGY

Q NO 161: A 34-year-old female presents with fever, dysuria and flank pain; a
urine culture grows motile Gram-negative rods in colonies that have a metallic A.
sheen on EBM agar and demonstrate hemolysis on blood agar. Which of the
following is the most important pathogenic bacterial factor responsible for this
patient’s condition?
Capsule
B. Heat-labile enterotoxin
C. O antigen
D. Fimbrial antigen
E. Lipid A
Explanation:
E. coli is part of the normal bacterial flora of the intestinal tract. Itis also a motile
gram-negative bacillus that is facultatively anaerobic and able to ferment both
lactose and glucose. E. coli grows well on blood MacConkey, and eosin methylene
blue (EMS) agar plates. EMB agar is a selective and differential medium used to
isolate and identity enteric pathogens from contaminated clinical specimens. On EMS
agar, organisms that ferment lactose such as F. coli bind to dye in the agar and
produce blue-black colonies that have a distinct metallic sheen. Most strains of F. cc
demonstrate beta-hemolysis on blood agar. F. coli is responsible for urinary tract
infections (UTI), neonatal meningitis, and several types of gastroenteritis. The
virulence factors expressed by a particular strain of E. coli will determine disease.
Adhesion is one of the most important virulence factors expressed by E. coli more
specifically; fimbriae (Type 1 fimbriae, P fimbriae and S fimbriae) permit the
adhesion of F. coli to uroepithelial cells (choice D). F. coli causes approximately8O%
of all urinary tract infections. UTIs are most common in females, owing to the
significantly shorter length of the female urethra. F. coli originate in the feces and
spreads to colonize the perineal, genital, and—most importantly—the periurethral
regions. During, for instance, sexual intercourse or bladder catheterization, F. coil
can be propelled into the urethra and bladder from the colonized periurethral region.
P fimbriae (pyelonephritis-associated pili) adhesion allow the uropathogenic F. coli to
colonize the bladder and ureters, causing cystitis and pyelonephritis.
(Choice A) F. coli strains that cause neonatal meningitis synthesize K-1 capsular
antigens. The K-1 antigen is considered the major determinant of virulence among
strains of F. coli that cause neonatal meningitis.
(Choice B) Enterotoxigenic E. coli produce heat-stable (ST) and/or heat labile (LT)
enterotoxins. In both structure and mode of action, the LT enterotoxin is very similar
to the cholera toxin: both act to increase intracellular cAMP by activating a
stimulatory G-protein in the gut mucosal cell apical membrane.
(Choice C) The O antigen is a cell wall outer membrane polysaccharide antigen used
to classify gram-negative bacteria. The O antigen is the most extracellular
component of lipopolysaccharide (see below).
(Choice E)The Lipid A in LPS is similar for all enterobacteria: Lipid A causes
activation of macrophages, which leads to widespread release of IL-1 and TNF-alpha,
which in turn cause the signs and symptoms of septic shock.
Educational objective:
F. coli is one of the dominant components of the normal flora in the intestinal tract
of humans and animals. F. coli cause approximately 80% of all urinary tract
infections. P-fimbriae are the most important virulence factor that uropathogenic E.
coli express. Without P-fimbriae, F. coli would not be able to bind to uroepithelial
cells and would not be able to ultimately infect the bladder, ureters, and kidneys.
Instead, the bacteria would simply be washed away during urination.

74
USMLE WORLD STEP 1 MICROBIOLOGY

Q NO 162: Autopsy of a 14-year-old male who died of septic shock shows extensive
hyperemia and infarcts of his internal organs. Purpuric cutaneous lesions and
hemorrhagic destruction of both adrenals are also present. The organism most likely
responsible for this patient’s death more commonly causes which of the following?

A. Urinary tract infection


B. Pneumonia
C. Infectious diarrhea
D. Endocarditis
E. Meningitis
F. Skin infection

Explanation:
N. meningitidis bacteremia may be accompanied by signs and symptoms of sepsis
including spiking fevers, chills, arthralgias, and myalgias, as well as purpuric
cutaneous lesions and hypotension that may progress shock and death. Waterhouse-
Friderichsen syndrome is a complication of meningococcemia that may involve
adrenal gland destruction, disseminated intravascular coagulation (DIC), and shock.
Although N. meningitidis often causes meningitis, N. meningitidis bacteremia
frequently occurs without meningeal involvement.
(Choice A) E. coli is the most common cause of urinary tract infection. S.
saprophyticus, P. mirabilis, Klebsiella and enterococci are other common causes.
(Choice B) Community acquired lobar pneumonia is most commonly caused by
Streptococcus pneumoniae. This organism is also the most common cause of
meningitis in adults.
(Choice C) Infectious diarrhea is frequently caused by viral gastroenteritis, especially
in children. In adults, the most common causes of bacterial infectious diarrhea are
Campylobacter, Salmonella, Shigella and E. coli.
(Choice D) Acute endocarditis is most frequently caused by S. aureus while subacute
bacterial endocarditis is most commonly caused by S. viridans. Though S. aureus
bacteremia can cause rapid-onset shock with circulatory collapse, neither of these
agents is a common cause of meningitis or bilateral adrenal hemorrhage.
(Choice F) Skin infection usually occurs at sites of where the stratum corneum has
been disturbed, such as wounds and areas of inflammation (e.g. eczema). Causative
agents are usually normal skin flora (e.g. staphylococci, Group A streptococci).
Group A streptococcal skin infection can potentially cause post-streptococcal
glomerulonephritis and S. aureus skin infection can potentially cause bacteremia and
acute endocarditis.

Educational Objective:
Meningococcal sepsis may be associated with disseminated intravascular coagulation
and hemorrhagic destruction of the bilateral adrenal glands. This constellation of
findings is known as the Waterhouse-Friderichsen syndrome. N. meningitidis sepsis
does not necessarily include meningitis.

75
USMLE WORLD STEP 1 MICROBIOLOGY

Q NO 163: A researcher finds that Streptococcus pneumoniae injected


intraperitoneally is uniformly fatal to mice. Antibodies with which of
the following in vitro effects on the bacterium would prevent this A.
outcome? Decreased
motility
B. Capsular swelling
C. Inhibited hemolysis
D. Loss of cytotoxicity
E. Impaired adhesion

Explanation:
Pathogenic species of S. pneumoniae produce an outer polysaccharide capsule that
prevents phagocytosis by the host immune system. This capsule is S. pneumoniae
major virulence factor; in laboratory experiments where itis removed, the organism
becomes nonpathogenic. Because the capsule is antigenic, S. pneumoniae infection
elicits anti-capsule antibody production and lasting immunity against that particular
strain. (Opsonization of the capsule by antibody and complement renders the
organism non virulent.) Unfortunately there are 82 different capsule types due to
variations in the polysaccharide, and immunity is only to the single infecting strain.
The pneumococcal vaccine generates immunity against the capsular antigens of
several important strains.
When viewed under a microscope, the capsule swells when specific anti-capsular
antibodies are added (Choice B), an effect known as the “quellung reaction”. This
reaction can be used to identify S. pneumoniae and to serotype the isolate.
(Choice A) S. pneumoniae is a nonmotile organism.
(Choice C) S. pneumoniae is an alpha-hemolytic organism, meaning that it causes
incomplete or “green” hemolysis on blood agar. This property would not be affected
by addition of an antibody.
(Choice D) Pneumococci can destroy ciliated epithelial cells via the toxin
pneumolysin. This protein also helps the pneumococcus survive phagocytosis by
suppressing the phagocyte’s oxidative burst. Pneumolysin is not the target of any
specific antibody nor is it a major virulence factor.
(Choice E) The adhesion of S. pneumoniae to epithelial cells is mediated by a poorly
described protein called adhesin. Adhesin is not a major virulence factor nor is it the
target of specific antibodies.

Educational Objective:
S. pneumoniae expresses a polysaccharide capsule that inhibits phagoc4osis by
macrophages and polymorphonuclear leukocytes. It is the primary virulence factor,
without which S. pneumoniae cannot cause disease.

76
USMLE WORLD STEP 1 MICROBIOLOGY

Q NO 164: Mature B Iymphoc4es exposed to a specific virus in vitro acquire the


ability to proliferate indefinitely while maintaining the ability to secrete A.
immunoglobulins. Confirmation of an in vivo infection with this virus is typically
established with:
Tzanck smear
B. Papanicolaou smear
C. Heterophil antibodies
D. Cryoglobulins
E. Circulating surface antigen
F. Hemadsorption in the cell culture

Explanation:
Epstein-Barr virus (EBV) commonly infects B cells, stimulating them to enter the cell
cycle and proliferate continuously (a process termed “transformation” or
“immortalization”). This process is accomplished when EBV binds to the cell surface
and the EBV-encoded oncogenes activate proliferative and anti-apoptotic signaling
pathways within the B cell. EBV-infected B cells can be propagated indefinitely in
vitro. In an immunocompetent host, EBV-induced B cell proliferation is held in check
by a vigorous cell-mediated and humoral immune response. The ability to secrete
immunoglobulins and B-cell activation products (eg, CD23) is maintained by the
immortalized B cell population, with very few of these cells releasing virus particles
at any one time.
One serologic means of diagnosing EBV infection is the Monospot test which detects
a heterogeneous group of 1gM antibodies that react with the heterophil antigens
present on horse red blood cells. Agglutination of these horse RBCs by human serum
is a sensitive and highly specific test for EBV infection in the B-cell compartment of
the human host. The means by which EBV stimulates this particular immune
response remains unclear; it appears that EBV either induces a humoral response to
heterophil antigens or stimulates a non-specific polyclonal activation of B cells.
(Choice A) A Tzanck smear of material obtained from the base of a
vesiculoulcerative lesion is used to diagnose herpes simplex (HSV) or varicella zoster
(VZV) infection. The epithelial cells scraped from the ulcer base are prepared with a
Wright-Giemsa stain and examined for multinucleated giant cells and intranuclear
inclusions.
(Choice B) The Papanicolaou (“pap”) smear is used to screen cervical cytology
specimens for dysplasia caused by oncogenic strains of human papilloma virus.
(Choice D) Cryoglobulins are cold-precipitable serum proteins that contain
immunoglobulins. The pathogen most commonly associated with cryoglobulinemia is
the hepatitis C virus.
(Choice E) EBV typically causes a latent infection of B cells that is characterized by
persistence of the full viral genome but no full virion replication, no Fytic release,
and sparse expression of viral proteins. Because normally only minimal amounts of
EBV-specific antigens circulate in an immunocompetent host, a blood test for such
antigens would be insensitive to latent EBV infection.
(Choice F) Hemadsorption by infected B cells would suggest that hemagglutinins or
glycoproteins with a high affinity for RBCs are being expressed on the host cell
surface. While EBV is not characteristically associated with hemadsorption, some
influenza and parainfluenza viruses are.

77
USMLE WORLD STEP 1 MICROBIOLOGY

Educational Objective:
Epstein-Barr virus (EBV) commonly infects B cells, stimulating them to enter the cell
cycle and proliferate continuously (a process termed ‘transformation” or
“immortalization”). EBV is an oncogenic virus that promotes polyclonal B cell
proliferation and heterophil antibody production. The Monospot test is a highly
specific test used to diagnose EBV infection; the test is positive when horse red
blood cells are agglutinated in vitro by heterophil antibodies.

78
USMLE WORLD STEP 1 MICROBIOLOGY

Q NO 165: A 5-year-old Caucasian male is brought to the ER with fever and a new
skin rash (see the slide below). One week ago, his younger sister had similar
symptoms that resolved spontaneously.

Which of the following is the most likely diagnosis given this patient’s symptoms?

A. Rubella
B. Scarlet fever
C. Roseola infantum
D. Erythema infectiosum
E. Hand-foot-and-mouth disease

Explanation:
The image above demonstrates the typical “slapped-cheek” rash of erythema
infectiosum. It is also called “fifth disease” because itis one of the six common viral
exanthems of childhood. This disease is caused by parvovirus B19 (PB19), a single-
stranded DNA virus that can be transmitted by the respiratory route, via infected
blood, and transplacentally. Infection with PB19 results in erythema infectiosum in
children aplastic crisis in patients with preexisting hemolytic disease and hydrops
fetalis if transmitted from mother to fetus.
Erythema infectiosum is a benign childhood illness that can occasionally affect
adults. The incubation period is 7-14 days, and infection with PB19 presents with
malaise coryza, headache and fever associated with a bright-red rash on the cheeks.
This rash is called a “slapped-cheek rash” and usually spares the nasolabial folds. An
erythematous rash in a reticular (lace-like) pattern also appears on the trunk and
extremities but spares the palms and soles. The symptoms of disease generally
resolve during the following week. Sometimes (most commonly in adult patients)
the disease is complicated by symmetric peripheral arthropathy of the fingers,
wrists, and ankles.
(Choice A) Rubella presents with mild fever and discrete maculopapular rash that
resolves in 3-5 days. Occipital and posterior cervical lymphadenopathy is
characteristic. The characteristic rash of rubella is maculopapular spreading from the
head inferiorly to the trunk and then the extremities.
(Choice B) Scarlet fever is a group A beta-hemolytic streptococcal infection. It
presents with fever sore throat and erythematous macular “sandpaper” like rash.

79
USMLE WORLD STEP 1 MICROBIOLOGY

(Choice C) Roseola infantum is caused by herpes virus type 6, affects children 6-15
months old and is the sixth common childhood exanthem. It manifests with a high
fever that lasts for 3-5 days and resolves after the appearance of a maculopapular
rash that starts on the trunk and spreads peripherally.
(Choice E) Hand-foot-and-mouth disease is caused by Coxsackie type A virus. It
presents with fever and ulcers on the tongue and oral mucosa as well as with a
maculopapular and/or vesicular rash on the palms and soles.

Educational Objective:
Erythema infectiosum (fifth disease) is caused by parvovirus B19 and is transmitted
most commonly via the respiratory route. Symptoms of this infection include a
bright-red rash on the cheeks (“slapped cheeks”) and “lacelike” rash on the trunk
and extremities.

80
USMLE WORLD STEP 1 MICROBIOLOGY

Q NO 166: A group of military recruits develops a disease characterized by high


fever, headache, nausea and hemorrhagic skin rash. Gram-negative kidney A.
bean shaped cocci are isolated from the blood of one of the recruits. The blood
concentration of which of the following microbial components would show the
closest correlation with these patients morbidity and mortality?
Cell wall lipo oligosaccharide
B. Outer membrane protein
C. Inner membrane protein
D. Capsular polysaccharide
E. Exotoxin

Explanation:
The lipo oligosaccharide (LOS) of N. meningitidis is analogous to the
lipopolysaccharide of enteric gram negative rods. The cell wall lipo oligosaccharide
(LOS) acts as an endotoxin and is associated with many of the toxic effects of
meningococcal disease. Plasma levels of LOS are closely associated with disease
manifestations and outcomes in meningococcal infections.
As in other Gram-negative infections, during the infectious process the growth and
lysis of meningococci causes the release of outer membrane vesicles (OMV)
composed of lipo oligosaccharide (LOS) into the bloodstream. The severity of the
meningococcal disease has been shown to correlate with increasing concentrations
of OMV-bound LOS. As with LPS from other Gram-negative bacteria, LOS causes
sepsis by induction of a systemic inflammatory response characterized by the
production of tumor necrosis factor alpha (TNF-a), interleukin-1β (IL-1β), IL-6, and
IL-8. Additionally LOS has been implicated as the cause of cutaneous petechiae and
hemorrhagic bullae found in meningococcemia. It is also believed to be the cause of
bilateral adrenal cortical hemorrhage characteristic of the Water house-Friderichsen
syndrome.
(Choice B) Outer membrane protein is not involved with the production of sepsis in
meningococcemia, but lipo oligosaccharide is a component of the outer membrane,
and as described above, LOS is the causative agent in the septic symptoms in
patients affected by meningococcemia.
(Choice C) Inner membrane protein is not a structure that is involved in the
morbidity and mortality associated with meningococcal disease. Inner membrane
proteins as well as outer membrane proteins are not known virulence factors in
meningococci; they principally act in cellular homeostasis for the bacterium.
(Choice D) The capsular polysaccharide of the group C and group A serotypes of N.
meningitidis elicit an immune response, but the group B serotype has been shown to
be a poor immunogen and is not covered by the meningococcal polysaccharide
vaccine. The capsular polysaccharide is not associated with the morbidity and
mortality attributed to meningococcal disease, although it does assist the
meningococcus in resisting phagocytosis. Individuals who become colonized with N.
meningitidis produce protective antibodies directed against the capsular
polysaccharide for that particular serotype.
(Choice E) Exotoxin production is not a known mechanism of pathogenicity for the
Neisseria species. Better known exotoxins are the Staphylococcal toxic shock
syndrome toxin and enterotoxins (food poisoning), Botulinum toxin (C. botulinum -
botulism), Diphtheria toxin (c. diphtheriae - diphtheria), Tetanus toxin (C. tetani -
tetanus), and Pertussis toxin (B. pertussis—whooping cough).

81
USMLE WORLD STEP 1 MICROBIOLOGY

Educational Objective:
Meningococcal lipoligosaccharide (LOS) is responsible for many of the toxic effects
observed in meningitis and meningococcemia. Its blood level correlates with
morbidity and mortality.

82
USMLE WORLD STEP 1 MICROBIOLOGY

Q NO 167: A 32-year-old Caucasian male develops profuse watery diarrhea with


abdominal cramps, nausea and vomiting during a trip to Mexico. The diarrhea A.
resolved in a couple of days without antibiotic treatment. This patient’s
symptoms are most likely related to:
Intestinal invasion
B. Stacked-brick intestinal adhesion
C. Shiga-like toxin
D. Cholera-like toxin
E. Endotoxin release

Explanation:
This self-limited diarrhea “traveler’s diarrhea” is characteristic of Enterotoxigenic E.
coli (ETEC). E. coli are gram- negative motile enteric rods that normally colonize the
human GI tract. The usual presentation of ETEC gastroenteritis is watery diarrhea
with abdominal cramping, nausea and vomiting, and possibly a low fever. ETEC
causes diarrhea in infants and travelers in underdeveloped countries or regions with
poor sanitation. The disease requires colonization by the ETEC, usually in the small
intestine, and subsequent elaboration of the heat labile (LT) and! or heat stable (ST)
enterotoxins. Both of these toxins are plasmid-encoded. The LT enterotoxin is very
similar to cholera toxin in both structure and mode of action. LT increases
intracellular cAMP in gut mucosal cells by activating the stimulatory Gs membrane G
protein thus activating adenylate cyclase. The ST enterotoxin is not inactivated by
heat because of its small molecular size. ST causes an increase in cyclic GMP in the
host cell cytoplasm by binding to and activating guanylate cyclase located on the
apical membranes of host gut mucosal cells. The action of both the LT and the ST
enterotoxins is increased secretion of fluid and electrolytes resulting in watery
diarrhea. Fimbriae (pili) are responsible for bacterial adhesion to the gut mucosa.
(Choice A) Intestinal invasion is a property of gastroenteritis caused by Salmonella
Shigella, Enteroinvasive E. coli, and Campylobacter jejuni as well as the protozoan
Entamoeba histolytica. Invasion frequently leads to bloody or hemorrhagic diarrhea
due to the fact that the mucosal cells invaded by the bacteria frequently die and
cause some amount of necrosis.
(Choice B) Stacked-brick intestinal adhesion is a characteristic of enteroaggregative
Escherichia coli (EAEC). These organisms adhere to human jejunal, ileal, and colonic
mucosa in an aggregative or stacked-brick pattern and do not invade. This strain of
E. coli is implicated in persistent diarrhea in infants in developing countries, but little
is known about the pathogenesis of disease caused by this organism.
(Choice C) Shiga-like toxin is produced by EHEC. This toxin shares many properties
with the toxin produced Shigella dysenteriae — Shiga toxin, It, like Shiga toxin, acts
by inactivating the 60s ribosomal subunit in human cells, thereby inhibiting human
cell protein production and leading eventually to cell death.
(Choice E) Endotoxin release is a mechanism of toxicity for all gram-negative
bacteria. Once they gain access to and multiply within the blood stream they
elaborate LPS that induces an inflammatory response mediated by TNE alpha and IL-
1 secreted from activated macrophages.
Educational objective:
Traveler’s diarrhea is most frequently related to ETEC that produces heat labile (LT,
choleragen-like) and heat stabile (ST) enterotoxins. LT activates adenylate cyclase
leading to increased intracellular cAMP, and ST activates guanylate cyclase leading
to increased intracellular cGMP. Both cause water and electrolyte loss and watery
diarrhea.

83
USMLE WORLD STEP 1 MICROBIOLOGY

Q NO 168: An 18-year-old college freshman is brought to the emergency room with


a high fever, confusion and headaches. Physical examination reveals nuchal rigidity
and a purpuric rash on his lower extremities. This infection could have been
prevented by a vaccine containing:

A. Heat-killed bacteria
B. Bacterial outer membrane protein
C. Capsular polysaccharide
D. Inactivated toxin
E. Live attenuated bacteria

Explanation:
This patient is experiencing the classic signs and symptoms of meningococcal
meningitis. Neisseria meningitidis is a gram negative diplococcus transmitted via
respiratory secretions. Close living conditions (e.g. college dormitories. prisons, and
military barracks) promote its spread.
N. meningitidis has several virulence factors, including a polysaccharide capsule.
(impairs phagocytosis of the bacteria) LPS endotoxin, pili (for attachment to the
respiratory mucosa), and IgA protease (cleaves secretory IgA that would otherwise
inactivate the pili). Antibodies against the polysaccharide capsule confer immunity to
N. meningitidis. The meningococcal vaccine contains many of the N. meningitidis
capsular polysaccharides and stimulates the production of anticapsular antibodies.
Because the protection provided by the vaccine is incomplete and transient, it is only
used in high-risk groups like military recruits and college students.
(Choice A) Heat-killed bacteria are included in the Bordetella pertussis, Vibrio
cholerae (rarely used), and Yersinia pest/s vaccines.
(Choice B) The Borrelia burgdorferi (Lyme disease) vaccine contains recombinant
bacterial outer surface protein. (Choice D) The Cotype bacterium diphtheriae and
Clostridium titan vaccines contain inactivated toxin (toxoid).
(Choice E) The BCG vaccine is used in some parts of the world (not in the USA) for
protection against Mycobacterium tuberculosis. It contains live-attenuated organisms
of a different mycobacterial species. Vaccines for Francisella tularensis and
salmonella typhl also contain live-attenuated bacteria.

Educational Objective:
Neisseria meningitidis can cause upper respiratory infection, meningitis and
meningococcemia. Immunity against these bacteria is provided by antibodies against
their polysaccharide capsules. The meningococcal vaccine contains immunogenic
capsular polysaccharides from four major serotypes of N. meningitidis and induces
production of protective anti capsular antibodies.

84
USMLE WORLD STEP 1 MICROBIOLOGY

Q NO 169: A sample of contaminated moist soil is heated to 100°C for 15 minutes.


Which of the following bacteria is most likely to be recovered from the soil sample
following heat exposure?

A. Streptococcus pyogenes
B. Listeria monocytogenes
C. Escherichia coil
D. Bacillus anthracis
E. Brucella melitensis

Explanation:
This question essentially asks which of the bacteria listed are found in soil and
capable of surviving past the boiling point of water 100°C (212°F). Members of the
Bacillus and Clostridium geniuses are commonly found in soil and are able to survive
high temperatures desiccation and chemical agents by forming spores. Spore-
forming bacteria can be killed by autoclaving.
(Choice A, B, C and E) These organisms do not form spores and are likely to be
killed if heated above 10000.

Educational Objective:
Spore-forming bacteria can survive boiling. Bacillus anthracis and members of the
genus Clostridium are potentially pathogenic bacteria found in the soil and capable
of forming spores.

85
USMLE WORLD STEP 1 MICROBIOLOGY

Q NO 170: A 38-year-old Caucasian female who works as a nurse at a large


community hospital has the following readings of serologic markers for viral Anti-
hepatitis: HAV
1gM Negative
Anti-HAV lgG Positive
HBsAg Negative
HBeAg Negative
Anti-HBsAg Positive
Anti-HBcAg 1gM Negative
Anti-HBcAg lgG Negative
Anti-HBeAg Negative
Anti-HCV Negative

This patient most likely:

A. Is actively shedding HAV


B. Is infected with HDV
C. Is an asymptomatic carrier of HBV
D. Has recovered from HBV infection recently
E. Has been vaccinated against HBV
F. Has a chronic hepatitis but is non-contagious

Explanation:
This is the typical profile for an individual who has both a remote history of hepatitis
A virus infection and vaccination against hepatitis B virus. Had she actually
experienced HBV infection, her anti-HBcAg lgG would have been positive.
The hepatitis B vaccine is a safe and highly effective recombinant vaccine that
contains HBsAg, a surface antigen that stimulates the production of anti-HBsAg in
the host. Anti-HBsAg is a protective antibody and confers immunity to vaccinated
individuals. Therefore seronegative individuals who receive the vaccination will
develop immunity and be positive for anti-HBsAg but negative for HBsAg.
Serological markers for the hepatitis B virus include the following:
HBsAg: The first virological marker detected in the serum after inoculation it
precedes both the elevation of serum aminotransferases and the onset of clinical
symptoms. It remains detectable during the entire symptomatic phase of acute
hepatitis B and suggests infectivity.
Anti-HBs: Appearing in the serum after successful HBV vaccination or the clearance
of HBsAg, this marker remains detectable for life. It serves as an indicator of non
infectivity and immunity. However, there is a time lag between the disappearance of
HBsAg and the appearance of anti-HBs in the serum, which is termed the “window
period.”
HBcAg: This marker is not detectable in serum as it is normally sequestrated within
the HBsAg coat.
Anti-HBc: Appearing in the serum shortly after the emergence of HBsAg, this marker
remains detectable long after the patient recovers. The 1gM fraction signals the
acute phase of disease whereas the lgG fraction signals recovery from the disease.
Because 1gM anti-HBc is present in the “window period1” itis an important tool for
diagnosis when HBsAg has been cleared and anti-HBs is not yet detectable. Thus
1gM anti-HBc is the most specific marker for diagnosis of acute hepatitis B.
HBeAg: This antigen is detectable shortly after the appearance of HBsAg and
indicates active viral replication and infectivity. It is associated with the presence of
HBV DNA. HBeAg tends to disappear shortly after aminotransferase levels peak and

86
USMLE WORLD STEP 1 MICROBIOLOGY

before HBsAg is eliminated and is followed by the appearance of anti-HBe. Should it


persist for more than three months there is an increased likelihood of chronic
hepatitis B.
Anti-HBe: This marker suggests the cessation of active viral replication and low
infectivity.
(Choice A) Anti-HAV 1gM appears in the bloodstream at the onset of symptoms and
is a marker of acute infection. As the 1gM titer increases, fecal shedding of the virus
ends. Because this patient has only anti-HAV IgO, she is not shedding HAV.
(Choice B) The hepatitis D virus is replication defective and depends on the genetic
information provided by HBV for multiplication. Because this individual is not
infected with hepatitis B virus (her HBsAg is negative), she cannot be infected with
HDV.
(Choice C) The HBV carrier state is defined by the presence of HBsAg in serum for
six months or longer after initial recognition. This patient’s HBsAg is negative,
however, which rules this possibility out.
(Choice D) Recovery from HBV infection would be signaled by the presence of anti-
HBsAg lgG or anti-HBc lgG.
(Choice F) While the persistence of HBsAg in circulation meets the criteria for chronic
hepatitis, it does not necessarily indicate that the virus is successfully reproducing.
Chronic replication of HBV is demonstrated by persistence of HBsAg, HBeAg, and
HBV DNA (often accompanied by anti-HBc and anti-HBs).

Educational Objective:
Seronegative individuals who receive the hepatitis B vaccination will develop
immunity and be positive for anti-HBsAg but negative for HBsAg.

87
USMLE WORLD STEP 1 MICROBIOLOGY

Q NO 171: Three cases of severe pharyngitis were reported in a community of


immigrants. The patients had thick pharyngeal exudates, neck swelling and
difficulty swallowing. One of them died from severe heart failure. The toxin A.
responsible has a mechanism similar to the toxin of which of the following
bacteria?
Pseudomonas aeruginosa
B. Staphylococcus aureus
C. Clostridium difficile
D. Clostridium botulinum
E. Bordetella pertussis
F. Vibrio cholerae

Explanation:
P. aeruginosa produces several extracellular products including exotoxin A,
collagenase, elastase, fibrinolysin, phospholipase C and DNAse. These substances
assist in its invasion and dissemination in human tissues. Diphtheria toxin acts in a
similar manner to exotoxin, though they are structurally different. Both diphtheria
toxin and exotoxin A ribosylates and inactivate elongation factor-2 (EF-2), halting
human cell protein synthesis and causing cell death. Exotoxin A is a major virulence
factor and responsible for the high mortality associated with P. aeruginosa
septicemia.

Educational Objective:
Diphtheria toxin and pseudomonal exotoxin A both act by ribosylation and
inactivating elongation factor-2, inhibiting host cell protein synthesis and causing cell
death.

88
USMLE WORLD STEP 1 MICROBIOLOGY

Q NO 172: A 46-year-old female presents to the emergency room with fever, chest
pain and hemoptysis. She came to visit her family from Pakistan where she works
in a goat wool processing center. Chest x-ray shows widened mediastinum. Sputum
and blood cultures reveal large gram-positive rods that form ‘medusa head colonies
on standard media. The bacteria isolated from this patient most likely produce:

A. Antiphagocytic D-glutamate capsule


B. Antiphagocytic polysaccharide capsule
C. lgG-binding outer membrane protein
D. Intracellular polyphosphate granules
E. Peritrichous flagella

Explanation:
The occupational history of the patient wool processing, is an important risk factor to
elicit on history to aid in making the diagnosis of pulmonary anthrax. Pulmonary
anthrax is also known as “woolsorters disease” because exposure from handling
animal products such as animal hair, infected animal hides, hide-based clothing
products or wool has been associated with infection by Bacillus anthracis. Goat hair
is the most commonly implicated exposure in contraction of anthrax.
The spores of B. anthracis are very small, once they are inhaled they enter the
alveoli and are ingested by macrophages. From the lung the organisms rapidly move
to mediastinal lymph nodes and cause hemorrhagic mediastinitis. Once the spores
germinate into vegetative cells they will begin to produce the three-part anthrax
toxin and clinical symptoms will rapidly follow. Symptoms initially only consist of
myalgia fever and malaise but rapidly progress to hemorrhagic mediastinitis
(widened mediastinum on chest x-ray), bloody pleural effusions, septic shock and
death. Bacillus anthracis produces an antiphagocytic capsule that is required for
pathogenicity. The capsule is unique in that it contains D-glutamate instead of
polysaccharide. B. anthracis is an aerobic organism that can be grown on standard
culture media and forms non hemolyzing adherent colonies. On microscopy it forms
long chains that are described as being “serpentine” or “medusa head” on
appearance. (Choice B) Capsule in B. anthracis is composed of D-glutamate instead
of polysaccharide. (Choice C) Protein A is a virulence factor produced by S. aureus
which does possess the ability to bind the Fc portion of lgG and prevent opsonization
and complement-mediated killing of the bacteria.
(Choice D) Intracellular polyphosphate granules are a characteristic of
corynebacterium diphtheriae that can be visualized on microscopy after growth on
Loeffler medium and staining with methylene blue.
(Choice E) Peritrichous flagella are not present on the surface of Bacillus anthracis as
this organism is nonmotile. The term peritrichous flagella refer to flagella distributed
uniformly over the entire surface of a bacterial cell and are characteristic of highly
motile organisms such as Proteus mire bills.
Educational Objective:
1. Pulmonary anthrax, also known as woolsorters disease, is caused by inhalation of
spores most commonly while working with goat hair or hides. Hemorrhagic
mediastinitis evident as widened mediastinum on chest- x ray is an important clue.
2. On microscopy it forms long chains that are described as being “serpentine” or
”medusa head” on appearance.
3. Bacillus anthracis produces an anti phagocytic capsule that is required for
pathogenicity. The capsule is unique in that it contains D-glutamate instead of
polysaccharide.

89
USMLE WORLD STEP 1 MICROBIOLOGY

Q NO 173: An 8-year-old Middle Eastern immigrant is brought to clinic with low-


grade fever and a skin rash that started on his face and spread down his body. A.
Physical examination reveals postauricular tenderness. Which of the following is
the most likely cause of this patient’s disease?
Herpesvirus
B. Togavirus
C. Reovirus
D. Paramyxovirus
E. Parvovirus

Explanation:
Many recent immigrants to the United States will not have completed the CDC-
recommended vaccination regimen at the appropriate intervals. Itis quite likely then
that this patient was not vaccinated against measles, mumps, and rubella (MMR) at
age 12-15 months. Among the acute viral exanthems, measles (rubeola) and
German measles (rubella) are characterized by a maculopapular rash that begins on
the face and spreads to include the trunk and extremities. Postauricular and occipital
lymphadenopathy is particularly common in rubella, which is caused by a togavirus.
Rubeola, in contrast, is caused by a paramyxovirus.
(Choice A) Varicella zoster virus tends to produce a maculopapular rash that begins
on the trunk and spreads centrifugally to involve the face and extremities. Roseola
(HHV-6) causes exanthem subitum (also called roseola infantum), which is
characterized by a transient maculopapular rash that appears for a few days on the
chest and trunk once the patient’s fever falls.
(Choice C) An orbivirus in the Reoviridae family is responsible for Colorado tick
fever. This infection occurs primarily in the Rocky Mountain states and is
characterized by fever, vomiting myalgias, and weakness. A maculopapular rash is
not typically seen.
(Choice D) Among the parapoxviruses of medical importance, only measles (rubeola)
typically produces a rash. Rubella is more often associated with postauricular
tenderness than is rubeola.
(Choice E) A parvovirus is responsible for fifth disease (erythema infectiosum). This
illness is first characterized by redness of the cheeks (the “slapped cheeks”
appearance) followed by the appearance of a maculopapular rash on the extremities
and trunk. Postauricular tenderness is not a common manifestation.

Educational Objective:
In a susceptible child, a febrile maculopapular rash that begins on the face and
spreads to include the trunk and extremities is suggestive of rubeola or rubella
(German measles). The additional finding of postauricular lymphadenopathy
indicates that rubella is the most likely etiology.

90
USMLE WORLD STEP 1 MICROBIOLOGY

Q NO 174: Microscopic examination of an exudate specimen from a 32-year-old


Caucasian male reveals a yeast cell with a single bud. When incubated in a A.
specific medium at 75.2° (24°C), a multicellular structure with branching,
tubular cells grows. The patient is most likely infected with:
Aspergillus fumigatus
B. Rhizopus species
C. Malassezia furfur
D. Candida glabrata
E. Cryptococcus neoformans
F. Blastomyces dermatitidis

Explanation:
Certain fungal species display different morphologic characteristics in different
environments. These fungi are “dimorphic” forming molds (with hyphae) in ambient
temperatures (25-30°C) and yeasts (single cells) at body temperature (36°C) pour
species of dimorphic fungi are medically important and tested on the USMLE Step 1.
A comparison table is provided below for study purposes:

(Choice A) Aspergillus fumigatus has a mold form only. It is seen in biopsy


specimens as septate hyphae that form V-shaped branches. Aspergillus mainly
affects immunocompromised patients.
(Choice B) Rhizopus, Mucor, and Absidia are the saprophytic fungi that cause
mucormycosis in immunosuppressed patients. The typical presentation is a
paranasal infection in diabetic ketoacidosis patients. Broad non-septate hyphae are
seen in tissue specimens.
(Choice C) Malassezia furfur causes a cutaneous mycosis (hypopigmented skin
patches). KOH preparation of skin scrapings shows short hyphae and spores
(“spaghetti and meatballs”).
(Choice D) Candida glabrata is a component of normal human flora. It causes
disseminated infections in immunocompromised patients. In tissue specimens itis
seen as a budding yeast with pseudohyphae.

91
USMLE WORLD STEP 1 MICROBIOLOGY

(Choice E) Cryptococcus neoformans causes meningitis in AIDS patients. This fungus


exists in yeast form only. India ink stain of CS reveals encapsulated yeast.

Educational Objective:
Dimorphic fungi grow as molds at 25-30°C and as yeast at body temperature (35-
37°C). Medically important dimorphic fungi include Sporothrix, Coccidioides,
Histoplasma and Blastomyces species.

92
USMLE WORLD STEP 1 MICROBIOLOGY

Q NO 175: A county hospital experiences an outbreak of methicillin-resistant


Staphylococcus aureus (MRSA) wound infections. You suspect that many A.
staff members are carriers of the bacteria. Which of the following would you
expect to be the most commonly MRSA-populated site in staff members?
Hands
B. Axilla
C. Anterior nares
D. Perineum
E. Oropharynx

Explanation:
Humans are a natural reservoir for S. aureus. Very few strains of S. aureus are
susceptible to penicillin, and more than 50% have become methicillin resistant
(MRSA). MRSA can be hospital acquired or community acquired. Asymptomatic
colonization of the nasopharynx is most common, and this is the site sampled by
epidemiologists wishing to study S. aureus carriage. A colonized individual can be a
carrier and can transmit the organism by direct contact. In the general population,
25-30% of individuals have nasal colonization with Staphylococcus aureus. Nasal
carriage increases the risk of infections following surgery, peritoneal dialysis and
hemodialysis.

Educational Objective:
In the general population at any given time, 25-30% of individuals have nasal
colonization with Staphylococcus aureus. The anterior nares are the most common
site of colonization for both methicillin-sensitive and methicillin resistant
Staphylococcus aureus.

93
USMLE WORLD STEP 1 MICROBIOLOGY

Q NO 176: A 32-year-old male presents to the emergency department with


progressive fatigue malaise and lymphadenopathy. When tested for HIV
infection, his blood samples are positive for viral protein p24. The protein A. pol
detected in this patient is a product of which of the following viral genes? B. env
C. gag
D. nef
E. rev
F. tat

Explanation:
A member of the lentivirus subgroup of retroviruses, the human immunodeficiency
virus (HIV) has a diploid genome that consists of single-stranded, positive-sense
RNA. The HIV-1 provirus (ie, the latent form of the virus that is integrated into the
host cell DNA) contains three structural genes (gag, pol, and env) and six regulatory
genes (tat rev, vif, vpr, nef and vpu). While the tat and rev genes are required for
replication, the vif, vpr, ne and vpu genes are not necessary for replication and are
considered accessory genes.
The structural genes code for polyprotein precursors that contribute to virion
assembly. More specifically, the gag gene encodes group-specific antigens in the
virion core such as nucleocapsid proteins p24 and p1, pol encodes several proteins
such as reverse transcriptase, and the env gene indirectly encodes the envelope
glycoproteins gp120 and gp41.
(Choice A) The HIV-1 pol gene encodes reverse transcriptase (which synthesizes
DNA through the use of genomic RNA as a template), integrase (which integrates
viral DNA into cellular DNA), and a protease (which cleaves several viral precursor
proteins).
(Choice B) The HIV-1 env gene encodes gp160, a precursor glycoprotein that is
cleaved into virion envelope glycoproteins gp120 and gp41.
(Choice D)The HIV-1 net gene product appears to enhance viral replication, and is
thought to do so through downregulation of CD4 and MHC class I expression.
(Choice E) The HIV-1 rev gene encodes a protein that facilitates transport of
unspliced viral transcripts out of the nucleus. Once the unspliced mRNAs are in the
cytoplasm, structural proteins can be translated in the later phases of viral
replication.
(Choice F)The HIV-1 tat gene encodes a protein that transcriptionally activates other
viral genes. Expression of this gene is thought to contribute to the virulence of HIV.
Educational Objective:
The DNA form of the HIV genome includes structural and regulatory genes.
Structural gene (gag, p01, and env) products include nucleocapsid proteins p24 and
p1 (from the gag gene) and envelope glycoproteins gp120 and gp41 (from the env
gene). Important enzymes are produced from transcription and translation of the pol
gene. The regulatory tat and rev genes are required for viral replication.

94
USMLE WORLD STEP 1 MICROBIOLOGY

Q NO 177: A 5-year-old male is brought to clinic with a several day history of fever,
irritability and refusal to eat. Physical examination demonstrates painful gingival
ulcers, swollen gums, and cervical lymphadenopathy. Microscopic examination of
the oral ulcer base scrapings is shown on the slide below.

This patient’s current condition most likely represents:

A. Primary infection
B. Virus reactivation
C. Latent infection
D. Abortive infection
E. Slow virus infection

Explanation:
This smear of an oral ulcer scraping is histologically described as a Tzanck
preparation, which requires the application of a stain solution (most commonly the
Wright-Giemsa stain) to epithelial cells that are scraped from an ulcer base. The
presence of multinucleated giant cells with some intranuclear inclusions suggests a
herpes simplex virus (HSV) or varicella zoster virus (VZV) infection, which is
consistent with the provided clinical history of ulcerative gingivitis and cervical
lymphadenopathy. Herpetic gingivostomatitis is the most common clinical
manifestation of primary herpes simplex (HSV-1) infection; it occurs most often in
children aged 1-3 years and results in fever vesiculoulcerative lesions of the oral
mucous membranes, and localized lymphadenopathy.
(Choice B) Reactivation of latent viruses is primarily seen with members of the
herpesvirus family (including HSV-1 HSV-2, VZV, and CMV). HSV-1 that persistently
resides in the trigeminal ganglia is the predominant pathogen responsible for
recurrent oral-labial herpes. In contrast to primary infection with HSV-1 recurrent
lesions are often confined to one side, and the area of involvement is usually one
tenth that of primary infection (and less extensive than the involvement described in
this patient). Herpetic fever blisters or “cold sores” tend to favor the facial skin
around mucosal orifice, including the lips and nose; in contrast, this patient has
gingivostomatitis that appears to involve most of the gums and much of the oral
mucous membrane. Reactivation of VZV that resides in dorsal root ganglia most
commonly produces lesions that are initially vesicular and distributed along a single,
unilateral sensory dermatome. These VZV lesions end abruptly at the midline and
most frequently involve the trunk or sensory distribution of cranial nerve V. Finally,
reactivation of an endogenous, latent CMV infection (as may occur in

95
USMLE WORLD STEP 1 MICROBIOLOGY

immunosuppressed transplant recipients) tends to produce interstitial pneumonitis,


retinitis, hepatitis, colitis and/or generalized disease.
(Choice C) Latency occurs when a full viral genome persists in host cells but
infectious virions cannot be recovered from those cells that harbor the virus.
Because no viral proteins are produced and the full virion replication and release
cycle does not take place during periods of latency a latent virus is not cytopathic.
Latent viruses appear to be protected from immune attack by humoral and possibly
cell-mediated immunity. Thus, a latent infection would not be directly responsible for
this patient’s ulcerative gingivostomatitis. A latent Epstein Barr virus (EBV) infection,
if chronic and transforming, could result in an EBV-associated B-cell lymphoma.
(Choice D) An abortive viral infection results in little or no change in the host cell
and no viral replication. Therefore an abortive infection would not be expected to
cause the cytopathic effects observed in this patient.
(Choice E) A slow virus infection is characterized by a prolonged incubation period
that lasts months to years. During this time the virus may persistently infect the
host and continue to multiply before gradually causing clinically apparent disease.
Slow virus infections are predominantly progressive degenerative disorders of the
central nervous system (eg, subacute sclerosing panencephalitis [SSPE] and
progressive multifocal leukoencephalopathy).

Educational Objective:
Fever, vesiculoulcerative gingivostomatitis, and cervical lymphadenopathy are the
most common clinical manifestations of prima infection with herpes simplexvirus
(HSV-1). Reactivation of a latent HSV infection in the trigeminal ganglia generally
results in more limited perioral blisters or “cold sores.” An abortive viral infection
does not cause significant cytopathic effects.

96
USMLE WORLD STEP 1 MICROBIOLOGY

Q NO 178: A 12-year-old male is treated with nafcillin for a skin infection and shows
little response after two days of therapy. Pus microscopy shows Gram-positive cocci
in clusters, and sensitivity testing demonstrates little response to nafcillin but good
response to vancomycin. Which of the following would most likely explain treatment
failure in this patient?

A. Enzymatic degradation of the drug


B. Poor interaction with binding proteins
C. Mutation in RNA polymerase
D. Active drug transport out of the cell
E. Mutation in DNA gyrase

Explanation:
Gram stain and culture of this skin wound shows gram-positive cocci in clusters that
do not respond to nafcillin but show sensitivity to vancomycin. This is a classic
methicillin-resistant Staphylococcus aureus infection. S. aureus is one of the most
common causes of skin infections in the United States. Most of these skin infections
are minor, but S. aureus also can cause serious infections such as surgical wound
infections, bloodstream infections in the presence of central intravenous catheters
and pneumonia.
Approximately 95% of S. aureus isolates in the United States produce β-lactamase
(penicillinase), which causes resistance to penicillin; however, many S. aureus
strains, while resistant to penicillin, remain susceptible to penicillinase-stable
penicillins such as oxacillin, nafcillin and methicillin. Strains that are oxacillin,
nafcillin and methicillin resistant historically termed methicillin-resistant S. aureus
(MRSA)I are resistant to all 13-lactam agents, including cephalosporins and
carbapenems. Nafcillin (methicillin) resistance is mediated by alterations in the
penicillin-binding protein (PBP) structure. PBPs are the enzymes involved in cell wall
synthesis. Altered PBPs, particularly PB2a, have greatly reduced affinity for all beta-
lactam antimicrobial agents.
(Choice A) S. aureus produces an extracellular enzyme β-lactamase, which
inactivates the β-lactam antibiotics (penicillins) by breaking the β-lactam ring,
rendering them ineffective.
(Choice C) A mutation in RNA polymerase causes resistance to the rifampin.
Rifampin is also effective against S. aureus.
(Choice D) Resistance to tetracycline and sulfonamides is due at least in part to a
decrease in the levels of drug accumulation due to decreased uptake and/or
increased efflux.
(Choice E) Mutations in DNA gyrase cause resistance to quinolone antibiotics.

Educational Objective:
Methicillin-resistant S. aureus (MRSA) is resistant to all β-lactam antibiotics,
including beta-lactamase resistant antibiotics, because it has an altered penicillin
binding protein in its peptidoglycan cell wall that does not bind Beta lactams as
effectively.

97
USMLE WORLD STEP 1 MICROBIOLOGY

Q NO 179: An 18-year-old male with fever myalgias and profound fatigue is found
to have splenomegaly on physical examination. Laboratory testing reveals
lymphocytosis, abundant atypical lymphocytes, and a positive Monospot. Which of
the following sources of infection has the patient most likely come into contact
with?
A. Vaginal secretions
B. Blood
C. Saliva
D. Urine
E. Arthropods

Explanation:
This patient demonstrates many of the typical clinical and laboratory features of
Epstein-Barr virus-induced mononucleosis, including fever, pharyngitis,
lymphadenopathy, hepatosplenomegaly, atypical lymphocytosis, and a positive
Monospot test (ie, positive heterophil antibodies). Typically, the Epstein-Barr virus is
transmitted from an asymptomatic virus shedder to a susceptible individual through
saliva transfer (eg, kissing).
(Choice A) Vaginal secretions can harbor pathogens such as HIVI Neisseria
gonorrhoeae, Chlamydiae trachomatis, and Trichomonas vaginalis.
(Choice B) Blood can harbor pathogens such as hepatitis B virus, hepatitis C virus,
and human immunodeficiency virus (HIV).
(Choice D) Urine can harbor pathogens such as cytomegalovirus, adenovirus,
Escherichia coli, Staphylococcus saprophyticus, and Klebsiella pneumoniae.
(Choice E) Arthropods (eg, ticks) can harbor pathogens such as Rickettsia rickettsii
and Coxiella burnetii.

Educational Objective:
Typical clinical and laboratory features of Epstein-Barr mononucleosis include fever,
pharyngitis, lymphadenopathy, hepatosplenomegaly, atypical lymphocytosis, and a
positive Monospot test (ie, positive heterophil antibodies). EBV is generally
transmitted from an asymptomatic virus shedder to a susceptible individual through
saliva transfer (eg, kissing).

98
USMLE WORLD STEP 1 MICROBIOLOGY

Q NO 180: A 45-year-old sailor presents to your office with penile lesions. He says
that he first noticed a papule on his penis. Soon thereafter, several draining ulcers
formed and his inguinal region began to hurt. He also complains of mild fever and
red eyes. Cell scrapings from the genital lesions show inclusion bodies. Which of
the following is the most likely cause of this patient’s condition?

A. Hemophilus ducreyi
B. Calymmatobacterium inguinale
C. Herpes simplex virus (HSV)
D. Treponema pallidum
E. Chlamydia trachomatis

Explanation:
Chlamydia trachomatis is a small, gram-negative, obligate intracellular bacterium
responsible for the most common sexually transmitted disease in the United States.
The multiple different serotypes of C. trachomatis cause a range of diseases.
Serotypes A through C cause an ocular infection (trachoma) in children; serotypes D
through K cause urogenital (sexually transmitted) infections and inclusion
conjunctivitis; and serotypes L1-L3 cause lymphogranuloma venereum.
Lymphogranuloma venereum is a chronic disease characterized by an initial small
papule on the genital mucosa that contains cells infected with C. trachomatis. This
papule is followed weeks later by swollen, painful inguinal nodes which coalesce,
ulcerate, and rupture. If left untreated, this condition can cause fibrosis and
anogenital strictures; the latter is frequently observed in women. Although
lymphogranuloma venereum is not often seen in the United States or Western
Europe, it is endemic in regions of Africa, Asia, the Caribbean, and South America.
Histologically, the lesions of lymphogranuloma venereum contain mixed
granulomatous and neutrophilic inflammation. Chlamydial inclusion bodies are
present in the epithelial and inflammatory cell cytoplasm. Lymph nodes demonstrate
granulomatous inflammation with stellate abscesses of necrosis and neutrophils that
gradually transition into nonspecific chronic inflammation and fibrosis. This fibrosis
can cause lymphatic obstruction and strictures.
(Choice A) Hemophilus ducreyi causes an ulcerative, sexually transmitted infection
termed chancroid. Within a few days of inoculation, a tender, red papule appears on
the external genitals. This papule then erodes to become a painful, irregular ulcer
with a shaggy base and yellow-gray exudate. The regional lymph nodes swell and
may become chronic ulcers themselves.
(Choice B)The encapsulated coccobacillus Calymmatobacterium inguinale causes the
sexually transmitted granuloma inguinale (donovanosis). If untreated, the infection
can lead to severe lymphatic obstruction and lymphedema (elephantiasis).
Donovanosis first manifests as a papule on the genitalia that eventually ulcerates
and becomes an exophytic, soft, painless mass with indurated borders. Scarring and
strictures can ultimately develop.
(Choice C) Herpes simplexvirus 2 is typically associated with genital herpes, a
sexually transmitted disease characterized by vesicles on the mucous membranes of
genitalia that soon thereafter become superficial ulcers encircled by inflammation.
Large pinkish purple intranuclear inclusions (Cowdry type A bodies) press the
nuclear chromatin to the edge of the nucleus.
(Choice D) Treponeme pallidum is the gram-negative spirochete responsible for
syphilis. The primary stage of syphilis is characterized by a firm, painless, raised, red
lesion (chancre) on the genitalia that heals without treatment within a few weeks.

99
USMLE WORLD STEP 1 MICROBIOLOGY

Educational Objective:
Serotypes L1-L3 of Chlamydia trachomatis cause lymphogranuloma venereum, a
sexually transmitted disease histologically characterized by chlamydial inclusion
bodies in the cellular cytoplasm.

100
USMLE WORLD STEP 1 MICROBIOLOGY

Q NO 181: A virus-encoded polypeptide detected in infected human cells has a


specific protease activity essential for generating several functional viral
proteins from a large precursor polypeptide. Which of the following viruses is A.
most likely studied in the experiment?
Measles virus
B. Rabies virus
C. Influenza virus type A
D. Rotavirus
E. Echovirus

Explanation:
All viral replication requires transcription of viral specific mRNA directly or indirectly
from the viral genome. The transcription allows viral proteins to be synthesized by
the host cell ribosomal machinery for translation. This question describes a specific
situation in which the viral specific mRNA is polycistronic (meaning it contains
multiple cistrons, or genes, within the same mRNA transcript) and produces a
polyprotein product or functional individual viral proteins to be generated, the
polyprotein product must be cleaved by a specific viral protease. The viruses that
demonstrate this particular method of viral replication include the single-stranded,
positive-sense, linear, nonsegmented RNA viruses. A SS+ virus has a genome that is
independently infective if transferred into a host cell and that functions as its own
mRNA. Among the viruses listed, only echovirus possesses an 33+ RNA genome.
(Choices A, B, and C) These viruses contain single-stranded, negative-sense, linear
RNA (the measles and rabies viruses are non-segmented, while the influenza virus is
segmented). Within the viruses is an RNA-dependent RNA polymerase that can form
viral specific, positive-stranded mRNAs that are smaller than the total size of the
viral genome. Thus these mRNAs are not polycistronic, and a specific viral protease
need not be present to generate individual viral proteins.
(Choice D) This reovirus contains double-stranded, linear, segmented RNA. The
virion contains an RNA polymerase that transcribes each segment into viral specific
mRNA. Thus, the mRNA is not polycistronic and a specific viral protease need not be
present to generate individual viral proteins.

Educational Objective:
All viral replication requires transcription of viral specific mRNA directly or indirectly
from the viral genome. When the viral genome consists of single-stranded, positive-
sense, linear, nonsegmented RNAI it serves as its own polycistronic mRNA and
produces a large precursor polyprotein product. This product must be cleaved by a
protease to generate a complete set of functional individual viral proteins.

101
USMLE WORLD STEP 1 MICROBIOLOGY

Q NO 182: A 7-year-old Caucasian male is brought to the emergency department


with a facial rash that looks like “slapped cheeks.” The mother says that the boy
has had some malaise and low-grade fever over the last few days. The infectious
agent responsible for this patient’s disease is most likely to replicate in which of
the following tissues?
A. Lymph nodes
B. Alveolar epithelium
C. Bone marrow
D. Kidney tubules
E. Intestinal epithelium

Explanation:
The prodrome of low-grade fever, headache, malaise, and upper respiratory
symptoms followed by the sudden appearance of red, flushed cheeks seven to ten
days later is characteristic of erythema infectiosum (“fifth disease”), a common
childhood exanthem. This exanthem is thought to be caused by the small DNA
nucleocapsid virus parvovirus B19. The blood group P antigen globoside is a
parvovirus B19 receptor that is expressed on mature erythrocytes, erythroid
progenitors, megakaryocytes, endothelial cells, placenta, and fetal liver and heart.
As a result, parvovirus B19 is highly tropic for erythrocyte precursors — particularly
pronormoblasts and normoblasts — and is found predominantly in the bone marrow.
The rash that follows the initial viremic period in erythema infectiosum is thought to
at least partly result from local immune complex deposition once serum levels of
virus-specific 1gM and IgS have attained high enough levels.
(Choices A, B, D and E) Lymph nodes, alveolar epithelium, renal tubular epithelium,
and intestinal epithelium are not significant targets of direct parvovirus infection.

Educational Objective:
A prodromal febrile upper respiratory illness in a child followed by the sudden
appearance of red, flushed cheeks approximately seven to ten days later is
characteristic of erythema infectiosum due to parvovirus B19. This virus is highly
tropic for erythroid precursor cells and predominantly replicates in the bone marrow.

102
USMLE WORLD STEP 1 MICROBIOLOGY

Q NO 183: A 44-year-old male with a localized rash on his chest is positive for
varicella (VZV) lgG antibodies. Which of the following is the most accurate
statement about this patient’s condition? A.
The patient is susceptible to chicken pox
B. The patient has primary infection with WV
C. The patient is susceptible to herpes zoster
D. The patient has never been vaccinated against VZV
E. Strong cell-mediated immunity against WV is likely

Explanation:
The clinical combination of varicella lgG antibodies and a dermatome-centered,
vesicular, painful rash is strongly suggestive of herpes zoster (“shingles”). Herpes
zoster results from the reactivation of latent varicella zoster virus (VZV) infection
within the dorsal root ganglia, and most frequently occurs in the elderly or the
immunocompromised.
Primary infection with VZV typically occurs in children and is a highly contagious
condition described as “chickenpox.” Clinically, children with chickenpox present with
fever, malaise pharyngitis, and a generalized vesicular and pruritic rash. The skin
lesions appear as successive crops on the face, trunk, and limbs, and most have
crusted within six days of onset. Children who have experienced chickenpox infection
are typically resistant to future episodes of chickenpox, but may develop herpes
zoster later in life.
The varicella vaccine is now recommended for all children aged 12 to 18 months
women of childbearing age, adults with sustained risk of exposure, and household
contacts of immunocompromised hosts. Vaccine-induced immunity appears to
dissipate overtime, however.
(Choice A) Once an individual has had a primary WV infection, circulating IgO
varicella antibodies confer a high degree of natural immunity against reinfection with
VZV. Thus, a future episode of chickenpox would be unlikely in this patient. The lgG
varicella antibodies do not, however, confer immunity against herpes zoster
(shingles) because they are ineffective against latent virus in the dorsal root ganglia.
(Choice B) lgG varicella antibody titers are usually not significantly elevated until ten
days after the symptomatic onset of chickenpox. This patient already has
measurable lgG varicella antibodies, which suggests that he experienced primary
VZV infection some time ago.
(Choice D) Varicella vaccination dramatically decreases the incidence of primary VZV
infection. On rare occasion, vaccinated individuals will still develop subsequent
primary VZV infection (and, potentially, herpes zoster). When this occurs, the VZV
infection is usually mild.
(Choice E) Varicella reactivation most often occurs in individuals with suppressed
cellular immunity.

Educational Objective:
IgG varicella antibodies suggest an antecedent primary varicella zoster virus (WV)
infection. These antibodies generally confer immunity against chickenpox re-
infection but not against herpes zoster.

103
USMLE WORLD STEP 1 MICROBIOLOGY

Q NO 184: A 35-year-old male who received a kidney transplant one year ago
presents to the ER because of pleuritic chest pain and cough. He is found to
have a low-grade fever. Chest x-ray shows an infiltrate in the lower lobe of
the right lung. The patient is started on broad-spectrum antibiotics, including
vancomycin, ceftriaxone and azithromycin. Despite these measures, his
condition worsens over the next several days. Mucicarmine staining of his
bronchoalveolar fluid reveals the following on light microscopy.

Which of the following is the most likely cause of this patient’s condition?

A. Rhizopus species
B. 4kspergillus fumigatus
C. Candida albicans
D. Cryptococcus neoformans
E. Blastomyces dermatitidis
F. Histoplasma capsulaturn
G. Coccidioides immitis

Explanation:
Mucicarmine stain is used to detect the polysaccharide capsule of Cryptococcus
neoformans. The polysaccharide capsule of C. neoformans is a major virulence
factor. The image above shows a red-stained capsule which is typical for this fungus.
C. neoformans is seen in tissue as round yeast cells with narrow-based buds.
C. neoformans usually affects immunocompromised patients (like the kidney
transplant patient in the above scenario). It is transmitted via the respiratory route,
and may cause pulmonary disease. Although usually asymptomatic, cryptococcal
lung disease can cause pneumonia-like symptoms. Patients complain of cough with
scant sputum production and pleuritic chest pain. Dyspnea and hemoptic is may also
occur. Chest x-ray findings are non-specific, and may show infiltrates or nodules.
The diagnosis is made by identifying Cryptococcus in sputum, bronchoalveolar
washings, or tissue samples. Methenamine silver (GMS) and mucicarmine stains are
used.
C. neoformans is a neurotropic fungus. Subacute or chronic meningoencephalitis is
the most common presentation of cryptococcosis.
(Choice E) Blastomyces dermatitidis can cause both lung disease and disseminated
mycosis. Its microscopic appearance in tissue is that of round yeast with broad-
based budding and a thick doubly-reflective wall.

104
USMLE WORLD STEP 1 MICROBIOLOGY

(Choice F) Histoplasma capsulaturn is a dimorphic fungus that causes tuberculosis-


like pulmonary disease. It can also cause disseminated mycosis in
immunocompromised patients. It is found intracellularly in tissue (within
macrophages), appearing as small ovoid budding yeast cells.
(Choice G) Coccidioides 1mm/Us causes lung disease in immunocompetent people
and disseminated mycosis in immunocompromised individuals. In tissue samples it
appears as large, irregularly sized thick-walled spherules that contain small round
endospores.

Educational Objective:
Cryptococcus neoformans is the only pathogenic fungus that has a polysaccharide
capsule. The capsule appears red on muci carmine stain and as a clear unstained
zone with India ink.

105
USMLE WORLD STEP 1 MICROBIOLOGY

Q NO 185: A 39 year-old paraplegic man with an indwelling bladder catheter


presents to the emergency room complaining of twenty-four hours of rigors
nausea, and vomiting. His temperature is 38.9°C (102.0°F). Urinalysis shows A.
3+ leukocyte esterase and numerous white blood cells. Urine and blood
cultures grow non-lactose fermenting gram-negative bacilli. Which of the
following pathogens is the most likely culprit?
Escherichia coil
B. Pseudomonas aeruginosa
C. Klebsiella pneumoniae
D. Staphylococcus saprophyticus
E. Enterococcus cloacae
F. Ureaplasma urealyticum

Explanation:
Patients with indwelling bladder catheters are at increased risk for urinary tract
infections caused by both typical and opportunistic organisms. Typical organisms
include Escherichia coil, Klebsiella pneumoniae, Staphylococcus saprophyticus, and
Proteus mirabilis. This question asks you to identify an opportunistic organism
according to microbiology. We can identify Pseudomonas aeruginosa as the
pathogen because it is the only non-lactose fermenting gram-negative rod listed. It
is also a common cause of urinary tract infections in patients with indwelling bladder
catheters.
(Choice A) E. coil is a lactose-fermenting gram negative rod and the leading cause of
urinary tract infections (UTI).
(Choice C) K. pneumoniae is also a lactose-fermenting gram-negative rod. In
addition to UTI it is a leading cause of spontaneous bacterial peritonitis and
nosocomial pneumonias.
(Choice D) Staphylococcus saprophyticus is a gram-positive coccus that typically
causes UTI in sexually active women. It has a seasonal distribution, being worse in
the summer.
(Choice E) An enterococcus cloaca is a gram-positive coccus in the Streptococcus
family. It is also a common cause of opportunistic UTI. Vancomycin-resistant
Enterococcus is a particularly fearsome opportunistic organism.
(Choice F) U. urealyticum is a known cause of urethritis but it requires special media
rich in urea and cholesterol to grow.

Educational Objective:
P. aeruginosa is a non-lactose fermenting gram-negative organism. It is a common
cause of urinary tract infections in patients with indwelling bladder catheters.

106
USMLE WORLD STEP 1 MICROBIOLOGY

Q NO 186: A 68-year-old female presents with a unilateral vesicular rash that


involves a single dermatome on her chest. The patient’s past medical history
is significant for hypertension controlled with metoprolol. She receives annual A.
influenza vaccinations. Which of the following is the patient most likely to
suffer from within the next six months?
Visual impairment
B. Recurrent skin rash
C. Persistent local pain
D. Normochromic anemia
E. Hepatomegaly

Explanation:
A unilateral vesicular rash localized on a single dermatome in an elderly patient is
most likely herpes zoster. Herpes zoster arises when latent varicella zoster virus
(VZV) infection is reactivated within a single dorsal root sensory ganglion.
Localized dermatomal pain that persists for more than one month after a zoster
eruption is termed post-herpetic neuralgia and is the most common neurological
complication of VZV infection. Post-herpetic neuralgia is typically described as
“stabbing” may be constant or intermittent, and often lasts for several months.
Although post-herpetic neuralgia affects only 10% of all patients after an episode of
herpes zoster, it affects 25-50% of patients older than 50 years, 50% of patients
older than 60 years, and more than 75% of patients older than 70 years.
(Choice A) Visual impairment can be a complication of herpes zoster ophthalmicus,
which is a condition caused by VZV infection of the trigeminal ganglion of cranial
nerve V1. However, this patient’s zoster is localized to a thoracic dermatome-- most
likely in the mid to lower thoracic region (T3—T12).
(Choice B) While herpes simplex is associated with recurrent vesicular skin rashes,
herpes zoster does not usually recur in immunocompetent hosts.
(Choices D and E) Normocytic anemia and hepatomegaly are not common
complications of VZV infection.

Educational Objective:
A unilateral vesicular rash localized on a single dermatome in an elderly patient is
most likely herpes zoster. Postherpetic neuralgia is the most common neurological
complication of varicella zoster virus infection.

107
USMLE WORLD STEP 1 MICROBIOLOGY

Q NO 187: A 54-year-old Caucasian male diagnosed with HIV infection two years
ago develops cough, low-grade fevers and hepatosplenomegaly. He has lost eight
pounds over the past month. A light microscopy image of a bone marrow
aspirate is shown below.

Which of the following is the most likely cause of this patient’s condition?

A. Rhizopus species
B. Aspergillus fumigatus
C. Candida albicans
D. Cryptococcus neoformans
E. Blastomyces dermatitidis
F. Histoplasma capsulaturn
G. Coccidioides immitis
H. Sporothrix schenckii

Explanation:
This image shows small ovoid bodies within a macrophage. Histoplasma capsulatum
is the only answer choice listed that can survive intracellularly and cause systemic
disease. In non-immunocompromised individuals, Histoplasma infection is
asymptomatic or may produce a self-limiting pulmonary disease.
Immunocompromised individuals can develop systemic histoplasmosis that may
prove fatal. Disseminated histoplasmosis causes hepatosplenomegaly because of its
predilection for the reticuloendothelial system. Ulcerated lesions on the tongue are
also very characteristic of disseminated histoplasmosis.
Chest x-ray of a patient with disseminated histoplasmosis may show diffuse
pulmonary infiltrates with hilar lymphadenopathy. In chronic lung disease, the
radiographic changes resemble those of pulmonary tuberculosis: cavitary lesions
form in the upper lung lobes, and calcified nodes and fibrotic scarring may also be
present.
Examination of lung biopsy specimens and bone marrow aspirates reveals oval or
round yeasts within macrophages. Culture on Sabouraud’s agar will grow hyphae (as
Histoplasma is a dimorphic fungus). Histoplasma antigen in blood and urine can be
detected by radioimmunoassay. Serologic tests (e.g. complement fixation
immunodiffusion) can be used to measure the level of anti-1-Iistoplasme antibodies.
(Choice B) Aspergillus fumigatus is a monomorphic fungus with only a mold form. It
is seen in tissue as septate hyphae with V-shaped branching.
(Choice C) Candida albicans is the most frequent opportunistic pathogen. It is
extracellular, and forms yeast and pseudohyphae.

108
USMLE WORLD STEP 1 MICROBIOLOGY

(Choice E) Blastomyces appears as round yeasts, with single broad-based buds and
thick, doubly-refractive walls.
(Choice G) Coccidioides immitis also causes disseminated mycosis in
immunocompromised patients. In tissue sections it is seen as large spherules
containing endospores.

Educational Objective:
H. capsulatum can survive intracellularly within macrophages. It causes a
disseminated mycosis in immunocompromised patients. The clinical features include
systemic symptoms (fever and weight loss), painful oral ulcers, lymphadenopathy
and hepatosplenomegaly.

109
USMLE WORLD STEP 1 MICROBIOLOGY

Q NO 188: A 45-year-old male presents to your office because of occasional


hemoptysis. His past medical history is significant for tuberculosis that was A.
effectively treated several years ago. Ultimately, the apical region of his
right lung is resected. Gross examination reveals a fibrous cavity partially
filled with a round mass of hyphae. This patient’s condition is best described
as:
Contagious
B. Necrotizing
C. Allergic
D. Colonizing
E. Invasive
F. Disseminated

Explanation:
Aspergillus fumigatus is a mold that is widely present in organic matter. It forms
septate hyphae that branch at 45° angles (V-shaped branching). The spores of
Aspergillus are inhaled with the air and are cleared by the mucus and ciliated
epithelium of the respiratory tract. In individuals with suppressed immune defenses
Aspergillus causes a number of diseases.
Colonizing aspergillosis occurs in old lung cavities (e.g. from tuberculosis,
emphysema, or sarcoidosis). Aspergillus does not invade the lung tissue, but grows
inside the cavity, forming a “fungus ball” or aspergilloma. This condition maybe
asymptomatic or it may cause cough and hemoptysis. On chest x-ray, an
aspergilloma will appear as a radiopaque structure that shifts when the patient
changes position.
Hypersensitivity reactions to Aspergillus occur in patients with asthma, producing a
condition called allergic bronchopulmonary aspergillosis (ABPA). Wheezing, fever and
migratory pulmonary infiltrates are characteristic.
Invasive aspergillosis occurs in immunosuppressed and neutropenic patients and is
characterized by primary lung involvement with symptoms of cough hemoptysis,
pleuritic chest pain and fever. Necrotizing pneumonia and granuloma formation also
occur. Lung biopsy shows characteristic hyphae invading the lung tissue.
Extrapulmonary involvement is common in invasive aspergillosis.
(Choice A) Aspergillosis is not contagious. Aspergillus fumigatus is present in the
environment, and causes opportunistic infections in immunocompromised patients.
It does not cause disease in immunocompetent individuals.
(Choice B) Aspergillus has a predilection for blood vessels and spreads
hematogenously. Invasive aspergillosis can cause interruption of blood supply and
tissue necrosis.
(Choice C) Allergic bronchopulmonary aspergillosis (ABPA) occurs in patients with
asthma. It presents with wheezing and migratory pulmonary infiltrates. Increased
titers of IgE and antibodies to Aspergillus are diagnostic.
(Choice E) Invasive aspergillosis develops in immunosuppressed patients, especially
those with neutropenia. The lung is the most common area of involvement, with
development of necrotizing pneumonia and granulomas.
(Choice F) Invasive aspergillosis can become disseminated. Aspergillus spreads
hematogenously and can affect any organ system.
Educational Objective:
Aspergillus fumigatus causes opportunistic infections in immunosuppressed and
neutropenic patients (invasive aspergillosis). Aspergillosis can also be colonizing
(aspergilloma) and allergic (ABPA).

110
USMLE WORLD STEP 1 MICROBIOLOGY

Q NO 189: A 45-year-old Caucasian male with advanced HIV infection undergoes


bronchoscopy for cough and chest pain. Mucicarmine staining of his
bronchoalveolar fluid shows budding yeast forms with thick capsules. Infection
with this organism most commonly manifests as:
A. Oral plaques
B. Esophagitis
C. Skin infection
D. Interstitial pneumonia
E. Meningitis
F. Sinusitis

Explanation:
Cryptococcus neoformans affects only immunocompromised patients, and is,
therefore, an opportunistic pathogen. It is present in the environment in soil and
pigeon droppings. C. neoformans is transmitted by the respiratory route, but it is not
acquired via person-to-person contact. Inhaled yeast enters the lungs and is cleared
in immunocompetent persons by macrophages and T-cells. In people with an
impaired cellular immune response, C. neoformans causes symptomatic disease.
Lung infection occurs first, but it is commonly asymptomatic. In some patients, C.
neoformans may cause cough with scant sputum production dyspnea or pleuritic
chest pain. The diagnosis of pulmonary cryptococcosis is made by microscopic
examination of bronchopulmonary washings and lung tissue. Cryptococcal yeast
appears red on mucicarmine stain.
In patients with HIVI sarcoidosis, or leukemia, and in those on high-dose steroid
therapy Cryptococcus commonly causes meningitis. Headache, nausea, vomiting and
confusion are the common symptoms. Diagnosis is made by examining cerebrospinal
fluid stained with India ink. The round budding yeast have peripheral clearings or
“halos” due to their thick polysaccharide capsules. Serologic testing (latex
agglutination) is used for the detection of C. neoformans capsular antigen in CSF.
(Choice A) White oral plaques are a common manifestation of Candida infection (oral
thrush). Oral thrush is associated with diabetes mellitus, immunosuppression (such
as HIV), and treatment with antibiotics and steroids.
(Choice B) Esophagitis in an HIV (+) patient is most commonly caused by Candida
infection. Esophagitis presents with odynophagia, and is diagnosed by endoscopy
with biopsy.
(Choice C) Cryptococcus may cause a cutaneous mycosis that manifests with
papules, pustules, nodules and ulcers. Cutaneous cryptococcal disease is rare
(occurs in 10% of cases).
(Choice D) Cryptococcal pulmonary disease causes granuloma formation (nodules).
It usually does not cause interstitial pneumonia. Again, meningitis is more common
than pulmonary disease.
(Choice F) Mucormycosis classically affects the paranasal sinuses. Mucormycosis
occurs in immunocompromised patients and is strongly associated with diabetes
mellitus and diabetic ketoacidosis. Aspergillosis fumigatus is another fungus that
may cause sinusitis.

Educational Objective:
Meningitis is the most common presentation of Cryptococcus neoformans infection.
It occurs in immunosuppressed patients and can be diagnosed by India ink staining
of the CSF. Cryptococcal pneumonia is diagnosed by mucicarmine staining of lung
tissue and bronchoalveolar washings.

111
USMLE WORLD STEP 1 MICROBIOLOGY

Q NO 190: A 62-year-old Caucasian male who recently underwent a mitral valve


replacement is having low-grade fevers. He also complains of dyspnea and
malaise. Repeated blood cultures grow Gram-positive cocci in clusters that are
catalase-positive and coagulase-negative. Which of the following is the best
initial treatment for this patient?
A. Penicillin 0
B. Nafcillin
C. Vancomycin
D. Ciprofloxacin
E. Erythromycin
F. Ceftriaxone

Explanation:
The coagulase-negative staphylococci, particularly Staphylococcus epidermidis, have
been identified as a major cause of infections in patients with predisposing factors
such as indwelling catheters or implanted foreign bodies. These bacteria have the
ability to colonize intravenous catheters and prosthetic devices (heart valves,
vascular grafts. peritoneal dialysis catheters) because of their ability to produce a
polysaccharide slime which allows adherence to the prosthetic devices. When
reported, it is initially difficulty to differentiate culture contamination from real
infection because S. epidermidis is a part of the normal skin flora but recovery of
organism from multiple blood cultures indicates infection. These organisms can be
resistant to multiple antibiotics and initial aggressive antibiotic treatment is a must
pending antibiotic sensitivity analysis. S. epidermidis isolates are susceptible in vitro
to vancomycin and rifampin and are frequently resistant to methicillin; therefore
vancomycin combined with rifampin or gentamicin or both is recommended for
therapy of serious infections caused by methicillin-resistant strains. S. epidermidis
can cause an indolent endocarditis after valve replacement (compared to the
aggressive endocarditis associated with S. aureus in IV drug abusers) but if left
untreated it can lead to intracardiac abscess formation dehiscence of the prosthetic
valve from the heart, and septic embolization.
(Choice A) Very few strains of S. epidermidis are susceptible to penicillin G. As with
any positive blood culture in a symptomatic patient proper treatment involves initial
empiric broad-spectrum coverage pending organism identification and antimicrobial
susceptibility. Subsequently antibiotic therapy can be tailored to the specific
sensitivities of the offending organism.
(Choice B) Semisynthetic β—lactamase resistant penicillins include methicillin,
oxacillin, nafcillin and a few other antibiotics. Staphylococcus isolates from hospital-
acquired infections are frequently resistant to methicillin.
(Choice D) Ciprofloxacin is a fluoroquinolone antibiotic. Ciprofloxacin-resistance
appears to accompany methicillin resistance in the Staphylococci, and the likely
mechanism involves the encoding of multiple genes conveying antibiotic resistance
on the same plasmid, which then can be spread to other bacteria by conjugation or
bacteriophages.
(Choice E) Clindamycin can be used for the treatment of certain infections caused by
oxacillin-resistant Staphylococcus. It is documented that constitutive and inducible
clindamycin resistance exists in some S. aureus strains that are resistant to
erythromycin. If an isolate is resistant to erythromycin and susceptible to
clindamycin, and the patient is exposed to clindamycin, the chances are that the
patient will quickly develop resistance due to an inducible mechanism of resistance.

112
USMLE WORLD STEP 1 MICROBIOLOGY

(Choice F) Ceftriaxone is a third generation cephalosporin therefore, ceftriaxone will


have increased anti-Gram negative activity and decreased anti-Gram positive
activity. Even if the question response was a first-generation cephalosporin such as
cephalexin, this response would still be incorrect because the mechanism of
methicillin resistance is an alteration of the penicillin binding protein—the same
protein bound by the cephalosporins. If an organism such as S. epidermidis is
methicillin resistant, it will also likely be resistant to the cephalosporins.

Educational Objective:
The treatment of a coagulase-negative staphylococcal infection involves initial
empiric treatment with vancomycin with or without rifampin or gentamicin due to
the widespread antibiotic resistance of S. epidermidis, especially in nosocomial
infections.

113

You might also like